Vous êtes sur la page 1sur 325

UNIVERSIDAD NACIONAL AUTÓNOMA DE MÉXICO

FACULTAD DE CONTADUR[A Y ADMINISTRACIÓN


PLAN DE ESTUDIOS DE LA LICENCIATURA EN ADMINISTRACIÓN
Programa de la asignatura

Denominación: Razonamiento lógico matemático para la toma de decisiones


rea o campo de conocimiento; eje, bloque, ciclos o
Semestre: No. Créditos:
Clave: 1424 tronco curricular: Matemáticas
8
Carácter: obligatoria Horas por clase Horas por semana Horas al semestre
Teoría: Práctica:
Tipo: Teórico 4 64
2
Modalidad: Curso Duración del programa: 16 semanas

Seriación (Obligatoria/Indicativa): Ninguna


Asignatura con seriación antecedente:
Asignatura con seriación subsecuente:

Objetivo(s) de la asignatura:
Objetivo general:
Al finalizar el curso, el estudiante dominara los fundamentos matemáticos a fin de desarrollar
habilidades de razonamiento lógico-matemático que le permitan analizar situaciones hipotéticas
y de la vida real para la resolución de problemas. Asimismo será capaz de acreditar
evaluaciones de razonamiento matemático y habilidades cuantitativas.

Objetivos específicos:
Proporcionar las bases teórico-prácticas para que el alumno:

Refuerce y domine los fundamentos de aritmética, algebra y geometría.


Analice y resuelva problemas por medio de la lógica-matemática.
Desarrolle habilidades de comprensión, análisis y razonamiento matemático para la resolución
de problemas de la vida real.
Aprenda a interpretar resultados de modelos matemáticos para sustentar la toma de
decisiones.

lndice Te mático
Unidad Tema Horas
Teóricas Prácticas
1 Fundamentos para el análisis matemático 20 o
2 Introducción a las evaluaciones de habilidades cuantitativas 4 o
3 Solución de problemas y suficiencia de datos 12 o
4 Algebra y tópicos espeéiales de matemáticas 16 o
Métodos cuantitativos aplicados a los negocios y la toma de
5
decisiones
12 o
Total de horas: 64 o
Contenido Temático

Unidad Tema y subtemas


Fundamentos para el análisis matemático

1. Principios del análisis aritmético


a. Resolución de ejercicios con:
i. Propiedades de los números.
ii. Fracciones y decimales
iii. Escalas y proporciones.
iv. Exponentes y radicales.
2. Principios del análisis algebraico
1
a. Resolución de ejercicios con:
i. Simplificación algebraica, polinomios y factorización.
ii. Ecuaciones lineales, inecuaciones, sistemas de
ecuaciones y ecuaciones cuadráticas.
3. Principios del análisis geométrico
a. Resolución de ejercicios con:
i. Líneas, ángulos, áreas y perfmetros
ii. Triángulos, polfgonos y circunferencias

Introducción a las evaluaciones de habilidades cuantitativas

1. Estructura y funcionamiento de las evaluaciones de habilidades


cuantitativas.
2
a. Estructura de los ejercicios Problem Solving
b. Estructura de los ejercicios Data Sufficiency

Solución de problemas y suficiencia de datos

1. Análisis, comprensión y resolución de ejercicios Problem Solving


3
2. Análisis, comprensión y resolución de ejercicios de suficiencia de datos
(Data Sufficiency)

Algebra y tópicos especiales de matemáticas

1. Construcción de modelos algebraicos.


a. Introducción al modelado
b. Convirtiendo texto en expresiones y ecuaciones
c. Representación grafica de ecuaciones lineales
4 2. Análisis cuantitativo
a. Definición del problema
b. Desarrollo del modelo
c. Datos de entrada
d. Solución y análisis de resultados
e. 1mplementación

Métodos cuantitativos aplicados a los negocios y la toma de decisiones


5.
1. Aplicaciones de modelos matemáticos a la solución de problemas y la toma
de decisiones.
Bibliografía básica:
l. AYRAJ.y R. Lardner, Matemáticas aplicadas a la administración y a la economía,
México: Prentice-Hall, Sª edición, 2009, 818 pp.
2. BUDNICK F., Matemáticas aplicadas para administración, economía y ciencias
sociales, México: McGraw-Hill, 4ª edición, 2007,1033 pp.
3. BURDEN L. y Faires J., Análisis numérico, México: Cengage Learning, 9ª edición, 2011,
888 pp.
4. KAUFMANN Jerome E., Algebra, México: Cengage Learning, 8ª edición, 2010, 920 pp.
S. RENDER Barry, Métodos cuantitativos para los negocios, México: Pearson Educación,
9ª edición, 2006, 731 pp.
6. SWOKOWSKI Earl W., Algebra y trigonometría con geometría analítica, México:
Cengage Learning, 12ª edición, 2009, 1032 pp.

Bibliografía complementaria:
l. CHENEY W. y Kincaid D., Métodos numéricos y computación, México: Cengage
Learning, 6ª edición, 2011, 792 pp.
2. DULAN Steven W., GMAT exam prep, lndianapolis: Pearson Education, 2007,418 pp.
3. GMAT Course Book, Ka plan, 2008,449 pp.
4. HARSHBARGER Ronald J., Matemáticas aplicadas a la administración, economía y
ciencias sociales, México: McGraw-Hilllnteramericana, 7ª edición, 200S, 9S9 pp.
S. MA~TZ Geoff, Cracking the GMAT, New York: Random House, 2007, S3S pp.
6. The official guide for GMAT review, Wiley, 12ª edición, 2009, 840 pp.

Sugerencias didácticas: Mecanismos de evaluación del aprendizaje de los


alumnos:
Exposición oral del profesor (X)

Exposición audiovisual del profesor (X) Exámenes parciales 70%

Análisis y resolución de ejercicios (X) Participación en clase 10%

Ejercicios de tarea (X) Asistencia y puntualidad 20%

Lecturas obligatorias (X) Total 100%

Perfil profesiográfico:

Profesor con grado académico mínimo de licenciatura preferentemente de maestría, con


experiencia docente, con conocimiento previo de la estructura utilizada en las evaluaciones de
habilidades cuantitativas.
®

MATH WORKBOOK

ENDER MARKAL, MBA, CFA

~
1:\1 tJ [l)~li:J

GMAT·· is a registered trademark of the Graduate Management Admission Councii~(GMAC"). This book does not contain actual GMAT items and is not
endorsed by or affiliated in any way with GMAC.
About the Author

Ender Markal is the founder of SFTutors, a San Francisco-based tutoring organization that helps students succeed
on standardized tests and gain admission to top graduate schools. He has tutored math for almost 20 years, helping
students overcome math anxiety and gain admission to top schools. He has a B.S. in mechanical engineering, an MBA
in finance, and is a CFA.

©Copyright 2011 by Barron's Educational Series, Inc.

Al! rights reserved.


No part of this publication may be reproduced in any form or by any means v.rithout the written permission of the
copyright owner.

All inquiríes should be addressed to:


Barron's Educational Series, Inc.
250 Wireless Boulevard
Hauppauge, New York 11788
www.barronseduc.com

ISBN: 978-0-7641-4534-6

Library ofCongress Control Number: 2010931205

Printed in the United States of America


987654321
CONTENTS

1 Getting Started ........................................................................................................................... 1


1.1 Laying the Foundation ........................................................................................................... 1
1.2 Welcome.to the GMAT ......................................................................................................... 2
1.3 How to Use This Book to Your Best Advantage ....................................................... 3
1.4 Say Helio to the GMAT......................................................................................................... 5
1.5 How the CAT Works .............................................................................................................. 7
1.6 Preparation Strategies .......................................................................................................... 9
1.7 Test Day Strategies ............................................................................................................... 12
1.8 Specific Tips for the Ouantitative Section .................................................................. 15
1.9 Registering for the GMAT ....................................................................................................18

2 Arithmetic .................................................................................................................................... 19
2.1 Properties of Real Numbers and lntegers ................................................................. 19
2.2 Ratios and Proportions ..................................................................................................... 50
2.3 Percentages ........................................................................................................................... 58
2.4 Descriptive Statistics ......................................................................................................... 66
2.5 Counting Methods and Probability .............................................................................. 74

3 Algebra ........................................................................................................................................... 91
3.1 Algebraic Expressions and Factoring ......................................................................... 91
3.2 Linear Equations and lnequalities ................................................................................ 99
3.3 Exponents and Radicals ................................................................................................... 110
3.4 Ouadratic Equations, Functions, and Symbolism ................................................ 120

4 Word Problems ...................................................................................................................... 135


4.1 Translation from Words to Equations and Basic Word Problems ............... 135
4.2 Rate Problems ..................................................................................................................... 148
4.3 Work Problems ................................................................................................................... 156
4.4 Mixture Problems ............................................................................................................... 162
4.5 lnvestment/lnterest Problems ...................................................................................... 169
4.6 Set Problems and Venn Diagrams .............................................................................. 174

5 Geometry .................................................................................................................................... 181


5.1 Lines, Angles, and Triangles ........................................................................................... 181
5.2 Quadrilaterals, Circles, and Solids ............................................................................. 200
5.3 Coordinate Geometry and Graphs ............................................................................. 219
5.4 Data lnterpretation ........................................................................................................... 234

GMAT MATH WORKBOOK iii


6 Data Sufficiency ............................................ :....................................................................... 241
6.1 Format .................................................................................................................................... 241
6.2 Two Types of Questions ................................................................................................ 243
6.3 lnterpreting the Stem .......................................... :........................................................... 246
6.4 Challenging the Statements ................ :........................................................................ 247

7 Model Tests ...............................................................................................................................275


7.1 Model Test 1 and Answers ............................................................................................. 277
7.2 Model Test 2 and Answers ............................................................................................ 295

iv GM.At .MATKWORKBOOK ..--·--·· ..


Getting Started 1
-+ 1.1 LAYING THE FOUNDATION
-+ 1.2 WELCOME TO THE GMAT
-+ 1.3 HOW TO USE THIS BOOK TO YOUR BEST ADVANTAGE
-+ 1.4 SAY HELLO TO THE GMAT
-+ 1.5 HOW THE CAT WORKS
-+ 1.6 PREPARATION STRATEGIES
-+ 1.7 TEST DAY STRATEGIES
-+ 1.8 SPECIFIC TIPS FOR THE QUANTITATIVE SECTION
-+ 1.9 REGISTERING FOR THE GMAT

SECTION 1.1: LAYING THE FOUNDATION


Gaining admission to top business schools is becoming harder and more competitive each
year. The 2008-09 test year, for example, set a record with more than 265,000 GMAT tests
administered, representing an 8% increase over the previous year. Today, aspiring graduate
school students compete with MBA hopefuls from all o ver the world for limited spots offered
by top schools.
The importance ofyour GMAT score, which will strongly influence your acceptance into busi-
ness programs, is contingent on a number of factors. If yo u are a recent graduate, the GMAT score
is especially important because you lack the advantage ofyears ofwork experience that can set
you apart among tens ofthousands of candidates. For older applicants, the GMAT is particularly
vital because admissions committees consider how long you've been out of school and willlikely
place more weight on your GMAT score if yo u have been out of school for sorne time.
Fortunately, boosting your GMAT score, as an integral part of your application, is still
within your control. With appropriate preparation, you can use the GMAT test to your
advantage and present a graduate school application that stands out among many others.
The GMAT is your only chance to "make up" for weaker parts ofyour application and further
enhance your application with a more recent and outstanding indication of your eligibility.
GMAT is most concemed with your ability to execute. This book is designed to introduce
you to fundamental mathematical concepts contained \<Vithin the GMAT. Because understand-
ing math concepts theoretically is one thing, and their execution is another, this book presents
a range ofproblems through which you can apply lessons that familiarize you with those prin-
cipies, bridging the gap ben.veen basic math concepts and specific GMAT questions.
The comprehensive examples in this book are designed to ensure that you fully under-
stand each concept before you proceed to self-practice. The extensive sample problems that
accompany each section allow you to test your mastery of individual principies and your
process as a whole.

GMAT MATH WORKBOOK


While none of these concepts is any more challenging than what yo u have already Iearned
in high school, you will see that connecting the dots bet\-veen concepts and solutions might
be difficult at first. The good news is that you will find many explanations in this book that
vvill help you establish a direct line bet\-veen basic concepts and solution applications. In
addition, most of the concepts that will be new to you will be those that have to do with
the structure of the test. Studying for this exam should be more a matter of refreshing your
memory than it V1rill be learning new concepts.
Your success will ultimately be the product of a proper GMAT preparation strategy. This
book will show yo u how to strategically approach h-vo different aspects of the GMAT: devel-
oping a preparatory system months and weeks befare you take the exarn, and mastering
strategies you can implement on test day.
Subject explanations have been prepared with the busy test-taker in mind and organized
by bullet points. There are reminder boxes throughout the book that will help yo u carry out
solutions easier without the aid of a calculator.
We present you this valuable and practical guide to reach your life-changing goal. Now it
is your turn to take control and accomplish it.

SECTION 1.2: WELCOME TO THE GMAT


The GMAT is an acronym for the Graduate Management Admission Test, a standardized
exam administered internationally by the Graduate ManagementAdmission Council (GMAC)
and taken by students applying to business schools. Almost 2,000 business schools-nation-
ally and internationally-consider the GMAT scores as integral components of candidates'
admissions materials and accept the exam as one legitimate indicator of subsequent aca-
demic performance. Most of the testing locations around the world (and certainly al! of the
testing si tes in North America) now administer the GMAT only via a computer program. Test
takers no longer have the prerogative to opt for the pencil and paper format. For those ofyou
who have just recently decided that business school is the appropriate next step or who have
¡:10t taken a standardized test since the SAT (and you've been trying to forget that experience
for a long time), you actually have very few reasons to feel any distress. The object of the test
is to appraise your ability to reason methodically and to apply the mathematical and verbal
skills you've accumulated throughout your career as a student and, if you've been lucky, in
your employment since graduating. In essence, mastering this test is a matter ofrecollection
and recall, as well as familiarizing yourself with both the format of the exam and the manner
in which the questions are posed.

The GMAT: Not Your Nemesis


Many of yo u might be under the impression that this exam is going to be the most difficult
hurdle to clear in your admissions process. This does not have to be so if you have allotted
yourself a considerable amount of practice time and if yo u allow yourself to be guided by
logic rather than by emotion during the course of your preparation. Although the GMAT
is taken specifically by individuals hoping to enter business school, it does not endeavor
to measure your comprehension of any particular business-related subject nor require
you to have any business experience. (Schools might require you to have business experi-
ence, which is something to keep in mind when making application decisions.) You will be
expected to have mastered the basics of "reading, writing, and 'rithmetic." The verbal seg-
ment will assume yo u understand the standard conventions of vvritten English and that yo u
are capable of presenting coherent and compelling arguments in twn analytical essays. The
math segment will assume yo u know the basics of algebra, geometry, and arithmetic and that
you can apply these concepts in situations that are slightly more challenging than those you
confronted in high school math class.
The GMAT essentially serves as an objective supplement to the more subjective gauges
utilized in the candidate selection process, such as letters of reference, transcripts from previ-
ous institutions, and in-person interviews. Admissions offices understand that every applicant
comes from a different background anda different accumulation of experiences-economically,
socially, educationally, and so forth-than every other applicant Thus the GMAT, business
schools maintain, is a safe gauge by which applicants can be assessed under similar conditions
and measured by the same standard. So you see, the G.MAT is not necessarily your enemy. In
most cases, it is an exam that simply requires you spend sorne time brushing off the cobwebs in
those recesses of your brain where you've stored your knowledge of syntax and linear equations.
(If yo u have completed college, yo u have undoubtedly been exposed to tl1ese skills befo re.) Half
the battle of conquering the GMAT is simply knowing what to expect on the exam. By commit-
ting to working tl1rough this book, yo u are already on your way.

Your Scores and the Admissions Process


The influence your score has on your acceptance into business programs is contingent on a
number of factors. Admissions committees will consider, for exarnple, how long you've been
out of school. They willlikely place more weight on the GMAT scores of students who have been
out of school for sorne time than on students who have recently graduated. Each institution yo u
apply to is also likely to have its own estimate of acceptable and preferable GMAT scores. Many
post averages of their entering classes on their websites. Be sure yo u check with the schools yo u
are interested in so that yo u can set your goals. If, for example, your GPA is a little lower than tl1e
average GPA of a recent entering class, your goal should be to score a little higher than that class' s
mean GMAT score in order to compensate. Keep in mind, too, that other factors all carry weight,
like your letters of recommendation, your work experience, and your essays. Be sure not to get so
caught up in the GMAT that yo u fail to give each of these its requisite consideration.

SECTION 1.3: HOW TO USE THIS BOOK TO YOUR BEST


ADVANTAGE
This book was designed not only to introduce you to the fundamental mathematical con-
cepts. It was also designed to present yo u with a wide range of problems through which you
can continue to familiarize yourself with those mathematical principles-through applica-
tion. Understanding something on an abstract or theoreticallevel is one thing; execution is
another. As you know, the GMAT is most concerned with your ability to execute. What we
should pause here to stress, then, is how important it is that you work out even the sample
problems, as opposed to merely reading the solutions. Reading is not going to help you
absorb and retain nearly as well as will constant application.
The comprehensive examples in this book will help ensure you fully understand each
concept before yo u proceed to self-practice. The extensive sample problems that accompany
each section will allow yo u to test your mastery of individual principies and your process as
a whole. Be sure to double-check the book once you've arrived at the solutions to these prac-
tice problems. We might be able to show you an easier or quicker way to arrive at that same
answer (and quick and easy is how you will have to play this one).

GMAT MATH WORKBOOK . 3


Although none of these concepts is any more formidable than what you have already
learned in your high school geometry and algebra classes, chances are high that you've been
balancing your checkbook and estimating your taxes with a calculator ever sin ce. The good
news is that most of the concepts that \Vill be new to yo u will be those that have to do with the
structure of the test. The data sufficiency questions, for example, may seem a little daunting
at first, but we'll talk more about those later. Studying for this exam should be more a matter
of refreshing your memory than it will be learning new concepts. Furthermore, m u eh of your
success is going to be a product of strategy. As we al! know, knowledge without systematic
application tends to be knowledge ill used. This book, will focus on how to approach two
different aspects of the GMAT strategically. It will help yo u develop a preparatory system for
the months and weeks befo re yo u take the exam .. It will also provide yo u with a number of
strategies to implement on test day as yo u sit calmly and confidently befare the screen. The
mastery, so to speak, will be in the method.

Chapter Divisions
The mathcovered on the GMAT has been broken dO\vn into five chapters in this book. A
chapter each is dedicated to arithmetic, algebra, and geometry. Chapter 4 gives special atten-
'tion to word problems-those perplexing questions that actually test more than mathemati-
cal aptitude. In fact, sorne require a fairly high degree of reading comprehension. Chapter 6
focuses on a type of problem that is likely to be new to you, the data sufficiency question.
Although these questions might initially seem discouraging, keep in mind that the only
intimidating aspect is that the way they ask yo u to reason is noveL Chapter 6 gives yo u a so lid
strategy that, if consistently applied in practice, is sure to earn you more points on test day.
In an ideal world, you've given yourself enough time between now and the day of the
exam to work thoroughly through this entire book. As a general rule, the more preparation
time yo u put in, the better you're going to perform on test da y. If yo u find, however, that time
is not on your side, this book is still structured to accommodate you. It is ordered by tapie.
So just skip the tapies you have mastered, skim the tapies you're relatively familiar with, and
submerge yourself in the chapters that contain concepts you're not quite comfürtable with.
The subject explanations ha ve likewise been prepared with the busy test taker in mind and
are organized by bullet points. Note boxes appear throughout the text to help you cany out
sorne of the solutions more quickly without the aid of a calculator. Consider incorporating
sorne of these tips into your solutions. We also suggest you prepare flashcards with sorne
notes and explanations as you go through the subject explanations. Flashcards allow for
more "mobile" study for those of yo u on the run.

Minimize Test Day Angst: Replicate Actual Testing Conditions


in Practice
Now is the best time to start simulating actual testing conditions. Because yo u are studying
at your own pace and will take the exam in a limited and unbroken time frame, you should
minimize the discrepancies between the studying and testing situations. Befare you begin a
practice section, get yourself sorne scratch paper and do al! of your calculations on that rather
than in the book. This will help brace yo u for test day when you'll have no option but to look
back and forth between the computer screen and the page on which you're doing your work
by hand. Similarly, when you're takin~ diagnostic or practice tests (both of which we cer-
tainly suggest you do befare test day), try the following. Set yourself a timer, turn your phone

4 GMAT MATH WORKBOOK


off, and let your roommates know they must not disturb yo u for the next four hours. Do not
revert to using a calculator, even in practice. Acclimating yourself to doing everything by
hand is certainly going to minimize the terror that the "no technology on test day" rule might
otherwise provoke. In fact, we advise yo u to start doingall ofyourdaily calculations by han d.
The more mathematical problems you process Without the aid ofa calculator, the better.
Ifyou are interested in simulating the exam even more precisely, GMATPrep test prepara-
tion software is available at www.mba.com. You have the option of dmvnloading the software
directly onto your computer or receiving a CD-ROM version of the software when yo u regís-
ter for the exam. Two full computer-adaptive practice tests are included as well as real-time
scoring. You have no reason not to take advantage offree practice exams, especiallywhen the
rule ofth.is test is "practice like you're gonna play."

SECTION 1.4: SAY HELLO TO THE GMAT


The GMAT is a 3lfz hour exam. In fact, a pre-exam computer tutoría! and breaks bring it to
a full 4 hours. The test consists of 78 multiple-choice questions and 2 short-essay writing
assessments. Formerly created by the ETS (the infamous originator of the SAT), the test
is now designed by ACT, Inc. The GMAT is developed in cooperation with the Graduate
ManagementAdmission Council (GMAC). The function ofthe GMAC is the implementation
of guidelines, procedures, and policies about the admissions process and the types of intel-
ligence business schools are looking for in applicants. The ACT then turns these guidelines
into the kinds of questions you will encounter.
Although this book concentrates on the math segment of the exam, we think it is only fair
to let yo u know what yo u can expect to see in each of the sections . .The. test contains 4 sec-
tions. Although the analytical writing portiohs can· appear in·either arder, the quantitative
segment will always precede the verbal segment.

1. Analytical Writing Assessment: Analysis of an issue (30 minutes)


2. Analytical Writing Assessment: Analysis of an argument (30 minutes)
3. i
Quantitative (Math) Segment: 37 multiple-choice questions- or more will be data
¡
sufficiency questions and or less will be problem-solving questions (75 minutes)
4. Verbal Segment: 41 inultiple-choice questions split fairly evenly among critica!
reasoning, reading comprehension, and sentence correction questions, although
there might be more sentence correction questions (75 minutes)

The kinds of verbal questions you'll encounter will not be covered by this book. The dif-
ference between the problem-solving questions and the data sufficiency questions is that:
problem-solving questions are the kind you've grown accustomed to encountering on stan-
dardized tests. Your task is simply to salve the problem you are presented with and choose
the correct answer from the 5 choices you are given. Feel free to skim the table of contents
ofthis book to get a sense ofwhat kinds of concepts we're talking about. Percentages, expo-
nents, coordinate geometry, and quadratic equations are all possible tapies. Data sufficiency
problems also test quantitative reasoning. However, they use a set of directions you're going
to want to have memorized for test day. Chapter 6 will discuss this in detail. In brief, though,
you will be given a question followed by two statements. Both, either, or neither statement
will be useful in determining the answer to that question. Your task will be to determine
whether either statement alone will sufficiently answer the question, whether both are neces-
sary to answer the question, or whether enough information is not in either of them to come
to a conclusion as regards the question.

GMAT MATH WORKBOOK·-··5------


Breaks and Experimental Questions
You will be offered an optional 5- or lO-minute break after both the analytical vvriting and
quantitative sections, which we suggest yo u take. Four hours is, after all, quite a long time to ask
your body to be still and attentive. So yo u can't say yo u weren't warned, yo u should knovy that
about a qua11er of the questions yo u will encounter on the exam are experimental questions-
questions that have no bearing on your score but that the ACT has slipped in to test how well
they will be received on future exams. Essentially, yo u are paying the ACT to be its guinea pig.
Even more unfortunately, you won't know which questions are the experimental ones. Just
assume that all questions count and treat each question as one that will influence your score.

Your Scores and What They Mean


After you've hit the Answer Confirm button for the last time, the computer will calculate your
scores for_everything except the analytical vvriting pm1ions. We won't briefyou on how the
testing screen will appear. You can access the tutorial on the GMAT website. At this point,
yo u will have the option either to see your score, which will make it official, orto cancel it and
never know how yo u fared. Of course, after all the studying you'll have done, yo u likely won't
need to opt out. The t\,vo vvriting sections will be read by actual humans. The official score
report-including the writing scores-will be mailed to yo u within lO days after the exam. As
we will explain in the next section, the scores aren't calculated by a straightforward "number
of questions you got right minus number of questions yo u got wrong" method. Sorne ques-
tions are worth more than others. Each question's value is contingent upon its leve! of diffi-
culty. Therefore, answering a difficult question correctly is worth more than answering a less
difficult question correctly. Your official score will include:

1. Your quantitative score (from Oto 60)


2. Your verbal score (from Oto 60)
3. Your inclusive score (from 200 to 800, in 10-point increments)
4. Your analytical writing score (from l to 6, in half-point increments)

In the official report you receive in the mail, you will also be given a percentile rank (from
0% to 99%). The percentile rank is a reflection of your performance relative to all GMAT
test takers in the most recent three years. A percentile rank of 70%, for example, means you
scored higher than 69% of the other test takers in this time period. It also means yo u scored
lower than the other 30%. As soon as your essays have been read and scored by the GMAT
readers, score reports will be forwarded to each of the business schools yo u designated when
yo u registe red for the exam. Your scores will be transmitted to up to 5 schools without charge;
sending then to each additional school will cost yo u a fee. Although most schools will simply
average reported scores ifyou take the test more than once, sorne schools have begun disre-
garding scores that are disproportionately lower than others. Sorne institutions promise to
look at only your highest scores, although this practice is becoming less common as it obvi-
ously favors those who can afford to take the test multiple times.

Good Scores vs. Bad Scores


The question we inevitably get asked regarding these reports is "What is a good GMAT score?"
The short answer is that a good score is the best score that you personally are capable of
achieving-something you'll definitely have a better sense of after taking a few practice tests.
The longer and equally frustrating answer is that whether a score is good or bad is pretty

''6'' 'GMAT MATH WORKBOOK


indeterminate. No one has yet established a number by which to assess whether someone
has passed or failed the Gl\1AT. Although there is no benchrnark, average grades have tended
to differ based on undergraduate rnajor. For example, physics and computer science majors
tend to score the highest. Most scores tend to fall between 390 and 620. The average Gl\1AT
score in recent years has been around 530 or 540. GI:vlAC publishes annual score reports for the
year prior ifyou're interested in the details. We suggest you don't get too caught up in other
people's averages. Instead, check in with the schools you plan on applying to. Each institution
is likely to have a different nurnber in mind for what scores their ideal candidates receive.

Time- and Stress-Saving Tips


To spare yourself any additional frustration on test day, yo u must learn now that yo u will not be
allowed to bring anything into the testing room except the paper or note boards handed to you
by the administrator. Cell phones, calculators (both handheld andona watch), pens, watch
alarms, and so on are prohibited. Ifyou have any questions about the test itself, you can access
a "Help"function at the bottom ofthe screen. However, your time will continue to expire even
while you're getting answers to these basic questions. You will have an untimed period prior
to the start of the test in which you are given an interactive tutorial to allow you to become
familiar with the program you are about to use. This tutorial indudes information about how
to use the word processing program that you'Il use to write your analyses. Presumably, you'll
have al! the time you need during this tutorial to go over the details. Ifyou're going to be sit-
ting there for 4 hours aiready, there isn't much of a point in spending an additional half hour
to learn the system. Free test tutorials that will acquaint yoil with the basics, such as entering
answers, are available at www.mba.com. Getting acquainted \vith the procedures prior to the
test is going to be critical-and is going to reduce your stress tremendously on the big day.
You might want to familarize yourselfwith
. the other testing procedures and . rules befare
arriving at the test site. They include fingerprinting, identification, disclosure prohibition,
and so forth. The list is long. If you're interested in the details, yo u can access all of them at
http://www. mba.comlmbalthegmat/testday.

SECTION 1.5: HOW THE CAT WORKS


CAT is an acronym for computer-adaptive test. Although the computer and the test aspects of
the name should be pretty straightforward, yo u should be most concerned with the adaptive
aspect. Adaptive essentially means that there is no predetermined order of difficulty to the
questions yo u wíll be given. This is the biggest difference between the CAT and a paper exam.
The computer tries to match the difficulty of each successive question to both your overall
.performance and to the question you've just answered. The computer takes into account
not only whether or not yo u answered the previous question correctly but also the difficulty
leve! ofthat particular question. Thus the better yo u perform, the more challenging the ques-
tions will get-and the more challenging the questions, the more points you will receive for ·
answering them correctly. In short, your score is not determined by the number of questions
yo u answer correctly but by the difficulty of the questions yo u answer correctly.
The first qucstion yo u will see is one that the computer considers of average difficulty-a
question 50% of test takers are likely to answer correctly and 50% are likely to answer incor-
rectly. Ifyou answer that question correctly, the computer is going to give you a slightly more
difficult question the next time around. If you answer it incorrectly, though, your second
question \vill be slightly easier.

GMAT MATH WORKBOOK 7


Although the computer assumes from the beginning that yo u are an average examine e,
it determines pretty quickly whether or not this is actually the case. For each subsequent
question, the program will continue to choose from a large pool of test questions, all
arranged by question type and difficulty. The particular question chosen depends on how
you seem to be handling the given questions. The more questions you answer, the better
refined the computer's interpretation will be of the difficulty leve! yo u are capable of nego-
tiating. By the time you're half\<Vay through the quantitative and verbal sections, the com-
puter should be choosing questions that are in accordance with your aptitude. This means
that most of the questions will seem challenging and you will get approximately as many
correct answers as incorrect answers. This will be the leve! the computer uses to determine
your scaled score.

What the Structure of the CAT Means for You as a Test Taker
You must understand a few crucial things about how this format differs from the kind you're
used to taking. Most importantly, yo u can't skip questions on the CAT. Yo u will be able to see
only one question ata time. Yo u cannot go back to change an answer once you've confirmed
it. Becaúse the computer predicates each succeeding question on your answer to the ques-
tion prior, yo u have no choice but to answer the question in front of you-even if it means
guessing-and let it go. 0Ne will talk later about strategic guessing so you can choose intel-
ligently when yo u don't exactly know the answer.) Do not get frustrated if the test seems to be
getting incrementally more challenging. If it do es, this means that you're doing well. The last
thing, though, that you want to do on test day is try to ascertain whether the qu'estion you've
got befare you is easier or harder than the last one. Trying to figure out how the computer
thinks yo u are doing is only going to take yo u away from your real focus, which is performing
your best in the moment and on the present question.

Giving the Early Questions More Time and Energy Than the
Later Ones
Probably the most critica! thing you need to keep in mind is that your final score is by no
means based solely on the last question that you answer. In fact, given what you now know
about the algorithm the test uses to determine each question, it is more important to answer
questions correctly early on than to answer questions correctly at the end. In fact, we highly
suggest that when you're allocating time, you spend a disproportionately larger amount of
time on the first 10 questions than on subsequent ones. You want to aim for 90% to 100%
accuracy, in particular, on these first 10 questio.ns. Why is answering the first 5 questions
correctly going to be more advantageous than answering the final 5 questions correctly?
Remember, the more difficult questions are worth more points. So if you start generating
the difficult questions at the beginning, you'll be getting more points for each question you
answer correctly both in the middle and at the end of the exam. If yo u don't start generating
difficult questions until the middle of the exam, you'll be receiving additional points only for
those questions you answer correctly at the end. Simply put, the harder questions are where
the points are. The more hard questions you encounter and the sooner you encounter them,
the higher your seo re will be. Also keep in mind that this do es not mean that if yo u miss the
first question, you're done for. The more questions you tend to answer correctly, the more
likely the computer is to recognize any silly mistakes you make as anomalies. So do not fear
a one step forward, two steps back progression.

8 GMAT MATH WORKBOOK


Leave No Question Unanswered
Please keep in mind that yo u will be penalized for all questions that yo u leave unanswered on
the CAT. In fact, each question yo u fail to answer will decrease your score by an even greater
increment than each question you answer incorrectly. This means that ifyou're running out
of time on a section, you're going to earn more points by mere!y guessing than by letting time
run out. We know that conceptually tllis is a little difficult to accept. \1\lhy answer a question
you've hardly glanced at? The GMAT is likely one ofthe few cases in which this is ever going
to be true. However, random guessing is, in this case, better than nothing.

Our Final Word on the CAT


As you've likely gleaned from this introduction to the CAT, it's gota few pros anda few cons
relative to paper-based tests. Among the cons of the CATare that you can neither skip nor
return toa question. Additionally, you can't make notes on the computer screen. You can't
see al! of the reading comprehension passage or the charts at once; you've got to scroll. A
compelling aspect of the medium is that test takers who have more experience using com-
puter programs will have a bit of an advantage. However, a fair number of pros essentially
tip the balance back. Because it is a computer test, you can take tl1e CAT at a time that is
convenient for you. You also learn your verbal and quantitative scores almost immediately
after yo u' ve answered the final question. Yo u have a timer on fue screen for purposes of pac-
ing and have a more personalized testing space (a computer alcove). Preparing yourself to
overcome the few cons, we think, will be worth your while.

SECTION 1.6: PREPARATION STRATEGIES


• First things first: take a diagnostic test. You will not be able to set your goals or be able
to organize a study schedule for yourself without having a sound idea of what leve!
you are at the start. There's no reason to spend all your preparation time studying
grammar if yo u already know everything you're going to need to know. A worse
scenario is finding out that much ofwhat you studied is not even material the GMAT
covers. Conversely, you might find out through a diagnostic test that you don't
remember anything about parallellines and transversals. Setting aside the time to
take a full diagnostic test will help yo u establish exactly where yo u should expend
your energy over the corning weeks. It will also give yo u a sensé from the outset of the
structure and the language you can expect on the exam.
• Sign up and set your goal. Another crucial step we strongly suggest yo u take befo re orga-
nizing a study plan is to decide on a convenient and suitable time to take the test and
actual! y register for it. Determining the timing of the GMAT is somewhat of a chicken
and egg problem where you will need to consider various factors. These in elude your
work schedule, how much time yo u estímate to spend working toward your goal and
the timing of the test. So make sorne reasonable estima tes on your time commitment
and register in advance. This will not only ensure that yo u take the test at your conve-
nience but vvill help you determine the details ofyour study plan.
Thinking about these factors together with the general score expectations ofyour
favorite schools will also help you set specific, achievable, and realistic goals for
your score.
• Create a realistic study plan for yourself and stick to it. Only yo u know how well yo u did
on the diagnostic test, and only you know how much time you have until test day. Get

GMAT MATHWORKBOOK___, 9
yourself a calendar and put it on the fridge, above your desk, or in any conspicuous
place. Keep in mind that yo u have alife to live and other responsibilities. Schedule an
hour a day ora couple ofhours tvvo days a week to study. You can even devote certain
weekends to certain concepts if necessary. Having this schedule is going to be useful
in two ways. First, it \Nill ensure that Giv1AT study doesn't get last priority in your life.
We all know how tragic that 'Tll just finísh these other things first" mentality can be.
Second, looking at the big picture will help you realize that, in fact, you have enough
time to prepare. Seeing your study schedule stretched out over a period of time will
keep it from seeming so daunting.
• Consider your shortcomings. Remember that your GMAT scores are going to the
institutions yo u are applying to in their entirety. You can't simply transmita section
of the exam. We know how easy it is to say "I understand 75% of this stuff; I'll brush
up on the tougher material a few days befo re the exam." We also know Murphy's law
states ·that the first questions you'll get on exam day will cover precisely the areas yo u
waited until the ~ast minute to study. Ifyou use this book, the GMATwebsite, and
all the other sources of kiformation out there, there's no way yo u can't know what is
going to be on this exam. This should be satisfying news. So once you've taken the
diagnostic test, focus first on the question types that gave you the most difficulty.
These questions made you pause the longest befare answering. You had toread them
twice befo re yo u understood them.
• Start doing all your calculations by hand today. We've aiready advocated this minar shift
in your daily life, so we won't harp. Practicing your multiplication tables rather than
relying on the calculator in the coming weeks is going to be the equivalent of many
precious GMAT minutes.
• Whenever you take a practice test, try to reproduce an authentic situation as closely as
possible. Use a word processer, ifyou have access to one, and use only the features
that will be available to you on the CAT. (The analytical writing section will be typed
into a similar program.) It vvi!l be good to have a sense from the get-go ofhowyour
body responds to sitting in a chair and keeping your focus intact for 4 hours straight.
This is definitely not the same thing as watching 4 consecutive hours oftelevision. You
really need to determine, for example, whether your pacing is where it should be. Also
remember ~hat any computer-based exams yo u can find are going to provide you with
more accurate conditions. So ifyou have a choice between practicing on the computer
and practicing in a test booklet, always choose the former. If yo u are computer
challenged, this is going to be crucial. Borrow a computer from a friend ifyou need to
get used to using a keyboard or mouse. Being comfortable with the medium is key.
• Think of the GMAT as a simultaneous ultramarathon and 200-meter sprint. You're going
to have to answer each question in about 2 minutes. We know that ifyou had al! the
time in the world to complete the GMAT, you'd likely score in the 99th percentile.
Unfortunately, the GMAT writers know that too, which is why they've introduced
the time limitation. Maintaining an aggressive and industrious pace is one of the few
things that will distinguish yo u from your fellow test takers. At the same time, the
GMAT requires a leve! of mental stamina that yo u vvi.lllikely need even more of once
yo u get to grad school. If yo u can strike a balance between these two m o des., you've
already outdistanced a good number of GrvlAT examinees.
• Know the directions well enough that you can recite them in your sleep. This is paramount,
especially for data sufficiency questions. The CAT will display the directions for each

-:::;-:_:-:,:-:::::;;:;-:::-:;:::;::.:;~=-
--·~"- .. -
~--".--·-·" .
-to."::::::GMM:MATH WORKBOOK
question type as it appears on the screen. However, the last thing yo u want to waste
your time on is reading the directions. If you've already familiarized yourself with
what each section is going to require of yo u, there will be no need to let the dock keep
running while yo u make certain you're doing the work the question actually entails.
• Now is not the time to make any (other) life-changing decisions. Don't try to quit or start
smoking, don't move to a new apartrnent, don't begin training for a triathlon, and don't
go through a breakup in the weeks befo re the exam. If at all possible, sign up for the
,GMAT ata time yo u know yo u won't have too many other responsibilities or priorities to
juggle. This exam is going to take quite a lot of mental stanlina and emotional stability.
Ensure that yo u' re burdened with as few things as possible during your preparation.
• Take the GMAT early enough that if you need to retake it for any reason, you have time
to do so. Most MBA programs admit students for only fall matriculation. November is
general!y the latest month yo u can take the GMAT if yo u want to ensure your scores
make it to all your schools in time. You might want to think about taking it earlier
than November, however, in case something goes awry with your November test. For
instance, you may come down with the flu, you may not do as well as you expected,
a family emergency m ay take yo u out of town, and so forth. The GMAT process is the
same as any other life process with deadlines. Don't wait until the last minute or the
last sitting. Always keep your "in case" mentality active.
• Know how to minimize anxiety. Are yo u worried about timing? Complete a few more
practice tests under exam conditions. Are yo u concemed about how your scheduling
and study habits compare with those of others? Join a GMAT study group. Are you ago-
nizing over whether you've given yourself enough time to preparé Talk to people who
have taken similar exams. They're out there, everywhere. You'll find your insecurities
will dissolve pretty quickly. Have yo u been studying for such long hours that you're
starting to get "easy" questions wrong? Get on your bike. Quiet your thoughts. Step away
from the desk for awhile. Ifyou need to keep the GMAT in the forefront ofyour mind,
go out for coffee with a friend and correct his or her grammar. Instead, yo u can choose
one of the possible "Analysis of an Issue" tópics and answer it in your head while you're
taking a walk through the park. Whatever yo u need to do, remember that you're still in
the world and that the GMAT is not the be-al!, end-all. Your mind is also not structured
to handle 12 hours of straight study. Break it up a bit. All of the above will help yo u
maintain a positive attitude, which is really the most important aspect of this process.
• Keep your scores and your expectations in perspective. If you're picking up this book
because you've already signed up for the GMAT orare about to, you've likely also
set your sights on a few colleges or universities you'd !ove to study at. You probably
also have a good idea what kind of scores those institutions require. If yo u haven't
looked at those schools' requirements, you should. Doing so will give you a sense of
where you are relative to the schools' general expectations. Knowing what your target
schools consider acceptable scores is a good idea. However, you shouldn't obsess over
those numbers. Don't sabotage yourselfwith the numbers the school provides for you
as a gauge. Looking at the scores ofyour diagnostic test is much more constructive.
With each new test yo u take, consider how yo u can raise your score a bit from last
time. Think of studying as a competition with yourself. There' s not too much val u e in
putting yourself against the rest of the GMAT test takers worldwide. Setting realistic
expectations, realizing what your limitations are, and so forth are going to be essential
to ensuring yo u neither disappoint yourself nor work yourself into a frenzy.

GMAT MATH WORKBOOK 11


• Know when you•ve reached your optimum score. A time will come when you've taken
a number of practice tests in a row and yo u just keep getting the same score, despite
the amount of studying you're doing. Ideally, this will be around the same time your
test is scheduled. Don't drag out the process by corúihuing to postpone the test so yo u
can get in more preparation. At so me point, your motivation, your interest, and your
performance will start declining. (So might other things, like your social skills.) You've
brushed up on the math and the grammar. You've rectified any bad test -taking habits.
You've learned how to pinpoint wrong answer choices instinctually. Yo u also know
you can finish the exam in a timely manner. What else is there? Take the test-for
your own salce-befare yo u start driving yourself crazy.
• The day before the test, congratulate yourself instead of cramming. We all know from
past experience how unproductive burning the midnight oil actually is. Ifyou've
given yourself the time to prepare, stuck to your schedule, and ea ten, slept, and
continued to breathe throughout the process, you're in great shape. Take yourself out
to your favorite restaurant the evening befare the GMAT or go with a friend who has
supported yo u during your preparation time. Eat the kinds of foods that make yo u feel
good in the morning. Relax and get to bed early. Remember the ultramarathon part
ofthé test. The exam won't slow down for you ifyou haven't slept the night befare
because you've convinced yourselfthere are a few more things you have time to insert
into your short-term memory.

SECTION 1.7: TEST DAY STRATEGIES


• Stick to your usual routine on the day of the test. If you typically have oatmeal and juice
for breakfast, eat that. On the morning ofthe GMAT, don't have pancakes and coffee
ifthat's nota usual breakfast foryou. Also be sure to time your pre-GMAT meal. Don't
eat right befo re the test because you don't want all your energy going to digest the
food in your stomach when it should be circulating blood to your brain. You should
eat about an hour or so befare the exam begins. Definitely bring a snack ifyou think
you'll need a midtest pick-me-up. You won't be allowed food at your alcove, but you
can bring it to the testing center to eat during one ofyour two short breaks.
• oevote special attention to the first 5 to 10 quantitative and verbal questions. The details
behind this strategy were previously described. Briefly, though, the CAT tends to
discover within the first questions of each section what difficulty leve! yo u are capable
ofmaintaining. The higher the CAT believes your competency leve! to be-and the
sooner-the higher your final score is going to be.
• Use your scratch paper, and use the tutorial time to prepare it. One of the most frustrating
aspects of the GMAT is that yo u spend most of the test looking back and forth between
the computer screen and your scratch paper. This should not mean yo u don't make
use of the paper. Doing work in your head almost always spells disaster in sorne forro
or another. Befo re the test starts, perhaps during the untimed interactive tutorial at
the beginning, divide your scratch paper into a chart to help you use the processes of
elimination. Obviously, you can't cross out answer options on the computer screen. So
the best way to combat this is to divide your paper into a chart with 5 sections. In each
box to the Jeft, write A, B, C, D, and E so that yo u can physically cross out the answers
you decjde to eliminate (leave a blank space at the bottom for formulas and such).
Using a paper chart is great way to keep track of the remaining feasible answer choices
as you workout the problern. It is rnuch better than covering the answers you decide to
elirninate by placing your fingers onto the screen.
a Process of elimination is your new best friend. We mean this. The sooner yo u can get rid
of the answer options that are definitely not right, the sooner you've gota significantly
higher chance of choosing the right answer. Frankly, deterrnining which answers
are wrong i.s often rnuch easier than deterrnining which answer is right. Sorne ofyou
rnight remernber the satisfaction of reading a question on sorne previous standardized
test and crossing out three oftheanswers immediately. Fevv-tllings are more gratifying
than knowing you've just increased your 20% chance of choosing the correct answer
to a 50% chance. So don't be afraid to come at the answers in this backward fashion.
Aggressive elimination is critica! to perforrning well on a timed test such as the GMAT.
We highly suggest this rnethod if for no other reason than that it gets tl1e wrong
answers out ofthe way-and out ofyour mind.
• Remember the verbal strategies. We'll give yo u a few exarnples even though iliis is a
rnath workbook. Note tl1at specific rnath tips are in Section 1.8. First, on sentence
correction questions, yo u can skip the first answer choice. The first answer option is
. always going to repeat the original. There's no sense reading ita second time ifyou
know the original is incorrecL Second, read passages for ilie general outline, not for
ilie details. Most of the questions are con cerned wiili ilie passage structure, not its
rninutiae. Keep iliis in rnind for these sections. Third, in ilie critical reasoning questions,
find the conclusion first. The GMAT !oves rnain point and prirnary purpose questions.
The conclusion will be ilie key to answering every question of tllis type correctly. Final!y,
have two solid exarnples for each ofyour analytical essays. Every possible prompt you
will respond to can be accessed on ilie GMAT website atwww.mba.com. Take a loo k at
sorne of these to get a sense of ilie kind of questions yo u will be asked.
• Maintain an active mental process. It beco mes a little too easy-especially an hour
or two into staring into the light of the screen-to let your eyes passively ro ve.
Maintaining an attentive and inquisitive rnind throughout should help with iliis. Ask
yourself the following when yo u encounter each new question.

• In what area is the cornputer trying to rneasure rny proficiency by asking me this
question?
• What skills and intelligence arn I being asked to apply right now?
• What are the data, and what is the unknown?
• How would a less careful test taker be flustered by a question like this?
• Which of these answers are meant to mislead careless testers?
• \A/hat is the most direct route I can
. use to find the answer to·-,-"this question?
.,

• Read each question and each answer choice thoroughly. We can't even begin to tell yo u
how many test takers we've watched answer questions incorrectly sirnply beca use they
read answer options A, B, and C but not D andE. Choice C sounded right, so they chose
it without looking at D andE. Unfortunately, choice D was ilie correct answer. In fact,
not reading everything fui! y is one of the leading causes of incorrect answers on the
exam: Remember, as "easy" as the multiple-choice forrnat rnight appear, all the answer
options are written by the test makers because they seern like they could be correct.
Every answer is going to seern Iike a possibility. Don't get caught up in an enticing
answer choice and disregard the others before you give thern a chance. Likewise, be
sure yo u know what the whole question is asking. Many of the quantitative problems,

GMAT MATH WORKBóOK 13 ··


in particular, have to be answered in two parts. If yo u answer only the first part, even if
you're doing the work right, your answer is going to be wrong. We guarantee that one
of the answer options is going to be that answer yo u arrive at when you're only halfway
there. It's one of the tricks the GMAT writers !ove to play on yo u.
• Move the keyboard out of your way once you are finished with the analytical writing
section. It will no longer be usefui to yo u. In fact, those unwieldy things are likely to get
in the way. The only two things you are going to need to focus on for the remainder
of the exam are the computer screen and your scratch paper. Keeping your alcove
otherwise clutter free will simultaneously de-clutter your mind.
• Pay attention to your timing and your pacing throughout the test. Remember the test is
designed in such a way that you'll always be working a little faster than is cornfortable for
yo u. Ideally, you'll have taken enough practice tests that yo u have a so lid sense of how yo u
keep up. Also remember that yo u' ve budgeted a little more time for those first 10 questions
and that you're going to have to pickup the pace significantly as you get into the second half
of each section. You absolutely should spend more than average time on each of these ini-
tial questions as long as you have worked out how to make up tl1at time. Do not let yourself
spend too much time on any one question in particular. At a certain point, you're going to
want to ma.ke your best guess and move on. Do not dwell too much on the dock. However,
glance at it after every 5 questions or so. You'll be able to adjust appropriately from there.
• Learn the art of intelligent guessing. Inevitably, you are going to have to guess
sometimes. Remember, the test is structured to give yo u precisely the questions that
are difficult for yo u. It may be a matter of not knowing absolutely what the question is
asking, having forgotten a formula, or simply running out of time. The more practice
tests and questions yo u do, the better yo u will get at maximizing your chances of
guessing correctly. The process of elimination, estimating, and recognizing the
answers that the ACT anticipates average examinees vvill eh o ose are all methods by
which to make prudent guesses. We'll cover so me of these in the following section.
• Guess wisely and move on. Remember two things. First, the next question you answer
correctly is going to bump you back up to a harder question and more points. Second,
more than 20% ofthe questions in both the verbal and quantitative sections are experi-
mental questions. You may feel better ifyou keep in mind there is a good chance that
any question you've had to guess on isn't ultimately going to affect your seo re at all.
• Make use of the breaks-but remember the clock is always running. Yo u might finish
a section of the test, determine that yo u are on a fabulous roll, and think you have
enough momentum to rush headlong into the next section. Remind yourself in these
moments that that next section is over an hour long. An unbroken 4 hours of anything
is a lo t. We recommend stretching to get your circulation flowing. Remember, your
mind won't concentra te if no blood is circulating to your brain. Grab a snack or focus
on sorne deep breath work. Don't use the breaks to mourn over questions you think
yo u missed; let go of those. Congratula te yourself on the portions of the exam you've
completed so far. Know that the minutes go by quickly and the test is going to resume
again after the break is over-whether or not you are there.
• Be careful about going anywhere near the "Section Exit" and "Test Quit" commands. Ifyou
confirm either, the test is as good as canceled. The only buttons you'll really need will
be the "Answer Confirm" and "Next" buttons, the ones on the right si de of the screen.
• lf for sorne reason the test does not go as planned, you will have the option to cancel your
scores at the end of the exam. You will never know what your seo res would have been.
as you workout the problem. It is much better than covering the answers you decide to
elimina te by placing your fingers onto the screen.
• Process of elimination is your new best friend. We mean this. The sooner yo u can get rid
of the answer options that are definitely not right; the sooner you've got a significan ti y
higher chance of choosing the right answer. Frankly, determining which answers
are wrong is often much easier than determining which answer is right. Sorne ofyou
might remember the satisfaction of reading a qtiestion on sorne previous standardized
test and crossing out three of thearrswers immediately. Few things are more gratifying
than knmving you've just increased your 20% chance of choosing the correct answer
to a 50% chance. So don't be afraid to come at the answers in this backward fashion.
Aggressive elimination is critical to performing well on a timed test such as the GMAT.
We highly suggest this method if for no other reason than that it gets the wrong
answers out ofthe way-and out ofyourmind..
• Remember the verbal strategies. We'll giveyou afew examples even though this is a
math workbook. Note that specific math tips are in Section 1.8. First, on sen ten ce
correction questions, yo u can skip the first answer choice. The first answer option is
always going to repeat the original. There's no sense reading ita second time ifyou
know the original is incorrect. Second, read passages for the general outline, not for
the details. Most of the questions are concemed with the passage structure, not its
minutiae. Keep this in mind for these sections. Third, in the critica! reasoning questions,
find the conclusion first. The GMAT loves main point andprimary purpose questions.
The conclusion will be the key to answering every question of this type correctly. Finally,
have two solid examples for each ofyour analyti~al essays. Every possible prompt you
will respond to can be ac.<;ess.ed on the.GMAT website at www.mba.com. Take a loo k at
sorne of these to get a sense of the kind of questions yo u will be asked.
• Maintain an active mental process. It beco mes a little too easy-especially an hour
or two into staring into the light of the screen-tó let your eyes passively ro ve.
Maintaining an attentive and inquisitive mind throughout should help with this. Ask
yourself the follmving when yo u encounter each new question.

• In what are a is the computer tryipg to measur~ m y proficiency by asking me this


question?
• What skills and intelligence am I being asked to apply right now?
• What are the data, and what is the unknown?
• How would a less careful test taker be flustered by a question like this?
• Which of these answers are meant to mislead careless testers?
• Vvhat is the most direct roúte I can use to find the answer to this question?

• Read each question and eaeh answer choice thoroughly. We can't even begin to tell you
how many test takers we've watched answer questions incorrectly simply because they
read answer options A, B, and C but not Dand E. Choice C sounded right, so they chose
it without looking at D andE. Unfortunately, choice D was the correct answer. In fact,
not reading everything fully is one of the leading causes of incorrect answers on the
exam: Remember, as "easy" as the multiple-choice.format might appear, all the answer
options are \NTitten by the test makers because they seem like they could be correct.
Every answer is going to seem Iike a possibility. Don't get caught up in an enticing
answer choice and disregard the others befo re you give them a chance. Likewise, be
sure yo u know what the whole question is asking. Many of the quantitative problems,

GMAT MATH WORKBóói< 13


in particular, haveto be answered in two parts. Ifyou answer only the first part, even if
you're doing the work right, your answer is going to be wrong. We guarantee that one
of the answer options is going to be that answer yo u arrive at when you're only halfway
there. It's one of the tricks the GMAT writers lave to play on yo u.
• Move the keyboard out of your way once you are finished with the analytical writing
section. It will
no longer be useful to yo u. In fact, those unwieldy things are likely to get
in the way. The only two things you are going to need to focus on for the remainder
of the exam are the computer screen and your scratch paper. Keeping your aleo ve
otherwise clutter free will simultaneously de-clutter your mind.
• Pay attention. to your timing and your pacing throughout the test. Remember the test is
designed in sueh a way that you'll always be working a little faster than is comfortable for
you. Ideally, you'll have taken enough practice tests that yo u have a solid sense ofhow you
keep up. Also remember that you've budgeted a little more time for those first 10 questions
and that you're going to have to pickup the pace significantly as you get into the second half
of each section. You absolutely should spend more than average time on each of these ini-
tial questions as long as yo u have worked out how to make up that time. Do not let yourself
spend too much time on any one question in particular. At a certain point, you're going to
want to make your best guess and move on. Do not dwell too much on the dock However,
glance at it after every 5 questions or so. You'll be able to adjust appropriately from there.
• Learn the art of intelligent guessing. Inevitably, yo u are going to have to guess
sometimes. Remember, the test is structured to give yo u precisely the questions that
are difficult for yo u. It may be a matter of not knowing absolutely what the question is
asking, having forgotten a formula, or simply running out of time. The more practice
tests and questions yo u do, the better you will get at maximizing your chances of
guessing correctly. The process of elimination, estimating, and recognizing the
answers that the ACT anticipates average examinees will choose are all methods by
which to make prudent guesses. We'll cover sorne of these in the following section.
• Guess wisely and move on. Remember two things. First, the next question you answer
correctly is going to bump you back up to a harder question and more points. Second,
more than 20% of the questions in both the verbal and quantitative sections are experi-
mental questions. Yo u may feel better if yo u keep in mind there is a good chance that
any question you've had to guess on isn't ultimately going to affect your score at all.
• Make use of the breaks-but remember the clock is always running. Yo u might finish
a section of the test, determine that yo u are on a fabulous rol!, and think you have
enough momentum to rush headlong into the next section. Remind yourself in these
moments that that next section is o ver an hour long. An unbroken 4 hours of anything
is a lo t. We recommend stretching to get your circulation flowing. Remember, your
mind won't concentra te if no blood is circulating to your brain. Grab a snack or focus
on sorne deep breath work. Don't use the breaks to mourn over questions you think
yo u missed; let go of those. Congratula te yourself on the portions of the exam you've
completed so far. Know that the minutes go by quickly and the test is going to resume
again after the break is over-whether or not you are there.
• Be careful about going anywhere near the "Section Exit" and "Test Quit" commands. Ifyou
confirm either, the test is as good as canceled. The only buttons you'll really need \Nill
be the "Answer Confirm" and "Next" buttons, the ones on the right side ofthe screen.
• lf for some reason the test does not go as planned, you will have the option to cancel your
scores at the end of the exam. You will never know what your scores would have been.

14 ·-GM:At=MATH WORKBOOK
However, ifyou are certain these aren't the scores you want going out, this is the price
you'll have to pay. Befare you hit the "Cancel" button, remember that the computer
is structured to give yo u questions that are difficult enough that you're bound to
miss sorne. The more trying questions mean you're on your way to a high score.
Many GMAT ta.kers think their performance was poorer than it actually was for this
reason. Chances are high that ifyou've put in the work, you're going to be pleasantly
surprised ifyou decide against score cancellation.
• Plenty of staff will be at the testing site should something go wrong. Thcse problems
include computer malfunctions, distractions, and so forth. Don't worry, though. The
designated centers are generally well run, so the risk of any of these is low. Know that yo u
can notify the staff with any questions or complaints and that reports can be filed online
to ACT. So yo u' re not without recourse in tl1e case of unanticipated complications.
• Congratulate yourself on the way out (and all that evening, and all the next day•. •). You have
just completed an exam that only a crazy few have the mental and emotional starnina to
take. Remember to thank al! the people that have constituted your support system in the
past weeks and months. Continue to remind yourself in the coming weeks when yo u' re
waiting for your official score what a truly impressive feat you've just accomplished.

SECTION 1.8: SPECIFIC TIPS FOR THE QUANTITATIVE SECTION


• Be particularly attentive to the two-part math questions. It's easy to forget that the
quantitative section actual!y takes a fair amount of critica! reading. Peruse these
questions as carefully as you do those in the verbal section. Here is an example of a
problem in which a simple reading error could get yo u in a lot of trouble:

If3x-17=151,thenx+4=
(A) 48¡
(B) 52
(C) 56
(D) 60
(E) 172

So you start solving the equation, right? Add 17 to 151 to get 3x = 168. Divide both sides by 3
to get x = 56. Bingo! We know what x equals. Furthermore, and this is where GMAT will get tricky
with its answers, the number 56 is one ofthe choices. Option (C) is obviously (?) the answer.
It's really arhazing how regularly this happens, especially when test day jitters are com-
bined with serious time limitations. An attentive test ta.ker will reread the question after find-
ing the answer to ensure he or she found the answer to the question actually asked. Reread
the question. It is not asking what x is but, rather, what x + 4 is. The correct answer is (D).
Notice the other answers are all answers you could potentially get ifyou weren't attentive
in your calculating. This is another reason why you must use your scratch paper. You might
have chosen (E) ifyou had added 17 to 151 to get 168, but had forgotten to divide by 3. You
would have determined that x = 168. You would instead have chosen (B) ifyou made it al! the
way to x = 56 but then subtracted 4 instead of adding it. Yo u would have arrived at option (A)
if yo u had subtracted 17 from 151 rather than added it. ACT expects that a certain number of
test takers are going to ma.ke mistakes of this sort on the math section. Be sure yo u are working
carefully but quickly. Don't find yourself trapped into thinking a particular answer looks good
because yo u incorrectly arrived at the exact same answer.

-- GMAT MATH WORKBOOK 15


• Don't be afraid to work backward from the answers on quantitative problem-solving
questions. Because your answer choices are all right there, the correct answer is necessarily
staring yo u in the fa ce. So make use of your options! Plug in each answer one ata time; one
of them is bound to fit. One thing yo u might have noticed, too, is that the answer choices
are always arranged in size arder. Startingwith choice (C) will often allowyou to cross out
the two choice above or the tvvo choices below and then move on. Here's an example:

Kumi is 4 times as old as Aimee. 3 years ago, Kumi was 5 times as old as Aimee. How
old is Aimee today?
(A) 9
(B) 12
(C) 15
(D) 45
(E) 48

This is the perfect example of a question that you could plug in answers. Who wants to start
pulling numbers out of thin air to se e if they work? Let' s start with (C) and assume that Aimee
is 15 years old today. Remember, that is what the question is asking. The first sentence says
that today Kumi is 4 times as old as Aimee. That would mean that Kumi is 60 years old. Our
next sen ten ce tells us that 3 years ago, Kumi was 5 times asoldas Aimee. So 3 years ago, Kumi
was 57 (60 - 3) and Aimee was 12 (15 - 3). Is someone who is 57 years old 5 times asoldas
someone who is 12? No. So we can cross out (C) and move on.
Now, if we're thinking clearly, we know that the numbers we arrived at for (C) are quite
clase to where we want to be: 12 times 5 is 60, and 57 is very clase to 60. So we're going to
want to stay in the range of 15. In fact, we can cross out answers (D) and (E) because they are
nowhere clase. This is called aggressive elimination, something that is going to be your best
friend for this exam. We could also consider that if Aimee is either 45 or 48, Kumi would be
clase to 200 years old. The GMAT isn't going to present you with impossible circumstances.
Let's instead work backward and try (B). If Aimee is 12 years old today, then Kumi is 48 years
old. So 3 years ago, Kumi was 45 (48 - 3) and Aimee was 9 (12 - 3). Is someone who is 45
years old 5 times asoldas someone who is 9? Absolutely. We know our answer is (B), and we
really had to consider only two answer options.
Remember that working backward from the answer choices may take considerably longer
than using an algebraic solution, especially when yo u cannot eliminate any of the answer
choices from the beginning. Work backward sparingly. Use the process of elimination first
where possible.

These are the fun ones, relatively


• Plug in numbers if the question gives you only letters.
speaking. You get to assert sorne kind of control by choosing the numbers you want to
use. Consider this method as an option whenever yo u are presented with a question
that is asking you about the relationship between any set ofletters. Here's an example:

If x + y= 24, and x- y+ z = 15, then 4x + 2z =

(A) 6
(B) 9
(C) 45
(D) 78
(E) 126

16 GMAT MATHWORKBOOK~~~
That' s a lot of letters and not a whole lot of numbers. These questions can be radically
disconcerting until you realize that you get to choose the numbers that you want to work
with. Essentially the problem is in your hands. This is also where choosing numbers wisely
comes in. You don't want to give yourself any more work than is necessary to,complete the
problem.
So we've got two equations with three unknowns since nobody has given us a single clue
as to what x, y, and z are. The only thing to do is to start plugging in your own numbers while
staying within the constraints yo u are given. The ilrst equation we've got is x + y = 24. Tliís
means our only constraint is that the two numbers we pick for x and y have to add up to 24.
Now here's what we mean by choosing wisely. A lot of potential number combinations will
get us to 24. However, it makes no sense to choose, say, 114 and -90. These numbers might
only complicate things for us further on. So let's let x = 20 and let y= 4.
We've let x = 20 and y = 4, and we know that x - y + z = 15. We can now plug in the
numbers we've determined for the first equation into the second: x- y + z = 15 can now be
, rewritten as 20- 4 + z = 15. Subtract the numbers on the left. We're down to 16 + z = 15,
which is a much easier equation to solve. Now we know that z = -1.
The hardest part is now complete. We've determined that one of the possible combina-
tions of x, y, and z are x = 20, y= 4, and z = -1. Now fill in the expression 4x + 2z. It can be
rewritten as 4(20) + -2, which can be solved. The answer is 78.
Alternatively, you can always use algebra to determine 4x + 2z. To answer the question,
yo u do not need to find x and z separately but just need to find 4x + 2z directly. Add the first
two equations side by side to eliminate the y-variable:

X+ y= 24
+x-y+ z = 15
2x + z = 39

Since the question asks for 4x + 2z, multiply both sides of the equation by 2 to get 4x + 2z = 78.
The more math practice you do, the more you'll enjoy questions made up almost entirely
of letters. You need to recognize the relationship among the set of letters the GMAT is giv-
ing you, pay attention to the constraints, and choose numbers that are going to make your
calculations as easy as possible.

• Remember that estimating can often eliminate a few answers for you right away. The
GMAT doesn't care how you get to the correct answer. All that it registers is whether
or not yo u got there. This means that estimating is not the equivalent of lazy math.
Rather, it is a smart and quick way of eliminating those excess (wrong) answers almost
immediately. Estimating is sure to save you loads of time. Let's take a look at this
question:

Amixture of red beans and black beans is to be prepared. The price of red beans is $2
per pound, and the price of black beans is $3 per pound. What is the ratio of red beans
to black beans if the mixture is to be sold for $2.75 per pound?
(A) 1:3
(B) 1:2
(C) 2:3
(D) 1:1
(E) 3:1

GMAT:MATtfWORkBOOK:~ 17
Applying a little common sense befare we really start messying the scratch paper With
unnecessary calculations will come in handy here. The price of the mixture is $2.75, which
is closer to the price of black beans ($3) than the price of the red beans ($2). That means the
mixture has more black beans than red beans. Sin ce the question is asking us the ratio of red
beans to black beans, our answer needs to be a fraction that is less than l. Thus we can get rid
of any number that is equal to or greater than l. Cross off (D) and (E). See how quickly we're
down to just 3 answer choices.
All we really mean by estimating is recognizing that your answer has to fall into a par-
ticular range of possible numbers. Once you come to that conclusion, everything that falls
outside ofthat set is suddenly no longer an option.

• Always look at the answer choices before you begin on the question. The answer choices
will give yo u a sense of how much work yo u need todo. For example, if all of your
answer choices contain 'IT, you're not going to want to spend your precious time
converting everything to 3.14 only to have to undo it again when it's time to choose
your answer.
• Know the data sufficiency answer choices cold before you walk.into the exam. Ultima te! y,
you're going to be able to approach each data sufficiency question in the same way.
We suggest you pay particular attention to Chapter 6 ifyou've never seen a question
of its kind befo re, which is likely if this is your first go at the GMAT.

SECTION 1.9: REGISTERING FOR THE GMAT


You can take the computer-adaptive test 10 hours a day, 6 days a week, 3 weeks a month at
over 400 testing centers worldwide. You shouldn't have too many scheduling conflicts vvith al!
of those options. You can register for the GMAT online at www.mba.com ifyou've gota credit
card handy. You can also call one ofthe manytest centers listed online at www.mba.com ifyou
prefer to register in real time with an individual. You may also call1-800-717-GMAT if you're
in the U.S. or Canada. The registration fee as of2010 is $250. Because ofthe nature ofthe CAT,
it's certainly possible to register for an exam even up toa few days befare. Remember that test
centers start filling up in October. Finding a convenient center beco mes increasingly difficult
in November as MBA applicants are finalizing their entrance requirements. Either way, we sug-
gest you schedule a couple of months or at least a few weeks in advance. You want to ensure,
after all, that your test scores reach the institutions you're applying to in time. Knowing how
much time you have befare taking the GMAT is going to be crucial when working out your
study schedule. It also gives you a definitive deadline by which to consider yourself prepared.
Lastly, consider your biological rhythms when deciding on the time of day that's best for you.
Ifyou're a morning person, schedule an early appointment. If you don't start waking up un ti!
noon, keep that in mind. Arriving at the test center at the time of da y when yo u are at your peak
is going to be the final strategy in a long line of pivota! decisions. Follow through on al! these
decisions while on the road to acceptance into business school.

18 GMAT MATH WORKBOOK


Arithmetic 2
-+ 2.1 PROPERTIES OF REAL NUMBERS ANO INTEGERS
-+ 2.2 RATIOS ANO PROPORTIONS
-+ 2.3 PERCENTAGES
-+ 2.4 OESCRIPTIVE STATISTICS
-+ 2.5 COUNTING METHOOS ANO PROBABILITY

SECTION 2.1: PROPERTIES OF REAL NUMBERS AND INTEGERS


Real Numbers
• Real nurnbers include all nurnbers on the nurnber line (0.2, 4, 333, 11, {3, -.fS, ... ).

Negative numbers o(
1
) Positive numbers

-3.5-15 0.2 13
t t t t
-5 -4 -3 -2-1 o 2 3 4 5 . 6

• Ratio na! nurnbers are all numbers that can be represented as a fraction
(t· 0.004, -12.3, ~~' ... ).
• Irrational nurnbers are all nurnbers that are not rational cV2,-,f3, .fS, 11, .•. ).

They cannot be represented as fractions.


• Repeating decirnals are rational nurnbers (0.3 = k• 0.66 = t).
• Integers are all nurnbers that ha ve no decimal or fractional cornponents
( -3, 878, -99, ... ).

Note that every integer is also a ratio na! nurnber. Integers are a subset of ratio na!
nurnbers.

ABSOLUTE VALU E
• The absolute val u e of any nonzero nurnber is always positive. REMEMBER
The absolute value of x is written as lxl. lf x is positive, then lxl = x.
lf x is negative, then lxl = -x.
Exarnple: 1-41 = 4,1171 = 17,101 =O For example,
if x = 5, then 151 = 5 and
• For any nurnber, J-xl = lxl. if x = -3, then l-31 = -(-3) = 3.
• Iflxl=7,thenx=7orx=-7.

GMAT MATH WORKBOOK 19


PROPERTIES OF REAL NUMBERS
• Commutative Property:
a+b=b+a and a· b = b ·a
3 + 12 = 12 + 3 17·5=5·17

• Associative Property:
a+ (b + e) = (a+ b) + e and a· (b · e) = (a · b) · e
13 + (12 + 14) = (13 + 12) + l4 22 . (2 . 6) = (22 . 2) . 6
l3 + 26 = 25 + 14 22. 12 = 44. 6
39 = 39 264 = 264

• Use commutative and associative properties to simplify certain calculations.


First add the numbers that add up to 10 ora multiple of 10.

Example: In (17 + 8 + 13 + 12), rearrange to perform easier additions:


(17 + 13) + (8 + 12) = 30 + 20 = 50.

Multiply even numbers and the number 5 first wherever possible (4 · 5 = 20, 2 · 5 = lO).

Example: For 2 · 3 · 15, multiply2 and 15 first.


2 . 3 . 15 = 2 . 15 . 3 = 30 . 3 = 90

• Distributive Property:
~
a · (b + e) = a · b + a · e = ah + ae

Example: 3 · (x - 5) = 3 · x- 3 · 5 =,}x - ~5

• The distributive property is also very useful when simplifying certain calculations.

Example: 12 · 19 = ?

Instead of trying to multiply by 19, vvrite 19 as (20 - 1).


12 . 19 = 12 . (20 - 1) = 12 . 20 - 12 . 1 = 240 - 12 = 228

Example: 63 · 11 = 63 · (10 + 1) = 630 + 63 = 693

lntegers
Numbers that are included in the set {... , -4, -3, -2, -1, O, 1, 2, 3, 4, ... } are called integers.
REMEMBER
The integer • { ... , -4, -3, -2, -1} are negative integers.
o is Tleither • { 1, 2, 3, 4, ... } are positive integers.
positive nor • The integer Ois neither positive nor negative.
negative; lt
is evein.
ODD AND EVEN INTEGERS
• Integers divisible by 2 are even integers: {... ' -4, -2, o, 2, 4, 6, ... }
• Integers not divisible by 2 are odd integers: {... ' -5, -3, -1, 1, 3, 5, ... }

Multiplication Addition and Subtraction


(even) · (even) = (even) -8. 2 = -16 (even) + 1- (even) = (even) 4 + 12 = 16
(even) · (odd) = (even) 4. 7 = 28 (even) + 1- (odd) = (odd) -12 + 9 = -3
(odd) · (odd) = (odd) 11. -3 = -33 (odd) + 1- (odd) = (even) 5 + 17 = 22

20 ---GMATMATH WORKBOOK
• There are no even/ odd rules for division. One number m ay not be divisible by
another number. Also, an even number divided by an even number could be either
even orodd.

Examples: 12 -7- 2 = 6 (even -7- even = even)


6-7-2=3 (even -7- even = odd)

CONSECUTIVEINTEGERS
• Consecutive integers are integers that {-3, -2, -1, O, 1} or {61, 62, 63}
follow in a sequen ce:
• Consecutive even integers are even integers {-12, -10, -8} or {44, 46, 48, 50}
that follow in a sequence:
• Consecutive odd integers are odd integers {-11, -9, -7} or{l47, 149, 151}
that follow in a sequence:

When solving word problems, consecutive integers can be syrnbolized by 11, n + 1, n + 2, ....
Consecutive odd or even integers can be symbolized by n, n + 2, n + 4, 11 + 6, ....

PRIME NUMBERS
• A prime number is an integer greater
than 1 and divisible by only 1 and itself: {2, 3, 5, 7, 11, ... }
• l is not a prime number.
• The smallest prime number is 2.
• 2 is the only even prime number.

MULTIPLICATION AND DIVISION OF SIGNED NUMBERS


(positive) (positive) = (positive) 8 2= 16
(negative) (positive) = (negative) -4 7 = -28
(negative) (negative) = (positive) .-ll -3 = 33
(positive) (negative) = (negative) 18 -2 = -36
(positive) -7- (positive) = (positive) 8 2= 4
(negative) -7- (positive) = (negative) -4 2 = -2
(negative) -7- (negative) = (positive) -4 -7--2 = 2
(positive) -7- '(negative) = (negative) 4 -7--2 = -2

ADDITION OF SIGNED NUMBERS


• Iftwo numbers have the same sign, add the absolute value ofthe numbers and keep
the sign.

Examples: (-5) + (-7) = -12


42 + 28 = 70

• Iftwo numbers have different signs, subtract the absolute value ofthe numbers and
keep the sign of the number with the higher absolute val u e.

Examples: (-1) + (3) = 2


7 + (-9) = -2

GMAT MATH WORKBOOK 21


SUBTRACTION OF SIGNED NUMBERS
• Change the sigo of the number that is being subtracted. Then add the numbers.

Examples: (-2)- (5) = (-2) + (-5) = -7


(-3)- (-4) = (-3) + (+4) = l
(7) - (8) = (7) + (-8) = -l

-1, O, 1 ANO NUMBERS IN BETWEEN

• a+O=a a· O= O ~ is undefmed %= Oas long as a *O


a .
• a·l=a T=a
• a· - l =-a __!!:___=-a
-1
• The reciproca! of a number is l divided by that number. The product of a number
l
and its reciproca! equals l. The reciproca! of 3 is The reciproca! of -~ is -l
The reciproca! is also called the multiplicative inverse.
• \AJhen you multiply any positive number by a number between Oand l, the number
gets smaller.

Examples: ! · 6 = 3

l.-ª-=--ª-
2 5 lO
----t ?o is less than both! and ~-
• The reciprocals of numbers between Oand l are greater than the original numbers.

Example: The reciproca! of ~ is i·


• The squares of numbers between Oand 1 are less than the original numbers.

Example: The square of ~ is 2\ .

• Ifo>~>l,then~>(~t>(~r

Exampie: ! > (!t > (!t ____.

• The reciproca! of numbers between -1 and Oare less than the original numbers.

Example: The reciproca! of -~ is -i- -~<


4
-1
5
• The squares of numbers between -1 andO are greater th:an the original numbers.

Example: The square of -~ is 2\.

• u -l > ~ > o, then (~r > (~r > ~


Example: ( -!r > ( -!r > -!

ORDER OF OPERATIONS (PEMDAS)


PEMDAS stands for
• Parentheses
• Exponents
• Multiplication and Division ·
• Addition and Subtraction

. ______ _::__--:----22'·-GMAT MATH WORKBOOK


INTRODUCTION TO EXPONENTS
• When a number k is multiplied by itself n times, it is represented as "-'". k is called the
base, and n is called the exponent.
2 . 2 . 2 . 2 . 2 = 25 k· k· k· k= k 4
• x1 = x 42 1 = 42 71 = 7
• xD = 1 13° = 63° = 2° = 1
Example: ((SZ - 22)3 .;-- 9 - 2)66 To start, calculate 52 •
((25 - 22) 3 7- 9 - 2)66 Subtract inside the parentheses.
( (3)3 7- 9 - 2) 66 Determine the inner exponent, 33 = 27.
(27 7- 9 - 2) 66 Divide first.
(3 - 2)66 = 166 = 1 Work inside the parentheses and then calculate
the exponent.
• lf there are consecutive multiplications and divisions in an expression, perform them
from left to right.
Example: 50_,_ 2 x 5 = 25 x 5 = 125 not 50·_,_ 10 = 5
See Sectfon 3.3 for complete coverage of exponents.

SAMPLE PROBLEMS

-3(-5) 2 - 7. 2 3 + 7. 3
3
=?
2 . 14 - 12 - 121

-3( -5) 2 - 7. 23 + 7. 3 Work the exponents first.


-ª-.
2
14 - 12 - 121 You can also perform the subtraction inside the absolute
value since absolute value signs are treated as parentheses.
-3 . 25 - 7. 8 + 7. 3
'ª-·14-1-101 Perform all multiplications, and replace 1-101 with 10.
2
-75-56 + 21
21- 10 Perform all additions and subtractions.
-75- 56+ 21 = -llO = _ 10
21- 10 11

The product of tour different prime numbers is an even number. Which of the follow-
ing is the least of the tour numbers?

(A) 1 (8) 2 (C) 3 (D) 5 (E) 7

All prime numbers except 2 are odd. An odd number times an odd number is always odd. So if the
overall product is even, there has to be at Ieast one even number in the mix. Therefore, the Ieast
number ofthe four is 2. Remember that lis nota prime number.

The answer is (B).

~ -~~ GMAT .MATI:LWORKBOO~ 23


lf O < b < a which of the following is always true?

(A)%> 1 (E) -b <-a

If O < b < a, then ~ < l. For example, if a= 3 and b = 2, then ~ < l.

(A) ~ > 1 is false because when ~ > 1, ~ < l.


(B) ~: > ~ is false because when ~ < l, its square is less than itself.

(C) ~ > ~: is true. Yo u can try numbers to verify your answer. If a= 3 and b = 2, then ~ > ~·
(D) ~ > ~ is false. Since ~ < l, it gets smaller as you take higher powers.
(E) -b < -a is false. Since b < a, -b > -a. For example, if a = 3 and b = 2, then
-2 > -3.

The answer is (C).

lf n + 1 is an odd number, which of the following must be an odd number?

(A) n (B) 2(n + 1) (C) n" (E) (n + 1)"

If n + l is an odd number, n must be an even number. For example, if n + 1 = 5 then n = 4.


Assume n = 4 and work your way through the answer choices.
(A) 4 (even)
(B) 2(4 + 1) = lO (even)
(C) 44 = 4 · 4 · 4 · 4 (even)
(D) 4 2 + 4 = 20 (even)
(E) (4 + 1) 4 = 5 · 5 · 5 · 5 = 625 (odd)

The answer is (E).

Factors, Multiples, Divisibility, and Remainders


FACTORS AND MULTIPLES
• The factors of a number are positive integers that divide the number evenly.

Example: The factors of 24 are 24, 12, 8, 6, 4, 3, 2, and l.

• Multiples of any given number are those numbers that can be divided by that number
evenly.

Example: 30, 45, 60, and 75 are all multiples of 15.


INTRODUCTION TO EXPONENTS
• When a number k is multiplied by itself n times, it is represented as Á-"''. k is called the
b<J.se, and n is called the e>qJonent.
2 . 2 . 2 . 2 . 2 = 25 k. k· k· k= k 4
42 1 = 42 71 = 7
13° = 63° = 2° = l
Example: ((5 2 - 22) 3 7 9 - 2) 66 To start, calculate 52 •
((25 - 22)3 7 9 - 2)66 Subtract inside the parentheses.
((3) 3 7 9 - 2) 66 Determine the inner exponent, 33 = 27.
(27 9- 2) 66
7 Divide first.
(3 - 2)66 = 166 = 1 Work inside the parentheses and 11'1en calculate
the exponent.
• If there are consecutive multiplications and divisions in an expression, perform them
from left to right.
Example: 50 7 2 x 5 = 25 x 5 = 125 not 50 7 lO= 5
See Section 3.3 for complete coverage of exponents.

SAMPLE PROBLEMS

-3(-5) 2 - 7. 2 3 + 7. 3
=?
23 . 14 - 12 - 121

-3(-5)2-7·23 +7·3 Work the exponents first.


l. 14- 12- 121 You can also perform the subtraction insíde the absolute
2
value since absolute value signs are treated as parentheses.
-3 . 25 - 7 . 8 + 7 . 3
l. 14 -1-101 Perform all multiplications, and replace 1-101 with 10.
2
-75- 56+ 21
21- lO Perform all additions and subtractions.
-75-56 + 21 -110
21-10 =-l-1-= -lO

The product of four different prime numbers is an even number. Which of the follow-
ing is the least of the four numbers?

(A) 1 (8) 2 (C) 3 (D) S (E) 7

All prime numbers except 2 are odd. An odd number times an odd number is always odd. So if the
overall product is even, there has to be at least one even number in the mix. Therefore, the Ieast
number of the four is 2. Remember that l is nota prime number.
The answer is (B).
lf O < b < a which of the following is always true?

(A)~> 1 (E) -b <-a

If O < b < a, then ~ < l. For example, if a = 3 and b = 2, then ~ < l.

(A) ~ > l is false because when ~ > l, ~ < l.


(B) ~: > ~ is false because when ~ < l, its square is less than itself.

(C) .~ > ~: is true. Yo u can try numbers to verify your answer. If a= 3 and b = 2, then ~ > l
(D) ~ > ~ is false. Since ~ < l, it gets smaller as you take higher powers.
(E) -b < -a is false. Since b < a, -b > -a. For example, if a = 3 and b = 2, then
-2 > -3.

The answer is (C).

lf n + 1 is an odd number, which of the following must be an odd number?

(A) n (8) 2(n + 1) (E) (n + 1) 0

If n + l is an odd number, n must be an even number. For example, if n + l = 5 then n = 4.


Assume n = 4 and work your way through the answer choices.
(A) 4 (even)
(B) 2(4 + l) = 10 (even)
(C) 44 = 4·4·4·4 (even)
(D) 4 2+ 4 = 20 (even)
(E) (4 + 1) 4 = 5 · 5 · 5 · 5 = 625 (odd)

The answer is (E).

Factors, Multiples, Divisibility, and Remainders


FACTORS AND MULTIPLES

• The factors of a number are positive integers that divide the number evenly.

Example: The factors of 24 are 24, 12, 8, 6, 4, 3, 2, and l.

• Mu! tiples of any given number are those numbers that can be divided by that number
evenly.

Example: 30, 45, 60, and 75 are al! multiples of 15.

24 GMAT MATH WORKBOOK


DIVISIBILITY

• A number is divisible by another number if the result (quotient) is an integer. The


remainder is O.

EXample: 32 = 8 · 4; therefore, 4 and 8 are factors of 32. So 32 is divisible by 4, and


32 is divisible by 8.

• If a number is divisible by two different prime numbers, then it must be divisible by


the product ofthose prime numbers.

EXample: 36 is divisible by 2, and it is divisible by 3. Therefore it is divisible by 6.

• If a number A is divisible by a number B, then A is also divisible by al! factors of B.


EXamples: 28 is divisible by 14. Therefore, it is also divisible both by 7 and 2 (14 = 2 · 7).
48 is divisible by 24. Therefore, it is also divisible by 2, 3, 4, 6, 8, and 12.
• Below is a list of common divisibility rules.
2-+ Al! even numbers are divisible by 2.
3 -+ If the su m of the digits of a number is divisible by 3, the number is also divisible by 3. REMEMBER
Example: To check if 1,458 is divisible by 3, add 1 + 4 + 5 + 8 = 18. 18 is lfa nGm~¡r'i~'
divisible by 3, so 1,458 is also divisible by 3. .divis[b!É;1 ~Y·
two différent ···
4 -+ If the last two digits of a number are 00 orare divisible by 4, then the number is primenll~befs,
divisible by 4. then .itn1úst ·
· be divisi~le by
EXample: 23,456 is divisible by 4 since 56 is divisible by 4. the product of
Alternatively, divide the last two digits by 2. If the result is even, then the number is those prime
numbei"s;
divisible by 4. Since the number is divisible by 2 twice, it is divisible by 4.

5-+ If the units digit of a number is O or 5, then the number i~ divisible by 5.


6 -+ If a number is divisible by both 2 and 3, then the number is divisible by 6.

Example: 546 is divisible by 6 because it is even (divisible by 2) and is divisible


by 3 (5 + 4 + 6 = 15, 15 is divisible by 3).

9 -+ If the sum of the digits of a number is divisible by 9, the number is also divisible by 9.

EXample: To check if 4,608 is divisible by 9, add 4 + 6 + O + 8 = 18. 18 is


divisible by 9, so 4,608 is also divisible by 9.

PRIME FACTORIZATION

• Every integer greater than 1 can be expressed as a product of a set of prime numbers.

EXamples: 30 = 2 · 3 · 5
124 = 2. 2. 31 = 22 • 31

• Prime factorization is finding which prime numbers multiply 108


together to result in the original number. To find the prime
factors of a number, start by dividing the original number by
0 / "/ " 36

a prime number. Write the resultas the branches of a tree as


0 {:;\/"
18
\!:.)9
shown on the right, 108 = 3 · 36. Keep moving down until you /"-
are left with prime numbers only. 00
108 = 2 . 2 . 3 . 3 . 3 = 22 • 33

GMAT MAtH WORKBOOK 25


• To find the total number of al! positive factors of an
integer, first use prime factorization. If A= a"' · bn : e',
the total number of al! positive factors of A is
(m+ 1)(n + 1)(t + 1).

Example: 288 = 25 • 32, so the total number of aii


positive factors of288 is (5 + 1)(2 + 1) = 18.

LEAST COMMON MULTIPLE "LCM" (OR LEAST


COMMON DENOMINATOR IN FRACTIONS)
• Tó find the LCM of two numbers, find<the prime factorization of eaeh number. Then
multiply one of each common factor and al! ofthe factors that are not common.
• Organizing the prime factors as shovvn below is a simple way to see the factors that go
into the LCM.

Examples:
24 = 2. 2. 2. 3 120 = 2 . 2 . 2 . 3 . 5
32 = 2 . 2 . 2 . 2 . 2 48 = 2 . 2 . 2 . 3 2
LCM = 2 · 2 · 2 · 3 · 2 · 2 = 96 LCM = 2 · 2 · 2 · 3 · 5 · 2 = 240

REMAINDER PROBLEMS
• A remainder is the integer left over when you divide two numbers. It is not the
decimal par:t of.the.quotient..

5 R= 2 quotient R = remainder
3) 17 17 7 5 = 3, remainder = 2 divisor) dividend
17 = (3. 5) + 2

• When an integer is divided by an integer larger than itself (for example 9 7 12), the
quotient is zero and the remainder is the smaller integer (quotient =O, rernainder = 9).

COUNTING PROBLEMS
• Inclusive means the two endpoints of a range are included in the set.

Exarnple: How rnany integers are between 75 and 152 inclusive? 152- 75 + 1 = 78

Inclusive: Final Nurnber - Initial Nurnber + 1

• Exclusive means the tvvo endpoints of a range are not included in the set.

Exarnple: How rnany integers are between 46 and 112 exclusive? 112 - 46 - 1 = 65

Exclusive:· Final Number - lnitial Number - 1

26 GMAT MATH WORKBOOK


PLACEVALUE

• The value of a digit depends on its place in a number. The places are named as
, follows:
~
.--<

8 ~
o
o
.-<
o e:
o
.--;
oo .--;
o o
.--;
e?,
::!'- .--< S ......
~ ..... ci ......
~ 8.--; "50
...... "50 "bb
"bb"50
......
:a ......
::!'- :a :a
:a :a ::'2 en
¡:::
·a ...... U)
U)
..e
U)
.... ..... "50 ..e ......
'"O
¡:::
U)
'"O "50 ·a o..
ca :0
......
'"O
'"O
¡:::
ro
U)
Cl)
..... :0 2 8 U)
Cl)
..... ro
U)
'"O
;:J
o ¡::: Ul
¡:::
U)
Cl) ·u -5¡::: '"O
¡::: ;:J
o
..e ;:J ¡:::
...... ..e Cl)
...... o
Cl)
'"O
Cl)
......
;:J
..c ..e
......

8 2 3 4 4 o 3

8,234.403 = (8 · 1,000) + (2 · lOO) + (3 · 10) + (4 · 1) + (4 · 0.1) + (0 · 0.01) + (3 · 0.001)

Example: 43.147 = (4 · 10) + (3 · 1) + (1 · 0.1) + (4 · 0.01) + (7 · 0.001)

• The place val u e of a number is the product of the number by the val u e of its position.

The place value of 3 in 365 is 3 · lOO = 300


The place val u e of 5 in 1,365 is 5 · 1 = 5
The place value of6 in 0.365 is 6 · 0.01 = 0.06

Example: If P and Q represent digits, the valu e of PQ can be calculated as PQ = 1OP + Q.


Similarly,

QP= 10Q + P
PPQ= lOOP+ lOP+ Q
335 = (3 · 100) + (3 · lO) + (5 · 1)

ROUNDING
• To round a number to a specific digit:

1. Check the digit to the right of that specific digit.


2.If the digit to the right is 5 or more, round your digit up one. If it is less than 5,
make no change.
3. If yo u are in the decimal places, elimina te all digits to the right of the specific digit.
Ifyou are rounding the whole part, replace al! numbers to the right with Os.

Examples: Round 23.058 to the nearest hundredths.

The hundredths digit of the number is 5, and the digit to the right of it
is 8. So round 5 up to 6, and eliminate 8.

23.058 = 23.06

Round 34,643 to the nearest hundred.

The hundreds digit is 6, and the digit to the right of it is 4. Make no


change to 6, and replace al! numbers to the right with zeros.

34,643 = 34,600

GMAT MATH WORKBo_O_K, 27


SAMPLE PROBLEMS

lf lJ; is an integer, how many different integer values can n have?


(A) 4
(B) S
(C) 6
(D) 10
(E) 12

Write 18 as a multiple of its prime factors using prime factorization: 18


18 = 2 . 3 . 3 = 2 . 32· Ci/ "'/"-.
-9
1. 18
2. 9
=
=
18
18
00
3. 6 = 18

The positive factors of 18 are 18, 9, 6, 3, 2, and l. The question does not specify
that the result is a positive integer, so we need to count the negative values as well,
-18, -9, -6, -3, -2, and -l. Therefore n can have 12 different values.

Alternatively, use the formula given earlier. If 18 = 2 1 • 3 2 , then the number of all positive
factors of 18 equals (1 + 1)(2 + 1) = 6. Since the negatives of all those numbers will make 1~
an integer as well, there are 6 · 2 = 12 values for n.
The answer is (E).

How many digits are used to number the pages of a 250 page book?

(A) 250
(B) 251
(C) 640
(D) 642
(E) 750

The question is asking us to count each digit. There are nine 1-digit numbers, ninety 2-digit
numbers (99 minus 9) and 151 (250 minus 99) 3-digit numbers. Therefore:
1·9 9
2. 90 = 180
3. 151 = 453
Total = 642

The answer is (D).

28 GMAT MATH WORKBOOK


' --:::'"·:· ,.._...._ "-::_.:~
---~--------~--- : _ _ _. ..
__;;;_,:.~...::..:~--=--:..-:-:.-.:.:
When x is divided by 5, the remainder is 2. Which of the following will result in a
remainder of 4 when divided by 5?

(A) X+ 1
(8) X+ 3
(C) X+ 5
(D) X+ 7
(E) X+ 9

The easiest way to sol ve remainder problems is to find a nurnber that works for the question.
Since the remainder is 2 when x is divided by 5, we could use 7 or 12 for x. Let's stick with the
smaller option, 7.
(A) 7+1= 8, remainder = 3
(B) 7+3 = 10, remainder =O
(C) 7+5= 12, remainder = 2
(D) 7+7= 14, remainder = 4
(E) 7 +9= 16, remainder = 1
The answer is (D).

lf 60 · n is the square of an integer, what is the least possible value that n could have?

(A) 6
(8) 9
(C) 12
(D) 15
(E) 60

Write 60 as a multiple of its prime factors: 60 = 2 · 2 · 3 · 5 = 22 • 3 · 5.

22 is airead y a perfect square. If we multiply 60 by another 3 and by another 5, then all prime
numbers will be perfect squares -7 2 · 2 · 3 · 5 · 3 · 5 = 22 • 32 • 52 •
The least number we could multiply it by is 15 (3 · 5). The number 60 also works, but 15 is
less than 60.
The answer is (D).

GMAT MATH WORKBOOK 29


Fractions
• A fraction stands for the division or the ratio oftwo numbers.
a-c- b, a and a:b al! mean "a divided by b."
b'
a-> numerator
b-> denominator
27 = 27
1
~ = undefined

EQUIVALENT FRACTIONS
• Ifboth the numerator and the denominator of a fraction are multiplied (or divided) by the
same nonzero number, the value of the fraction does not change. The resulting fraction
and the original fraction are equivalent. In other words, they have the same value.
a_a·c_a-c-k
b- b·c- b-c-k

Example:
12 _ 12 -c- 6 _ 2 _ 2 · 5 _ lO
T8 - 18 -;- 6 - 3 - 3 . 5 -15
12 3'
T8' . lent f ractwns.
2 an d lO are eqmva .
15

MIXED NUMBERS
• Mixed numbers contain an integer anda fraction portian. 3~ is 3 and ~· which is
essentially 3 + ~·
• To convert a mixed number into a fraction, multiply the denominator by the whole
number and add that product to the numerator.
thcn add

c:"2 (5 · 3) +2 17
C3= 3 = 3
firsl multjpJy

• To convert a fraction into a mixed number, divide the numerator by the denominator.
The quotient beco mes the whole number portian, and the remainder beco mes the
new numerator. Note that the denominator never changes.
19 3 R= 4
5 5 )lg

REDUCING FRACTIONS
• Reducing a fraction means dividing the numerator and the denominator by the same
nonzero number until they have no common factors. The resulting fraction is said to
be reduced to lowest terms.

Example:
72 - 72 -;- 2 - 36 - 36 -;- 6 - 6 - 6 -;- 2 - 3
48 - 48 -;- 2 - 24 - 24 -;- 6 - 4 - 4 -;- 2 - 2

• You can divide by any nonzero number in any arder as long as you divide the numer-
ator and the denominator by the same nonzero number.
• G!'vlAT questions vvill require you to reduce fractions as muchas possible to save time
and effort. In addition, the answer choices will always be given in their lowest terms.
ADDITION ANO SUBTRACTION OF FRACTIONS
• To add or subtract two fractions with the same denominator, add or subtract the
numerators and keep the denominator.
-ª+s;_=a+c
b b b
a c_a-c
b b- -b-
Examples: ~--'-1=~=1.±
5 ' 5 5 5
_]__ JL = _ _l_
15 15 15

• To add/subtract two fractions with different denominators:

1. Start by finding the LCM ofthe two denominators.


2. Expand each fraction so they all have a common denominator.
3. Add or subtract the numerators, and keep the denominator.
2 5 Find the least common multiple of 14 and 21.
14 + 21
14 = 2. 7
Expand both fractions to make their
21 7. 3
denominators 42.
LCM = 2 ·7 · 3 = 42
2 3 5 2- 6 10 - 16 Simplify.
14 . 3 + 21 . 2- 42 + 42 - 42
16..;.. 2- 8
42..;.. 2- 2f

MULTIPLICATION OF FRACTIONS
• To multiply two fractions, multiply numerators by numerators and multiply
denominators by denominators.
a b_a·b
c'd- c·d
Example: ~. 14 . .§_ = 168
7 3 5 105
Divide both the numerator and denominator by 21.
168 ..;.. 21 =
105..;.. 21
ª-5 = 1"ª-5 REMEMBER
Always simplify before you multiply.
Note: A better approach is to simplify before you multiply. When
49 15 ~ 15
3

multiplying fractions, any numerator can be simplified by any . 25 . 14 = ;a5 . J4'


S 2
denominator. 21
= 10
2 2

Example: J·lJ ·~ y·y·~


l l
= = ~ = l~
= 2.1

DIVISION OF FRACTIONS
• To divide tvvo fractions, take the reciproca! of (flip) the divisor (the second fraction)
and multiply.
g_,_s;_=g·Q.=a·d
b d b e b·c

GMAT MATH WORKBOOK 31


Examples:
NOTE
ll _,_ ~ = ll . H = ll . ~ = 22 = 4 ~
2

~ a
3 · 6 ~z s 1 s s s
b b a 1 a -3 ...,... 12 -_ Z
1
- . - 1 -_ -1 . -1 -_ - 1
e: = f. = b. e = be 7 7 ~4 7 4 28
1
a 13 7 l = 1l. . l = 39
a. T a e ae 3 l 1
b=b=T·b=b
e e

Note:}o multiply or divide mixed numbers, convert them into fractions first.

51 7 1 = 46 7 1 = 46. t= 46. z. = 92 = 30 ~
9 6 9 6 39' l 3 l 3 3

CONVERTING A FRACTION INTO A DECIMAL

• To convert a fraction into a decimal, simply divide the numerator by the denominator.
0.8
Example: ~ = 0.8 5 )4.0
4.0
o
CONVERTING A DECIMAL INTO A FRACTION
• To convert a decimal into a fraction, remove the decimal point and divide by a power
of 10, based on the number of decimal places. For 1 decimal place, divide by 10.
For 2 decimal places, divide by 100. For 3 decimal places, divide by 1,000 and so on.
Then reduce the fraction.

0.42 2 decimal places, so divide 42 by 102 (100) 42 -21


lOO - 50
1.025 3 decimal places, so divide 1,025 by 10 3 (1,000) 1,025 - 41
1,000- 40

"SPLITTING" FRACTIONS

• If a fraction has one term in its denominator andan addition or subtraction in its
numerator, it can be written as the addition or subtraction of two fractions.
a+b=Q+]z
e e e

• If the denominator is a su m or difference of two terms, yo u cannot split the fraction.


_a_*Q+Q
b+e b e

Example: _ l L
12- X
* ll12 - ll X

COMPLEX FRACTIONS

• A complex fraction has a fraction in the numerator and another in the denominator.
• Vlhen simplifying complex fractions, start by carrying out the operations in the
numerator separately from those in the denominator.
SAMPLE PROBLEMS

=?

~--------------------------------------------J
_g_+l First perform the operations in the numerator and in the denominator
3 6
1.ª- of the complex fraction.
7 2
Make the denominators equal in the numerator by multiplying the
1+1
6 6
_1_2_ first fraction by~-
14 Multiply the fractions on the bottom.
5
6
u To divide the fractions, flip the bottom fraction and multiply.
14
7
!2.. 14 = 35 Simplify befo re yo u multiply.
H 12 36
3

=?

Solution l:
l+l First carry out the operations in the numerator and in the
9 6
43 denominator ofthe complex fraction.
9 2
l . .f.+l.ª-
9 2 6 3 Expand the fractions to make the denominators equal.
4 2 3 9
9·2-2·9
2 3
18+18 Add the fractions in the numerator, and subtract those in
8 27 the denominator.
18-18
5
18 To divide the fractions, flip the bottom fraction and multiply.
-19
18
¡

2 · M = _2._ Simplify.
1
M -19 19

GMAT MATH WORKBOOK----33


Solution 2:

First find the least common multiple of all denominators:


LCM (2, 3, 6, 9) = 18.

Multiply both the numerator and the denominator by 18, and


distribute the 18.

COMPARING TWO FRACTIONS

• If two positive fractions have the same denominator, the one with the greater
numerator is larger.
8 5
u>u
• In general, to compare fractions, make all denominators the same. The fraction with
the greatest numerator is the largest.

a=~
3
a=~=.lQ_
3 105
70 63 60 -7 a>b>c
105' 105' 105
• If two positive fractions have the same numerator, the one with the smaller
denominator is greater.
8 8
u>n
• If a positive fraction has a greater numerator and a smaller denominator than another
positive fraction, the former is greater.
8 5
T7 > 19
• If only two fractions are to be compared, you can cross multiply.

27x28
9 2 When comparing 9 to ~~ multiply 3 and 9, and write 27 on the left.
14
14 3 Multiply 14 and 2, and write 28 on the right.
9
14 < 2
Since 28 is greater than 27, ( 4 < ~-
3

SA1v1PLE PROBLEMS

3 4
-;¡--=¡ .Jl-?
5 9 26- .
17 + 14
Convert the mixed number into a fraction.
Find the LCM ofthe denominators: LCM(2, 7, 14) = 14.
Multiply both the numerator and the denominator of the
complex fraction by 14.
Distribute the 14 and simplify.
-ª-.
2
14- ±. 14
7 .ll Simplifythedenominatorandthenumeratorofthecomplex
ll. 14 + JL · 14 26 fraction.
7 14
3 . 14 4 . 14
- 2 - - -=¡ .11 = 21 - 8 .11 = l-ª-.11 = l._:_l = l
12 . 14 9 . 14 26 24 + 9 26 33 26 3 . 2 6
-7-+14

. 6 1 4 10 7
lf the numbers , 3• , 7' and were ordered from least to greatest, what would
19 13 2 15
be the third number from the end?

6 4 .!Q 7
(A) 19 (C) 13 (D)
27 (E) 15

The third number from the end is the middle number.


If yo u loo k dosel y, al! the fractions are very e! ose to l
An efficient way to compare al! five fractions is to compare each fraction to l
• To compare 6 , expand 1-_ 6
. S.mce they have the same numerator, 6 < T8
6_1
- .
19 3 18 19 3
6 .ts less tl1an 1 .
19 3
4 l 4. 4 4_1
• To compare TI' expand 3 = u· Smce they have the same numerator, TI < TI - 3"
4 ts
TI . less th an 1 .
3
lO .
• To compare Z7' expand l -_ 97' Smce
. ·
they have the same denommator, lO
> 9 -_ 1.
3 2 27 27 3
lO.ts greater t l1an l .
27 3

• To compare 7 , expand
15
i =
5 . Since they have the same denominator, 7 > 5
15 15 15
= l
T5 ts greater th an 1.
7 .
3
Two of the fractions are to the right of (greater than) i• and two of them are to the left of
(less than) l
Therefore, the middle fraction is t-
The answer is (B).

Note: In this case, you did not ~eed to compare the two smaller fractions,
1~ and 19 .
6
76 78

If need be, you can compare fractions by cross multiplication


4 6.
1~x169 .
Therefore, TI<
19

GMAT MATH WORKBOOK 35


lf ;, is a number between ~ and ~· how many of the possible values of n are prime?

(A) 3 (B) S (C) 7 (D) 9 (E) 11

Make all denominators equal for easy wmparison: ~ · ~ = and ~ if 6 ·l-""'


2 1. Therefore,
6 < ~ < ~f. Son could be 7, 8, 9, 10, 11, 12, or 13. The prime numbers among the possible
21 2
values are 7, 11, and 13.
The answer is (A).

('
-··.·, .._ ..,·,
\ Decimals
RE:MEMBER
• Decimals are another way to represent fractions. Think of them as fractions with
Ádding zeros
denominators that are multiples of 10.
to the ~ight.of · ·
thed:ecimal 0.7 170 - 12 - 12
= 0.12- lOz- lOO
does not affect ·
itsvalue .. · • When comparing decimals that have a different number of decimal points, add zeros to
·.. -.· < ··.
0.03 == 0,030 .· ...·•. the right ofthe dec~mal points so each number has an equal quantity of decimal places.
····= 0.0300 Example: Anange 0.03, 0.023, 0.12, and 0.008 in descending order.
' /
Write them in a column, making sure each has an equal number of decimal places.
0.030
0.023
0.120
0.008
This way, it is easier to see that 0.120 > 0.03 > 0.023 > 0.008.

ADDING AND SUBTRACTING DECIMALS


• To add or subtract decimals, align along the decimal points.
Example: 0.007 + 0.1 + 17.013 + 2.001 =
0.007
0.1
17.013
+ 2.001
19.121
NOTE
To multiply a decimal by 10 MULTIPLYING DECIMALS
quickly, move the decimal1 • First multiply the decimals as if there are no decimal places. The
place to the right.
number of decimal places in the product is equal to the total number
12.34 . 10 = 123.4
of decimal places in the original numbers.
lf multiplying by 100, move
the decimal 2 places. Example: 0.03 · 12.5 = ?
lf multiplying by 1,000,
2 decimal places + 1 decimal place = 3 decimal places •
move it 3 places and so on.
125 · 3 = 375 So 0.03 · 12.5 = 0.375
Example: 0.04 2 = ?
NOTE
0.04 2 = 0.04 . 0.04
To<divide a decimal by 10
2 decimal places + 2 decimal places = 4 decimal places qtiickly,move the decimal1
place tp th,é left.
4 · 4 = 16 So 0.04 2 = 0.0016 =.
1.23 7 10 .0.123
.. ..
by
lf dividing 100, m ové the
DIVIDING DECIMALS · deciírlal 2 places.
• First multiply each number by multiples of lO until the divisor beco mes lfdivié:ling by 1000, move 1t
3 places andso on.
an integer. Then complete the division regularly. If the dividend still
has decimal places, use the same number of decimal places in the
quotient.
0.5
Example: 0.125 + 0.25 = (0.125 · 100) + (0.25 · 100) .= 12.5-:- 25 25 Jl2.5
12.5
0.0
Alternatively, \Nrite the division as a fraction, expand to convert the decimals into
integers, and simplify.
0.125. 1000 = 125 = l =o 5
0.25 1000 250 2 .

SAMPLE PROBLEMS

0.0045 . 0.09 - ?
0.015 . 0.27 - .

Solution l: Write each decimal as a fraction. Then "flip" the fractions in the denominator
and simplify.
_±L._j)_ 1
.a- 1 1 1
10,000 lOO 4B -9- 1,-GOO lOO _ 1 _
--·-·--·----01
_l§_. 27 lWlOO lOO lli E lO .
1,000 100 JO 1 1 .a-
l

Solution 2: Multiplyboth the numerator and the denominator by a power of lOto change
all decimals to integers. The numerator has a total of 6 decimal places. So multiply by
10 6 = 1,000,000.
1 1
0.0045. 0.09 1,000,000 = .45 ·-&- = _!_ = o l
o.o15. o.27 1,ooo.ooo ..J3· m 10. ·
5 .¡;-2

Solution 3: Fill in zeros to the right of each decimal as necessary to make the numerator and
the denominator have the same total number of decimal places. Then elimina te the decimal
places.
1
l5 1
0.0045 . 0.09 = 0.0045 . 0.09 = 4B ·-&- = _!_ = o 1
0.015 · 0.27 0.0150 · 0.27 J-W ·E lO .
lO z1

·· GMAT MATH WORKBOOK · 37


1.69 - ~ _,_ 0.04 - ?
1.3 20 . (0.4) 2 - •

Remember to perform the division befare subtraction (PEMDAS).


4
1.69 - lá _,_ 0.04 = 1.69 - .12 . Jkt(J
1.3 20 . 0.16 1.3 20 .DJM l
l
1.69 _ _lá .-1 = 1.69- u
1.3 52{1 1 1.3 5

Find the values of each fraction and subtract.

l. If O< x < 1 which one of the following is the greatest?


(A) X
(B) x2
(C) .JX
(D) x3
(E) x.JX
2. 12 randa m integers are picked among the numbers between 1O and 70. If 2 is added
to the tens digit of each number, by how much does the sum of these 12 numbers
in crease?
(A) 240
(B) 120
(C) 24
(D) 20
(E) 12

3. KTrepresents a 2-digit number. If John is I\.7years old and his brother is TKyears old,
which of the following could represent the age difference between the brothers?
(A) 11
(B) 16
(C) 17
(D) 18
(E) It cannot be determined from the information given

4. mis a positive integer and (m 3 + 5)1 is even. Which ofthe following must be odd?
(A) 3m+ 3
(B) m 2 +5
(C) m· (m+ 1)
(D) 2m+ 2m
(E) mm
5. vVhich of the following numbers are reciprocals of ea eh other?
I. 5 and ..f5
II. 2 an d 3
3 2
III. 11 and - 11

(A) I only
(B) II only
(C) III only
(D) II and III only
(E) I and II only

6. (0.5)2 is how many times the number 6?


2 0
(A) 0.2
(B) 8
(C) 20
(D) 40
(E) 50

7 0.007 . 0.42 -
. 2. 0.0049 -

(A) 0.3
(B) 0.7
(C) l
7
(D) 3
(E) 7

39 -
8. 201-

(A) 0.183
(B) 0.194
(C) 0.205
(D) 0.251
(E) 0.515

9. 1.?.+zl
7 3 =?
3 17 .
3 5- Til

(A) _li
21
(B) .?_1
19
(C) 1.li
21
(D) 76
21
(E) 76
7

--GMAT MATH WORKBOOK 39


0.03 . 32
10.
1 -:- ~121 =
0.06 + 20

(A) 3
3
(B) 27
3
(C) IT
9
(D) IT
(E) l
3
11. When mis divided by k, the quotient is 5 and the remainder is 26. What is the least
possible value of m?
(A) 57
(B) 130
(C) 143
(D) 161
(E) 184

12. E and m are integers. When E is divided by 22, the remainder is m - 4 and the quotient
is m. What is the least possible value of E?
(A) 18
(B) 22
(C) 26
(D) 88
(E) 111

13. If both ~ and ~ are positive integers, x can be how many different val u es?
(A) 2
(B) 3
(C) 4
(D) 5
(E) 6

14. If 20; x is a positive integer, what is the sum of all possible val u es of x?
(A) 5
(B) 6
(C) 21
(D) 22
(E) 42

15. When Kis divided by 5, the remainder is 2. Vvhich of the follovving must be
divisible by 5?
(A) 2K +2
(B) 3K + 1
(C) 2K- 2
(D) 3K- 1
(E) SK + 1

. 40..... GMAT. MATH WORKBOOK


-----··-·'
... -
. =·-~· -
16. 6PQ is a 3-digit number (P and Qare digits) that is divisible hy 4. vVhen 6PQ is
divided by 5, the remainder is 2. How many different values can P take?
(A) 6
(B) 5
(C) 4
(D) 3
(E) 2

17. Amir waters his lawn once every 12 days, and Masa waters his Iawn once every 15 days.
In how many days will they water their lawns on the same day after the first time they
water together?
(A) 3
(B) 15
(C) 30
(D) 60
(E) 90

18. rt-t 7 ~+~r=


(A) l
5
(B) l
4
(C) 1

(D) 4

(E) 5
1
1 +1+3
19. - - - +(1r-
- -
2 1-l 6
3

(A) l
3
(B) l
2
(C) 6t

CD) st
(E) 9

20 0.02 + 0.0021 + 0.42 =


. 0.2 0.21 420

1
(A) 1110

111
(B) 1000

(C) 1110
1000
(D) 111
100
(E) 1100

GMAT MATH WORKBOOK 41


21. x= 130 y= 152 z= i~
\IVhich of the following must be true?
(A) z<x<y
(B) y< x< z
(C) z< y< x
(D) x< z <y
(E) x< y< z

22. ( 1 + ! J. (1 + i) .(1 + ¡l ..... (1 + Al =

(A) 23l
2
(B) 23 l
23
(C) 23
2
(D) 22
23
(E) _l
23

23. (-º- + 55 + 5555) . -ª-


3 33 3333 5
(A) 5
3
(B) lO
3
(C) 5
(D) 8
(E) 13

24. m= 40 k= 400 n = 4000


39 399 3999
\IVhich of the follmving must be true?
(A) m>k>n
(B) n >k> m
(C) m> n> k
(D) 11 >m> k
(E) k> m> 11

15
25. 16 16 17 61 62
+ 17 + 18 + ... + 62 + 63
In the addition above, each numerator and each denominator ís íncreased by one. The
new sum ís how much greater than the sum above?
63
(A) 64
15
(B) 64

(C) 41
3
(D) 64

(E) _l
64
26. A bank prints a 3-digit serving number ranging fram 001 ta 999 far each custamer.
Rajiv's number was 962. Bianca arrived after Rajiv and received number 016. How
many customers received numbers after Rajiv and befare Bianca?
(A) 50
(B) 51
(C) 52
(D) 53
(E) 54

27. The sum of 5 different positive 2-digit integers is 130. What is the highest possible value
af the largest of these integers?
(A) 88
(B) 84
(C) 78
(D) 74
(E) 68

28. x and y are positive integers, and x 3 = 24y. What is the least possible value of x +y?
(A) 600
(B) 225
(C). 18
(D) 15
(E) 12

29. Wand Vare positive integers. When Wis divided by (V- 3), the quotient is
(V+ 3) and the remainder is 5. Which af the follovving could be the val u e of lV?
(A) 30
(B) 40
(C) 45
(D) 55
(E) 61

30. Letters A, B, C, D, E, F. G, and H are equally spaced an a number line in that arder.
If the distan ce between A and H is l, the distan ce A Gis haw m u eh langer than the
distance DH?
5
(A) 7
(B) 0.7
(C) 0.5
(D)-ª-
7
(E) .f_
7

GMAT MATHWORKBOOK-·- 43
1 2 3 4 5 6 7 8 9 lO 11 12 13 14 15 16 17 18 19 20

e A D E B E A B e E D D e D D B D E D B

21 22 23 24 25 26 27 28 29 30

E e D A D e B D e E

l. C Pick a number that fits the definition and compare. For example, let x = t (or use
0.25 ifyou feel more comfortable with decimals).
(A) x=l
4
(B) x 2 = _L
16

ccJ rx ={f =!
(D) (tt = l4
x3 =

(E)xf.X =-k · {f =t ·! =k
Choice (C) ! is the greatest.
Alternatively, compare the powers. Since O < x < 1, the number with the smallest
power has the greatest value.

2. A If l is added to the tens digit of an integer, its value increases by 10:


28 -138 38 - 28 = lO.
Ifyou add 2 to the tens digit, the value ofthe integer increases by 20:
28 ~ 48 48 - 28 = 20.
For 12 numbers, the sum increases by 12 · 20 = 240.

3. D If KTrepresents a 2-digit number, the value of this number is 10K + T. Similar!y, the
value of TKis 10T +K
For example, 25 = 2 · 10 + 5 and 52 = 5 · 10 + 2.
The difference is (lOT + K) - (lO K+ T) = 9T- 9K = 9(T- K)
Therefore, the difference must be a multiple of 9. 18 is the only m u! tiple of 9 in the
answer choices.
Alternatively, try numbers to discover the relationship.
21- 12 = 9
42- 24 = 18./
51- 15 = 36
9, 18, and 36 are al! multiples of 9.
4. E If (m 3 + 5)' is even, m 3 + 5 must be even since al! powers of odd numbers are odd.
If m 3 + 5 is even, then m 3 must be odd since odd + odd is an even number.
Final! y, if m 3 is odd, m must be odd. Check the answer choices individually to find an
odd result (see below).
Alternatively, plug in numbers and try.
Assume mis even. Plug in 2 (m 3 + 5)1. (23 + 5)1 = (8 + 5)7 = 137 is odd, so mis not even.
Assume mis odd. Plug in 3 (m 3 + 5)1. (27+ 5)1 = (32)1 is even. So m must be odd.
Let m= 3.
(A) 3m+ 3 = 3 · 3 + 3 = 12 even
(B) m 2 + 5 = 3 2 + 5 = 14 even
(C) m(m + 1) = 3 · 4 = 12 even
(D) 2m + 2"' = 6 + 23 = 6 + 8 = 14 even
(E) 33 = 27 odd

5. B The reciproca! of a number A is 1 div:ided by A. In other words, the product of a num-


ber and its reciproca! equals l.
Only option II contains reciprocals: 1 ..;- ~ = ~ and ~ · ~ = l.

6. E (0.5) 2 = o.s · o.s = 0.25 and 6 = 0.005


2 0
0250 = 250 = 50 times
0.005 5
f'lternatively,
0t = 0.25 . ~
2 0
= 50 times
200
You could also set up an algebraic equation:
(0.5)2 = n · 260
0.25 = n. 26o
n = 200 · 0.25 = 50

7 . A 0.007 · 0.42
2. 0.0049 One way to avoid dealing w:ith decimals is to expand the frac-
tion w:ith a power of 10 so that all decimals turn into integers.
In the numerator, there are a total of 3 + 2 = 5 decimal places.
In the denominator, there's only 4. Pick the larger one and
multiply both thenumerator and the denominator by
10 5 = 100,000.

0.007. 0.42 . 100,000 To multiply by multiples of 10 quickly, move the decimal


2 . 0.0049 100,000 places to the right as many times as the number of zeros.

7. 42 Divide both the numerator and the denominator by


2. 490
7 and by2.
-ª=l=o3
70 lO . Divide by 7 again.

GMAT MATH~WORKBOOK- 45
Alternatively,

0.007. 0.42 Add zeros to the right of decimal places to make all decimals
2. 0.0049
have an equal number of decimal places. If there are integers,
introduce a decimal point and place zeros to the right.
0.0070 . 0.42 Now that the numerator and the denominator have an
0.0049 . 2.00
equal number oftotal decimal places, you can re111ove the
decimal places.
1 1 3 1
70. 42 = ;(·J.lf·.-tr-7, =--ª--=o 3 Simplify to get 0.3.
49 · 200 ;r.;r.z. Jff ·lO 10 .
1 1 1 1

8. B A straightforward way is to divide 39 by 201 using long division.


Alternatively, (and much more quickly) use approximation and elimination.

]g1 is approximately equal to 2~ 0 l Since the denominator of fg1 is slightly larger


0
=

and its numerator is slightly smaller, ]g1 must be slightly less than ~ or 0.2. The closest
answer is (B) 0.194.
l_g_+ 2l Q+ 2
7 3 7 3
Convert all mixed numbers into fractions.
9. e 3"ª- - 11. .!§. 17
5 lO 5 10
27 + 49
21 21 Expand the fractions, and make the denominators equal.
36 17 Add the numerator, and subtract the denominator.
10 lO
76
21 Flip the denominator, and multiply.
19
lO
4
:76 .lO =_!_.lQ Simplify, and then multiply.
21 19' 21 1

40 = 1li
21 21

10. E 0.03 1 ..,_ ~


0.06 + vl21 2
Convert ~ into a decimal: lo = O. 05. ~ 121 = 11 and 32 = 9.
20
Add 0.06 + 0.05 = 0.11. Flip the second fraction
0.03 7~=
0.06 + 0.05 11 and multiply.
1 1
0.03 11 _ Z H _ l Expand the first fraction by lOO to get integers, simplify
O.ll ·g-- H. Z - 3 by 11, and multiply.
3

5 R = 26
11. D k)m
5 ·k+ 26 =m
If the remainder is 26, the least possible val ue of k is 27 sin ce the divisor is always
greater than the remainder.
If k= 27, then m= 5 · 27 + 26 = 161.
m R=m-4
12. D 22)E
22m+ (m- 4) =E
23m- 4 =E
To minimize E, m must be as small as possible. Since the mínimum value of a
remainder is zero, m - 4 = O. m = 4 is the mínimum value of m.
So the least possible value ofEis 23m- 4 = 23 · 4- 4 = 88.

13. C From the second fraction, we conclude that x has to be a factor of 48. 48 = 24 • 3.
The number ofpositive factors = (4 + 1)(1 + 1) = 10.
Factors of 48:
1 48
2 24
3 16
4 12
6 8
If ~ is a positive integer, x has to be a multiple of 6. Among the factors of 48, the
multiples of 6 are 6, 12, 24, and 48.

14 . D 20 X- X = 20 _
X
_! =
X
20 _ 1
X
2
2must be an integer. Therefore, x has to be a factor of 20.
The factors of 48 = {1, 2, 4, 5, 10, 20}.
2
2- 1 needs to be a positive integer. All factors work except 20 because ~~ - 1 = O
is nota positive integer. 1 + 2 + 4 + 5 + 10 = 22

15. D Find a number Kthat fits the initial definition, such as, 7. When 7 is divided by 5, the
remainder is 2. Then try the answer choices.
(A) 2 · 7 + 2 = 16 16-;- 5= 3 remainder = 1
(B) 3 · 7 + 1 = 22 22 -;- 5= 4 remainder = 2
(C) 2 · 7 - 2 = 12 12 -;- 5= 2 remainder = 2
(Dl 3 · 7 - 1 = 20 20 -;- 5= 4 remainder = O
(E) 5 · 7 + 1 = 36 36 -;- 5= 7 remainder = 1
Alternatively, write K as K= 5 n + 2 and plug it into the answer choices.
(A) 2(5n + 2) + 2= IOn + 6 Remainder = 1
(B) 3(5n + 2) + 1= 15n + 7 Remainder = 2
(C) 2(5n + 2) - 2= IOn+ 2 Remainder = 2
(D) 3(5n + 2) - 1= l5n + 5 Remainder = O
(E) 5(5n + 2) + l = 25n + ll Remainder = l

GMAT MATH WORKBOOK 47


16. B If the remainder is 2 when divided by 5, Q is either 2 or 7. If a number is divisible by
5, the ones digit must be 5 orO. If the remainder is 2, the ones digit is either 5 + 2 = 7
orO+ 2 = 2.
The 3-digit number is divisible by 4, so it is an even number. Therefore, Q must be 2.
If 6PQ is divisible by 4, the 2-digit number PQmust be divisible by 4. P could have
5 different val u cs. The Iast two digits of 6PQ could be 12, 32, 52, 72, or 92.

17. D This question is simply asking for the least common multiple of 12 and 15.
12 = 2. 2. 3
15 = 3. 5
LCM = 2 · 2 · 3 · 5 = 60

Alternatively, start writing the multiples of 12 and 15 until you find the first common
multiple.
12--24--36--48--60
15--30--45--60

To divide first, multiply -t by the reciproca! of ~-


1

[---·-+
l l 5 1 )"" Simplify and multiply.
5 5 4 4

lt-t+tr Add and subtract the fractions with the same denominator.

l
(-5
¡-!--5 Take the reciproca! since the exponent is -1 and a- 1 = ~-

1
19. D -1 + -+1
- + (1)-¡
- = Add and subtract the fractions in the numerator and
2 1 -1 6
3 denominator of the complex fraction first.
4
~+ ~ + lir To divide fractions, multiply the reciproca! of the
3 denominator by the numerator. (irwith 6.

l2 + 2 + 6 = l2 + s = al2

20 B Rewrite as 0·02 + 0 ·0021 + ---º.:1L Expand each fraction to convert decimals


. 0.20 0.2100 420.00
into integers.
2 21 42 Reduce each fraction.
20 + 2100 + 42,000

l ' 1 + 1 Make every denominator equal 1000.


T6 T lOO 1000
100 1o ' 1 - 111
1000 + 1000 T 1000 - 1000
21. E Ifyou compare each number to !· it is easier to notice that }~ is the largest.
3·¡ 1 1 5 3
10 !S ess than 2 because 2 = 10 > 10·

5·¡
TI 1 1_6 5
IS ess than
2 because 2 - 12 > 10.
12 > z
19 1 because 19-18- 2 W h'!C h !S
12 - 12 - 3' . greater than z·
1

To compare f0 with t2 , make the denominators equal. The LCD is 60.


3 _ 18 and 5 _ 25
10- 60 12-60

.1-ª. < 25
60 60
3 5
10<12
x<y<z

22. C Add a few of the terms to see if there is a pattern:

(~ + ~) · (i + ~) · (t + i) ····· (~~ + J2J = ~ · 4·% · ···· ~i ·~~


No ti ce that the numerator of the first fraction is the same as the denominator of the
second and so on. Simplify all the matching numbers up to the last fraction. Simplify
3 by 3, 4 by 4, and finally 22 by 22.

Whenever yo u
notice calculations
23. D Simplify the fractions inside the parentheses before adding. that might takea
very long time, look
55 ..;.. 11 =
33..;.. 11 3
ª-
and 5555 ..;.. 1111 = !2_
3333..;.. 1111 3
for a pattern that
can simplify your

(~ + ~ + ~) . ~ = 135 . = 8 i work.

24. A Write each fraction as a mixed number.

m= 40 = 1_l_ k= 400 = 1-1- 11


- 4000- 1 1
39 39 399 399 - 3999 - 3999
Since the integer parts are equal, the one with the largest fraction is the greatest.
m> k>n

25. D Do not attempt to add the given fractions. In questions that are seemingly very
tedious like this one, write out a few steps until yo u recognize a pattern.

If each numerator and each denominator is increased by one, we get


16 17 18 62 63
u+ 18
-L
' 19 + ... + 63 + 64'

GMAT MATH WORKBOOK 49

1•
Sin ce we need the difference, subtract the original from the new series of additions.
Realize that most terms cancel each other.

26. e After Rajiv, there were 999 - 962 = 37 customers. Befo re Bianca, there were
15 customers. The total is 37 + 15 = 52.

27. B Ifyou are trying to make the largest number as large as possible, the other 4 numbers
should be as small as possible. The 4 smallest 2-digit different integers are 10, 11,
12, and 13.
130- (10 + 11 + 12 + 13) = 130- 46 = 84

28. D First find the prime factorization of 24: 24 = 2 · 2 · 2 · 3.


x3 = 2 · 2 · 2 · 3 · y
To make the right side a perfect cube, y needs to be 3 · 3. That way, the right side
becomes 2 · 2 · 2 · 3 · 3 · 3, which equals 23 • 33 = 63 • So x = 6 and y= 9, and x +y= 15.

V+3 R=5
29. e V-3~
W= (V- 3)(V + 3) + 5
W= V 2 - 9 + 5
W= V 2 - 4
Whas to be 4 less than the square Óf an integer, 45 = 49 4. So W could be 45. Other
answer choices do not make W an integer.

30. E There are 7 sections between A and H, so each section is +long. AG is 6 sections long,
and DH is 4 sections long. So AG is 2 sections longer that DH.
2 sections = 2 · + = ~.

SECTION 2.2: RATIOS ANO PROPORTIONS


• A ratio expresses a mathematical relationship between two quantities. Specifically, the
ratio is the quotient of two quantities.

Example: lf the ratio of girls to boys in one class is 5 to 4, we can express it as i or


5:4 or 1.25.
• A proportion is an equation where t\vo ratios are equal. ~ = ~
• The easiest way to salve a proportion with an unknovvn is to cross multiply.
If-ª = .f then b · e= a· d.
b d'

····-·-::·::.::::·::.:::so:.::.-:GMAT-MATH WORKBOOK
• When setting up proportions, make sure the units of each side match each other.

Example: The ratio of the length of a rectangle to its width is 9 : 7. If the length of the
rectangle is 72 in ches, what is the perimeter of the rectangle?
length _ 9 9_ 72
\vidth - 7 7- width 9·w=72·7 width = 729· 7 = 56
Perimeter = 2(w + l) = 2(56 + 72) = 256

Direct Proportions
• Quantitics x and y are directly proportional if their ratio is constant: Ix = k
y .
(or y= k· x), where k is a constan!. In other words, x always has thc samc
nurncrical value.

Example: x and y are directly proportional. When x = -6, y= 8. What is the val u e
of ywhcn x = 3?

Set up the equation as y= k· x, and find k using the initial conditions.


y= k·x
k=I=--ª-=-4.
X -6 -3
.v=-±
-3 · x
Plug in x = 3 to find y:
REf'ltEMBER
.v=-±
-3 · 3 = -4
lf an increase in
Altcrnatively, sct up a proportion since i is always constant: one variable causes
an increase in
Y1 _ Yz
x 1 - x2 another variable,
these two vari-
_li Cross multiply. ables are directly
-6
proportional.
24 = --G · y2
y,= -4

lnverse Proportions
11 Quantities x and y are in verse! y proportional if their product is constant: x ·y= k,
where k is a constan t. In othcr words, x ·y always has the same numerical value.

Example: x and y are inversely proportional. Whcn x = 2, y=~-


What is x whcn .v = ,) i-?
Sct up thc equation as x ·y= k, ancl find k using the initial conclitions.
x·y=ck
ff an increase in
one variable causes
a decrease in
x ·r = s another variable,
Plu<>b in .r '-= J- to fi11d x:
~) .
these two variables
are invcrsely
r. L -5 proportional.
- 5

l
Alternatively, set up an equation sin ce x · y is always constant:
xl · Yt = xz · Yz

2·~=x2 ·!
xz = 25

SM1PLE PROBLEMS

4 n
lf 5 = 35 then n=?

n · 5 = 4 · 35 Do not multiply yet. Divide both sides by 5 to simplify.


7
1l = 4. 35 = 28
).)1

REMEMBER lf a lawyer charges $3SO for half an hour of her time, how many hours did she work
Since the if her invoice was $2,450?
GMAT is a
no calculator
test, simplify
whenever
$350 - $2,450
0.5hr-~
possible
before you $2,450 . 0.5 = $350 . J¡
carry out the 7

calculations. lz = $ 2 ·~~~ 0·5 = 7 • 0.5 = 3.5 hrs


1
Alternatively, find her hourly rate. If she charges $350 for half an hour, her hourly rate is $700
perhour.

The total time can be found by dividing the totai fee by the hourly fec: $:;~~º- = 3.5 hrs.

reads 100 pages in 12 minutes. How many pagr's can he read in one hour?

12 min _ 60 min
Ioo¡J"ag<:~s- lzpagcs
5
n ~_Q ~; OQ = 500 pages
..v~ 1

52 Gi\1AT t•VUH WORKBOOK


lf m · n = k · t, which of the following fractions are not equivalent?

(A) m -.!
k- n
k n
(B) m= t

(C) nk = Tm

For cach answcr choice, cross multiply and see if it matches the given cquation:
171
(A) = l_ --7 m · n = t · k
k ll

(B)
k
In = t12 --7 t· k= m· n

(C) lll = mt --7 t ·k= nz · n

(D) !l: = m --7 t· n= m·k


k t
(El .1_ = !i --7 t · k= m· ll
m k
The answcr is (D).

lf a : b = 3 : S and b :e = 3 : 4, what is a : e?

Thc qucstion can be written as follows:


{/ -- --
-- b -- -3 ti 1en
3 an d -- ; -{/ --- "~
b 5 e 4 e
Solution l: Cross multiply both ratios.

5({ = 3l; and 4/J = 3c Sin ce wc are looking for a ratio of ato e, we nced to climinate
/J. Soh-c for l; in the first cquation.

5a = 3b Plug this h into the sccond equation.

4·~r=3c !VIultiply both sides by 3.


20o ,~ 9c Divide both si des by 20c to obtain ~.
2_9_{_1 = -~e_ --7 (/ = _9_
20c 20e e 20

Solution 2: f\Iultiplr thc two ratios to eliminatc !J ami get thc third ratio,~­
{1 . J¡ o: _a
/¡ e e
~~. 3 = 9 -- (/
5 .¡ 26- (~
SECTION 2.2...:PRACTICE PROBLEMS

l. If fl =
1J
12e = 5, what is thc value of s_cz?

(A) ll
5
(B) l
5
(C) 1

(D) 5

(El 25

2. If ~~1 = ~= H· what is thc value of 111


~Js.?

(A) 31
(8) l
2
(C) 5
6
(D) ¡l
6
(E) ¡l
3

3. The ratio of brown eggs to white eggs in a basket is 1.5 to 1.8. If there are fewer than
25 brown eggs in the basket, what is the highcst possible number of total eggs in
thc basket?
(A) 22
(B) 24
(C) 44
(D) 55
(E) 60

4. If -]; = ~· then ~~-t3_f! = ?

(A) 4J

(B) 3

(C) 2~
(D) 2J
5
(E) lk
5. Thc ratio of fiction books to nonfiction books in Abi's library is ,¡ : 5. If shc buys 6 more
books of e;1ch kind, thc ratio would bcconw 5: G. Hmv many fiction books docs Abi's
library currcntly han~?

(A) 20
(!3) 2·1
(C) 25
(D) 30
(E) 3()

54 GN/\T MATH WORKBOOK


6. If x ~y= 1-, what is the value of ~?
(A) 2
3
(B) 3
4
(C) k
(D) - 1
4
(E) -4

7. If -4a = Jl = -6Sami a + lJ + e= 60, what is the val u e of j~_+.


~ a e
~?
(A) 3
(B) 6
(C) 8
(D) 12
(El 24

8. Three teenagers aged 12, 13, ancl 15 share $240 proportional to their ages. How much
does the youngest get?
(A) $6
(B) $12
(C) $40
(D) $72
(E) $78

9. If a machine can fill 1400 soda cans in onc hour, how many can it fill in 84 minutes?
(A) 1540
(B) 1750
(Cl 1960
(0) 2040
(E) 2240

·t 1' - , . .m·v·t,
10. It _:_==-=:::=J. what 1s the valuc ot - - · --?
m k r x·k·z
(A) 27

(B) 9

(C) 3

(D) _l
3
(E) ~

l J. Thc ratio of mountain bikcs to road bikes in a parking lot is 3: 5. What pcrccnt ofbikcs
in this parking lot are road bikc::;'?
(A) 32.5";,
(ll) el()'!(,
(Cl GO'Ji,
(D)
(E) ;¡n·
{ •.) o
12. If a~ b = a Zb, which of the following must be true'?
(A) a ís equal to b
(B) a is 5 times b
(C) a is 3 times b
(D) bis 3 times a
(E) bis 2 times a

SECTION 2.2-SOLUTIONS.

l
·
E -ªb = 5 and l!. = S
e
To find-ª, multiply -ªb by l!.. The b's wil! cancel ea eh othcr.
e e
Q.Q=!i.!i
b e l 1
-ªe = 2S

1
If = S, then a = Sb.
instead of b.
If *
Alternatively, cross multiply each ratio, solve for b, and substitute.
= S, then b = Se. Substitute Se into thc first equation

a= S· (Se)
a= 25e
-ªe = 25

2. E Split the fraction.

m+k=!!l+l:.
n n n
1
Find ~ ancl ~ separatcly and thcn acle!.
If m = !l thcn I'i = l.
3 6' 1l 6
If-~ = !.1 , thcn_k = .§_
5 6 11 6
!!l
n
+ _t;_ = l + Ji=
n6663
-ª = .1 = ll
3

Alternativcly, Jet~[= ~ = -~ = t, a constant number. We can rcprcscnt 111, k, ancln in


tcnns of t.

Ifli = 1, thcn 111 == 3t. Similarly, k= 51 ami 11 =-' 61.

!!l__~-_t;_ = }_t_:-_1:- 5 t = §_[ = Q = .1 = l l_


n 6t 61 6 3 3

56 GMAT i'lATH 'v'/ORKBOOK


3. C The ratio is fv
= i:~ = i~ = ~ since the number of eggs need to be an integer.
The highcst multiple of 5 that is lcss than 25 is 20. So there can be a maximum of
20 brown eggs.
5- 20 Cross multiply.
6-w
15W= 120
W=24
Total = 20 + 24 = 44

4. A Split the fraction.

2a + 3b = 2a + 3b = 2a + 3
b b b b
Sin ce~ = ~, plug it in the equation.
2. 3+ 3 = §_ +-ª- = §_ + 15
5 515 55
= ª = 41
5
AlternaÜvely, let a = 3x and b = 5x siüce !lb = ~ = 3 X
::J 5X
2a + 3b = 2 · 3x+3 · 5x = 6x + 15x = 21x = n = 4 1
b 5x 5x 5x 5 5

5. B Let the number of fiction books be 4x. The number of nonfiction books then
l
beco mes 5x sin ce thcir ratio is This means ~ = ~~-
4x + 6 _ 5
5 x +6 - 6 Cross mu 1tiply.

6 · (4x + 6) = 5 · (5x + 6)
24x + 36 = 25x + 30
36 =X+ 30
6=x
Thc numbcr of fiction books = 4x = 4 · 6 = 24.

6. E Remcmbcr that
X
x+y_3
--x-- 4 Cross multiply and distributc thc 4.
4x + 4y 3x Subtract 4x.
4y -x Divide by -y lo gel y·
~ = -4
J'

7. D Let ¡ =-" ~ =~ = k, a cons1an1 numher. \Ve can rcpresent a, lJ, ami e in terms of k.

a ~ 4k, lJ 5k, and e= 6k

If tl + b + e= 60, then 4k + 5k + Gk = 60.


15k~-·60

k= !)Q = 4
15
If k 4, tllen a== 1lk 16, b = 5k
:ce o-c. 20, ancl e Gk = 2-L
!1 · e ·- 20 · 24 ¿Q ·]_::l == 12
r1 +e- T6-~t--n 4o

G1AT rlAn·l WORX:BOOi{ 57


8. D lf the 12 year old gets 12x, the 13 year old gets 13x and the 15 year old gcts 15x.
The total is 12x + 13x + 15x = 40x.
40x = 240
x = $6 per year
The youngest one gets 6 · 12 = $72.

9. C Set up a proportion.
bottles _ 1400 _ x
minutes - (3() - 84 Cross multiply.

84 · 1400 = 60 · X Do not multiply yet. First divide both sides by 60.

X= 84 · 1400 Divide by lO and then by 6.


60
14 . 140 = 1960
1
m· y· t
lO. D x· k· z
Split the fraction into the multiplication of3 fractions, ~~ · ;; · { .•
x -
If m _ 3, t 11en x _ 1.
m - Y = 3.
We are given k If tz = 3, t h en zt = 1.
3 3
m.I.l=l.3.l=l
X k Z 3 3 3

11. D Let thc number of mountain bikes be 3x. Thc number road bikes beco mes Sx sin ce
MEMORIZE their ratio is 3:5. Then the total numbcr ofbikes is 3x + 5x = 8x.

~ == 12.5% The ratio of road bikes to total bikes ~~ = ~-


To convert to percentages, divide 5 by 8 and multiply by 100%.
~ == 37.5%

S ~ · lOO% = 5 ~ 0 % = 62.5%
8 == 62.5%

~ == 87.5%
12. B a; a b = 2 b Cross multiply.

3(a -- b) = 2(a + b) Distributc.

3a- 3b = 2a + 2b SubtrJct 2a ancl aclcl 3b.

a= 5b This mcans a is 5 times b.

SECTION 2.3-PERCENTAGES
¡¡¡ A pcrccntagc is a way to rcprcscnt a fraction with a denominator of lOO.

% (pcrcent) mcans "divide by 100" or lOO lll% mcans rWo


2 °'
/() -- Too
2 - O 02
-- · 2~1 0/O' -
--
27 - O 2~1
Too - · 2"'Jr:Or
0 /O
_
- Tero - .;
235 _ 2 3r:
J

a To fine! a pcrcentagc of a number, multiply that numbcr by the decimal cquivalent of


thc pcrcentagc. \Vhcn you scc "of," think multiplication.

Example: What is 4l'J;¡ of2S?


1
O.H · 25 = ll or _:Hr.5 '
--Hh
= :U_
4
= 11
4

58 GMAT ¡,¡_,\TH '1/0Hi<BOOi<


Alternatively, you can set up a proportion:
Part _ 44 _ x
Whole- lOO- 25 Cross multiply.

l
X = 4~5 = ~1 = ll
4
• To fin el what percent one number is of another number, simply divide the numbers
ancl convert the result into a percentagc by multiplying it by lOO% because 100% = l.

Example: B is what percent of 25?


4
~0%=32%
l .
8 is 32% of 25.
Alternatively, you can set up a proportion:
Part _ x _ 8
Whole- 100- 25
4
X = JOO' . S = 32
25
1
B is 32% of 25.

Percentage lncrease or Decrease


. New - Original
% mercase or decrcasc = . . · l 00%
0 ngma 1
Examplc: A supermodel slims clown from 120 lbs to 105 lbs for a show. What is thc
perccntage decrease in her weight?

Nev\'_-:-. ~rig!_~~. 100% = 10:¿_:::-_l_~Q. 100% = -15. lOO%= 12.5%


Ongmal 120 120
Note that thc ncgative sign rneans a decrease.

t<h.dtiple Percentage Changes


111 DO NOT add or subtract pcrccntage changes. Calcubte each separately.
111 Use 100 if the question clocs not givc yo u a number to start with.

Examplc: lhc population of Smallville incrcasccl by 20% from 2000 to 2001 and by
30'!{, from 2001 to 2002. By what pcrcent did thc population incrcasc from
2000 to 2002?

Assumc the population in2000 was 100.


The population incrcasc from2000 to 2001 is 100 · 0.2 = 20. So in 2001, thc population
was 120.
From 2001 to 2002, tlw increasc is bascd on 120. So, the increase was 120 · 0.3 ""3G.
lhe population in 2002 was 120 + 3G = l5CJ. Since we startecl at lOO, tlw last t\\'o digits
of 1;22 gin~ us the Óverall percentage increase: 56%. Note that this incrcase is grcater
than 20'ió + 30%.
_2()0_Q 2001
100 100. 1.2 = 121) 120·U=,I5G

+20% +30'~;,
MEMORIZE

1 1 - 2
100 = 0.01 = 1% 6 = 0.16 = 163%

1
50= 0.02 = 2% t = 0.2 = 20%

1
25 = 0.04 = 4%

1
*
1
= 0.25 = 25%

- 1
20 = 0.05 = 5% 3 = 0.3 = 333%

1~ = 0.1 = 10% t = 0.5 =50%

81 = 0.125 = 12.5% 1 = 1.0 = 100%

St\l\IPLE PllOBLEMS

,---------------CZH!Q!D
The price of a handbag is $204 after a discount of 15%. lf Aisha bought the handbag
at a 20% discount off the original price with her club card, how much more did she
save compared with the regular sale?

(A) $48
(8) $36
(C) $24
(D) $12

(E) $6

L ______________________ ------·------

We can set up a simple proportion to find the original price. hrst, calculate what percent of
tlw original price is $2!H. lOO%-- 15'Y.J = B5'/G. Thercfore. $204 is B5% of the original pricc.
Pa rt !E) 20,1
Cross multiply.
\-vhote 1oo x
2{) 1 • ] {)() ce B5 • X Di,·idc both sides by B5.

Simplify and didde. $240 is thc original price.

Tlw regular Sa\·ing is $240 mim1s $204. \Vhich equals S3Ci.

lf tlw original pricc is $240, tlw sa\·ings \\·ith a 20'\, discount could be found in orw of
t \\'0 \\·;1\·s.

20 __ 20 . 240
·¡ (J() -
X
'¿.f¡) x ·=.e - j()¡j- = Ji' lB or

:\islta san·d S 12 ( ~= S·lB- SJfi) mort· ,,·ith lwr club c;ml, ans\\·cr {IJ).

60 GMAT MATH WORKBOOK


Yao's investments gained 20% after one year and lost 15% the next year. lf he made
$12,000 profit overall for the two years, how much was his initial investment?

(A) $1,200,000
(B) $900,000
(C) $800,000
(D) $600,000
(E) $300,000

Since the initial investment is not given, assume it is lOO units. After one year, it increased
to 120 (100 · 1.2) units. In the second year, the loss of 15% is appliecl to 120 units. So Yao lost
0.15 · 120 = 18 units.

Overall, the investment went from lOO units to 102 units (120- 18). Therefore, the overall
increase is 2%.

Then set up a proportion.


Part _ 2 _ $12,000
Whole - lOO - X

X= $12,0020. lOO = $600,000

The answer is (D).

60% of 30% of a number is equal to 125% of 180. What is 200% of the number?

(A) 650
(B) 1,250
(C) 1,300
(D) 2,500
(E) 2,600

Translate the question into an equation.

Heplace GO% by O.G, 30% by 0.3, ami 125% by 1.25. Heplace "of" with multiplication.

O.G . 0.3 . X '= 1.25 . 180 Oi\·ide both silL·s by O.G · 0.3.

.
r = L~:) ·JBO
O.G · O.:l
r>lultiply by -l§{i to con\·ert all decimals into intcgers.
x = -\-.-x
PS · JílO =-P5
~--
· 1-
O ~e
1
-
1.2~)0 Simplify.

")()()''"
~ - u
'F)[) = -21()()
tll'] .~. ()() · j .~.>
'V[) 2 · 1,250 ce 2,SOO

Tlw answcr i:; (D).


SECTION 2.3-PRACTICE PROBLEMS

l. \Vhat is thc original pricc of a laptop computcr that is on sale for $4:{5 aftcr a 13'!ú
discount"?
(A) $448
(BJ $4Gl
(C) $500
(D) $513
(E) $530

2. t\lanju t1nishes JO<j¡> of her assigtunent on thc 11rst day. \Vhat percent of the rernaining
portion does she ha ve to finish on the second da y so that only 45'){, of the en tire
assignment remains to be done?
(A) 40%
(B) 45%
(C) 50%
(D) 55%
(E) 60'7h

3. A grocer buys applcs for 63a: per pound. If 1O'i'o of the apples goes bad and he still wants
to make a 20% profit ovcr his purchase price, what should be the sales price?
(A) 66<t:
(B) 70<t:
(C) 75<t:
(D) 77<t:
(E) 84<l:

4. Andre drives ata constant speed of 6•1 miles per hour for 4 hours ami covers 80% of the
total distance he is travel_ing. What is the remaining distance?
(A) 64 miles
(B) 128 miles
(C) 256 miles
(D) 320 miles
(E) 512 miles

5. Kumi is taking a test that has GO multiplc-choicc c¡ucstions. She answers GO% of thc
first 20 qucstions corrcctly. If shc \\·ants to scorc 70');, on this test, \\·hat pcrccnt of thc
rcrnaining qucstions docs shc nccd to <111S\H'r corrcctly?
(i\) 80'){,
-..r:O-'
(B) (,),o

(Cl 70%
(D) 65'){,
(E) 60%

62 Gi'iAT MATH \'JORKBOC>;<


. 5
6. vVI1at IS 40% of-;¡-?

(A) 25
(13) .§.
8
(C) ~
4
(D) l
4
(E)~

7. A beauty products company sells hair extensions. They increased the price of
extensions by 20% from year 1 to year 2, and their sales numbers decreased by 20%.
By what percent did the yearly earnings change? (earnings = price · quantity)
(A) Increased by 4%
(13) Increased by 2%
(C) No change
(0) Decreased by 2%
(E) Decreased by 4%

8. A store receives a shipment of the la test Shoot 'Em Up video game. 70% of the games
is sold at 30% profit, and the remaining 30% is sold ata Ioss of 20%. vVl1at is the store's
percentage gain or loss from the sale of this video game?
(A) 10%
(13) 15%
(C) 20%
(D) 25%
(E) 30%

9. 16% of a numbcr is 1.12. What is 125% of that numbcr?


(A) 8.75
(13) 7.50
(C) 7.00
(0) 6.75
(E) 5.25

10. The lenglh of a rectangle is increased by 40'~0. By what percent does the width need to
be changed so that the arca deet·cases by 2%?
(A) Increascd by 2'1(,
(B) Decreascd by 42%
(C) Decrcased by 40%
(D) Decrcasecl by 38%
(E) Decreascd by 30'/(,

l
11. Lea starts working on lwr homework ami finishcs BO'Yt, of thc work in one hour. Sin ce
she is gctting ti red, slw finishes BO'){, of the remaining homework in tlw next llour. What
percent of tlle originallwmework remains un!lnished after two hours?
(;\) 1()o;,
(B) H%
(C) 4%
(D) 2%
(E) 0'7;,

¡·J A certain hand sanitizer claims to kili~)()'!{, of bacteria with ea eh use. If Su e applies thc
sanitizcr twicc in a row, what percent of tlle original bacteria vvill rema in on ller hands·?
(A) 10/
/0

(B) 8.1%
(C) 9%
(D) IO'Yu
(E) 11%

SECTION 2.3-SOLUTIONS

l. e If it is on sale for 13%, its valuc is 87% of the original, 100% - 13% = 87%.
$435 is 87% of thc original price. Setting up a proportion is an easy way to find the
original price.
Part _ 87 _ 435
\VI){¡Ic - Tüt)- -:x- Cross multiply.

87 ·X= 435 · 100 Sol ve for x by dividing by 87 and simplifying.

.t\lternatively, 5et up a!1 algehra1r equation.

O.B7 · JI o~ 435 JI

:!. e Lct the entire assignment be lOO units. Shc finishcs 10% on the first day. so 90 units
rema in. She wants to lca\·c only 45'Jú (45 units) undone aftcr thc sccond day. So shc has
to !lnish 90 45 "" :15 units on thc sccond day. S he has to finish ~~ = ! = 50% of thc
remaining assignmcnt.

3. D Sin ce 1O'?í, of thc appll's wcnt bad. he cndcd up paying G3c: for 0.9 lb. !le purchased
G·\c
thc sellable app!es for {).~t)¡T) = 70lf per pound.

Ifhe \\·ants lo make a 20"(, profit (70 · 0.2 ~, J.Jc). the pricc slwuld be GJ( +¡.¡e= //C.

Altcrnativcly, sincc he \\·ants to makc 20'!(, profil, he nccds to makc G3C · 20'!(, = 12.GC
profil pcr pouml purcllasecl. Note that he o ni;; has O.~J pounds lefl tose!! aftcr !()"(, \\·ent
had. flis prof:t per pound nccds to be _!G:~;l~ = HC. The sales pricc is ()Jc + HC '" 7/C
pcr pound.

64 GI\'!A.T f;L'TH WORKSOOK


4. A distance = rate · time

He traveled 64 · 4 = 256 miles so far. The total distance can be found as follows.

If the total distan ce is 320 miles, 320 - 256 = 64 miles remain.


Alternatively, salve for the total time required since thé speed is constant.
80 - 4
100- x
4 · ~00 = 5 hours
x =
8
5 hours is the total trip. 5 - 4 = 1 hour remains. Since distance = rate · time, the
distance rcmaining is 1 · 64 = 64 miles.

5. B 60% of20 = 0.6 ·20 = 12

She airead y answered 12 correct. She wants 70% of 60 questions correct. Kumi's goal
is 0.7 · 60 = 42 correct.
She needs 42 - 12 = 30 more corrcct answers out of 40.
~~ · 100% = l5%

Alternatively, you can set up a weightcd average equation. (See Chapter 4 "Word
Problems.")
60% · 20 + X· 40 = 70% · 60
0.6 · 20 + 40x = 0.7 · 60
12 + 40x = 42
40x = 30

x= 75%

6. E To translate, replace 40% by eithcr 0.4 or ~~ and replace "of' by using multiplication.
1
40% of "º- = _':!_Q_ ·-º. = _4_.-º. = l
4 lOO 4 lO 4 2

7. E Sin ce no numbcrs are given, lct's assumc thc original price was $1 O and the cornpany
sold 1O u ni ts in year l.
Total carnings cqual $10 ·lO= $100 in ycar l.
Thc ncw price cquals lO+ (0.2 · 10) = $12.
The ncw sales ccgwl 10 -- (0.2 · 10) = 8 units.
Total earnings cqual $12 · 8 = $9G in ycar 2.
% change = lli
1
--¡0
¡}tJQ · lOtJ'!í, ~~ -4'!ó ~" 4'Yc, clecreasc

8. B Assumc 10 gamcs are dcli,-cred ata cost ofSIO cach ($100 total purchasc price).
So 7 of them (70%) are sold at S 13 (30% profit), and 3 (30%) of tllem are sol el at $8
(20'i(, loss.)

Total eamings cqual7 · 13 + 3 · 8 ~, 91 + 2·1 = $115. Since the total purchasc price is
$100, th: total perccntagc gain i; 15%.
Altcnntivcly, u:;c 0.7 · 0.3- O.:i · 0.2 "-· 0.21 -- O.OG = 0.15 = 15% gain.
~l. A St•t up a propon ion for the first part.
1() 1.12
Hiü x Cross multiply.
112
]()

Tlw nuinlwr is /.
l2S"i> of 7 = 1.25 · 7 ·ce B. 75

10. E :\ssunw tlw initial sides are/. 10 ami\\'' 10. So tlw initial arca 100.

The length is increased by -10";,, tlwrl'fore tlw lll'\\ /. = 10 · 1.4 1-1.


The new arca is 2"(, kss, tlwrdore the lll'\\' .-\ ·=· tl.~lB · 100 9B.

New :\rea = Ncw /_ · New 1\'


~lB 14 · N ew 11'
New \ \' = ~)ª =· 7
14
Sin ce the original width was 1O, the ¡wrcent decrease is D.Qr~; D. = 30%.

11. C Let the amount ofhomework be 100 units. She finishes 80 units in thc first hour, so
20 units rcmain after the first hour.
She finishes 80% of 20 units in the second hour. So 20 · 0.8 = 1G units are finished in
the second hour.
The remaining work equals 100- 80 1G = 4 units
..'L_ = 4% .
So thc remaininob work is 100

12. A Lct thc number of bacteria be 100 at the beginning. Sin ce the hand sanitizer kills
90%, 1O bacteria will be lcft after thc first application.
The second application will kill10 · 90'X, = 9 more, and only 1 bacterium will be lcft.
Thc answcr is 1%.

SECTION 2.4: DESCFUPTIVE ST ATISTICS


, Average (Arithmetic Mean)
a The average (or arithmctic mean) is thc su mofa sct of val u es clividcd by thc numbcr
~~-EMBE~ of\·alues in the set.
1 T.-~:·ivide by 5 ' :\\craoc = Slll1_l_~2LillJe_l'l}l_s
quickly, divide " Numlwr o! terms
the number by · 1 -JheaH'rageofB.
· · H,22,ancl40Is
,, · B -f 1'1 ! 1:1 + 2'J + ·lO % = J:L.:.
Lxampc: 12. ____ '::_:____ ;:-----'=-----=.o..r:
10 and multiply ,) ,)

by 2. a \\'(ml pro!Jil.'Im u:-,ually imoh'e either comhining or scparating t\'.·o groups. lnthese
cast·s. start by c:dculating the su m of thc qu:u1til)' for each group siillT group mTrages
cannot be addecl or suhtracted.
Sume~ 1\\'Cragl'. Numlwr ofterms

Exa111ple: Tlw an·r~¡ge grad\' ofall girb in :1 Spanish class is Bti. ami tlw a\-crage grade of
;¡ll bo~·s is 7!1. lftlwre are 12 girls aiHl B hoys in class. \\·hat is tlw class a\Tragc'?
The S\llll of al! girls' grades~- !l(i. 12 = un2.
The su m of all boys' gradl'S ~· 7B · B ~~ G2·L

66 G~·iAT MATH WORKBOQ;(


The su m of all grades in the dass = 1,032 + 624 = 1 ,656 .
.. . . l l ,G.J_56_ =
The su m of al! grades in the class _____
111e average ora(
0
e = -- - 82 8
Total number of students 20 · ·
• lf a number n is added to all the valucs in a set of numbers, the average of REMEMBER
the numbers increases by n. lf you increase (or decrease)
Example: The average of 4, 7, 12, and 15 is 4 + 7 + 12 + 15 = 3 8 = 9.5. each number in a set by n,
4 4 you increase (or decrease)
If 3 is addcd to ea eh number in the set, the average beco mes
the average by n.
4 4
°
7 + 10_:1- l5_±.11l. = 5 = 12.5, which is 3 more than 9.5. ''-----,---------------'
Median
• In a list of val u es arrangcd in incrcasing (or decreasing) order, the median is the
middle value.

Example: The median of the set {3, 3, 4, 5, 5, 5, 7, 8, 8, 9, 12} is the 6th number, which is 5.

• If the list has an even number of val u es, the median is the average of the two middle val u es.

Example: The median of the set {5, 5, 5, º'-.fi, 8, 9, 12} is the average of the t\'lo middle
(f + 8)
numbers, ~-- = 7.
Note that the median mayor may not be one of the elements of the original set of
numbers.

M o de
111 In a list of valucs, the modc is thc value that appears most frequcntly.

Example: The mocle of thc sct {3, 3, 4, 5, 5, 5, 6, 8, 8, 9, 12} is 5.

Range
111 Range is the simples! measurement of spread \dispersion) of elata. It is the clifference
betvveen thc greatest val u e and the leasl val u e.

Example: The rangc of the set {-7, -5, 0.25, 4, 12, 14, 21} is (21 - ( -7)) = 28.
------\
lil If a number 11 is added to al! the val u es in a sel of numbcrs, the rangc do es
REt 1Et•HlER
9

not cbangc.
lf you increase (or decrease)
Example: The rangc of --4, 7, 12, ancl 21 is 21 - ( -4) = 25. each number in a set by n,
lf3 is acldcd to each nu¡¡:lJer in t!Je sct, the set becomes the range is unaffected.

-1, 1O. 15, 2·1 and thc rangc is 2·t ( -1) = 25.

a Standard de\·iation is a commonly uscd mcasurc ofthc spread (dispcrsion) of data.


Thc higher tlw standard cleviation, the more sprcad out the data.
¡¡¡ Standard dtTiation shm\·s huw much \'ariation there is from thc average. The furthcr
tlw data are spread zm-ay frum the nwan, thc highcr thc standard de\·iation.
m Standard dc\iation is also used to indicatc thc dillcrcncc bct\\·cen a munber ami thc mca11.

Examp!c: In a sct ofrcalnumbcrs. thc mean is 7 il!ld thc standzm! de\·iation is Ll.
Thcrcforc, :J.G is '1 stambrd de\ ialions more than tllc mean (7 + 2 · Ll == 9Ji).
In addition, :1.1 is 3 stamLtrd LL~\iations lcss tha11 thc mean (7- 3 · 1.3 c.c 3.1).

GMAT r•JATH WOfii<BCOl< 67


• lJndcrstanding ami intt·rpreting tlw standard dt'\'iation is llll>rc importan! than
calculating it by hand sirwe it is unlikely that you willlw asked to cakulate it.
• Two data spts \\'Ílh the sanw nH•arr m;ry kl\'l' n·rs diiTerent standard dt·\·iations.
Example: :\ 11, :l, :J, 7, 91 arrd U' ::l. 1, :J, ti, 71

Both sets ha ve a mean of :J. llm\·t·n·r. SL'l :\has a higlwr standard deviation since its
elemcnts are more dis¡wrsed around (far ;1\\·ay from) S.

• To calcula te the standard de\·iation of a sct of numlwrs, first find thL• a\·erage of tlw
sct. Second, find thc di!Tercnce of each number from the mean ami take thc square
of each differcnce. Tiren add tlll' squarL'S of eadr dillercnce. Divide that su m by the
number of data points. Finally. take tire squarc root.
Examplc: Both A 11, :l, :J, 7, ~JI ami H = ¡:l, 4, :i, !i, 71 ha ve a meanof 5.
1:(1 5)'=JG
3: (3- 5)' = 4
5: (5-5)'= o
7: (7- 5)" = 4
9: (9 - 5)" = 1G

3: (3- !))2 =4
4: (4 - 5)" =1
5: (5-5)"= o
G: (G- 5)" =1
7: (7- 5)2 = 4 Standard deviation ~e lFf±_J___~-&/ f+ j}_ = 1.41
As you can see, set B has a much lower standard deviation than setA.

SAMPLE PHOBLEMS

Which of the following set of numbers has a higher mean than median?
(A) {3, 4, 5, 6, 7}
(8) {1, 3, 5, 7, 9}
(C) {1, 1, 5, 6. 7}
(D) {3, 4, 5, 7, 8}
(E) {2, 3, 5, 6, 7}

(A) !3. 4, 5, 6, 71 mean'" median 5 sincc the numlwrs are consecutive, thc middle numller
is also the mean.
lBl ! 1, 3. S, 7, ~JI mean = median = S sin ce tlll' rHrrJrhcrs arl' consecuti\'C odd, the middie
numlwr is also tlw nwaiJ.
(C:) 11, 1, 5, G, ll median= 5 buttlrc mean is less than 5 sirru· thc su m of this sct is less thanthat
oftlre sets abo\'t' or thirrk ofir lwing "liglrter" on thc ldt of5. Tire mean ¡¡·ill slide
t(l\\'ard tlll' lcft (srnalkr) simT 1 is furtlwr <m·ay frorn 5 comparcd to () or 7.
(D) !:l. tl, :i, 7, H} median c., 5 <111d tlll' mean is slightlv lrighcr since this time it slides lOi\·arcl thc
right. Tlw numlwrs on thc riglrt ;¡re funhcr a\\ ay from5.
(E) 12. 3, 5, G, /} samc as C.

Tlw airS\\·cr is (D).

68 GMAT NATH WORKBOOK


5 is added to each of the numbers in a set of 12 integers. Which of the following must
be true?
(A) The median stays the same.
(8) The range increases by 5.
(C) The mean stays the same.
(D) The standard deviation does not change.
(E) The mode does not change.

(A) False. The median increases by 5 since each number increases by 5.


(B) False. The range stays the same. Both the Iargest and the smallest numbers increase by 5.
(C) False. The val u e of the mean in creases by 5 sin ce ea eh number in creases by 5.

(D) True. The spread of the numbers stays the same.

(E) False. The mode increases by 5 since each number increases by 5.

The answer is (D).

The annual average salary of 50 employees is $68,000. lf 40 of these employees


have an average salary of $65,000, what is the average salary of the remaining
10 employees?

(A) $60,000
(8) $68,000
(C) $74,000
(0) $78,000
(E) $80,000

Average Number Total


-
68,000 50 employees 3,400,000
65,000 40 employees 2,600,000
X lO employees 800,000

3,400,000 - 2,600,000 = 800,000


Thc remaining 10 crnployccs ha ve a total salmy of $800,000.
-¡-¡ lCir
. average sa1ary 1s $800,000
. -¡¡y--- = $"0
u ,
(lOO

Thc answer is (E).

Alternatively. set up a weightccl average cquation.


40 · GS,ODO + 1O · x
-- ---50 ----- = 68,000

2,600,000 1 lOx = 3,400,000


lOx o~ 800,000
X=- 80,000
SECTION 2.4-PRACTICE PROBLEMS

l. Tllt• a\·t·ragt• of l!i numlwrs is 1;. lf B mort• numlwrs witll a Slllll of l:l:! are addcd, \\·llat
ís tllc an·r;tgt• of tlll' :!·1 nu1nlwrs'~'

(!\) 1:>
B
(B) :!:1
H
(( :¡ ~l:l
B
(D) B
(F} :lO

·¡ 13¡wople llave an average age of27. What wílltheir a\·erage agc be in J years'?
(A) :HJ
(B) 33
(C) 39
(D) 40
(E) 43

3. Hube!lo spenl a total of 12 days studying for the GI\IAT math section. On each uf the
first 4 days, he solved 80 questions. During the next 5 days, he solved 50 questions every
day. For the last 3 d<iys, he sol ved a total of 54 questions. I-Iow many questions die! he
sol ve per da y on average'?
(A) 48.3
(B) 52
(C) G 1.3
(D) G2
(E) GS

4. The average (arithmetic mean) of a certain normal distribution is 22, and the standard
deviation is 4. What is the diiTcrcnce bctwccn the valuc that is 2.5 standard dcviations
lcss than thc mean and thc \·al m~ tllat is 2.5 standard deviations more than tlw mean'?
(Al 4
(B) 8
(C) 10
(D) 20
(El 30

(;\) 42
(B) :.lG
(C) 32
(!)) ¿.¡

(E) 17.5

70 G:U\T M;.\TH WORKBOOK


6. There are 21 students anda driver on a bus. Six ofthe stuclents are 13 years old, seven
of them are 14 years old, and the restare 15 years old. Ifthe driver is 34 years ole!, the
mean age of everyone on the bus is how much more than the median age?
(A) O year
(B) 1 year
(C) 7 years
(D) 14 years
(E) 15 years

7. The average of six numbers is 15.5. lf another number is added to the list, the average
becomes lO. What is the nevv number?
(A) 5.5
(B) o
(C) -5.5
(D) -22.5
(E) -23

8. On Tuesday, Manu's Bookstore made a profit of 20% on 60% of the items so le! and lost
10% on 30% ofthe items sold. If al! books have the same sticker price and the bookstore
had no profit or loss on the remaining items, what was the average profit on Tuesday?
(A) 5.0% profit
(B) 7.5% profit
(C) 9.0% profit
(D) 10.0% prot1t
(E) 12.5% profit

9. A printing shop has lO printers working ata rate of 2,500 pages per printcr per da y on
average. The management is considcring two different new high-speed printers. Printer
A has a capacity of 5,500 pages per da y. Printer B has a capacity of 8,500 pages per da y.
How m u eh more would their daily printing average be if thcy invest in two printer Bs
versus two printer As?
(A) 100 pages per printer per da y
(B) 200 pages pcr printer pcr day
(C) 300 pages per priillcr pcr day
(lJ) 400 pagcs per printer pcr day
(E) 500 pagcs pcr printcr pcr da y

10. G, -1, l, 2, -3, 3, -3, 4, -2, O, 5

Which of thc follrming must be true for thc list or numlwrs above?
(¡\) modc <mean< mcclinn
(B) mean< modc ·<median
(C) median< mean< modc
(D) modc ·< nh·dian < mean
(F) median modc mean

G;'>1AT f·L\TH WORKBOO?\ 71


11. Thc averagP of x and y is k. Tlw averagt> of 111. 11, and ¡1 is l. What is tlw averagt> of x, y.
111, 11, and p in terms or k and t'?
(k~ t)
(:\)
2
(k+ t)
(B)
5
(3k + 21)
(C) --·-·--s·----"

(D) (2k + 3/l


5

(El
5(k + t)
_(:) ___
12. J>, 13, 2, 4, -2, B, 1, o
J> and B are prime numbcrs, and
P + B = 12. What is thc median of the list of numbers abo ve;'
(A) l
(13) 2
(C) 3
(D) 4
(E) 5

SECTION 2.4-SOLUTIONS

Ul 2
A 1
13
3

EH+f:±Hf8
1. D The su m of thc first 1G numbcrs is
10
D
11
D L_
]

Numbcr oftcrms ·Average= 16 · ~~') = GO


Afler adcling thc ncw numhers, the new total is GO + 132 = 192.
lt)'J
Since we nmv hm·e 24 numhers. thc tww average is -;J,L. "'' B.
L."t

2. A Since each person will be 3 years olclcr, the average will incrcase by 3 and beco me :~o.
Alternativcly, do thc follmving cakulations.
Su m = 13 · 27 = 351
lncre;¡sc = 13 ·3 = 39
Thc llC\\' surn = 35l + 39 ce 390

Thcrc are still 13 peoplc.


AVCI",\0(' = ~-3_,~Q :~()
b 13 .

3. B First find thc totalmunlwr of quPstions soh-ed.


(4 · B\l) + (5 · 50) + 54 "' 3:!() ~¡ 25() + 5! .~e ()¿.¡

.' . totalnumbcr or qw·stions . ()é'·l -· f:')


[he <l\'Crage IS tlw -- -llllllllÍc¡: cJTcE't~-:-;;-- - lT -· .J~.

72 GMAT ¡··iATH WORKBOOK


4. D The value that is 2.5 standard deviations Iess than the mean is 22 - (2.5 · 4) = 12.
The value that is 2.5 standard deviations more than the mean is 22 + (2.5 · 4) = 32.
32- 12 = 20

5. A The average of a, b, and 12 means adding the tluee terms ami then dividing by 3.
a+ b + 12 = 4?
3 ~

a+ b + 12 = 126
a+ b = 114
The average of a, b, 4, and 50 means adding the four terms and then dividing by 4.
a+ b + 4 +50 = 114 + 54 = 168 =
42
4 4 4

6. B The total age can be found as:


(6. 13) + (7 . 14) + (8 . 15) + 34 = 330
l
The average is 3 Q = 15.
2
Since 22 people are on the bus, the median age wi!l be the average of the ages of the
llth and 12th persons once we list the ages from least to greatest. Both of those peop!e
are 14 years old. So the median is 14.
Mean - Median = 15 - 14 = 1

7. E The original sum =Average· Number oftenns = 6 · 15.5 = 93.


The sum aftcr the new number is added =Average · Number of terms = 7 · 10 = 70.
The number must be 70 - 93 = -23.

8. C Let the total number of items be 10 and each book have a sticker price of $10.
Thc total value is $10 · lO= $100.
1
6 items wcre sold at 20'1{, profit: Profit = 6 · $10 · 0.2 = $12
3 items were so le! at lO'!'iJ loss:Loss = 3 · $10 · 0.1 = $3
1 Total profit on 10 books = 12- 3 = $9
!
Percent proflt = -$9
- - = 9%
$100
Alternativcly, use thc fol!owing equation.
0.6. 0.2- 0.3. 0.1 = 0.09 = 9%

9. E Option 1: Buy 2 printcr !ls:

Averane ="
Total pagcs (lO · 2,500) -t (2 · 5,500)
---·-=--:··-·--- = ------------, ---· --------- =
25,000 + 11.000... = 3,000
0
1! o! prl!lt('rs 12

Option 2: Fluy 2 printcr Es:


(lo . 2,500) + (2 . 8,500) 2S.OOO + 17,000 _
12 ------r:r------- == 3,:loo

Thc diilcrcnce is 3,500- 3,000 = 500 pagcs pcr printer pcr day.

l
Alternatively, look at tlw inrreaSL' in capacity only.
~ · S,SOO 1 1,000
Opt ion 1:
1~ 1~

~ · ll.SOO 17' 000


Option ~:
1~ 1~

17. ()()() 11'()()() (),()()()


Tlll' dillewnn• ··
1~ 1~ 1~
so o

1O. D First list tlle numlll'rs from kast to greatest.

1 · :1. -:l. -~.···l. O, 1, ~. 3, ·1. S. ti

Tlw nwdian is tlle middlc numlwr. \\'llicll is l.


To find the mean, add alltlle numlwrs. Start with adding ... l ami l. -2 aml2, ami so
fortll. The sum is 12.
·¡·¡ 1e mean =
12
TT = l Ti.
l

The modc is ·-3 since it repeats twicc.


mode < median < mean

ll. D The average of x ami y means add x and y and then divide by 2.
x+y
2
k X+ J' = 2k
The average of 111, n, and p means add 111, n, and p and thcn divide by 3.
111 + 11 + p '
--3--.. ={ 111 + JI 't' jJ = 3f
Average ofall = (x + l.L.:::J~~..'.:_+ p) = 2k + 3t
5 5

12. C If P + n 12 and if P and B are prime numbcrs, thc only possible pair is S ami 7.
List al! numbers from least to greatest. -2, -,1, O, 2, 4, 5, 7, 8

Sincc there are an cven numhcr of terms, two numbers are in the middlc. To find thc
median, average the two middlc terms.

1
·¡ lC ¡· . (2
lllCLiall IS --...
+--
4) ')
:>.
2

SECTION 2.5: COUNTING i't1ETHODS ANO PROBABIUTY


Fundarnental Prindple o·f Counting
111 lf onc evc·nt can hap¡wn in 111 ditlercnt ways ami anotlwr evcnt can llappe11 in 11

di!Tcrent ways, the l\\'O l'\l'Ilts togctlwr (111 illJcl n) can happcn in 111 ·JI di!Tercnt ways.

Examplc: lf yotil'<IIl cltuuÓ>L' 1 pan h. J S\\'L'ater, and J pair of sliocs from .¡ pants,
5 swcatcrs. and :~ p;1irs of shoes. hm\· many di!Terent outfils can you crea le?
.¡ . s . :¡ c.., ()0

11 lf Olll' C'\'l'lll can happ('ll in 111 di!T('rcnt \\·ays allll anotlwr c\·cnl canlwp¡wn in 11

di!Terent \rays. eitlwr 111m 11 can hap¡wn in (111 i n) diiTnenl ways.

Example: lfyou Clll clwose eitlH·r 1 S<llllh\·i,·ll or 1 soup from a list of-1 s;md\\·iclws
and :í soups. hm\· many di!Tnent options do you han'?
.¡ f 5 ~)

74 GriAT í"íATH v'IO:zi<BOOK


Factorial
• lf n is an integcr grcater than 1, n factorial (n!) is defined as the product of all the
integers from 1 to n.
3! = 3. 2. l
6! = 6 . 5 . 4 . 3 . 2 . 1
n! = (n) · (n - 1) · (n - 2) · ... · 2 · 1

• By definition, 1! = 1 andO! = l.

Ordering
• The number ofways a set of objects can be ordered is found by n!

Example: How many different ways can 5 di!Terent books be ordered on a shelf?
5! = 5 . 4 . 3 . 2 . l = 120

Example: In how many ways can 4 people sit on a bench?


4! = 4 . 3 . 2 . 1 = 24

Permutation
• A permutation is an arrangement of a group of objects in a particular order.
a lf the arder in which the members are chosen crea tes distinct arrangements, such
counting problems involve permutations.

Example: At school, 4 candidates are running for 3 different positions, president,


vice prcsident, and secretary. How many differcnt outcomes are possiblc?
Ask the follovving questions:
l. In how many differcnt ways can I pick the prcsident among 4 studcnts? 4
2. Once I pick the presiclent, in how many clifferent ways can 1 pick thc
vice presic!ent? 3
3. Once 1 pick the presidcnt ancl vice presiden t. in how many clifferent ways
can 1 pick thc secretary? 2
Multiply these numbers together beca use a presidcnt, anda vice presiden t. Llli.d a
secretary must be chosen:
4. 3. 2 = 24

a The following formula can be used to fine! thc number of possible permutations of
k objccts choscn from a set of n different objects when n "?:.k.
P(n, k) = ___il!_
(ll k)!
In thc previn1rs cxamplc, thc number of possiblc outcomcs are

P(1l, 3) =
0C~L~f)! = i · 3 /-~-l = 24.
Examplc: If G athlctes are running in a r:1cc, in how many ways can thc golcl, sil ver,
ami l>ronze mcdals be distriiJutcd?
In lhis case, ordcr clcarly matlers. Use pcrmutation.

P(CJ 3) '~. _<)! ,., 6-~-~-·1.: 3. ·.?-~ 1 = G · 5 · 4 = 120


,, (()·3)! 3·2·1
Combination
• A combination is grouping of objects in which tlw onkr or arrangenwnt of tlw objeus
is irrek\·ant. For example, ifyou are pil'king four toppings for your pi1.za, tiH' ortiL•r in
whirh you pirk them is not important.
• lf tlw order in which the memlwrs are clwsenmakes no dillert'IKL', such rounting
probk•ms inn>lve combinations.

Example: llow many diffnent groups of 3 representatin~s can be eh osen from a


group of ::i students·~

Letthe students be A, B. C. D. aiHI E. A SL't of representati\L'S can be


selected as follows.
ABC ACD BCD CDE
ABD ACE BCE
f\BE ADE BDE
There are lO combinations.

• Thc following formula can be used to find the number of possiblc combinations of k
objects chosen from a set of 11 clifferent objects when n 2: k.

C(n, k) = (n -'~)! . /. .~
In the previous example, the number of groups are
' r: ' - 5! - 5.4.3.2. l -
((_), 3)- (5- ~3!- (2~0.(3~- 10

Example: In a certain test, stuclents are allowed to pick any 4 questions out of
6 questions provided. In how many different ways can Kim selcct her
questions?
In this case, the ordcr shc picks hcr questions does not mattcr. Use
combination. ·

C(6 4) _
·
6!
- w4J! . 4! --
__
-:z-=-n ~-= ·3 . 5
G· 5 · 4 · 3 · 2 · 1
¡z--:-rFcT-:3 1."')

Discrete Probability
• Probability is the chance or likelihood that a ccrtain event will happen.
a Thc probability of an event happcning is thc ratio of thc numbcr of ways thc C\'l'Ilt
can happen to the total number of possiblc outcornes.

, ___ Numbcr of outcomcs giving thc clesired rcsult


I (f·) - ---------- ···----- - -- .. ··-···----· -- - --- - --- -
, Numbcr of all possible outcomes

a The probability of an cn'nt is a numhcr bet\\TC'Il O amll inclusive, O :e:: I'(L) s: l.

Examplc: What is the probability of rolling a-l in onc thrmv of a die'?

A regular die has G faces, sn tlwre are G possible outcon1cs.


\Ve only \\'ant thc numher -1, so the numbcr of.cksirccl rcsults is l.

!'(!-')
. =· lG
Examp!c: The probability of picki11g a bluc nwrblc from a bag that contains 7 bluc
mar!Jlcs allCI 1B red marblcs is l'(lj) 'l:i·
76 GN!\T MATH WOíiK800K
• The probability of an event not happening is P(not E) = 1 - P(E).

Example: The probability of not picking a blue marble from a


REMEMBER
bag containing 7 blue marbles amll8 red marbles is
lf you have only three colors of marbles
J>(not B) = l - _]_ = .lli in a bag, the probabilities of picking
25 25'
each of them separately will add up to 1.
• P(E) = 1 represents a certain event.
For example, if there are 4 blue, S green,
Example: When a fair die marked from 1 to 6 is rolled, the and 7 white marbles in a bag,
probability of a number less than 7 showing up is l. P(B) + P(G) + P(W) = 1
lt is a certain event. 4 S 7 16
16 + 16 + 16 = 16 = 1
• P(E) = O represents an impossible cvent.
Example: When a fair die marked from l to 6 is rolled, the
probability of an 8 showing up is O. lt is an impossible evcnt.

Example: In a certain car lot are only cars, trucks ancl SUVs. If a vchicle is sclectecl
randomly, the probability that it will be a car is 9 ancl the probability that
it will be a truck is i~· lf a total of 98 vehicles are in thc parking lot, how
many of them are SUVs?

The probabilitics of selecting a car, a truck andan SUV acle! up to 1: P(C)+ P(T)+ P(S) = l.

t + f4 + P(S) = 1 Acld the fractions.

4 5
T4 + T4 + P(S) = 1

9 - + P(S) = l
Subtract ~ from cach sic! e of the equation.
14 1
P(~) = 1 - -~- = 2
14 14
If tlw prohability of picking an SUV is - 5 , you can sct up a proportion.
14
5 _ X
H- 98 Cross multiply.

X = 9íl . -i~T = 35
There are 35 SUVs in the parking lot.

ProbabHities of Two or í\'lor<e Events


INDEPENDEiYr EVENTS

1!1 lflhL'lJCClliTCIICC' ofen'lll [;'docs not affeclthe probability that C\'CI1t Foccurs, L:'and r
are indcpendent C\'Cnls. Thc pn Jbability of both E ami Fhappening is the product of thcir
indi\iclual probabilities.

P(f and F) = P(r) · P(J-')

Examp!c: A scarf is picked randomly from a dra11·cr fui! ofscarn~s ami rcplaccd.
There are 30 scan·es inthc drawer. 15 an~ square shaped. 7 <n~ rvctangu-
lar <PHI B are triangular. \Vlwt is the probabi!ity of picking l\\'0 triangular
SC<ll'\'es in succession?

Sincc thc sean-es are rcphced after picking. tlw probabilitics of each pick are
indv¡wndcm frum ea eh otlwr. In othcr \\'ords, l he first pie k do es nut <lf'fL'Cl thc
Sl'Ulilll pick.
Thc probability of picking a triangle in tlw first pick is P(T1) ' :w·
B

Tlw probability of picking a triangll• in tlw pick is also /'( T,l H


S('Ullld
:w·
The probability of picking 2 triangular scarn:s in succcssion is
Jl{'/'). /'('/')-e B . B . ().¡ J()
1 ' •. :10 30 - ~)()()

• Thc probability of eitlwr l:'<n Fhap¡)('ning is gin·n by the formula:


l'{l:'or F) = {'(/:') 1 P(F) 1'(/:'and ¡:¡

Examplc: A card is drawn randomly from a regular dcck of cards (52 cards in total,
1:l in cach of four suits). \Vhat is thc probability that tlw card is cithcr a
1O or spadcs'?
Noticc that these two scts are overlapping. A can! can be a 1O ami a spadc al the sanw
time. There are a total of four 1Os ami only o¡w 1O of spades.

P(lO) # of !Os _ 4
-52--52 P{Spade) = ~-1 P(lO and Spade) =,A,-
,)L.

Add the probabilities of selecting spadcs and !Os. To avoid double counting. subtract
the probability of selccting the lO of spadcs.
P(lO or S¡Jade) = P(lO) + P(S¡Jade) P(IO ami Spade) = r.:~> + J-~
,)L. :J¿
1 16 4
52 -- 52 -- 13

MUTUALLY EXCLUSIVE EVENTS

• If two events are mutually exclusive, they cannot both happcn. lf yo u pie k a dance
class at rancio m, it cannot be both a tango and a salsa class at the same time. This
mcans P(Tand S)= O.
• Thc probability of either one or the othcr of two mutually exclusive events happcning
is given by thc following formula:
P(Tor S)= P(T) + P(S) beca use P(Tand S)= O

Examplc: A total of 5 tango classes, 4 salsa classes, ami 7 swing classes are olTerecl at
a dance studio. If a class is selccted randomly, what is thc probability that
it will be either a swing class ora tango class?
Tango ami swing classcs are mutually exclusive. Thcreforc. picking citlwr tango or
swing is:

P('for S) = P(T) + fl(:l) -~- + J = 12. =}


1G ]() lf) ,¡

SAf\IPLE PHOBLEi\IS

The faces of a cube are marked with the letters K, K, L, W, M, and M. lf the cube is
rolled, what is the probability that a K will turn up?

'--·-··--·----------· -·---------------------· .... -·---- --------


.. ..

1\ cuiJe has G faces, so tlH· numhcr of al! possihlc out comes is G.

\\'e 1\'ant 1\to silO\\' up and tlwrc are twn 1\.s. so tlw numlwr of d('SÍrcd ITSttlts is 2.

/'( /\.) 0..7 -~ = :\

78 Gl'iJ.\T l";.\TH WORKBOOK


In the spinner shown qelow, the white region is 90°, the other two patterned regions
are each 60°, and the tinted region is 150°. What is the probability of landing on the
white or the dotted region?

A circle has 360° in total.

P(white) = Jl_Q_
360
P(dotteti) = 60
3GO
9
P(white or dotted) = 3 cib + 36~) = ~~~ = 152

A sandwich shop has 4 types of bread, S types of meat, and 6 types of cheese. Each
sandwich can be prepared either hot or cold. lf a sandwich is selected at random,
what is the probability that it will be a hot sandwich with Swiss cheese?
1
(A) 240
1
(B) 120
1
(C) 40
1
(D) 20
1
(E) 12

The total number of possible sandwiches can be founcl by multiplying the numbcr of eh o ices for
each selection: 4 · 5 · G · 2 = 240 different sandwiches. So therc are 240 possible outcomcs. Note
that the 2 in thc equation stancls for the hot versus cold option.

Whcn thc desirecl outcome is a hot sandwich with Swiss cheese, 5 typcs of rneat ami 4 typcs
of bread can be choscn. So there are 4 • 5 = 20 sandwiches that are hcated with Swiss cheese
(desired outcorne).
P(Svviss all(! Ilot) = _D~!il!:~c!_QtJKll_!J1Q~ _ = -~()__ = _L
Al! poss1blc outcomcs 240 12
Alternativcly, sin ce we are intcrcstcd in only our choice of chccsc ami hcat, we can ignore thc
bread and mcat choices. Wc can only pick the chccse (t) and thc hcat ( J). Thc probability of
gctting a hot sandwich with Swiss cheesc bccomcs:

P(Swiss amlllot) = t ·-1 = -Ó~


The anS\\ cr is (E).

Gi'-J.a:r i'·i;\i H WORKBOOK 79


lf two different numbers are selected from set E = { -2, -1, 2, 3}, what is the prob-
ability that the sum will be positive?

(A) 1
2
(8) 1
3
(C) ~
3
(D) 14
(E) ~
4

Therc are six diffcrcnt ways to pick a sct of two numbcrs. Sincc order does not matter, thc
quickest way is to list all your eh o ices: (- 2, -1), (- 2, 2). (- 2, 3), ( -1, 2), ( -1, 3). (2, 3). Writc
your answers systematically so as not to omit any possible pairs. Remember that (- 2, -1) is
equivalent to ( -1, -2), so do not double count.
Out of the 6 eh o ices, 4 ha ve positive sums: (- 2, 3), ( -1, 2), ( -1, 3), and (2, 3).

P(Positive sum) = ~ = ~
The answcr is (C).

A 6-sided die is rolled twice. The first roll is recorded as m, and the second roll is
recorded as k. What is the probability that m;:; k will b~ greater than 2?
S
(A) TI
(8) 1
2
(C) 13
(D) 2!_
36
(E) lZ_
36

------------------------------------------------------

First, simplify the givcn cxprcssion to gct more insight on tbc qucstion.
m-+k m k k
--ííT- = ·¡¡¡ + ·¡¡¡ = l + líl
\\'e want l + -~ > 2,

So subtract l from both sidl'S.

l+-k>2 k
111 -
-) Tíi > l
i\lultiph- both sicks by 111 sittcc 111 is a positivc number.

k> /11

80 Gi-íAT NATH WOr<KBOOK


Once we subtract 1 from each si de of the inequality, vve see that the ratio of k to m needs to
be greater than l. Since both k and m are positive integers, we can conclude that the question
is asking for cases whcre k> m (whcre the sccond rol! is grcater than thc first roll).

List all the potential cases. For example, ifthe first roll is l, there are 5 cases where the second
roll will yield a larger numbcr (2, 3, 4, 5, 6).

First Roll (m) Second Holl (k)


1 5 possible
2 4 possible
3 3 possible
4 2 possible
5 l possible
6 m
Desircd outcomes = 5 + 4 + 3 + 2 + 1 = 15

Total possible outcomcs (die rolled twicc) = 6·6 = 36


P(k > m) = 15 = _§_
36 12
Thc answcr is (A).

SECTION 2.S~PRACT!CE PROBlEtvlS

l. Sct A = {2, 4, 8, 16}. Set Bis formed by multiplying each number in setA by 2. A number
is selected ranc!omly from each set to form the fraction ~ wherc a is from setA and bis
from set B. What is thc probability that the fraction will equall?
l
(A) T6
3
(B) T6
(C) l
4
( 0)
. ~i
4
(E)

2. In a regular deck of cards (52 cards in total, 13 in each of 4 suits, 3 face cards in cach
suit), if onc card is choscn at rancio m, what is the probability that it is a face carel or
hearts?
27
(A) 52
(B) 25
52
(C) 1 l
26
(DJ T~
(E) ¡'~

(i?eiAT ¡-.JATH WORKi300K 81


3. A keyboard contains only 26 keys (one for each lctter in the English alphabct). If 3 kcys
are randomly prcssed, what is the probability of typing !\IBA?
(A) l
3
3
(B) 26
(C) u6r
(Dl _L
26
(E) u6r
4. There are 20 goats, 25 shecp, 3 dogs, aml12 chickens in a barn. If one animal is picked
randomly, what is thc probability that it will have only 2 lcgs"?
(Al 1o
(B)l
5
(C) l
3
5
(D) T2
(E) 1
5
5. A law firm has 15 lawyers that will be ranclomly assigned to 15 offices, 2 of which are
comer offices. If everyone has an egua! chance of getting a comer officc, what is the
probability that both Jasmine ancl Margaretta will get comer offices?
(Al /5
2
(B) 15
2
(C) 225
1
(D) 105
1
(E) 225

6. If a number is picked from al! positive integers less than 100, what is thc probability
that it \vil! ha ve 5 in only onc of its cligits?
l
(A) 1T
lO
(B) 99
18
(C) ~EY
18
(D) 1üü
(E) 19
· TlYo

82 GMAT i'c1ATH WORKBOOK


I Iow many triangles are in the figure above?
(A) 6
(B) 9
(C) 10
(D) ll
(E) 12

8. A bag contains blue, yellow, and red balls. The probability of picking a yellow ball is k
Which of the following cannot be thc total number of balls?
(A) 12
(B) 18
(C) 26
(D) 42
(E) 420

9. There are 20 girls and 16 boys in a class. 6 of the girls and 8 of the boys play soccer. If
one stuclent is picked randomly, what is the probability that it will be either a soccer
player ora girl?
7
(A) T8
(B) 97
(CJ 1}¡
(D) 95
(E) ~

10. Katya has 7 pairs of shoes. In how many different ways can she pick a pair that does not
match'?
(A) 49
(B) 42
(C) 41
(D) 3G
(E) 30

Gi\i/\T MATH vVORKBOO:< 83


11. Among the attcndees of an intcrnational marketing conference, 55% speak only one
language, 25% speak 2 languages, 15% speak 3 languages, and the rest speak 4 or more
languages. If one person is selected randomly, what is the probability that hc/she
speaks 2 or more languages?
(A) l
4
(B) .?_
5
(C) Z~
l
(D) T8
(E) 178

12. 8 people got togethcr for a meeting, aml each person shook hands with everybody else.
How many handshakes were there?
(A) 8
(B) 16
(C) 28
(D) 56
(E) 64

13. Ea eh Ietter of the word MISSISSIPPI is written on a picce of paper and placed into a ha t. If
one piece of paper is selected randomly, what is the probability that it will have an S on it?
1
(A) TI
2
(B) TI
(C) l
4
(D) "ª-
4
(E) 141

14. If two six-sided dice are rolled, what is the probability that the product of the two
numbers is odcl?
1
(A) 4

(B) 91
(C) -lG
1
(Dl T8

(F.} :-lG
15. Among 50 familics survcycd, cach owncd a car, a housc, or both. 2B oftlw familics
owned a housc, ami ,¡2 of thc familics owncd a car. If 1 family is sclectccl ranclomly,
wklt is thc probability that it will mm both a housc ami a car?
B
(.'\} 5iJ
(B} H
50
(CJ 20
50
2')
(!)) _:-:_
50
- 2B
(LJ so
84 GMAT MATH WORKBOO;<
16. Therc are 200 balls that are numbercd from l to 200. AH balls are placcd into a bag,
and l is selected randomly. What is the probability that the number on the ball will be
divisible by 6?
G
(A) 200
16
(B) 200
17
(C) z()o
33
(D) 200
GG

17m
(E) 200

A square dartbo.ard is divided into 1G squares as shown. If a dart is thrown randomly


and lancls on the board, what is the probability that it willland on a dark square?
l
(A) T6
(B) 61
(C) i
(D) ~
8
(E) lfi
16

18. In a race of 8 swimmcrs, how many diffcrcnt vvays could the gold, silvcr, and bronzc
meclals be distributcd?
(A) 216
(B) 336
(C) 340
(D) 343
(E) 512

19. Thcre are red, whitc, ami bluc balls in a bag. Thc proliabi!ity of picking a red ball is
twicc as muchas picking a whitc ball ami G times as muchas picking a bluc ball. \Vhat
is thc prob:lhility of picking a bluc ball'?
1
(A) Tü
(B) 1
5
(C) 2
9
l
(D) j

(E) ~-
20. A group of six friends wants to ha ve thcir picture takcn with the bride ami groom at
a wedding ccremony. lf the bride and groom stand next to each other ami everyone
forms a single line, how many different arrangements are possiblc'?
(A) 8!
(B) 7! · 2!
(C) 7!
(D} 6! · 2!
(E) 6!

SECTION 2.5-SOLUTIONS

1 2 3 4 5 6 7 8 9 lO 11 12 13 14 15 16 17 lB 19 20
B e E B D e e e B B e e E A e D e B A B

l. B A= {2, 4, 8, 16} · ancl B = {4, 8, 16, 32}

4 · 4 = 16 Fractions can be formed


Three of the fractions equal 1:
Probability = 3
16

2 e P(F) = # of face carcls = l.:_±= g


. 52 52 52

P(li) = li
52
Thcse two sets are overlapping. There are 3 carcls which are both hcarts and face cards.
To avoid clouble counting, subtract the probability of sclccting one of thcse carcls.
P(l-1 or F) = P(fl) + P(F) - P(H ancl F)

lZ. + JJ. _ _,1_ = 22 = ll


52 52 52 52 26

3. E The number of 3-lcttcr words that can be crcatccl from 26 lettcrs is 26 · 26 · 26 = 2G 1•


On !y 1 of thesc words is i\ fBA. P(M /3;\) = (
23
,) ¡
Altcrnativcly\ builcl the word one lettcr at a time.
The probability of pressing M first is ;]¡~· of pressing B second is 1~· and nf prcssing A
1 2
r1111. ·el 1::.
. . z(j·

Prcssing M, B. andA in ordcr is :fe~· tcj ·2~ = (;}¡J·


4. B Only thc 12 chickcns out of 20 + 25 -t 3 + 12 = GO atJilllals han· 2 lcgs.
P(2 \('aued)
1)t)
== 12_
60
= l5

5. D Grcak this questiou into easier parts. Lct's assumc Jasminc goes first ami ran-
t\\'O

domly picks an officc. Thc probahility that shc will g<'t a corncr otlicc is 2 .
15
Once Jasminc picks and gl'ts a comer officc, tlw probability thatl\larg;¡rctta will gct a
comer oiTice is \ (l comer o!Ticc is left out of H ofliccs). Thcrcforc,
1
') l ') 1
PU <md i\ ll =- -rs .
Tf e~ 2Tü = TF~·
Altcrnativcly, the first comer office can be assigned in 15 different ways. Once it is
assigned, the second comer office can be assigned in 14 different ways. Overall, the two
comer offices can be assigned in 14 · 15 different ways.
Our desircd outcome is for Jasmine and Margaretta to have comer offices. That can be
accomplishcd 2 differcnt ways:
Comer 1 to M Comer 1 to J
Corncr 2 to J or Comer2 to M

6. e There are 99 positive integers that are lcss than 100. There are 99 possible outcomes.
lB of these numbers have 5 as only one of its digits (cxclude 55).
5
15
25
35
45
50, 51, 52, 53, 54, -, 56, 57, 58, 59
65
75
85
95
P(5) = _lli
99

7. e Count the triangles systematically starting with .6ABCto thc left .


.6ABC .6ABD .6ABE .6ABF
.6ACD L\ACE .6ACF
.6ADE úADP
.6AEF
Thcrc are 1O triangles in total.

8 e P(}') = l = !'lumbcr ofyellow ba!I2_


· G Total number of balls
Sin ce thc numbcr of balls must be an intcgcr, it has to be a multiplc of 6. The only
ans11·cr choice that is nota multiple of Gis 2G.

9. B P(G) = ~f2 Probability of picking a girl is 20 out of 36.


36
P( S l = ti:tt)}
,) )
= ~-(~-
;¡)
Probability ofpicking a soccer player is 14 out
of3G.
P(S and G) =A~~ Probability of picking a fe mal e socccr player is
6 out of3(i
P(S or GJ' ~~ ~(GJ 14 G 2B 7
~, · + :3ci -- 36 = :16 = 9

GNAT t•iAnl1NORXBOOK 87
1O. B Break down tlw prohkm lo picking a shoe for llw h·fl fool amlthen for tlw righl fool.
Katya C<lll pick 1 dill!'ITlll lefl siHH'S sinn· tlwn· are no n·strictions Y!'l. Afler picking tlw
ldt slHll', slw can only pick ti dillerenl s!HH's for tlw righl foot sinn· slw is lrying 11ot to
match. There are 1 · ti • .¡~ dillerent ways.

1 1. e Since tlll' totalnumlwr of people is not gin•¡), aSSlllll!' tlwre an· lOO peopll' to simplif)·
your calculations.
1 language: :i5
~ languages:2:i
3 languages: 1:i
·l or more languagcs: 100 ~ (:iS + 25 1~ 15) ~· 5

The number ofpcople who s¡wak 2 or more languages is 25 + 1:1 + 5 = ·15.

fl(2 or more) = _15_ = .Jl~


100 20
Alternativcly, 2 or more languages means not only 1 language. To find the probability
that the pcrson eh osen speaks 2 or more languages, subtract the probability of spcaking
onlv 1 languaae from thc entirc grrou¡J 1 - li~~- 4
= lOO~~~ = .JL
' o ' lOO 20·

12. e Thc first pcrson shook hands with 7 people. The sccond pcrson shook hands with
G people because he or she already shook hands with the first person. Makc a list of
everybocly's uniquc handshakes.
1st person shook 7 hands
2nd pcrson shook 6 hancls
3rd pcrson shook 5 hands
4th pcrson shook 4 ha neis
5th pcrson shook 3 hands
6th pcrson shook 2 hands
+ 7th pcrson shook~}__!_l<~~----
2B handshakcs in total

13. E Thcre are 11 leller~ in total..¡ nlwhich are::,.

P(S) = ít
l·L A For tlw product to be odd, both llU!llbers necd lo lw miel.
The probability thal tlw 1st die wi!! show an odd numbcr is ~~ or} since tlwrc are 3 odd
numbers on a die ami it !J;¡s G faces. Thc probability of an odd numhcr for the 2nd die is
also} Thc probability th<ll both wiil shm\ an odd number i~ Jl(Odd) ~· ~ ·} -~ ~1·

15. e Once \n' add the number ofhousc owm•rs and r<1r m\·ncrs (·12 + 2B ~~ /0), \\T rcalizc
that thcre is an owrlap. /0 50'~ 20 peoplc mm both.
fl(Both) ~ 2il
50

88 GMAT 1\-JATH •lRi<T:ooK


16. D first find out how many numbers between 1 and 200 are divisible by 6.
The smallcst number divisible by 6 is 6. The largcst number divisible by 6 is 198. lt is
divisible by 198 because it is divisible by 2 and 3.
l9B- 6
6
= 192
6
= ,)_
.,.)

Since we nced to count them inclusively, there are 32 + l = 33 numbers.


P(6) = 33_
200

17. C There are 16 small squares on the board, so the total number of potential outcomes
is 16. Therc are 6 dark squares.
P(Dark) = j)_ = -'2
16 8

lB. B 8 di!Terent svvimmers could get the gold. Once one gcts the gold, 7 different swimmcrs
can gct the silvcr ancl then 6 different swimmers can get the bronze.
B · 7 · 6 = 336

Alternatively, use the permutation formula;


P(B, 3) = B! = -ª-:.J~l = 8 · 7 · 6 = 336
(8 - 3)! 5!

19. A Let the probability of picking blue (B) be x.


The probability of red is 6 times the probability of bluc, so R = Gx.
The probability of red is 2 times the probability of white, which mcans the probability
of Wis half of R, so W = 3x.
Sin ce there are only three diffcrent colors of balls in thc bag, the probabilitics must add
up to l.
R+W+B=l
6x + 3x + x = l

lOx = 1
X=
.
_L
1()
P(Blue) = x =
1
b
20. B Since the bridc ami the groom stanclnext to each other, consider thcm as onc person
initially. Iftherc are 1 pcople to arrange in a liiw, thcre are 7! possible orderings. In addi-
tion, the bride and the groom can arrange thcmsclves in 2! ditTercnt ways (BG or GB1.
Thcrefore, the total numbcr of arrangelllcnts is 7! · 2!
Algebra

_. 3.1 ALGEBRAIC EXPRESSIONS ANO FACTORING


_. 3.2 LINEAR EQUATIONS ANO INEQUALITIES
_. 3.3 EXPONENTS ANO RAOICALS
_. 3.4 QUAORATIC EQUATIONS, FUNCTIONS, ANO SYMBOLISM

SECTION 3.1: ALGEBRAIC EXPRESSIONS AND FACTORING

Algebraic Expressions
111 In algebra, you work with unknown quantitics that are reprcsented by various sym-
bols. These symbols are usually Icttcrs of the alphabet. These unknown quantities are
called variables. They are used as plac~ holclers for unknown quantities.
111 \Nhen letters are placed next to numbcrs without any operators between them, it
means multiplication. Each series (product aml/or quoticnt) ofvariables and num-
bers is callcd a term.

"2x" means "2 times x" = 2·x

2xz' = 2 · x · z · z

3mR '" 3 · n: · R
kr k· r r k 1
111 = -¡¡¡- = k · 111 = -,,-, · r = k · r · Tll
111 A pattern ofvariables and numbers is called an expression. lt is made up of constants,
variables, anc! mathcmatical opcr::tions. 1\11 expression may includc onc
or more tcnns. NOTE
111 -· 27 Expression witll two tenns: m ami 27 lf there's no coefficient in
3111 + 5y -- 4z Expression with tlucc terms: 3m, 5y, a!l(! ,1;:
front of a monomial, the
coefficient is equal to 1.
¡¡¡ t\n cxprcssion with onc tcrm is callcd a monomial. Thc numbcr in front ax = lax
of tlw variable is callcclthc cocfl1cicnt of tlw term. lf there are no powers, the
power is equal to 1.
'v\ r:J.\
~----Tb--c--~----1-b-'c_'_______ . . ~
Exam¡)lcs: C2\r,
~ Str)',
u:~ Cf7'xllz
~ ~ ~

\ t / ~-
cocfficient~ A
• An cxpression \\·ith more than eme term is called a polynomial.

Examplcs: n+3 3(r- k) 3x2 Sx + ·1


• Since al! \·ariablcs reprcsent a number, all arithmctic rules n~\ie\\'ed in Chapter 2
apply to variables and expressions as well.

ADDING ANO SUBTRACTING ALGEBRAIC EXPRESSIONS


• Tenns that have equivalent variable parts are called like terms.
NOTE
lf there's a negative sign in Examples: 4x and 7 x, 2711m 2 and 7 lllll 2 , and 5.5mfC and
front of parentheses, it changes 13nzfC are like terms.
the sign of every term that is 7zt and zt are likc tcnns. zt simply means lzt.
within the parentheses.
5.\}' and 12yx are also likc terms since yx = xy (commutative
-(m - 3) = -m + 3
property).
''------------.--------~
·x and 5x3, · and 3xy and 3xy 2 are not like tenns.

• Only like tenns can be added or subtractcdo

Example: 3x + 5y + 6x = 9x + 5y
Example: ~.=..fu~+ 7v Identify like terms, and mark themo

6xy 2 + 42oxf- 5y + 7y Group Iike terms together. Remember to


m ove the terms along with their signso
48xy 2 + 2y Combine likc termso

Example: 111/l + m2 - (3mn- 4m 2 ) To remove the parentheses, multiply both


3mn ami -4m 2 by- l.
+ m 2 - 3mn + 4m 2
11111 Group like termso
mn - 3mn + m 2 +4m 2 Add like termso
-2mn + 5111 2

MULTIPLYING ALGEBHA!C EXPRESSIONS


• Whcn multiplying two terms, multiply the numerical parts togethcr and the variable
NOTE parts togcther.

3m· 2m* 6m Examples: 2c · e= 2 · e· l · e= 2 · 1 · e· e= 2c"


3m· 2m= 6m 2 3X · 5)' = 3 · 5 ·X· J' ~, l5X]'
511111 · 12111 = 5 · 12 · 111 · 111 • n 1
=--= G0111: 11

111 Wlwn multiplying t\YO polynomials, use thc clislributi\'c property.

Examplc: 41/1 · (2- 511) =e 4111 · 2- 4m· 511 Bm ·- 201/111


3m+ 2m= Sm
9m- 3m= 6m
~~
Exarnplc·: (!ÍI+ 2) (ll - 3)
o

~~
117. (11 -- 3) + 2 o (11- 3) l\lultiply each tcnn of the first cxpression by
ca eh term of thc sccond cxprcssiono

11111- 3/J/ + 2n -· G Thcre <1rc no likc lcrms, so this is the simples!


form of tlw cxprcssion.

92 Gi\<lAT fYU,TH WORKSOOK


DIVIDING ALGEBRAIC EXPH.ESSIONS
• \Nhen dividing two terms, divide the numerical parts with cach other NOTE
and divide/simplify the variable parts with each other.
The denominator of a fraction
can never be zero since
Examples:
dividing by zero is undefined.
Simplify thc m's since ~~~ = l.

FACTOH.ING AND SIMPLIFYING ALGEBH.AIC EXPH.ESSIONS


• Factors of a tcrm (ora polynomial) are simpler tcrms that when multiplied together
are equal to the original term (or polynomial).

2x 2 + x + 5 2·x·y
~ ~ ~ ~ ~ ¡
Terms Factors

Examplc: Two of the factors of 6m 2 are 3m ami 2m sin ce 3m· 2m= 6m2 •
6m2 = 2 · 3 · m · m

• The process of rearranging ka + kb to k· (a + h) is called factoring or taking out


the common factor.

Example: 5 and (x + 3) are factors of 5x + 15 beca use 5 · (x + 3) = 5x + 15.


~

Example: a k+ at + an = a · (k +t+ n) Each term has a factor of a,


so a can be factored out of
the parcnthcses.
Example: 6111 2 -- 3m + 9m:< = 3m · (2m - 1 + 3m2 ) Each tcrm has a factor of
3m. Factor out 3m from each
term. While you are factoring
out 3m, you are dividing each
term by 3m. Thercfore, the
remaining tcrms are
2!.!.~ = 2 ~]!.!.~ = -1 ancl
3m ' 3m '
9m'
3111
Example: 8111'' + 41'11 2 - 16111' = 4111' · (2u +u- 4) Each tenn has a 4, 11, ancl 11 in
it, so 4111' can be factored out.
Thc rcmaining terms are
§J!J(. = 2 l' 1J..~{.: = u a ncl
4!1!' ' 4!111 J

-16111'
-~r¡¡;,- = -4.

Example: 2x- 5 "'' 1


-· "" "-"- If thc same cxprcssion
2x -- 5
appears in thc numcntor and
the dcnominator of a frac-
tion, thc: quoticnt cquals 1
pruvidcd tlFtt thc cxpression
itsc!f is not cqual to zero.
Example:
First factor out the common term of 5a in the
numerato r.
5a(a 5)
-----~---
a-::¡
= r: ,)(7 .
S1111p ¡·¡·
1 y 1llC tcrm (a- .l
") SlllCC
. (a- 5)
·-(---·;=-) = l.
{l-:J

Example:
3x- 6
L-x Factor out a 3 in thc numerator.

3(x- 2)
-2-----x- 2 - x is equivalen! to - x + 2 or (x- 2).

3(x- 2) = _
3 Divide out (x- 2), leaving l in the
-(x- 2)
denominator.

EVALUATING ALGEBRAIC EXPRESSIONS (SUBSTITUTION)


• An algebraic expression can be evaluatcd by substituting in known values of all the
variables when they are given.

Example: Evaluate 4ab + 3a 2 - 6 when a= -2 and b = 3.

Replace al! a's with 2 and all b's with 3.

4(-2)(3) + 3(-2) 2 - 6 Use parentheses foral! variables initially so


it is clear which variable is replaced by which
number. Doing this also helps vvith determin-
ing the order of opcrations.

-24 + 3. 4 6 Do the multiplication in the first term and the


exponents in the second term. nemcmber that
(-2) 2 = (-2). (-2) = 4.
-24 + 12 - 6 -18

Sf\rVIPLE PHOBLEMS

Multiply (a + 2) by (a - 7).

~
(tt + 2) · (rt - 7) i\lultiply each tenn ofthe first cxprcssion by each term ofthe scconcl
~-y
expression.
{/ . (/ -- 7 (7 + 2 ({ -- 14 i\lultiply, add, am! subtract like rcrms.

94 Gi"JAT i"LO.TH WORi'{BOOK


4fr:f.1M!t»--------------------,
1 Simplify x(x - S) - 16(x - 5).

X(X- 5) - 16(X- 5) Notice that (x 5) is a common factor, so it can be factored out.

(x- 5) (X- 16)

r-------------------~814!art--------------------.
Factor 6m 2n- 12mn + 18mn 2•

6111 2 11- 1211111 + 18mn 2 Notice that 611111 is a common factor. Factor out 6mn.

6mn(m - 2 + 3n) The remaining terms are


6nz2lz = m -12m!~= _ 2 and l8mn2 = 311 .
6mn ' 6mn ' 6mn

SECTI

l. What is the simplcst form of2(x + 3)- 3(x- 2)?


(A) 10
(B) 12
(C) 6x
(D) 12- X
(E) X-!- 12

') 'lJJl.k__¡_¡_~ = 7
l - 2k .
(A) -k
(13) -m
(CJ
(D) m
(El k

3. If x =--o ---2, what is the val u e of -- 2.\-c -- :-'>.r:'·?


(i\) -2-l
(B) -16
(C) -B
(D) 1()
(E) 32
·L Which of the fullowing is equivalent Lo (k+ t)(Jk -t 5) -2k- 2t"?
(¡\) 3(k+ t)(k+ l)
lB) (f..: T t)tk + 3t)
(C) (k- t)l3k + 5)
(D) 3(k tl(k + t)
(El (3k + 5)(k + 2t)

, 5. Tlw difference of (x + 1J' and (x- 1)2 is how many times greater than x?
(¡\) -2
(B) 2
(CJ 4
(D) 3x
(E) 2x + 4

6. What is the value ofthe expression -2m'n -3mn2 when m= -1 and n = -2'?
(A) -IG
(B) -8
(C) 4
(D) 8
(E) 16

7. The exprcssion (x- 3)(2x + 5) - x(2x- 1) is equivalent to which of the following?


(A) -15- 2x
(BJ -2x
(C) -15
(0) x' - 2x- 15
(E) -15 + 2x

xyz + x yz + xy 2 z + xyz 2 _
2
8· ---- xyz --- - ?

(A) -\l'Z
(B) 4xyz
(C) x +y+ z
{[)) 1 +X+ J' +Z
(E) l + x + i' + z 2
2

9. Wh¡ll is lhe value of cm~¡f en if n °" 5?


(A).¡
(B) 3
(C) 2
(D) l
(E) lt cannot be dctermined from thc information given.

10. (l.r -- 2)(2.r + 1)- (13x 2 +. 3.r- 2) =,?


(;\) X

(B) x(.r G)
(CJ 2'7x'- l
(IJ) x:'
(E) x!- 4

96 Givl/\T NATH )Rl<tJOOK


ll. (n - 2)(x + 3) is how much more than -O n)(x + :m
(A) 2nx + 6n + 2x+ G
(B) 2nx + 2x + 6
(C) 2x + 6
(IJ) 6
(E) 2

12. If 2x + 3y = 12, what is the value of 1/ +y+ 4x 2 + l2xy + 9y 2 - .Jx- Gy'?

(A) 4
(B) 16
(CJ 44
(D) 124
(E) It cannot be determined from the information given.

SECTION 3.1~SOLUTIONS

l. D 2(x + 3) - 3(x- 2) Distribute 2 and -3.


2x + 6- 3x + 6 Add and subtract like terms.
-x+ 12= 12-x

2. B Z!_n k_-:_!!!:. Factor out an m in thc numcrator.


l - 2k
m(2k- 1)
--T=--2k Hcwritel 2kas-(2k-1).

m(2k- 1)
-=--r2 k:-=-·n = - m

3. O Substitutc -2 for each x. It is a good idea to use parcnt!Jescs in the first step.
2(--2) 2 - 3(-2)J (-2)' = 4 (-2)' = -B
-·2(4)- 3(-BJ Multiply.
-8 1· 24 = lG

,1. A (k+ t)(3k ..:. S) - 2k -- 21 Factor out -2 frum -·2k 21.


(k+ 1)(3k t 5) - 2(k + 1) (k+ l) iwcomes a common tcnn. Factor it out.
(k+ t)(3k + 5 -- 2)
(k+ t)(:i!: + 3) Factor out a 3.
3(k + t)(k + 1)

5. C First fincJ the diffctCilCC of (X+ 1)' aml (X-- J)c.

rx + 1r - (x- 1 r Square and subtrdct or use thc di!Tcrence of l\\'O sc¡uares


formula from Scction 3..!.
x2 +· 2x + 1 -- (x' ·- 2x + l)
x' + 2x + 1 --- x2 + 2x ·
o.- 4x
{x == 4 Divide· 4x by x to find llow lll<illY tinws greatcr the
X
diffcrcncc is th;cn x.
GH.·\T f•lATi-l WORK300K 97
' 6. E Plug in~ 1 for m ami ~2 for 11.

~2(~1) 2 (~2) ~ 3(~1)(~2) 2

~2 · 1· r~2) ~ J( IJ. Hl
-!+12=16

7. e (X~ 3)(2X + 5) ~ X(2X 1) Multiply sincc no common terms can be


factored out.
~x
2
+ 5x - 6x ~ 15 ~ ~x 2 + x Add and subtract like terms.
-- 2x + Sx ~ 6x + x ~ 15 =
2 2
2x 15

8. D Notice that xy..:: is a comm9n term in the numerator.


xyz + x 2yz + xy 2 z + xyz 2
xyz Factor out xyz first.
xyz( l + x + y + z)
---xyz _ __ Simplify xyz.

=l+x+y+z

9. A The numcrator is e· a· n ~ e· n. So en is a common factor that can be factored out.


cn(a ~ l) ~ _
ne ~a 1

Plug in 5 for a.
a - l = (5) - 1 = 4

10. D (7x- 2)(2x + l) - (13x 2 + 3x- 2) Multiply the first term.


14x + 7x 2
~ 4x- 2- (13x 2 + 3x- 2) Removc the parcntheses by multiplying each
term by -l.
l4x 2 + 7x- 4x- 2 - !3x 2 - 3x + 2 Group thc like terms, and then acld ami
subtract.
14x2 ~ 13x2 + 3x- 3x- 2 + 2 = x 2

11. e Fine! the diffcrence bctwcen ~he two terms by subtracting thc second from the first.
(n- 2)(x + 3)- [-(4- n)(x+ 3)] Multiply the tcrms. Two negativcs will turn
into an adclition.
nx+ 3n·- 2x G + 4x+ 12 ~ nx- 3n Group thc like tenns, and thcn <tdd and
subtract.
nx -- nx + 311 - 3n -- 2x + 4x- G + 12
2.r +G
Altcrnatin?ly,
(n- 2)(x + 3) + (:J ~ n)(x + 3) Factor out thc (x + 3).
(x + 3)(11 -- 2 + ,¡- n)
(.r + 3)(2) = 2x + 6
12. O Noticc that the certain tcrms in the given equation can be written in terms of2x + 3y,
which equals 12.
?r
~J +y+ ü' + 12xy + 9y" ·IX- Gy

B e
2 2x + 3¡• l'J
A: -{ + y= - 3= 3~ = 4

B: 4x 2 + 12xy + 9y' = (2x + 3y)' = 122 = 144


C: -4x- Gy = -2(2x + 3y) = -2 · 12 = -24
2{ +y+ 4x 2 + 12.q + 9y 2 - 4x- 6y = 4 + 144- 24 = 124

SECTION 3.2: LINEAR EQUATIONS AND INEQUALITIES


Linear Equations
SOLVING A LINEAR EQUATION WITH ONE UNKNOWN
• An equation states that two algebraic expressions are equal. For example, "sixteen
more than 2 times a number is equal to 5 less than half the same nurnber" is an
equation in plain English that translates into 2rz + 16 = ~- 5.
a The solution of an equation is the value of al! stated variables that rnake the statement
true. In other words, tbe solution is al! numbers tbat satisfy the equation.
Example: In the equation 3m + 2 = 8, replacing m with2 gives you 3(2) + 2 = 8,
which is true. Thcrefore, 2 is the solution of the cqu:ttion 3m + 2 = 8.
• To sol vean equation with one unknown, isolate the variable on one si de of the equa-
tion. Isolating a variable means it remains alone on onc side of the equatiün. When the
variable is alonc, it has a coefi1cient of l. The side the variable is on c:oes not matter.
Examples: n = -3

~ = x (which is the same as x = ~)


• To isol~1te a variable, use the following properties.
,·1rl< l i 1ion /S 1il; 1raer i o; 1: You can add (or subtract) the same quantity to (or from)
each sic! e of thc equation without affecting the cquation.
Examples: x ~' G 11 4=5 n+2=5
x+3=G+:l n--1+4=5+4 n + 2 -- 2 =5-2
x-+3=9 n=9 11=3
Mili! ipl icnr ion(J)iuision: Yo u can multiply (or divide) ca eh si de of the cquation by
the san1c nonzcro quantity without allecting thc equation .

Examplcs: x= G .!1=5 711. = 21


4
3·x~3·G 4-!l 5·4 711_21
-~r---r
4
3x = 18 11 = 20 11=3
í REMEMBER
Whatever you do to one side of the equation, you must do the same to the other side of
the equation. The goal is to isolate the desired variable.

THREE-STEP PHOCESS TO SOLVE UNEAR EQUi\TIONS WITII ONE UNKNOWN

1. EV:\l.ll.YIT. t•acll side or tlll' equation as lllliCll as pos-.,ihl(• by lllllltiplying l'\¡JOIH'll(S,


simplifying parentlwses. ami comhining liKl' terms.
2. l\IOVE al! \'ariables lo o m• si de and allnumlwrs lo tlll' ollH'r sidc of lhe equal sign
using llw propcrtics prl'\Íously descrilwd, aml simplify holh sidcs again.
3. ISOLJ\TE thc dcsired variable.

Example: 5 t- 4(11 2) + 3(11 + 5) = 2(11- /) +1


l. Evaluatc

5 + 4(11- 2) - 3(11 + 5) = 2(11- 7) + Use the distributive propcrty to removc the


parenthescs.
5+ 4n- 8- 311- 15 = 211 -- 14 + 1 Group like terms togethcr.
411- 3n + 5 8 -- 15 = 211- 14 + 1 Combine likc terms.
11- 18 = 211 -- 13 Once each side is simplified, go to steps
2 aml 3.
2. Move
11 - lB = 2n - 13 A del 18 to ca eh si de of the equation so the
numbcrs are all on thc right side.
11 = 211 +5 Subtract 2n from each sidc so the variables
are ali on thc lcft.
-n 5
3. Isolate
-11 = 5 l\lultiply each sidc by -l.
11 = -5
zr Ifyou cmlup \\·ith a r~1lsc ~l~l!cnwnt at tlw cl1l! uf::nur solution, citlwr thc cquation
docs not ha\'l' a solution (docs not \\'Ork for an)' lllillllwr) or you llave madc a mistakc.

Examplc: :~(.r ¡ el) - ]() = 27 + :lx Distrihutc :1 on the lcft side.


3.\ -- 12 - 1(í = :!.7 ' :Lr -\dd likc tcnm.
:\x -1 " 27 ' 3.r Su!Jlract :\.r from both sidcs.
--- -~ = 21, whicll is falsc This nH'ans thc original cquation has
nn snlutirlll.

111 lfyou cnd up \\·ith a truc slatemcnt at tlw cml but 110 \·ariablcs. tlw cquation works for
all rcalnumlwrs.

Examplc: :l(.r -'- -1) lt)- :h .¡ 1 listribute :l on thc left side.


:1.1 ' 12 - 1ti = :lx -- -1 :\dd likl· tcrms.
:lx -1 - :lx ·· ,¡ Suhtracl Jx from both sidcs.
-1. \\·hich is truv This me;ms tlw cquation is trtH' for
any real numlwr.

100 Gf'1AT ¡··iATH WORX:BOOK


SOLVING TWO LINEAn EQUATIONS WITII TWO UNKNOWNS
• Two independent equations are necded to solvc for two unknmvns.

Example: 2x + 3y == 6
7y- 5y =
.:...
11

• Two common mcthods can lw ust'd to soh·e thesc equations simultaneously,


substitution ami elimination.

S u úst i tu t ion:

1. Use one of the equations to isolate one of the variables. Pick thc variable that
is easier to isolate.
2. Substitute the isolated variable into the second equation to transform it into
a single linear equation with one unknown.
3. Solve the equation using the three-step process described previously.

Example: 2x + 3y = ll
5x- y= 2

Isolate the y in the second equation. It is relatively easy to isolate sin ce it has
a coefficient of -l.
5x- y= 2 Subtract Sx from both sides.
-y= 2- Sx Multiply both si des by -l.
y= -2 + Sx Substitute this y into the first equation as follows.
2x + 3y= 11
2x + 3( -2 + Sx) = 11 Distribute the 3.
2x 6 + 15x = ll Acle! the Jike terms.
l7x- 6 = ll Add G to both sides.
17x = 17 Divide both sicles by 17.
x=l
Aftcr solving for one of the variables, substitute it into onc of the original equations to
sol ve for thc second Yariablc. Use the first equation for exJmplc.
2x + 3y = ll Plug in l for x.
2(1) + 3y = 11 Subtract 2 from both sides.
3y= 9 Divide both sicles by 3.
y=3

This mcans the pair x = l imd y= 3 is thc solution of both cquations. Check _1uur
answcr by suhstituting these Yalucs in lo thc original cquations.
2X + 3J' o·. 11 2(1) + 3(:1) 11 11=11.1
5.r- .1' = 2 5(1)-3=2 2=2.1

l:!illlinurion:

a Flimination is ba~t·d on tlw fact that two cquations can be addccl togcthcr to crea te
a third equation.

2.r f .Jy = Ll

5.r + 2y = 8
• To eliminare one of the variables, the coefficients of the variable you are trying to
eliminare should ha ve the same absolute value. lf they do not, you can easily multi-
ply one or both of thc equations by a constant number to make them thc same.

Example: 2x + 3y = 6 Sin ce the coefllcients of y are 3 and -3,


lx- 3y = 12 adding these equations side by side will
9x +O = 18 eliminare y.
9x = 18 Divide both sides by 9.
X=') Plug x = 2 into thc t1rst equation.
2(2) + 3y = 6 Subtract 4 from each side.
3y = 6- 4 Divide both sidcs by 3.
.v= ~
3
Example: 2x + 3y = 14 Ifyou multiply the seconcl equation by 2, xcan
-x+ 5y=6 be elimina red using acldition.

2 · (-X+ 5y) = 2 · (6) Multiply every term by 2.


-2x + lO y= 12 Add this equation to the first equation to
eliminate x.

2x + 3y= 14
-2x + 10y = 12
o+ 13y = 26

l3y = 26 Divide both siclcs by 13.


y=2 Plug 2 into the first equation for y.
2x + 3(2) = 14 Subtract 6 from each side.
2x= 8 Divide both sides by 2.
x=4

m If two cquations are m u! tiples of ea eh other, there are infinitely many solutions.
In othcr words, an infinite number of (x, y) pairs satisfy the equation.

Example: 3x- 5y = 21
9x- l5y= 63

Ifyou multiply thc first equation by 3, 3. (3x- 5y = 21) = 9x- 15y e= 63, you get the
second equation. This means thesc two equations are equivalent. In this case, there
are inflnitcly many (x, y) pairs th;:t satisfy both equations, such as x 2, y= -3
or x = 7, y= O.

Ifyou do not notice this from the beginning ami attempt to soln: thcsc types of equa-
tio:Js, you gct a true statcment with no variables.

3x-5y=21 beco mes 9x- 15y = 63


9x-- 15y= G3 beco mes 9x- l 63 Subtractthe cqu;Hions.
O'""' O The rcsult is'truc, which mcans therc
are infinitely many solutions.

n Hthc cquations prescni contradicting results, thcrc is no solutiun.

Examplc: 3.r- 5y = 21
9.r- 15y = (i2

10.2 GMAT HATH WORKB '0K


If you multiply the t1rst equation by 3, 3 · (3x- Sy= 21) yo u get 9x- 15y = 63. The
left-hand side bccomes the same as that of tlw sccond equation, but it is equal toa
different constant. This means thcse two equations present contradicting informa-
tion. In thcse cases, no (x, y) pairs satis!}· hoth equations.

Ifyou do not notice this from the beginning ami attempt to solve these types ofequa-
tions, you get a false statement with no variables.

3x 5y = 21 beco mes 9x- 15y '= 63


9x- 15y= 62 beco mes 9x- l5y = G2 Subtract both sides.
O= l Thc result is false, which means there
is no solution.

Linear lnequalities
• An inequality compares two algcbraic expressions. For example, "three times a num-
ber is greater than 5 more than t\vice thc same number" is an inequality in plain
English that translates into 3n > 2n + 5.

SIGI"~S Of INEQUALITY
'f:. not equal to
> greater than
2: greater than or equal to
< less than
=s less than or equal to
'--------------------------------------

1:1 Solving a linear inequality with one variable is esscntially the same as solving an equa-
tion with one variable. The only c!iffercncc is that whcn you multiply or divide both
siclcs of an inequality by a ncgativc. nonzero numbcr, the incquality sign reverses.
Everything else is the samc as working with cquations.

Example: 3 2x =:::: 5 Subtract 3 from both sides.


-3 -3
- 2x 2: 2 Divide both si des by --2 ancl thcrcfore rcverse
x s: l thc incquality sign.

Example: -4 21-:r- ,::-c_i l\lultiply both sidcs by --5 and


) Whcn you multiply or
revcrsc the inequality. divide both sUes of an
(-5) · (--4) S ( 4 ·~~~_1)· (-5) Simplify thc --5s on the right side. inequality by a negative
20 S: 4.r- 4 Add 4 to hoth sides. number, the inequality
24 S 4.r Dí vide both sic! es by 4. si9n reverses.
6s.r x can he any re ! numbcr greater
than or cqual to 6.
• l\\'o ÍIH'qualitil's l'aiilll' <tdded ~idv by side if tlll' inequa!it\· signs han·tlw

rl: :: :::: :::: ~ :~ :~
REMEMBER ·¡ sanw dirl'ction.
1 1 ! {/ f¡
/J
(/ • ( /J • ti
'-.

S.\:'\ll'l.li'I\OBI.l.\1S

.-------------------------44:o;tQtJ•------------------
I
Find x if 4 + 3(x- 4) = -x- (2x- 16)

(A)~
1 4
(B) 1

(C) .1_
3
(D) 3

(E) 4

4 +3(x-4) = -x- (2x-l6) Evaluate the contents of thc parenthescs first. Distribute
3 aml -l.
4 + 3x - 12 = - x - 2x + 16 Add and subtract like terms.
3x···B=-3x+lG Adel 3x to both sides.
()x 8 = 16 Add 8 to buth sidcs.
Gx = 2·1 Divide both sides by 6.
x=4
The answcr is (E).

¡-·---------------··-··· - - - · · - - -

Solve for x and y if x - 2y = 12 and 2x - 3y = 25.


- - - - - - · · · · · · --· - - - - - - - - - - - - - - - - - - - - - - - - - · ·------------------·----·J

Use substitution ~inn· x can be casily isolatcd in thc firstcquation.


x 2y =' 12 Add 2y to both si des.
X~' 12 + 2y Substitutc tlw C'l]\IÍ\"alent or X from this equation into thc second equation.

2\x) -- 3y = 2:1
2(12 + 2\') - 3F' 2:1 Distrihutc 2.
2·1 + ·ly - 3y = 25 Add like tcrms.
2-1 + .\' =-- 25 Suhtr<ll't 2-1 from both sides.
.\'e: ]
Substitute y= 1 iillO the first cquation to find x.
x-2(l)=c12
X-- 2 =- 12
.\ =e 1 1
r-----------------ltüW!tt----------------,
lf 3 lbs of apples and S lbs of oranges cost $3.15, 15 lbs of oranges and 9 lbs of apples
cost $9.45. How much does 1 lb. of apples cost?

(A) $1.10
(8) $1.05
(C) $0.90
(D) $0.80
(E) lt cannot be determined from the information given.

First translate the two equations.


3lbs of apples and S lbs of oranges cost $3.1S translates as 3A +SR= 3.1S
1S lbs of oranges and 9 lbs of apples cost $9.4S translates as 9A + 1SR = 9.4S
Use elimination since neither A nor R can easily be isolated. Multiply the first equation by 3
to match the coefflcients of R and then subtract.
3 · (3A + SR= 3.1S) · 3
9A + 1SR = 9.4S Bring o ver the secorid equation and subtract.
9A + 1SR = 9.4S
O= O True These two cquations present the same information.
There are infinitely many solutions. Therefore not enough information is provided to salve
the problem.
The answer is (E).

l. Find x if .:! - 2 x.±_l + g_=--.:-! = §.:::-..1~


5 3 15 15 .
(i\) --15
(B) -12
(CJ --9
(DJ -5
(F) --3

2. IfJ(n -- 1) + 2(2n - 3) = 3(2a- 3) + n, what docs o cqtd'?


(!\) N o solution
(!3) --1
(C) O
(D) l
(E) J\ll real numbcrs

3. lf _ _lll __ = G - ---~---, wbat docs 111 cqtL1l?


171 - 3 /11 -- 3
(i\) 3'
(Bj 4
(C) 5
(D) G
(E) 7
·l. 11 :l.r
., 1.11 11a11~
. l 1lt' gll'<tll'~l
.lllll'gt·r \<1
1lit' t¡ ¡· .\!.,
2
1.·\) ')

11\i
1() ()

( 1)¡ 1
11) -
')

.l. Tlm•(' adulh 1\'ilil [\\() childtl'll p;lid S 1:_>-;- Íll total rm l'Otlrl'll tickt•ts. 11 a child's ticket
costs lt·~s tllan an adult's tickl't a11d if tlm·l' cllildrl'll allll l\\o aduhs paya lota! ofSll:l.
IHJ\\' llllll'h are thc tickl'lS rm OIH' adult with Olll' child'{
(:\) S lO
(B) s-m
(C) S50
(D) $()()
([) SBO

2 J -¡
ti. lf O.; a< f¡ ami k= _rL_f¡!.... ~. which of thc following must be true?
(A) k< 2
(!)) k< 7
(C) k< 9
(D) k> 9
(E) k> 11

1. If -4 < r1 < 2 and -() < LJ <:l. what is th~ largest integer va!ue of a 2 + b 2?
(A) l2
(B) 3c!
(C) 45
(IJ) 51
r:: ·¡
(E) ,)_

. 1 n. vVhat is thc value of X if X- 2x J :\x -- 4x + 5x- Gx + ... + llx -· !2x = - 1!3'?


(t\) - 3
(B)
(Cl o
{!)) 1
([) :1

') l
~l. [f 1 . < ')' how tllé\11\' intt•t;t•r 1·aluc•s cm x lake'?
9 X .. l > ·' . •
(¡\) ()
(ll) B

(Cl ]()

(!))ll
(El 12
lO. lf x. ~--;¡ ·'
,_ = ,1 =,~J~x which of the following cannot be the val u e of x'?
(.\) o
!B) l
(C) 2
([)) 3
(E) ·1

11. 2x + 3y + 1 = O ami -1 < y< 2. \Vhat is the greatest integer value of x?


(:\) -J
(!3) -2
{C) O
(D) 1
(E) 2

12. If A = 4x + 3 and B = 3x -· 4, which x results in the smallest value that lA - El can take?
(A) -7
{!3) -1
(C) O
(D) 1
(E) 7

1. e 15(~- b~¡.J. + i..iTJ.:I) = (


8 ¡ix)l5 Multiply both si des of the equation
by 15 to simplify the denominators.
(15 · ~- 15 · 2 -' :f_l + 15 .!.¡:;·~) = (SL¡t'l: · 15) Multiply each tenn by 15 and
simplify.
3x- 5(2.r+ 1) + 4-- x= 8 -7x Distribute -5.
3x- 10x 5 + 4 - x = 8 7x Simplify.
-8x -- l = B -- 7x Add Bx.
--1=B+x Subtract B.
-9 X

2. E 3(a - 1) + 2(2a 3) = 3(2a- 3) + a First distribu te 3 ancl 2 on the left ami


distribute 3 on thc right.
3a -- 3 + -la- 6 = Ga- 9 +a Add ancl subtract like terms.
7a-9=7a 9 Subtract /(1 ancl add 9.
o o
Since you cnded up \\'Ílh a tniL' st<ltcmcnt, thcrc are infinitely many ~olutions. In this
case, the cc¡uation is satisficd foral! n·;¡J \'alucs of {/.

i1i<BOOK 107
~illt'l'
111
:l. e 111
;1· and
' 111
. '1. alre;id\' han· tllt•
. ' .
c.;lllll' dt•Jlolllin;¡tor. add
111
. :1 to hotll
.
-,idt''>.
111
.\dd Illl' fr;wiiull".
111
)
,) 111 :~
111 •· -; h
111 - :1
(i·IIJI-:Il 11111'71 lli-,trillute.
li111 lB · 111 ' -; Sulllr;Jct 111 fnHll hotll -,idt·'·
':i111 1B
5111 :.'':i
111 oc ':i

·L B :l.r 2 ~.: - 1 i\lultiply both sides by 2, and rcmemhcr to re\·crsc thc


2
incquality sign.
(-3.r- 2). -2 :S (-+l. -2
G.r +4s 1 Subtract 4.
6x :s -:l Divide by ti.
.\ -
. < -l2 The greatest intcgcr that satisfics x :s-lis l.
2
5. B Translatc the statements first.
Threc adults with two children paid $127 is :l:\ + 2e = 127.
Three childrcn amltwo adults paid $113 is 2A +:le= 113.
The qucstion is A + e= ?
When a combination of x and y (su eh as x + y) is asked instead of x and y indiviclually.
it is often quicker to gct to the result directly by adding or subtracting.
3A + 2C= 127
+2A+3e=l13
sA +se= 240 Divide both sides by 5.
A+e=·Hl

e . 2n + 7/;
¡)\C\Vl'He 2n 7h · · . .. 2a -
G
). ---~J-· ·as·'¡}· + -/¡', \\' 1l!C 11 Sllllp 111lt'S to /)' + 1.

Sincc O< a< IJ, f~ must be lcss than ; sincc bis greater than o: -Z < l. i\lultipl)' both sidcs

by. 2 to get 2/Jr1 < 2.

lf k= ?:a

-+ 7 aml ?f¡11 < 2, k !JH.Ist be lcss than lJ.

7. D SincP squares or IH\g;:lli\•l) 11l1Il1bcrs tlrC positi\'l\ pick (111 (/anda !.J \\'ith tht greatcst
absolutc \'alue fromcach intcn·al. Sincc -..J < o and (i ·: /1, pick a' -- .¡ ancl/; ·--Gas
limiting \'alm•s.
Your choice for a ami/¡ do 110t lll'cd to be intcgcr~;. Tlwy can he any rcalnumhcrs tliat
are closc to --4 ami ... G to makc o"+ /¡:' both a;. bq't' as possihlc ami an intcger.
( -4)' + ( --())' = l(i : :J(i 52. You kilO\\' that OC. (/e: ' .¡ allCI/¡ ,.:e --(i. Pie k tlw iwxt illlcgn
that is lcss than 52, \\·h:ch is 51.

108 Gi"lXf MATH WORKSOOK


ll. E Group the x's on the left so that the sum of each group is the same for each calculation.
x 2x+3x-4x+5x-Gx+7x-Bx+9x-10xt 11x-12x= -18
-x --x -x --x -x --x = -18
-6x = -18
x=3

9. D Expand the t1rst ami the third fraction by 2 to make the numerators 2 for easy
comparison:
_2__ <" _2_ < ~
113 ·x-6 6
(x- 6) must be a number between 6 and lB. Therc are 11 numbers between 6 ami
18 (lB- 6- l = 11).
lf (x- 6) can take 11 ditTcrent intcger values, then x can also take 11 dilferent integer valucs.

e 2 -4 Cross multiply.
lO. -
x--2=-
- 4
2x
2(4 - 2x) = -4(x- 2) Distribute.
8- 4x = -4x + 8 Add 4x ancl subtract B.
0=0 True statemcnt.
The solution is all real numbers except 2 beca use 2 makes the denominators equal zero.
Denominators cannot be zero sin ce any number divided by zero is undefincd.
Altcrnativcly, check thc denominators of each fraction befare you start. Remember that
x- 2 and 4 - 2x cannot equal zcro beca use dividing by zero is undefined.
x-2*0
x*2
and
4- 2x *O
-2x -Lt *
11. e The easiest way to approach this problem is to try answer choices for x (starting from
choice (E) sin ce yo u are looking for thc grcatcst valuc) and sec which onc results in a y
that is between -1 and 2.
r::
Try x = 2 2(2) + Jy + 1 = o 4 + 3y + 1 =o y=--º-
3
Not in the range

Try x = 1 2(1) + 3y + 1 = o 2 + 3y + l =o y=--ª-=-1


3
Not in thc rangc

Try x =O 2(0) + 3y + 1 =o o+ 3y + 1 =o )' = -l In the r<tngc ./

So x =O.

12. A lA-- Bl "'' 14x + 3 - (3x- 4)1 -~- l'lX +3 - 3x + 41 =-= lx + 71


lx + 71 cannot be ncr,, ti ve. The lcast val u e it can take is O.
I\lakc (x + 7J equal to zero to sol\ e for x.
x+7=0
x= -~-7
SECTION 3.3: EXPONENTS ANO RADICALS
Exponents
• \\'ht'Il a IltJinlwr ~- i~ Inultiplit;tl by iht·lt' ntiilll'~. it i~ l'l'jll'l'~t'llll'tl a~ k'. k is callt>tl tlw
ba~t·. <llld 11 is calktltlw exponen!.
'). '). '). '). ') ')
Loo .... - ........

k·k·k·k--k'

• .rt .\' ,\ll\' llllllllwr to tlll' pmwr ot' 1 l't¡ual~ ihl'il.


4~: .¡~ -¡
í --
.,_ -1

• x~~ . 1 .-\ny nonzcru numlwr to tlw pmn·r o!' O cquals l.


1:l" (J:l"
O" = undcflned
• 1f x"' =, x" tlwn 111 = n except \\·hcn x = -l. O. or l.
If 12'" · 1 = 12". thcn m+ 1 = B ami 111 = 7.
• All po\\'ers ~Jf positi\·c numlwrs are positivc.
7 1 = 343
T = 49
• Evcn powers of negative numbcrs are positive, and odd powers of negative numbcrs
are negative.
(-7) 1 = -343
(-7) 2 = 49

i\1ENORIZE
21 = 2 33 = 27 32 =9
22 = 4 4 3 = 64 4 2
= 16
23 = 8 5 3 = 125 52 = 25
2 4 = 16 62 = 36
2 = 32 5
72 = 49
2 6 = 64 82 = 64
7
2 = 128 9 2 = 81
2 8 = 256 10 2 = 100
1F = 121
12 = 1442

13 2 = 169
14 2 = 196

'------'------------------------------------------------ ------------ - - - - - - - - - - - - - - - - - - -
15 2 = 225

l\iUU'iPUC/HWN Ai'L! DIVISWN


m To multiply •, lwll tlll' bases are thc sanw. add thc cxponcnts.

¡¡¡ To di\·itk \\·hcn tlw bases are the sanw. suhtract tlw cxponcnt oftlw dcno1t1inator
from tlw cxponcnt of tlle nur1wrator.
:~:;: e=. X'' 1•

110 Gi'c1 AT i'V\TH WORKBOOK


Examplcs: 4 2 • 4 12 = 414
5'' -;-5 1 = 52

• To multiply whcn thc cxponents are the same, multiply the bases and kccp thc
exponcnt same.
x" ·ya = (x ·y)''
• To divide whcn tlw exponcnts are the same, divide the bases and keep the
exponent samc.

t:; =(y)"
Examples: 2:~ · x 1 = (2.\'Y
(12)" = (2 · 2 · 3) 2 = 2 2 • 2 2 • 3 2 or 4 2 • 32

~~ = r~r
NEGATIVE EXPONENTS
111 Negative exponents take the reciproca! of thc base ami thcn bccomc positivc.
x-" = (1)"
X
= _l_
X"

(~f" = (J,J"
Examples: s-z = (~r = fz = tr=;
l~r" = r~r = ~~ = ~¡- = 6 t
EXPONENTS OF EXPONENTS
¡¡¡ To take thc cxponcnl of an exponen t. multiply thc cxponents.

Examplcs: (2')' = 2'· 2l· 2' · 2' · 2' = 2 1"


(3 2) 1 3[l

((~ l T H" 1r= ( = ( = -~~ ~~ fi\j

7 2 + 7 1 cf. 7" :'? · T 'F 12·'


7·7+7·7·7·74-7'' 5·5·7·7'*12·12

¡:¡ Thc squarc root of a positivc' numbcr x is thc numhcr that whcn multiplicd by
itsclf cquals x. The squwre root of a
fiG == ¡,¡ :·:r = .r:f' = -l nega::ive numbcr is not
m Equations in the fonn of xc = positiz·e constont havc two solutions (two roots). defined in real numbers.
In other vvords, you
Ifxc=49, thcn x=7 or x= -7 (orx= 2:7).
cannot ta!<e the squarc
¡¡¡ Thc sqtwr(' root of a positin• numhcr is only thc positivc root. root of a negativo
ff5
nr~
= ,¡
,¡, -- 1 l:'~=t~~~--- ----· -~-·
SIMPLIHING H.AmCALS

• lf tlll' 11\lllllwr undl'r tlw -,qu~m· rotll is a multipl<' of a ¡wrlt•ct -;quarv. it canlw simpli-
lil'd. 1 ir..,l fi1Hltlw prilllt' f;¡¡·tori/~llÍilllol tlw llllllllwr li!Hit·r tlw ..,qua!\' root.llwn look
lor p~lil·..,oillll' -,anl\' prillll' llllllllll'r i¡wrfl'l"l'>ljlldiTSí. <IIHI takt• onc oftlH'lll out oftilv
-,qu;·m· root ilwratl't' ,:l · :l - :\1.
\liS

, IWI

ADDITION AND SUBTRACTION 01: RADICALS


ms han• the same numlwr undcr thc square roo!, the1· are considered like
.. ~. Therl'fore, you can add or subtract thcm.

Examplc: 3{5 + 7 .[5 - l3.f5 = (:1 + 7 - I:lh:J' = :l/5


m lftlw terms h:11'1• d:rr.,rent numbers unckr tlw square root, you cannot add or

,[3 does not simplify any furthcr.


First simplify the radicals as much
as you can.
+ l7D- 7..f5- 6[3-::3-.--3
-.[5;- 7[5- 3. GD Add the like tcrms.
17f3'- JB.[i
-4~.) l
'
.. )

MUI;nPUCATWN AND DIVISWN Of RADICAtS


¡¡¡ To multiply t\\'l) radicals, multiply thc m!mlwrs uncler the square root sign.
D' . .¡y = .¡T;_¡,
m To di1·ide (\\'O radicals, divide the 11\llllhcrs undcr tlw square root sign.

-~r == H
Examplcs:

n . r12 = n . n == 1:11j = ()
2[S · . ¡.[7 = B · n5 = Bf:\'5

tScc tilt• "llill<'rt'I\CL' ofT\\·o Squarc•s" on pagc 1~2 fm a quid-.er solution.\

112 Gi"<1AT MATH \.VORK800K


REMEMBER
Although ( -:-4) 2 = 4 = 16, fi6 equals only 4.
2

However, when x 2 = 49, x = 7 or x = -7.

• Hadicals can also be represcnted as fractional exponents (L'xponents in the form


ofa fraction). /.----__1_---------...
l:ri'; = xi; NOTE
The square root of a number
.d ="'X is that number raised to the
~Sl = 5~ 1
power of2.
3} = V3
.¡g = (6)!

COMMON ERROR§
vx' + y' =F x +y
-.J25 + 16 =F ..{25 + {16
(xl _ y6)t = ~(x4 _ y6) =F xz _ y3

SAMPLE PROBLEMS

(S - s-1) . 2-L
a++a-2-3
1
=?

(s -- s- 1) • JLi:JL:_
2 +2 2 3
Work out the cxponcnts first. H.cmember x- 1 = ±-
8 -i- J
(s -- ~). T __ sf To add thc fwctions, first makc the denominators cqual.
---- + ---3
22 2

Add and subtract thc fractions.

Divide thc t'ractions. flip thc fraction in thc dcnominator


ancl multipl)' it by the frartion in the numerator.
7'. 1 + 7'
---=?
7' + 7' . .

Split till' L'\jlllllL'Ilh using tlll' ruk x · ' .r · · .r ·.

htl"lor out-;· fnllll hotiltlll' tlllllll'r;ttur alllltlll' dl'llOlllinatur.

Simplit\ -;·, alllln·plan• 7 '\\·itil ~.


1

l\lakc tlll' dL'nominators cqual in order to add tllc fractions.

_jL=a·Ic~/ Flip tlle fraction in tlw denominator. amlmultiply.


B B
-
1

What is the va fue of x if 8' · 16' + 1


= 4' + 7
?

(A) 1

(8) l2
(C) 2

(D) ~
2
(E) 3

To multiply exponenti;tl functions, tlw bases necd lo be cc¡ual.


B'. · 1G' · 1 = ·f' · ~ Heplan: B ·~ 2 ', 1G = 21, ami ·l = 2~.

Tu find tlw e\pom·nt of anexpom•nt, multiply.


2l' . 2 ,, . 1 ~· 2~' . 11 To mtdtiph·, add the e\poncnts.

If tlw bases are <'qua!. tlw P\ponents must also he equal.

/x+-1 =2x-+ 1-! Suhtract 2x from cach sidc.


S.r .; .¡ ~. ¡.¡ Subtract -1 from cach sidc.

S.r= JO Di\·ide ll\ 5.

x=2

114 GcJ.b,T MATH WOR~(300K


SECTION 3.3-PRACTICE PROBLEMS

l. ~ G4 -.:-::n:; ~ ·~)

(.-\)
(B) .[2
(C) 2
(D) 2/7
(E) lO

2. l l l
x = - (o2J2 Y= -·¡o.3) 3 z= - o.m
Which of the following must be true?
(A) z<y< X
(B) z <X< y
(C) x <y< z
(D) x< z <y
(E) y< x< z

3. vVhich of the following must be positive if zy 1 < O, x 2y 3 > O, and x<lz 2 < O?
(A) x
(B) y
(C) z
(D) xy
(E) zy

4. ( ..f5 - 2) . ( .J5 + 2) = ?
(A) 1
(B) 2
(C) 2.f5
(D) 3
{El 4Is-

5. lf m 1 = G-l, wllich of thc following is nota possiblc val u e of m - k?


(!\) -· -1
(D) 1
{C) G
{D) 12
(E) G:l

(¡\) ~J
el

([)) ~~
ccJ n·
(D) :¡[j
(E) ,¡[:¡
1. If23'·5 1 =25 3' 2
whatisthevalueofS'?
(;\) o
(13)
(C) ')

(D) 25
(E) 125

8. Which of the following is thc best approximation of ~(f.ohGs ?

(A) 81
2
(B) 25
(C) 8
(D) 12.5
(E) 125

9. Which of the following is falsc?


(A) (3')2 = 38
(B) 53 • 5 2 = 5"

(C) r¡r=t
(D) 53 + 54 = 57
7 6
(E) -
7-3 = 7-
3

10. c5-l + 2-IJ-1 =?


(.\) -7
7
CBJ To
(C) LQ
7
(D) 7

(E) 10

(A) m
cm H
(C) :v53
52
(D) 32
(E) ~

1í6 G~iAT 1\lXf;{ 'liORKB00K


12. ?_5~ =?
55 tJx '

(:\) 5
(B) 5'
(Cl 5 1'
(D) 5 1'
(E) 5 11 12,

13. 111 ami k are positi\·e integl~rs, and m* k. If mt = k"', what is the value of m· k?
(A) 4
(B) 6
(C) B
(D) lO
(E) 12

14. 10• 3 is greater than which ofthe following?


(A) 10-1

(Bl _L
l 03
2
(C) lO
(D) _l_
0.1
(El 0.001

15. nr 1 + n· 1 =
5
12-and nr
1
n 1
= T~· vVhat is the val u e of --,~?
(A) 3
5
2
(B) 3

(C) 65
(D) ~
(E) 35

16. fH2t) is hcJW m;~ny times grcater than 125Il25·?


(A) 5
(B) 2.5
(C) 1.25
(l)) 0.05
(E) O.fJ.t
SECTION 3.3-SOLUTIONS

1 ')
~ !: .'\ .¡ S ti -
1 B 9
1

_I)_l __ ~\_j_ B A () e D D D r
l. O lknwmber that ~li-l :l(i t:- H;.t -[~ti. Subtract first aJHI tlwn simplify.

Jii-i -~--:lti ~~~3 ,_ r1 . -: = 2t;.

:>. A .\-
-
= 1
{i.{j:j
1
J' =e - 0.02/ Z =- l
lJ.(f]
Tlw negatÍ\'l' numlwr with the smallest absolute value is thc largest.
Look for the fraction with the smallest absolute value, meaning the fraction vvith the
largest dL'nominator. x is the largest, ami z is the smallest.

3. B zy' <O either z or y is negative.


x'y' >O x' is always positive, so y must be positive. This alw tclls us that in thc
first statement, z must be negative.
x'z 2 <O x is negative since z' is always positivc.

4. A Remember that -15 · -15 = 5. Multiply each term.


(-15 - 2) (.[5 + 2) = 5 + 2-/5- 2-15 4 = 1
Alternatively, use the difference of two squares.
(a - ú) (a + b) = a' - ú 2
(-/5-2) (-{5 + 2) =[5 2 - 2 2 =5-4= 1

5. O Al! possible va!ucs of 111 ami k are:


82 = 64 m-k=6
4] = 64 m-k
2'; = G4 111- k -4
G4 1 == G4 111- k= G3

Thcrcfore, m- kcannot be 12.

G. C Simplify thc first fraction:

/3 .f.~ .f:l
y-r¡~ = 7ft~ = -::¡
Simplify the second fraction:

Acld thc fractions:


'. O First writc each tl'flll as a pmver of 5. For example, replarc 25 by 5 2•

2x +4 = Gx - ·l
B = 4x
x=2

1
8 . 0 O.OOGS . 1
can 1Je wntten as ()j)(j(i5 =
1
-
.
== -
Jo,ooo . ·r o apprtmmatc
. 1o,ooo , rcp 1acc
65 65 65
-10,000
65 by G4 since ..[64 = 8.

rro;ooo = r:I.Qo2 = 10<2


Ví34 f8 2 B
= 12 <;
·v

9. o
(A) This is an exponcnt of an exponent, so multiply.
(B) These have the same bases. To multiply, add the exponents.
2
2) -z
(C) (3 = 322 = 49
(D) 5' +5 1 :¡:. 57

These cannot be addecl without first finding 5 3 ancl5 1•


-G
( E) -7 = 7-s- (-3) = 7-<l
7·-3
These ha ve the same base. To divide, subtract the cxponents.

10 · e e)r·-t + 2 -t l--t -__ (s1 ,-, 2l ¡·-t _


- (51 . 22 + 2l · 5s¡-t --_ (1o
2 5 ¡-t _ ( 7 ¡-t _ lO
+ To - To -- -7-
Note that es-t + 2 1) -t *5 1 + 2 1 • Yo u can clistributc the powcr only vvhcn therc is a
multiplication ora division opcration insiclc thc parenthcscs. For examplc,
es-!· 2 1) 1 =5 1 • 2 1 = 10.

12. A Hcplace 25 by 52 •

13. C The C<l'-'ÍCst \\·ay lwrc is to try nuillbcrs liiltilthc equation is mct: 111' = k"' works only
for·F=2 1
m· k= B
1
1-1. A He\Hilt' lll as l:l'
10 01
r Loll\'l'rl ;di ans\n'r dwices to fractions to see wllich OIH'

is il'ss til<lll / :
11 00

[ \) IIJ 1() :)()() ¡~ ],·~~ thdll 1()(Ji).


1

1 1
11)()() (·qua 1t o IO'oo
ill)
lO

iC) 10 1
lO illl is grt'<ltl'r than 1()()().
1
1

0~ 1
IDJ IOisgreatnth;111 1 .
10 00
1
(E) 0.001 l 0 0<i is cqual to T<fütr

1:J. D Hcplace 111 1


= ¡h and 11 1
= f¡.
Add tlll' two t'qttations sidc by sidc:
1 . ¡__ :i
m'¡¡- TI
..¡_ l - 1- 1
' /Ti IÍ - T2
2 _ G
¡¡¡--u
Gm = 24
111 =4
Plug 111 = 4 into thc first equation to gct n:
l l - 5
4 + ·¡¡- T2
1_5 1_5 3_2
Ti-- L-·:¡--12 T2-T2
l- 1
¡¡- 6
n=G
1/_6_3
m-:¡-z-

1<1. E Thc phrasc "ho\\' 111<111)' times grcatcr" translatcs into division. Di\'idc ['lT25. by
125fí~5.

J:ft25
125.f1;[!í

SECTION 3.4: QUADRA'fiC EQU;'\Tl NS, FUNCTIONS, AND


SYr-1BOUSN

Ouadratk ons
11Quadratic cqu;Hions ha\-c an x' tcrm, \\·!Jich mc;ms thcy are sccond-dcgrcr cquations.
m The stand;trd fonn of a q.t;Jdratic e;;uation is ax' + ln + e O.
a, b, ande are rt•;tiJJttlllhcrs \\ lwrc a-tO.

Examplcs: Jx~· -- 5.r- 90 = O, -·L{' + 3 =O, .r' -t .r'


~
SOLVING QUADRATIC EQUATIONS
NOTE
• To solve a quadratic equation, take the following steps.
Factoring will work for most l
1. l\love all terms to the left side uf the equation to get a zero on the of the equations you will ~
right. encounter on the test. Using
the quadratic formula is your
2. Simplify the left side if necessary by combining like terms.
back-up plan.
3. Try to use factoring to solve the equation as explained below.
4. If the equation does not factor easily, use the quadratic formula.

• To factor a quadratic equation in the form of x 2 + bx + e= O (where a l), follow


these steps.

1. After setting the equation equal to zero, find two numbcrs (n 1 and n) that add to lJ
arid multiply to c.
x 2 + (Sum)x + (Product) = O
2. lfyou can find such numbcrs, factor the equation as (x + n 1) • (x + n) =O.
3. Finally, set each factor equal to zero, such as (x + n 1) = O, (x + n) = O. Then solve
for x.

Example: Solve x 2 - x- 6 = O

First, find two numbers where n 1 • n2 = -6 and n 1 + 11 2 = -l.


n 1 = -3 and n2 = 2 because -3 · 2 = -6 and -3 + 2 = -l.
- x- 6 = (x- 3) · (x + 2) = O.
2
Next, factor x
Finally, solvc for x. Since x- 3 =O and x + 2 =O, x =' 3 and x = -2.

Check: (- 2) 2 - ( -- 2) - 6 = 4 + ¿ - 6 = O .r
(3) 2
- (3) - 6 = 9 3- 6 = o .r

lf the product of two terms is zero, at least one of them has to equal zero.
lf p · q = O then p = O or q = O.

m lf factoring is not possible or ifyou cannot scc it quickly, use thc quaclratic formula.

Exarnplc: x2 - x- G = Ó
n=-c l,IJ= -l,ancl e= -6

x=
l) :+: ff=·n:.--=.-4oTc=&1
------:z~-¡---·---- ------
n-+-:z:r __
_ 1 :2:: --2----- 1 =5
--2·-- NOTE
All Cj\:Jdratic equatlons
--l-+5
-2 -- = 3 ami.1-5
-;r-' == 2. The solutíons sct is S= {3, --:Z}.
can be solved by using the
quadratic formula. In sorne
mi If ~;:•- 'loe is lcss than z.ero, thc quadratic cqtn' ion has no real soJ¡;•ions. cases, applyi. g t: e~ formula
takes a bit longsr, but lt
Example: 3x 2 - 2x + 9 ~' O
a!ways givcs a soiution.
lJ"- 4(/c= (-2f' -- 4(3Wl¡ = ,¡ lOB == -10·1 <O
• lf /1· ·lile i:-equal tu ;eru, tlll' quadratic t>quation ha~ a :-ingle solution. This is
actual!\· a doulllt· root 11IH'n' ti\'() -,ulutions arL' l'ljll<tl to ead1 other.

Ex a m p le: .\ {l.\ . ') ()

/1' ·luc 1 ti)'· -lill(~ll :llí· :\tí O

• lf /J· ·111!' j;, gn·atvr than zem. tlw quadratic equation has t11·o distinct real solutions.

Exampil': x' tü ~l O
/1' ·ll/C ' tí) . .¡ ( l )( ~)) , :Hí ~ :lh 72

DII:FEHENCE OF TWO SQUAHES

• Thc ditlen•nn• of 1\I'O squares can be factored as follows:

a~ /;·' = (a- /J) · (a+ IJ)

Examplcs: .r' -- 49 = (X - 7) · (.r + 7)


ltü' -- 9/J: = (4.r- 31J) · (4x + 3b)

PEHFECT SQUAB.ES
• Thc squarc of a binomial in thc form of (a+ !J) can be quickly determined with
"lir::,t tcrm' + (2 x first tcrm x second term) + second term'."

Examplc: (3x + 4/J)·' = (3x)" + 2 · 3x · 4/J + (4/J)" = 9x 2 + 24xh + 16//


m The squarc of a binomial in th" form of (a- b) can be quickly detcrmineJ with
"first teiw'- i.2 X tlrst tcrm '< second tcrm) + sccond tcrm'."

Examplc: (5t- 4/Jf "" (5f)'- 2 ·51· 4/J + (4lJF 251 2 - 40tlJ + 1Gb"

FACTORS AND HOOTS Of QUADHATIC EQ'Ui\TIONS

a Jf (.r- k) is a factor of rn.:~ ·+ In+ c. tlwn ax" + ln + e is divisible by (x k). In other


words. thcrc's anotlwr hinollli<d in thc form of (ox 111) where (x -· k)· (ax- m) =
axc + In i c.

Examplc: (x- G)is a factor of x'- 4x- 12.


So thcrc is <liHJth('r binomial in thc form (x- f..:) wherc
x'- 4x- 12 = (.r- tí)l.r- /.:).
In this case. k~-· 2 beca use ( -6) · (·-·k) must equal -12.

111 If (x- k) is a factor of ox' -t In+ c. thcn substituting k makcs bot!J cxprcssions zcro.

Examplc: (.r '1) is a factor of x' + 2x- 24.

So if yo u makc x- ·1 ~ O and solw for x. x = 4 is a root of x' + 2x- 24.


(:\ ro<•' is a solution 11·hen a qt1<1dratic cquation is sct cqual to zcrn.)
x: + 2x- 24 •-• (·1)'; L(l)- 24 ,,. JG +S-· 24 =O
Functions
• A function can be thought of asan equation ora rule that tells yo u how to associate
the dements in one set (the domain) with the elements in another set (the rangt!).
A function transforms one number into another.

• Functions are usually represcntcd by a letter followed by the variable used in the
expression.

Examples: f(x) = 4x 2 +3 g(t) = tl- 3t+ 2

1~ WHAT IS f(x)?
N ame of Name of f(x) does not mean ftimes x.
The function the variable lt means that the name of the
function is f and its variable
is x.

11 /(3) indicatcs that 3 should be plugged in for each variable. Ifj'(x) = 4x 2 + x, thcn
/(3) = 4(3)2 +3 = 3G +3= 39.

11 Thc domain of a function is thc sct of all thc val u es for which the function is defincd.
Thc rangc of a function is the sct of al! val u es that are the output, or result, of applying
thc function.

Examplc: y= x 2 - 4 is a function. Ifyou input x = 3, you get an output of y= 5.

Symbolism
m Qucstions with symbols define sr ::cific rules and use arbitrary symbols to define certain
opcrations. Since these are nonstandard functions, each qucstion starts by clefining the
symbol.

Example: 1f @x = 5x- 3, what is thc val u e of @7?


07 ~~ 5 . (7) - 3 = 32

Exampl" w
\]7 is dc>finrd '" x' · t -- y· '- \Vlwt ;qhcvahw nf (ID
l -'
1

This funclion tclls you to take thc u¡,pn ldtnumber, square il, ami mulliply by thc
l<mcr righl numller. Thcn subtrcrct thc product of uppcr right am!lO\\·cr left numbers.

4 2 • 3 - 7 · l = 4B - 7 = 41
Sequences
• i\ sequenn· is a st•rit•s of ord!'red nurnlwrs following a rult•. :2. ,1. B. 1!i. :c. tí·l. ... is a
sequt·m·c. Lach lll!Illlll'r in tiH· Sl't¡lll'llC!' is called a ll'nn of tlw sequt·nn·. lile first
Illlmlwr i~ tlw fir-.t tt•nnlti 1J. tlw second numlll'r is the second tt'l"lll (u). and so 011.

Tht• gt'IH'ral term of a Sl't¡lll'llce i'i 11".

• Sequenn·s are represcnted eitlll'r b\· a few of their tt•rms or by tlw rule describing
t lll' terms.

Example: a". 1 (a,y + :lmcans each term is found by squaring the pre\"ious tenn
ami adding ]. hll" example. if (/1 oc· 4, (/é ~ (ar + 3 = (4)·' + :1 = ¡~¡
a, e= (aY + 3 = ( 19)' + 3 = :l!·d

ARITHMETIC SEQUENCES

• In an arithmetic sequence, each successivc number differs by a fixed amount from thc
previous number. Thc difference of consecutive terms is constan t.

Examples: 72, 78, 84, 90, ... 78 - 72 = 6, so 6 is the common difference (d)
2, -4, -10, --16, ... d = -4-2 = -6

a The nth term of an arithmetic sequence can be found by a" = a 1 + (n - 1) · d, where d


is the common difference a!l(l a 1 is the first term.

Example: What is thc 50th ~crm in the series 5, 9, 13, 17, ... "?
NOTE
d=9-5=4
Consecutive numbers form an
arithmetic sequence with a a,,0 = ({ 1 +50- 1 · d = 5 + (50- 1) · 4 = 201
common diffc:rence of l.
• lhe su m of n te:-ms ol :m arithmctic sequencc is half the numbcr of
Consecutive even and terms times the su m of the first ter m and thc last tcrm.
consecutive odd numbers each
form an arithmetic sequence Sum of 11 tenns = ~- (a 1 + a,)
with a common difference of 2.
Examplc: What is the sum of alleven numbcrs bctwccn 4 and 62,
inclusive?
Thcrc are :·HJtcrms bctl\"l'Cll 4 a!J(I6:2 including G2 and -1 ( 62-; 1+ 1 = 30 J·\ So 11 = 30.

Sum = ~(4 + 62) = 990

GEOi\IETRIC SEQ' ENCES

m In a gcomctric scqucncc, each succl'Ssivc numbcr is a fixed m u! tiple of thc prcvious


numbcr. Thc ratio of consccuti1·c terms is constan t.
")1
Examplcs: 7, 21, G3, 189, ... ~- = 3, so 3 is thc common ratio (r)
1

l 1 r= 3=1
9, 3, l.}' 9' ... 9 3
a Tlw nth ter mofan gt>omctríc scqucncc can be found by a,"'" a 1 • r" 1 11·here r is the
common ratio and a 1 is thc first tl'rm.

Fxample: What is the l2tll tnm in thc series 5, 15, 75, ... '?

124 GMAT PL\fH I.NORXBOOK


UEPI~, TING SEQtH:NCES
• In a rcpcating scqucncc, a fixed set of numbcrs rcpeats, such as
2, :l, l, -1, 2, 3, l, -1, 2, :l, ...

Example: In t!Jc repe<lting scqucnce below, \\hat number is in the 953rd place'?
5, 2, -2, -3, 1, 5, 2, -2, -3, 1, 5, 2, -2, ....

5 numbers are repeating. Di\'ide %3 by 5 ami find thc rcmaindcr, wbich is 3.


The 3rd number in the scquencc is also in the 953rd place: -2.

Si\l\lPLE PHOBLEl\IS

r---------------------------~~~r~~~~-,~·:-~·L~-~~~[t-------------------------------l

lf (X - 7) is a factor of x 2 + tx - 14, what is the value of t?

(A) -5 (8) -3 (C) 1 (D) 3 (E) S

Solve x - 7 = O anci find its root: x 7. So x 7 is also a root of x 2 + tx - 14


(i.e., makcs it zcro).

72 + t(7) - 14 = o Multiply.

49 + 7t- 14 =o Acid the like terms.

7t + 35 =o Subtract 35 from both sides.

7t = -35 Divide both sides by 7.

The answcr is (A).

lf x = 0.35, 0.2x 2 - Sx + 0.7 is how much greater than 0.2x 2 - 7x - 0.3?

(A) O (8) 0.4 (C) 1 (D) 1.7 (E) 2.7

·---------- - ----·---·-------- --------- ---------------------···------------------------------.J

SincP tlw qucstion is "how much grcatcr," subtract thc sccond polynomial from thc first onc.
Carry out thc suhtraction lwforc plugging in.

0.2x 2 - 5x + 0.7 -- (0.2x'- 7x- 0.3) Distributc thc ncgatiw· sign.

0.2x'' -- 5.r + 0.7 O.::'x' + 7x + 0.3 Group tlw JikP tcrms.

0.2x 2 -- 0.2x' -· 5x + 7x ·1 0.7 + O.:l i\dd ancl subtract Jikc tcrms.

2x +l Plug in O.:r> for X.

2(0.?15) + l ~ l.-:

Tlw <111S\\'l~r is (D).


~~---------------q -•U:tWA!t»--
1 ! lf 4a 2 - b 2 = 27 and 2a - b = 3, what is the value of 2a + b + 1?

~~ (A) 3
(8) 4
(C) 9
(O) 10
(E) lt cannot be determined from the information given

L
Whenever yo u see a difference of two squares in a problem, yo u should factor it.
NOTE
4a"- IJ:' = (2a- b) · (2a + ú) = 27 Since you know that 2a- lJ '"" 3, plug that in.
Whenever
you see a 3 · (2a + ú) = 27 Divide both sides by 3.
dífference of
two squures (2a + b) = 9 Add l to both sides.
in a problem,
2a +b+ 1= 9 +1
factor it!
2a + b + 1 =lO
Thc answcr is (0).

lf a (cj> b = 2a 2 +banda# b =a · b\ what is the valuc of 3 @', (5 # 2)?

(A) 118
(8) 56
(C) 38
(0) 28
(E) 14

"----------- - - - - - - - - - - - - - - - - - - - -

[valtwte tllc contcnts of the parentheses first. Sin ce a !f b ~ a· !J 2 , 5!! 2 = 5 · 2 2 = 5 · 4 = 20.

Tlw qucstion reduces to 3@ 20. Since a@ lJ = 2a 2 + IJ, 3@ 20 = 2 · 3 2 + 20= 2 · 9 + 20 ~, 313.

The answer is (C).

l. x = ~- is onc of thc solutions of Gx 2 + mx- 4 = --2. What is thc' val u e of 111?


(A) -1
(B) o
(e:) J
2
([)) l
(E) 2
·¡ ti4sz - l =1
'"'· Bs + l ·
(:\) -8s
(B) Bs- l
(C) 1 - Bs
(D) Bs + l
(E) 8s

3. x 2 + x 5a + 3 = O. a is one of the roots of the cquation. a could be which of the


following?
(A) -2
(B) -3
(C) O
(D) 2
(E) 3

4. If x · (x- 12) = -35, what is the difference of the values of x that satisfy the equation?
(A) 2
(B) 5
(C) 7
(D) 12
(E) 22

fA) 9
(B) 3
(C) 1

(D) 31
(E) l
9

G. For what intcger \'alue of x is 2x 2 5x = -3?

(A) ;}_
2
(B) 1
(C) O
(D) -1
(E) -2

(A) 2B9
(B) 187
(C) Ei3
(D) 17
(E) 9
1
B. lf x 2 + mx 15 is divisible by (x- 5), what is the value of m?
(:\) 5
')
(B)
(C) -2
(D) -3
(E) -5

9. \ Vhich one of the following is a root of 2~" - -i~x: + ±= O?

(:\) -2l
(B) 2l
(C) 23
(D) 2
(E) 3

10. If 4c 2 - d" = 19 and 2c + d = 19, what is the value of d + e?


(A) 1
(B) 5
(CJ 9
(DJ 14
(E) 20

11. What is tbc product of tvm numbcrs that sadsfy tbe equation ~x + 3 - x = 3?
(A) -6
(B) -2
(C) O
(D) 3
(E) 6

12. If eme of tlw roots of x 2 - (2m- l)x +3m+ 2 =O is equal to 1, what is thc othcr root?
(:\) lO
(B) 4
(C) -1
(D) -10
(E) -13

13. 1f x- 2y = 8 and xy = 5, what is the val u e of x:• + 4i''?


(A) 4,~

(B) 54
(C) 61
(D) 14
(E) 8.1

123 GMAT r:ATH WORJ<3;:10K


14. \Vhat is the val u e of 52' - 4W - l ~F "?
(.\) 33
(BJ 35
(C) 37
(D) 39
(EJ 41

15. lf x 2 + Jl, = 31, what is the val u e of x- ;_\3.?


x- .

(A) 36
(B) 25
(C) 9
(D) 5
(E) 3

16. For any integcr greatcr than 1, «11» is dct1ned as thc remainder when 11 is divided by 5.
If <<11>> = 3, what is the valuc of <<311>>?
(Al
(B) 2
(C) 3
(D) 4
(E) 9

Answcr qucstions 17 and 18 based on the figure bclow.

~
For any integcr, ~ = ox + ay+ az.

~
l..

A
(A) 18
7
- .?
-

(13) 51
(C) 152
(D) 213
(E) 630
B. C If x 2 + mx 15 is divisible by (x 5), thcy havc thc samc roo t.
Set x- 5 = O and find its root, x = 5. Substitutc 5 for x in thc first equation to find m. j'

52 + Sm- 15"" O
25 + 5/ll- 15 = ()
5171 + JO= o
5m = -10
m= -2
Altcrnatively. you can plug in thc answcr choices for this problem.

9. D This is one ofthc problems wherc you can plug in thc answer choices. Try each one
until you find a number that works. The challenge is that the answer choiccs are frac-
tions. Plugging in might be very time consuming. To solve it algebraically, multiply
both sides of the equation by 4x 2, thcreby simplifying thc dcnominators. 4x 2 is thc lcast
common m u! tiple of al! denonünators.

4x· 2 (-1__ l + l)
2x 2 4x 4
= (0)4r 2
·
Multiply and simplify.

(4x 2 •
2 ~2 - 4x
2
• ±)
ix + .4x 2 • = O
2- 3x + x 2 =O Rearrangc into standard form.
x2 - 3x + 2 =O Factor (sum = -3, product = 2).
(x- 2)(x - 1) = O
x-2=0 x-1=0
x=2 x=1

10. D First factor 4c 2 - d 2 as (2c- d)(2c + d).


(2c- d)(2c + d) = EJ You are given 2c + d = 19.
(2c- d) · 19 = 19
2c- d = 1

Solvc the cquations by e!imination:


2e d= 1
+ 2c + d = 19
------
4e = 20
e= 5
Plug e= 5 into one of thc cquations to find d.
2c +d 19 =
2. 5 + d = 19
10 + d = 19
d=9
d+e=9+5=1-l

132 Gi'•1AT r•l;YfH VIORKBOOX


ll. E {i-+-3-x=3 Add x to both sides so that thc square root is isolatcd.
lxT3=3+x Squarc both sidcs.
(¡:¡-:;:::s r (3 + xr
'-=

x + 3 = 9 + Gx + x 2 Subtract x ami 3
O= G + Sx + x 2
Hc\\Titc.
x 2 + Sx +()=O Factor (sum = 5, product = 6).
(x + 2)(x + 3) = O
x= -2 x= -3 Sincc any numbcr under a squarc root must be
positivc or zero, make surc that you check the zeros
you t1nd by plugging them into thc original cquation.
In this case, both -2 and -3 work.
Thcrcforc, ( -2)( -3)=6

12. D Find the val u e of m by substituting l for x.


F - (2m- l) · 1 + 3m + 2 = O
1 - 2/ll + 1 + 3117 + 2 = o
111+4=0 111 = -4
Therefore, the original equation is:
x2 - (2(-4) -1)x + 3(-4) + 2 =O
x 2 + 9x 10 =O Factor (sum = 9, product = -10).
(X+ lO)(x- 1) = 0
X+ 10 = () X- 1= 0
x=-10 x=l

13. E The question is asking for the Sl!ll1 vf the squarcs of two tcrms: x and 2y.
It is possiblc ancl quickcr to get to x 2 + 4y 2 without t1nding x and y separately. Square
both sidcs of x- 2y = 8.
(x- 2y)2 = 64
x 2
- 4xy + 4y 2 = G'l x 2 + 4y 2 appcars on thc lcft.
x2 -!- 4y 2 -- 4xy = 64 You are givcn that xy = 5, so 4xy ~~ 20.
x + 4y~ - 20
2 = G4
x:~ + 4Y:' = B4

l-1. D You can squarc each tcnn and subti·act Alternativcly, for the first part, 52 2 - 1íF, use
1

tlw dilTerencc of two squares.


52:~ - 48' = csz + 4B)(52 - 4fll = 100 · 4 = 400
So thc qucstion reduces lo -100- 19 2 = 4r'O -- 3G1 = 39.
Or, ,¡oo -- 19' = 20' -- JCF = (20- 19)(20 + 19) = 1 · 3:J = 39
13. D Notice that if you square x- i.
you get v0ry similar terms to the first expression.
Yo u can find the square of the original expression and take the square root at the en d.

(·r - 1)2
x ""' ·r" G+ _9_
x" =
·rz + ~-
x2 - 6 .t' + _;2 = 31 is given.

(x- x!3'2 = Jl
.
- b
'
25

Y-1=~r:)
·' X -·

16. D •<11» = 3 means when n is divided by 5, the remainder is 3. Pick a number for n that
works for this case. For example, whcn 8 is divided by 5, thc rcminder is 3. Usen = 8
to answer thc qucstion. 3n = 3 · 8 = 24. When 24 is divided by 5, the rcmainder is 4.

17. B ax + ay= az can be written as a(x + y+ z) by using factoring.


a(x +y+ z) = 3(5 + 6 + 7) = 3 · 18 = 54

18. C If a· (49 + 90 + 48) = O then a must equal zero becausc the value inside the
parentlwses is not zero.

19. B Start by calculating the parentheses (9 \' 7) = 92 - 72 = (9 - 7)(9 + 7) = 2 · 16 = 32.


The question reduces to 32 Í 64 J- = ~~: = ~~ : i-% = t :t i- =

20. B If33s = 18, then al! v's get rcplacecl by 3. 33s = 32 - 2 · 3 + 3s = 18.
9 6 + 3s = 18
3 + 3s = 18
3s = 15
le S
r~

-+' 4.1 TRANS!..ATION FROi'<l WORDS TO EOUAilONS ,<\ND 1

BAS!C WOI~O PROBLEt<1S


-? 4.2 Rt\TE PROBLEMS

-> 4.4 MIXTURE PROBLEIV1S


-+ 4.5 INVESTMENT/INTEREST PROBLEMS
-+ 4.6 SET PROBLEMS ANO VENN D!,'\GRf\í-1S

S~c·noN 4.1: Tf.V~~~SLAT~ON !f;RCM '\;VO~U)S TO EQUi!·'l>TIONS


AND B.ASiC 'IVOr'tD PROBt~MS
Many students find it hard to comprehcnd or intcrpret word problcms. Luckily, you can
use a too! to unlock most word problems v\·ithout memorizing specific formulas or solution
methods for each type-of word problem. Students who prefer the latter method usual! y get
discouraged easily or baffled quickly when they encounter a slight change in wording or a
derivative of a standard problcm.
The mcthod cxplained hcrc is commonly callecl translation. We will start with ccrtain
ground rules to translate or convcrt vvord problems into algcbraic equations that can
be solved easily. If you have skippecl the prcvious chapter, which revievvs nccessary
algebra concepts, go back now and revicw that chaptcr. Focus especially on the sections
in which wc discuss linear equations with one unknown and linear cquations with two
unknowns.

Lct's revicw the fundamental translation first. This is where mo"t students have troublc.
Wc will put it all togethcr at the end of this section. Take a look at tilc following examplcs.

Sixtcen more than a numbcr Eight more thzm fivc times a numbcr
'--~'--~'---y-__J '-y-·_)'--.-----''------~----..-/

16 + X= 16 +X 8 + 5·n= 8 + 5n

________________
Eigl :1 lcss than six times a nurnber
-............__
'-·,--''------.----__)'-----v-----'

~'-...
G· 11 8 = Gn - 8

. ·---·---··-----·---::_ "
:\oiÍlT 111at 'li'S'- lil<lll ..... ,\ ilt llr.·~ liH· pt>~ÍiÍ<Hl uf tÍur lllllnlwiS in 1lw lr;uhlali<nl. \\'llcn \\"l'

'><!\ ··t·igll! !t·~ ... tlliill ;¡ llllliliH'J ." ,,,. 1111'<111 /i ¡, ... uiltractt·d i'roiél ;¡ llillllh't or til<Jl íllllliht•r is

!Ít't'l'l'il'·l'd ll\ 1\.

~.;, jil'i'<'t'!l! o! si'l''t'll

¡, li
¡¡; í ),()(; . 1¡;
1í ii) liill

IIH· lollm\ÍJJg ¡;¡!Jit' \iilll:t•!¡; \i>ll ll'illl,i;lll' l'l'rlilÍll •'\pir,:.iuth Í!liu :,\IJJI)(lb/o¡H'l'<ttioiJS.
1\\ \li!lchillg lor iht•st• '.\{)rd" Í!l '.V>rd prtl:,¡ ... ,ns. \Olí t :u1 tt'i!tLi<tlt' liH·tJI into corn•sponding
pan-. oln¡u;ttiolts.

Bob's age is: 8 =


A number, which, what n, x, t. . . a number is the sarne as: n =
what is: x =
ls, equals, was, has, will be, Two times a number is 6: 2 · n =6
=
costs, the result is, is the same os Jean's age was: J =
More than, plus, added to, Five more than a number: x +5
+
combined, total

11 less than a number: n - 11


7 is decreased by a number: 7 - a
Less than, minus, subtracted
The difference between Ali's and
from, decreased by, difference,
Ken's ages: A - K
the increase from
Five years ago, Jim's age:j- 5
The increase from ten to T: T- 10

21 o f a number:
1
2 ·m or 0.5 · m.
Of, product of, times,
The product of a numb0r and 9:
mu 1'iplied by
k· 9 or 9k

The ratio of 5 to 6: -~
Divided by, ratio of, pcr, out of 1 or +
A number is divided by 12:
1 1 or n + 12
12
+100 12 percent: lOO
Percent, v ···at percent X
Too What percent of 60 is 15?: lb6 · 60 = 15
n, n + 1, The surn of three consecutive integers
Consecutive integcrs
n + 2, . . . equals: x + (x + 1) + (x + 2) =
Consecutive odd i;~ tegers, n, n + 2, The surn of two consecutive odd intcgers
consecuéive even integers n + 4, . . . equals: x + (x + 2) =
1 A number plus twice its reciproca!:
Reciproca! of a num~·er 1
d d+ 2. d
5 more th2:1 the opposite of a
Opposi'..? of a number -n
numbcr: 5 + ( -n) or 5 -- n
Qll!CK PR\CTICE

Translate cach of the following into an algcbraic form. Do not solve.

1. Five more than the proúuct of x ami y


2. :2:i is n lcss than 3G
3. The ratio of x to y is thc same as the ratio of z to 5
4. Kim's age 5 years frotn now
5. The sum of thrcc consccutive even intcgers
6. The su m of twice a numher and tbrcc times a number
, . Twicc tbe sum of a number ami thrce times tbe number

¡\NS\\'EHS TO QUICK PllACTICE

l. 5 + xy
2. 25 = 3G- 11

3.
x_z
y-~

4. K+5
5. n + (n + 2) + (n + 4)
6. 2x+ 3x
7. 2 ·(m+ 3m)

Now let's proceeú wilh translating full sentcnces, which produces equations.

Five times a numbcr is twelve more than the numbcr


~~'----~-...--.-~'-~'~

5 11 12 + n 5n = 12 + n

Since the second parl states "thc number," it rcfers to thc samc numbcr c!efincd in thc first
part of thc statement. Therefore, \Ve use the same lettcr (n) for both numbcrs.

\Nhat numbcr equals twcnty-two more than three times itself?

lil 22 + 3 /ll m= 22 + 3m

The sum of lV\'O consccutivc int''gers is 65. 11 + 11 + 1 = 65


3x _ 7
The ratio of 3x to 5y is equal to thc ratio of 7 to 12. 5)1 --u
1 \Vhat numbcr equals sixtccn \ess than twice itsclf? X= 2x- lG
'1¡ Scn~nty-fivc clividcd by a number is thrcc times the numbcr. L'i = 3k
k
Thc sum of two consccutive cvci1 numbers is 30. z + z + 2 == 30

The kngth of a rect:mglc is twicc its \\idth. 1= 211'

A trianglc with sidcs n, n + 2, ancl 22 has a pcrimctcr of 11·1 (1 ·f {/ + 2 + 22 = 4rJ


Jason is eleven years oldcr than Karen. J=ll+K

Fivc years ago, Jason was twice asoldas Karen J -· 5 = 2 · lK - 5)

In the last word pmblem. 5 years agu hoth Jason and Karen \\ere 5 year<:; youngcr than
they are !lO\'.'. So \\e use(/-- 5) and \K .... 3) fnr lhcir current ages.

When time changes in age problerns, everybody is affected. Two years ago means
(current age ·- 2) for each person.

Two years frnm nmv, Tom will be lhrec times asoldas his brotlwr T + 2 = 3 · (B + 2)

After kll gives $30 to Kim, he will han~ half Lhe moncy Kim has. J 30 =~(K+ 301

------------------------------

When two parties exchange objects (or money), subtract the amount from tlle giver and
add the same amount to the receíver.

\fany word problcms will reveal thcmsclves easily aftcr a straightfor.· .r,l translation. We
will cuver such problcms in this scction and rcvicw other word problems tl1at requirc simple
formulas in the following sections of this chapter.

STEP-BY-Srm~ PL\N FOR SOlLVl!NG WDRD PJROBLEMS

~,~l
@F_D Head thc question vcry carefully to understand thc problem in its cntircty.
@E_::>J) Identify thc unknown quantities, and assign variables (a onc-lctter "name") to
1 ::; 1
' --:: 1 cach quantily. Express ea eh quanliiy/unknown in tcrms ofonc variable ifpossible.
Ji
Kyle's age: J(
Bcn is lwice asoldas Kyle: 2K
r!C time Al len takcs to unish thc las k: f Yc 1 s~ - _:k~ narr~c :;1e vari~1blcs using
:'.,,; lnra finishes the same task in ez;c.::y iJ•:ntHiable !eUers (likc the
fkst letter of the unknown) rather
2 hours more than i\llen: t +2
\_ than a!w~ys ca!ling them x or ~--
(ir_§P 3) Translatc the sen lcnces, wh!ch are
esscntially algcbr<Jic equations, inlo
V
u their m<Jthcmatical equívalcnts.
<11
.e
u @_Zf:D Solvc the algebraic cquations to find thc quantitics you identificd in Step 2.
0
@iCP 5) Go back to tlw question, read thc last st1temcnt and makc sure yo u amwer thc
1 ~ right qnestion. rvlakc sure your answcr makcs scnsc and that it is rcJsorEtblc

1
L~ bascd \ n tite filcts presented in thc qucstion.
SAl\lPLE PHOBLEMS

Find the largest of three consecutive integers whose surn is 84.

\Ve can start by translating parts of the question.

Let"s name the tlm~e consccuti\e integers as 11, (11 + J l. ami (n +- 2).

The su m of tluee consecutive integers beconws 11 + ( 11 + 1l + (n + 2).

\Ve complete tlle translatiun as 11 + (11 +- 1) + (11 + 2) = 84.

]n+3==B1
3n = Bl
ll = 27
11 +l = 28
n+2 = 29

The largcst consecutive integcr is 29.

Always go back to the question and find out what it is asking once you've sol ved Jor
tlle unknown.

Alternativcly, questions about thc sum of consecutive integers or consecutive oclcl/even


integers can also be solved using the following shortcut.

If the su mofa set of integcrs is given, divide the su m by thc numbcr of integt~rs. The rcsult is
alvvays thc micldle integcr.
In thc examplc above, ~l = 28 is the middlc numbcr. So tlw Jargest cons~~cutive intcger is 29.

iil If an even number of irttegcrs is g;ven, dividing the sum by thc nurnhcr of integcrs will
give you a num!,er between thc two middle numbers.

For example, let's say the su m of six consecutive evcn integers is 102.
Divide 102 bv 6, JtlQ 2 = 17.
. )

This means the midclle t\vo cven numbers are lG ami lB. So thc consecuti\·e even intcgcrs are
] :>, 1!1, lG, 18, 20, a:,cl22.
Katherine is 4 years o!dcr than Julia. 2 years from now, Julia's age will be ~ of
Katherine's age. How old was Katherine 5 years ;:¡go·'

lcd~!\": /\ .¡

l<atherine Ju!ía
Toda y J+4 J

2 years from now J+6 J+2

') ')
J -!· 2 ~- ~ ( J ; ()) )uiia's agl' willllc;; of 1\.a!l¡crine's age in 2 ycars.
• ) l ~
,)

i\lultiply both sides by J.


3 . (} +2) 2 . ( J + (j) Distributc.
3l+G=-:U+ 12 Hearrangc the terms.
J= (j Julia is G years olcl today
K=]+4 Katlwrine is 4 ycars oldcr tllan Julia
K=G+4=10 Kathcrine is lO years olct toclay.

Kathcrine was 10 - 5 = 5 years old 5 years ago.

Sophie has twice as much money as Nate does. After Sophie gives $5 to Nate, she
will have 1.5 times as much as Nate does. How much momcy did Nate hi'JVe originally?

! et Sophie's age be S and Natc's age be N

Chigin<11ly: S= 2 ·N

Befo re 2N N

Aftcr 2N- 5 N+ 5

2N- 5 = 1.5 · (N+ 5) Sophic's moncy is 1.5 times as muchas Nate's.


2N- 5 = l.5N + 7.5 Distribute 1.5.
0.5/V= 12.5 Hcarrange the terms.
·N= S25 Multiply both sidcs by 2.
N ate has $25 today.
~3-
1-=
1
tdaogle'• ,;de• ace x, x + 3, and x + 5. lf ;" pec;mete' equalo 59, how long;,""
longest side?
[___________________ _
Translatc thc "sum of" into addition.

X+ (X L :l) t- (X+ 5) = 59

3x + B =59
Jx =51
X= 17
X+ 3 = 20
X+ 5 = 22

Hovvevcr, thc answer is not 17. Take a moment ro go back ro thc question ami doublc-chcck
wbat is being askcd.

The sidcs are 17, 20, and 22.


The longest side is 22.

l. Sebastian was 6 ycars old 5 years ago. How old will he be 12 years from now?
(A) 13
(B) 17
(C) 18
(D) 23
{E) 28

2. Hannah is four years oldcr than her sister. Two years ago, he1 sistcr's age was l of
Hanrwh's age. How old will hcr sistcr be in 5 years?
CAl 14
(B) 18
(C) El
(D) 21
(E) 23

3. Sonny is 5 times asoldas .l\Iichacl. Thrcc ycars from now, he will be t\vice asoldas
· I'vlichacl. How old is Michacl'?
(¡\) 1
(Bl 4
(C) 5
(D) 9
(E) 11
·1. \\\·ndy is Hi yt>aro.; oidl'r tilan lwr i>rotilt•r ()lll'lllÍil. Thrt•t• n·aro.; lro!n llll\\', siH• \iillllt'
3 tinH•s as old ¡¡<; lll'r hrotlll'l. lltl\\ old i-; ()lll'IHÍn·i
l \ i :_:¡
1111 :.!1
((¡ B

!lll :>
11 i :l

:J. Thl' tot~d prin· of ;¡ p;tir of s!tot•c, anr.l ~~ '>1\'t'~lll'r is S:lli:J. \\'ht•n luli:tl'\l'ilangl'll ti!l'
S\\ l';ttl'l' for :1 Ch\':t¡Jl'l' lllll'. till' lll'\\ total prin• lll'l'<lllll' S:.!~J:l. lf tlll' Jli'Íl'(' of tiH' !ir-.;t
S\\l'illl'l' ÍS 1,() lÍilll'S tlll' Jli'ÍL'l' of tilt• StTOlHI S\\l':lll'l', \\!Jat Ís the jli'ÍlT of the sil!Jes'(
(:\) s-·¡
... 1;..... ¡
1
(B) Sl:.!O
1
(CJ Sl7:1
(U) $192
1
(El $2,15
1
G. 5 roommates ~;han• their rent equally. lf one of the roommates moves out, they would
each have to pay $21:i0 more. How much is the total rent?
(A)$1,300
(B) $2,340
(C) $2,GOO
(D) $3,120
(E) $5,200

7. Joe and Tricia are waiting in a line. Tricia is ahead of Joe, ami there are 5 peoplc
between them. If the sum of the number of pcople in front ofTricia ancl the total
numbcr of people in front of Joe is 22, how many people are in front ofTricia?
(A) G
(B) 7
(C) 8
(D) 9
(EJ 10

i
8. A bicycle's front tire has a diameter that is of the cliameter of the back tire. If the back
tire rnakes 90 revolutions, how many rcvolutions does the front tire make?
(A) 10
(B) 30
(C) 90
(D) 270
(E) 810

9. Thc largest of the 4 consccútivc cven inkgcrs is 150'~(, of the smallest. What is the
average of the two middle integcrs?
(¡\) ll
(B) 12
(C) 13
(D) 14
(E) 15
1O. The square of an even mnnbcr equals 6less than 5 times the number. \v1lich of the
following is the square of the number?
(,\) 2
(B) 3
lC) .J
(!)) 6
tE) 9

t 1. Thc su m of tlw smallest ami thc largest of 7 consccuti\·c integers is 12B. What is the
aYerage of the 7 inll'gcrs·?

(:\) ()()
(B) 61
(C) 62
(D) G:1
tE) 64

12. If the dillerencc oftwo positivc intcgers is 4 ami the difference of their squares is 80,
what is the su m of tlwir squares?
(A) 8
(B) 12
(C) 20
(D) 144
(E) 20B

13. :\tan k contains a certain amount of water. If x gallons of water are adcled, only! of the
tank woulcl be ernpty. If x gallons are removed, only {- of the tan k is full. What fraction
of the tan k is full?

(A) l
8
(I3) l
4
(C) ~-

(D)!

(E) ;3
4

14. The cost of renting a villa in Puerto Vallarta is $P, whi( h is to be sharecl equally among
B pcople. If two more people are invitcd tojo in, how m u eh less vvill ea eh of the original
8 pcoplc pay in tcrms of P?
p
(A)
·s
p
cm To
p
(C) !m
.,, p
(rJ) 4ü
' p
CLJ 2o
SECTION 4.1-SOLUTIONS

f-+-W}, s ¡ ! TB_:Lti_I<l~t
1 •
~Le A!lltc¡E\c¡n~lc¡!U_~nln
1 6 1 12¡13 . 14

l. D lf Sebastian \\·as G years old 5 years ago, he is 11 now.


12 years from now he will be ll +- 12 23 years uld

') e
flannah: H
Sister: S
Toda y: H ,_~ S+ 4

1-iannah Sister
Toda y S+4 S

2 years ago S+ 4-2 S 2

S- 2 = i(S + 4 - 2)

s- 2 = -ª(s
4
+ 2J Multiply both si des by 4.
4(S- 2) = 3(S + 2) Distribute 4, anci distribute 3.
4S- 8 = 3S + 6 Rearrange.
S= 14 Toda y
S= 19 5 years from now

3. A
Sonny: S
Michael: íH
Today: S=5M

Sonny 1\'Hch<Jel

Toda y 5M M

3 years from now 5M +3 M+ 3

Sm +3= 2(1/1 + 3) Distributc 2.


Sm + 3 = 2m+ 6 Rearrange.
3m = 3 Divide by 3.
m = 1 J\Iichael is l year old today.
4. D
Wendy: W
Qucntin: Q
Toda\: t\ioc Q +- lG

\N'endy

Toda y Q + 16 Q

3 years from now Q + 16 + 3 Q +3

Q + 19 "' 3 · l Q t- 3) Distributc 3.
() +- El = 3Q +9 lkarrange.
lO= '2Q Divide by 2.
Q=5

s. e
Shoes + Sweater 1 = $36:=i
Shoes + Sweater 2 = $293

Note that the dillerencc betwccn the two totals is also the differencc betwcen the prices
of thc two sweatcrs.
The pricc of the first sweater is 1.6 times the price of the sccond sweater.
SI = l.G. 52
The price differcncc bctween the sweaters is
$365 - $293 = $72.
This means 5 1 - S, = 72.
Heplace 5 1 by l.GS¿ and solvc for 52 •
l.GS2 S2 72
O.GS2 = 72
0~z -- ü.G -$PO
72
- ·· ~
If thc second sweater is $120, the shocs are ($293 - $120) = $173.

6. E Lct the total rcn t be R.

If 5 peoplc are sharing tlw rcnt equally. cach pcrson pays -~.

If 4 pcople are sharing thc rent cqually, each person pays -~.

The increasc in rcnt pcr pcrson is the diffcrencc bctween the above quanti: s
B-
4
B
5
= 260
f!li -- 1R = 2GO
20 20
ll = 260
20
Hcnt 20 · 2[)0 "" $5,200
- e .\ ~impll' diagrall\lll<l\ lll'lp Íll thi-; prohlt'lll

J ·· ·· ·· ·.: 1 11 ¡wopk O
Lvt tlw tltltnht•r 11! pt'tiplt> i11 lnJill (JI /lw 11.

IIH' lllllllilt·i· ol lh'tiplt• Íll lnl'lt ol j lH'L'liltll'" u .>.


·)'\
llit' llllllllil'l' ol pl'O¡Jl\' Íll !!'11111 oi' f <l!Hl /i:-. i1 11

'.!_ ¡¡ (i .,.,

:!.11 l(j

11 B
Thl'rc are B ¡wopil' i11 lrtllll ul lrit·ia.

B. O Uearly, hoth tires \\'ill tran·ltlw ~atlll' di-;t<IIH'l'. \\.lll'll a tirt· lll<tkl's UJH' rt'qJlutioll, it
lllll\I.'S fom·ard a;, !'<tras ÍIS l'Írl'll!llfl'rl'lll'l'.

Circumkn:nec· 2r.r = diamcl.:r • r.

Dianwter of the front tire = ti


Di a meter of thc rcar tire = 3d
i\ftcr 90 revolutions, the back tire tnwels:
Circumference · revolutions = TI · 3d· 90
The front tire will travel the same distance:
d · " · front revolutions = TI • 3d· 90
Front revolutions = 3 · 90 = 270

9. E Let the 4 consecutivc intcgers be


11, (11 + 2), (11 + 4), ami (11 + 6)

The largest is 150% of thc smallcst tracslatcs into:


11 .,.. G = l.S11
6 = 0.511
11 = 12
So thc numbers are 1:2, l·L lG, ancll8.
Tbc average of the two middle numbcrs is l~L;_L§ = 15.

lo. e \Ve can translate the problcm as follows


x" = 5x- G Quadratic cquation
.\'" - 5x + G == O factor or use thc quaclratic formula
(x -- 3)(x --· 2) O
x = 3 or x = 2
Since we are tole! it is an cYcn number, x "' 2.
Thc anS\\Tr is the sq u are of 22 ,= 4.

t

1 l. E Let the 7 ronsccutive integers be


11, (n 1- 2), ... , (11 ¡._ G)

T!Je c;um uf tlle !argest and smallest is 128.


11 -r n + () = l2B
:!.n "" 1:!.2
11 = Gl
So the numbers are() l, G:2, !)3, G4, bS, ()(), and ()7.
Thcir an~ragc would he the middle nu¡nbcr, G'L
A!tcrnative!y, you could simply add them and divide by 7.

12. E Tlw dilTerence of two positive inwgcrs is 4 translates into x -- y= 4.


Thc dilTcrencc of tlwir squares is 80 translates into x:· -y" = 80.
i\otice tlnt tlll' second equation is a di!Terence of two squares ancl can be factorcd
as follows.
x" -- y' = (x - yl (x + y) = 80

From tllc tlrst equation, \Ve know that x -- y= 4. So we can plug it in.

4 ·(X+}')= 80
X+ J' e~ 20 1\ow WC havc IWO first-c!cgrce equations that WC can solve together.
X+ J' = 2()

x-j ~:_-±.__ Hecall the first equation here.


Add the two equations sic! e by si ele.
2x = 24 Divide both sicles by two.
X= 12
y= 12- 4 = 8
x' + y 2 = 12 2 + 8:' = 14<1 + G4 208

13. D -l- empty means -~ full.


Let the initial amount ofvvater be t.

t + x '= ~ of the tank

+ t- x '"" t of thc tJ.nk Wc can add the cquations side by sidc to eliminatc x.

21 ~,% of the tank

t = -} of the tan k

Alternativcly, draw a simple tlgurc to hclp.

} add x

} !\'rilO\\? X

1
l
. 1)
1 l. 1) ilw ori;~in~tl rvnt pn ¡wr-;on is ".
()

fl
ilw n•JJ! ¡wr p<'l"'i(lft ~tlin :..' joiiJ<'d in is
J()
/' /) ~-., í) 1/'
" !(1 itJ )(!
"

l ht' \\(lit! "pe! .. rt IIIÍi ds ll~ tiLi! \\\' <lrt' \\Orking \\·j¡!J ;¡ ratv. 'litis j-. SÍlllpl\ a LI!ÍO of l\\ll
'.i'.léiltiitit·~. •.1 ;:1\ tli!it'it'ill unit~ .. Thinking uf ratc~ as lr;wrions Illa\ simplii\ o m lllldt•rstalllling
()! lill''>l' !\ [í'-'' uf prPlllcms. ¡:or t'\a111pil', a ronsult:!IJt's r~Jll' is the doll<tr ;JilltJtlllt slw chargl's
¡wr fwur. S!ltr. Saks ¡wr IJJ<nltii for a grocery storl', llliles 1wr gallon for a car. or fe·¡• ¡wr lllil<·
lrH a call :tre al! <'h'Iyda\' rates \H' are bllliliar 11ilf!.

ltl gvneral. any rate prolJlt•m can fw set up as follows:

Hale: (quantity :l per quantity 11) · Quantity 13 "' <Juamity A

í\liles pcr gallon times gallons used =Miles drivcn _I!li.l_<;? · gallons = miles
gallon
!\ate of service ($ per hourl times lwurs = Total fee --~- · hours = S
hour
lf a certain bookstorc sells 127 books every two days, at this rate how many books
will it sell in 14 days?
Books per day · Number of days =Total books sold

-127- · 14 = 889 books


2
Alternatively. set up a proportion.

Thc fundamental rclationship to solve rate problcms irwolving motion is:

Rate ·Time = Oistance or r·t=rf .!_l¡j]c2 · hours = miles


hour

In other worcls, clistance traveled D by an object is the procluct of its average specd r allCI
the total time t of motion. The average speed (ratc) is the ratP of cb;mge of clistann· or th~:
distan ce per tirm?.
i\nothcr easy way to remcmber this relatiomhip is from the spccd limit signs we use on tlw
higlnvay every dav. f~atc =~ 1!.li (miles per hour).
• - 111"
mi . . distancc (mi)
So rmust equal--, wlllch h - - . - ------
hr tm12 (iu)
\Ve can also rearrange thc r · t =' lJ formula to CJlculate time or rale.

D r·t
. [)
t= T
,. = J2.
l

No tire how the units work. In our basic r · t = D formula, 1~{¡ ht · =· mi.
Si\:VIPLE PHOBLEí\IS

Helen bikes from home to work at an average speed of 15 miles per hour. On the way
back, she runs at 9 miles ppr hour. Hl~r run takes 20 minutes longer than her bike ride.
How far is her work from home?

Notice that inthis problcm, rhe distance covered in both legs ofthe trip are thc same. So d, d,.
Lct thc time Helen takes to bike to work be t
Since 20 minutes equals} l10urs, the time Helen takes to run back lwcomes 1 + }.
Rate X Time Distance
Bike 15 t 1St

Run 9

Because the distances are the same, set d 1 = d 2•

l5t = 9 · ( l + ~) Distribute 9.
l5t=9t+3 Subtract 9 t from both sic! es.
Gt = 3 Divide both sides by 3
t =! hrs Biking time is! hour or 30 minutes.

D = r X t = 15 X ! = 7.5 miles Use the main distance formula.

A car leaves City A and heads toward City 8, which is 240 miles away, traveling at .
an average speed of 45 miles per hour. lf another car leaves from City 8 and heads
toward City A at the same time and travels atan average speed of 35 miles per hour,
how far away from City A do they meet?

Notice rhat in this problcm, thc total distancc is covered by two cars together. As a rcsult, we
will sct up the final equation as d 1 + d2 = d10 ,, 1•

Sincc they travel un ti! thcy mcct, thc time of travel for both cars is the same, t.
Rate X Time Distance
Car from A 45 t 45t
Car from 8 35 t 35t
Total 240

d 1 + d2 = c(,,., 1
45t + 35t = z,w
801 = 240

t == 3 hours
Now check what thc qucstion is asking.
D1stamc fiOm Cit) A o~ Ha te of Cn A t1mes tune -- 45 · 3 - Lb miles.

¡
."'-.
~

REMEMBER 1
r Do not A boat travels 72 miles to an is!and at 24 miles per hour and returns at 12 miles per
average given l hour. What is its average speed for the entire trip?
1 rates. Use:

Average rate =
1 Total distance t
\
= !J
r,
= lZ
2-1
= 3 hours
traveled
Total time
t2 = ~ = t~ = G hours

Averane rate = '[g_tal <iL~an~__t.!_<~~ed = 72 ·-~ = 16 miles per hour


b rotal time 9

l. A bicycle lert point A and traveled north at 12 km/hr. Two hours later, a motorcycle left
e
the same point riding south at 34 km/hr. lf at the mi of t hours, they are 162 km apart,
how long was the motorcycle's ride?
(A) 7 hc_mrs
(B) 6 hours
(C) 5 hours
(D) 4 hours
(E) 3 hours

2. A car travels 240 miles from Town Ato Tovvn B at an average spced of 60 miles per
hour. At what speed did it travel on the way back if its average speed for the whole trip
was 48 miles per hour?
(A) 16 miles per hour
(B) 24 miles per hour
(C) 36 miles per hour
(D) 40 miles per hour
(E) 48 miles per hour

3. Two cars leave City A and travel in opposite clirections. The first car travels at 65 miles
pcr bour, and the second car travcls at 55 miles pcr hour. How many hours pass befo re
they are 300 miles apart?
(A) 2.0 hours
(B) 2.5 bours
(C) 3.0 hours
(D) 3.5 hours
(E) 4.0 hours
4. Karen dri\·es the llrst lBO miles of her trip atan average specd of 60 miles per hour.
1f slle clrives the remaining 12.0 miles atan average speed of :lO miles per hour, the
average speed for her •mtirc trip is dosest to which of the follmving?
(A) 42 miles per lwurs
(B) 43 miks pPr lwurs
(CJ ,¡:;miles pcr hours
. ( D) -17 miles ¡wr homs
(E) 30 miks per hours

S. ,\ company produces its ceil phoncs in two different countries. The cost to manufacture
une phone in counrry "\ is 20% more than it is in Country /3. The company produces
GO.OOO plwnes in Country ,\ and 'ln,OOO phones in Country B. If the average cost pcr
plwnc is S 1O, \\·hat is the cost pcr phone in Country A?
(,\) $9.00
(BJ S9.20
(C) S9.BO
(D) $10.20
(El $10.80

G. Carl and Jen's houscs are 1S miies apart. IfCarl Jeaves his huuse biking 12 miles per
hour tovvard Jen's house and Jen leavcs hcr house at the same time running toward
Carl's housc at 8 miles per hour. How much time passcs until thcy meet?
(A) 1 hour 1S minutes
(B) 1 hour
(C) 45 minutes
(D) 30 minutes
(E) 15 minutes

7. A tortoise starts walking due north ata rate of 2 kilomcters per hour. 2 hours later, a
harc starts running in thc same direction from the sarne point ata ratc of G kilom~;tcrs
per hour. How long afler the tortoise's dcparture will thc hare catch up to thc tortoisc?
(;\) 1 hour
(B) 1.5 hours
(CJ 2 hours
(D) 2.5 hours
(E) 3 hours

8. JV!ark chargcs e clollars ¡wr h hours of consulting scrvices. At this ratc, how m u eh would
it cost to hirc I\lark for a project that is cxpectccl to take him k hours?

(Al -r-
c·lz

e· k
(B) -,¡--

k· h
(C) -e
(D) c·fl·k
--60___
__· ke·____h
(E) CO
~- At the same time, two cars rakc off from City A and travcl towarcl City B with awrage
speeds of .!Q miles per hour and SO miles per hour. The fasrer car rcachcs City !3 15 min-
utes earlier than slower car. 1low far is City B from City A?
(r\) l o tll il ("i
( 13) ·lO miles
(C) 50 miles
(D) GO miles
(E) 90 miles

1O. Daría has k fricnds on thc FaceS pace online community. lf she adds friends ata con-
stan! rate of m friends pcr wcek for thc ncxt 11 days, how many fricnds \Vill she ha\·c at
the end of n days?
(A) kmn
(13) k+ 11111

(C) mn rí'
7
(0) k+ 17111 i¡e
7
(E) k+ n + 111 ¡'
1

Camitle bikcd to work atan average specd of 14 miles per hour and returned home l
using the same route atan average speecl of lB miles pcr hour. If the return trip too k [
20 minutes less than the first trip, how long was hcr total commute?
f
(A) 6 miles
(B) 12 miles
(C) 14 miles.
(0) 21 miles
(E) 42 miles

12. On a 90-mile circular track, two cars start driving from the same point in opposite
clirections. Car A's average speed is 120 miles per hour, ancl Car B's average speecl is 150
miles per hour. How many miles will CarA ha ve traveled by the time they meet for the
first time?
(A) 30 miles
(B) 35 miles
(C) 40 miles
lDJ 45 miles
(E) 50 miles
l. E
Rate X Time = Distance
~-~
-""-- "--- ····· --~------~~

Bicycle 12 t 12t

fv1otorcyclc 34 ( t - 2) 34(t- 2)

Total 162

The time tr<t\ek'd by tl!e motorcyde is ( t - 2) because it takcs ntT two hours aftcr
the bicycle.
Since the total distancc bctwccn tbem is given and they tr<n·et in exactly opposite
directions, we can add ea eh distan ce travelcd to makc it equal to 1G2.

di + d2 "' d""'li
121+ 3-l(/ - 2) = 162 Distribute 3-l.
121 + 34t- 68 = J(-)2 Hearrange.
4Gt = 230 Divide by 46
t= 5
(t- 2) = (5 - 2) = 3 hours

The motorcycle traveled for 3 hours.

2. D For the tlrst part of the trip:

t 1 = f~ = _g¿~o = 4 hours

Let the time spent for thc rcturn be t¿:

Average ratc IQt<ll.<ii~Ji!l}~~travel.Q_Q = ~40 .:2 = 48


Total time 4 + t2
_:1_ª9- = 48
4 + t,
480 = 4B · (4 + t) Divide by 48.
10 = 4 + t,
t2 = G

D. 2L10
r 2 = t • = -- -- = 40 miles pcr hour
2 6
3. B d 1 + d2 = d,,,,¡
rl . t + ,.2 . t = d!n<.d Notice that t is the same for both.
G51 + 55t 300
120t = 300
t = 2.5 hours
4. B t= -~
The time for part l:

The time for part 2:

Rate x Time :::: Distance

Part 1 60 3 180
Part 2 30 4 120
------·----------
Total 7 300

Average rale =Total dl.~_tanc~lr_;l_l{_elc;_Q


I ola! tune
Average rale = l2Q = 42 g""' 43
1 ¡

:J. E Let thc cost lo produce in Country B be x. The cost to produce in Country A is 20°/r)
higher than x, which is x + 0.2x = 1.2x.

Average cost per phone = -.--.J_Qt;¡J__l~J_s~----


1 o tal number of phones

Average cost per phone = 60,00(_)_.:__~:2x_-J::_j~OOO · x = 10


60,000 + 48,000
72,000x + 48,000x = 108,000 · 10
120,000x = 1,080,000
120x = 1.080
x = $9 is thc cost pcr phone in Country B.
1.2x = 1.2 · 9 = $10.80 is the cost per phone in Country A.

6. e
Rate x Time = Distance

Carl 12 t 12t
Jen 8 t 8t
Total 15

12t+8t=15
20t = 15
15
t= :;mhours

}g ·60 = 45 minutes
, . E \Vlwn the hare catchcs up with thc tortnisc, they are at the same distance from their
starting point.

d, = d2
Let the time the tortoisc walked be t.
Sincc the hare started 2 hours later, its time is (t- 21.
d¡ = d2
ri . r occ r" . (t -- 2)
2t = G(t 2)
2/=()t-12
·ll = 12
3 hours

9. e
Rate x Time Distance

Fast 50 t 50t

Slow 40 t _;_' 1
4 40(t+~)
Total

di= d2
50t=40·(t+i)
50t=40t+l0
lOt = 10
t = 1 hour
If thc fast car travelcd 1 hour belwcen the citics:
D r·t=so!!Ü 1 hr = 50 miles
hr

10. D Avcraue rate = ftj_QJ~C~S_<lcJQesi


b Inne
r == E= ::::-!_LZ ___ = !2.1 friends pcr day.
r í days 1

Sincc we are looking at a pcriod of 11 days:

Total friends added = !J


1
• 11 = il,v-~

DaJ :a had k fricnds lo start. So the total m!Jl_:;cr of fricnds al the cnd of 11 days is:
k+ !.!L'-!
1
11. E d 1 -= t/2
Lct the time to go to work be t.
Since shc spent -~ hours less on tlw way back, 1\. t -- A-1 is the time ir took her to go back home.
,) ,) 1
d, = i/2
r·t=r·fr~l\
l :.: \ 3)
1 1\
l-It= lB ·(t-- })
1-!t = lBt -- G
(:) = 4 t

t =!!.
4
t = 1.5 hours
D = r· r 1.5 · 1·1 = 21 miles (orwway)
Commute ,= 21 · 2 = 42 miles

12. e
r·t+r·t=d
1 .! llll,ll
Notice that t is the same for both.
120 . f + 150 . [ = 90
270 . t = 90
t= l
3
They mect after kof an hour.
D= r·t = 120 · l = 40 miles
3

Work problems are essentially rate problems. They can be solved using the same formula.

Ha te · Time = Work done

The rate for work problcms is work completed pcr unit time.

l'l.2~ls. · Hours = Work done


Hour
Notice that time and rateare inversely proportional. As the rate incre;¡ses, the amount oftime
ncedccl to complete the work dccreases.

In most work problcms, the rate al which a certain job can be complctcd by certain machines
or persons is given and thc rate at \\·hich the work can be complctcd in collaboration nceds
to be computecl.
In ordcr to combine the work done by each machi m~. \\'C calculate thc portion (fraction)
ofwork complctcd pcr unit time for each machine.
For example, pipe A can fill a pool in 4 hours. Thereforc, it can fill ~} of the pool in l hour.
For exarnplc, pipe B can fill thc samc pool in G hours. TherefClr(', it can fill i of the pool in
l hr
We can acle! the two ratcs to determine \\·hat fraction of thc pool can b~ fi!lccl in l hour.
If the pipes work to¡;cther, + = - 3 + i i }¿
= . 5 of the pool can be fillcJ in 1 hour.
12 12
Jftlw pipes can fill¡~ of the pool in 1 hour, they cm fill the entirc pool in-~?_= 2 ~- hnurs.
proportion below sl10\\'i you lww to calculate the time.
5 12
T:I _ 12
·¡ - -.{'
Note that the proportiDn '.\'ill always giH~ you thc reciproca! ofthe surn ofthe portions of
done.
The fundame'nlal rl'latiun~;llip to sulve t!wsc ty¡w~ tJfwork problcms is

·• ~~ 1 CC<el in" ni ll<e jnb lh<>l '"" br "" iYI<eJ by bnlb i" 11 ,¡, 1i me
L..---->- Fractiz'n of th.: job tltat can be: rinishcd by ''y" in unit time
L-------,_ frac·tion nf thc job that can b.: fini'>h.:J by "x" in unit ti m.:

In this relationship, the x is how long the 11rst machinl'lpcrson needs to finish the job
alone. The y is how long thc sccond n1achinc/pcrson nccds tu finish thc job alonc. The t is
how long both nccd to finish the job togcthcr.

SAMPLE PHOBLEi\ !S

~lf
.,____
Jooo oao 'ead 30 page; io 24
\ · has 360 pages?

-
mio=~•oold n take to fioi;h a book that

-----
J
"Rate ·Time = Work done

Juan's rate = ~~- = lj. pagcs/minute


lj. t= 360
. } = 360 · ·fb = 3G · 3 = 108 minutes

,·Altcrnatively, sincc thc numbcr of ¡ugcs is directly proportion<l: to the rcading time, we
· .. could also sct up a proportion:
Pagcs 80 _ 360
Mir1uie -7 2'4 - -r Cross mullip!y.

80 . t = 24 . 3(}() Di\·icle by 80 befare multip!ying in order to simplify.


t- 24 · 360
- --·80 -·- =
3 · 3G
---r- .
= 1os nllmrtes
lf Terry can type 5 pages in 7 minutes and Kerry can type 6 pages in S minutes, how
long would it take thern to type 335 pages if they work together?

Terry can type 5 pages in 7 mins. Tlwt nwans he can type ~ of a page in onc minute.
i 5 pages \ :- .
1 ~---,oc __ 1 -~ '2- paue per llllllUte
\ i 111llll!lCI i "'

Kcrry can type () pages in 5 míns. That mcans slw can type ~ pages pcr minute.
{ 6 pages \ G . . ,
F p,l"l' per mtnute
1, -;::----e-------¡--~
:J mm u te :J ,.,

To find how many pages Terry and Kerry can type per miHute,
5 ' ()F --
"7 -;- -
25
-3-r + 42
l-;.;
7
--- -3_--:=
67
1 ;) J ~J ;)

Together thcy can type ~~~ pagcs ín l minute.

At this stagc, we can eithcr use the general ratc formula or sct up a proportion to find the total
time to typc 335 pagcs.

Option l: Ha te · Time = Work

~I · t = 335 Multiply both sides by '3)-5-.


67
35
t = 335 · ~-~ = 175 minutes

67
Option 2: 35 - -¡-
-¡-- 335

Multiply both si des by ;?-º-~ .


%Z · t =
J;)
335
67
t = 335 · ~ = 175 minutes

l. Printing press A can finish a certain job tluec times as fast as printing press B. vVhen
;vork!ng togct!Jcr, thcy can finish thc job in 12 hours. How many hours vvill it takc press
B to finish tlle job when workii':<; alonc?
(A) 4
(B) 16
(C) 3G
(DJ 48
(E) G:J
2. Machinc A working alone can complete a job in 2}¡ hours.l\Iaclline H working alonc
can do the samc job in3} hours. llow long will it takc for both machines working
together at tlwir rc::;pccti\·e constant ratcs to complete the job?
(:\) l lwur 30 minutes
(ll) l hour ,¡5 minutes
(C) 2 huurs
(DI 2 lwur l.:J minutes
(E) 2 hours :10 minutes

3. Faucet 1 working :done can fill a tub in 20 minutes. Faucet 2 working alone can do the
sarne job in 15 minutes. Tlw hole at thc bottom ofthe tub can empty a full tub in :JO
minutes. Ir both fauccts are 1n>rking at thcir constant ratcs given above ami thc lwlc is
nqt covcrcd, hol\' long dm•s it takc to rill an empty tub?
(,\) 5 minutes
(B) G minutes
(C) 1O minutes
(D) 12 minutes
(E) 20 minutes

4. vVhen working alone, Dominic takes twice as much time as Nick does to mow a lawn.
When working together, they can mow half of the same lawn in 15 minutes. How long
does it take Do minie to mow the lawn by himsclf?
(A) 90 minutes
([3) GO minutes
(C) 45 minutes
(D) 30 minutes
(E) 20 minutes

5. Austin takes 5 minutes to prepare and sea! one wedding invitation in its envclope. lf his
fiancée FE'licity can do thc same job at 3 minutes pcr inVitation, hmv many envclopcs
can thcy prepare in 30 minut~?s togcthcr?
(AJ 8
(B) 10
(CJ 12
(D) 14
(E) 1G

6. Bottling l\lachine AlOOO can fill small bott! sin 2 scconds or largc bottlcs in 3 scconds.
A planl is to be dcsigncc! to 111! a mínimum of 20,000 small bottks aml15,000 large bottles
in OIK 8-hour shifl. 1\ssuming no downl imc, how many A lOOOs ~:re nceclcd?
(:\)
(l3) 2
(CJ 3
(D) 4
(E) 5
7. l\lachine ,\can finish a job in 20 huurs. l\lachine B can finish the same job in 15 hours.
l low long would it take tu finio.;h halr ihe job if 2 machine A's and :l machine Ws aw
used simultanenusly"?
( t\) (j
(B) 5
[C) .¡
')
(D) J
.,
(E)

n. lf S software engilwcrs working fl llours a da y can linish coding a computer program in d


days, how many days would it takc R engincers working i bours a da y to finish the same job?
(!\) Shd
Ri
Shi
(B) Rd

(C) Si
Rdlz
(D) Jld
Shi
(El _BL
Slzd

B= ~ 1 1 ;-fB±t-"~Iftl
l. D Let the time it takes 11 to finish the job be t. Press B would finish thc job in (3t) hours.
Sct up thc basic relationship for work problcms.

* +~ = 1+ o}¡= 112
Makc the denominators equal by expancling the first fraction by 3

3+.l=-L Acle! thc fractions.


3l 3t 12
4 - 1
3t- n Cros;, multiply.

12·4=31 Divide both si des by 3.


12. 4
-3·--- = t Si!llplify.
t=4·4=1G
This mcans prcss /l can llnish the job in 1G hours. So, press B can fiuish thc job
in 16 · 3 = 'W hours.

2. ;:!>. J. + )_ = ___1 __ + __j_ =o _l


• A B 21. 3;l t
2 ' 4
j_ + __ l = l_ !,__
5 15 t
2 4"
_? --L _4 == _()- + .'L = _lll_ = _? = J
5 ' 15 15 15 15 3 t

2
t = 'l boms
~ hours =-= 1 l>ov 30 minutes
l
30
Notice we are subtracting the water let out by the hole. Convert tlw fractions so they al!
han~ a cornnwn denominator.
:2 .. :) 1 l
60- ()() 12

t"" n.1 12 minutes

·l. A \\"lwn \\orking together. thc entire lawn takes 30 minutes to mow.
LL:t tlw time l\ick takes tu mow the entire la\\'11 be t.
The time Dominic takes lo mow the emire la\\'11 becomes 2t.

1 _J=J ... J"" L


lJ ' N 2! . 1 JO
J. + _2_
2t 21 30
:l .. l
·ii- :m
21 = 90
t"' 45 minutes

Dominic would take 2 · t = 2 · 45 = 90 minutes to mo1.v the lawn by himself.

5. E Do not use the long formula. Just set up simple proportions in this case because thc
amount of vvork is bcing asked ami the total time is given.

i\ u sti 11: 'º-.DJÜ! = 1Q 1~1Ü!


l env x
x = 6 envelopes in 30 minutes
3 min __ 30 min
Felicity: Teriv - --y--
y= lO envelopcs in :lO minutes
Total envelopcs = 16

Altcrnatively, if yo u prefer u si ng the foúnula:

Ratc ·Time =\York done

Austin: J.Qq.lf. · 30 min ~ G cnvelopcs


5mm
Felicity: J-ell~'- · 30 m in =·= 1O envelopcs
3mm
6 + 1O =~ 1G envelopes

6. C The total time necdcd to fill 20,000 small bottles is


Hatc X Time = \Vork clunc
- Lb.01U.r~. - · 1= 2o,ooo
2 scconds
t == 2 · 20,000 == 40,000 seconds

Thc total time necdcclto fill 15,000 largc bottles is


t = 3 · 15,000 == 45,000 seconds
Orw t\1000 can finish this job ill 85,000 scconds.
1
-~ ~.~;:,; JHlll!"')
; ,! j( 1ljl ~
1

')

~ J!i;ll'llilll'. \'-. (';111 lilli._,J¡ :~¡, til tiH' jtih Íll 1 il<llll.
1
1 !l!;ll.'llÍIH' /! l ;¡¡¡ lillÍ...,JI . o!' iill' j1d1 i11 1 IHJIII'.
1,)

. : 1!l; 1el! i ll\' 1)'" e 111 fi 11 i "! 1 :) {)


1,)
r 111<' i1dl j 11 1 ll 1 llll".
'
., ')
,) 1 ,) ,)

~1) l."i j() i.í ,;1) :w


So alllll<tl'llilll'S l'<tll fini:-.h tlw joi> i11 t) lwur--;.

Tlw qtwstion is ;¡sking lor tllv IÍllll' to fiJii-,!J half of tlw joh, which is :l hours.

B. A ¡:jnd tlll' total amotlllt ol\1·ork tlr~t.

S ~~n~inecrs · /¡ b()_L.!L~ · d da\'s = SI ~ti hours nf work


' clay ·
Tlw llCI\' number of engimTrs is 1? ami thc workday is i hours long. l.ct tbe ncw numbcr
of days be x. Thc total amount ofwork stays tlw sarnc.
Shd Rix
\' =.e ,')_/1(/
. ¡¡¡

In mixture problcms. usually two or more products with diffcrent characteristics are mixcd.
Ccrtain characteristics of the rcsultant mixture are a·;ked abe u t. In so me problems, the com-
position will be changecl Yia adding or remo,·ing one of thc ingrr:dients.

The fundamental rclationship tu solve mixture problcms i:;

1 % of product · ,\mmlllt of mixlllre ,.. 1\mount of product in that mixture

~ Once you mix, you get the following equation:

l'roduct in mixture l + l'roduct in mixtut-e 2 ~-Total product in tlw rcsulting lllix

- - - . -~ -- ~~~-ll-~xam-plc·:- I low many litcrs of a 2:-i% salt solution necd to be mixcd witl! l5litcrs of !O'V., salt
___ . _ _ ___ ~ solution to ct-catc a 1()"(, salt solutiun'?
-
1\iOT!CE % of salt x mixture Amount of salt

Mixture problems are m<Jth- Solution 1 25% m 0.25m


ematically very similar to the
Solution 2 10% 15 1.5
rate ¡Y ;b!ems v:e reviewed in
Section 4 2, whcre d, + d2 = d,. Mixture 16% m+ i5 0.16(m + 15)
Salt in solution l + Salt in sulution 2 """' Total salt

0.25171 + !.:) ~ 0.16(111 + 15) \lultiply cach term by 100 to avoid decimals.
25m+ 150 = Hi(m + l'i) Distribute lG.
25m+ 150 = 1Gtn 2iil Hearrange the terms.
:!.::'>m- llim = 210 I.:iO
9/l/ = 90 lli\ ide by 9.
m= 10 liters

If tlw mixtun' problem is about cost, thcn use the following equation:
Price 1\mount Cost
Once you mix, you get:

Cost, + Cost 2 "' Total cost

SAlVIPLE PflOBLEf-.IS

The; cost of cashews is $8.50 per pound, and the cost of peanuts is $5 per pound. lf a
grocer wants to produce a 10-pound mixture that costs $6.40 per pound, how many
pounds of cashews should she use in the mixture?

L _____________ -------------------

Price x Amount = Cost


Cashews $8.50/lb .a 8.5a
Peanuts $5.00/lb (10 - a) 5(10 - a)
--- --- ---------------
Mixture $6.40/lb 10 $64

Cost, + Cost 2 =Total cost


8.5a + 5(1 O a) =--' 6-1 Distribute S cncr thc parcnthc:;cs.
8.5rt -+- 50 5a ==- ti-1 Combine like tcrms, and subtract 50 from cach siclc.
3.5a = 14 Divide by 3.5.
(/ e~ 4 pounc!s of cashcws

-· --~P~~gy~:n•,-c~ntains 50 coins consisting of quarters and dirT;2S. Their total value is


$7.40. How many quartcrs are thcre?
[

Price x Amount = Cost


Ouartcrs 251/piece x 25x
Dimes 101/¡~. :ce (50 -X) 10(50 -·X)
Total 5J 7401
Co<;t 1 + Co~t 2 = Total cost \Vrite the total cost equation in cents to avoiJ decimafs.
25x + 10(50 - x) = 7·10 Distribute lO ovcr thc parentlwses.
25x ~.. 500 -- l Ox ~o 7 !O Combine like tcrms. and subtract 500 from each side.
15x e~ 2 10 Dilide lJ\' l ).
x -, [() qu:Jrtcrs

l. Ciant brand sports drink contai11s 15% fmit juice by volume. I Iow rnany liters uf pure
fruit juicc need to be added to 20 liters uf tlw sports drink ro increase the juice percent-
age to ::!0%?
(,\) 1 liter
(13) l i liters
(C) l ~ liters
(DJ l ~ liters
(EJ 2 liters

2. 25liters of JO% salt water are mixcd with l5liters of20'1h salt water. The lO liters of
\Vater are added to thc mix. What is the resulting percentage of salt?
(A) 10.50%
(13) 11.00%
(CJ 13. 75'!{¡
(0) 15.00%
(E) 15.25%

3. A drink is 12% alcohol by volume. How much water do you need to adcl toa
300 ce drink to decrease the alcohol content to 8% by volume?
(;\) 50 ce
(BJ 75 ce
(C) l 00 ce
(D) 125 ce
1
1
(E) 150 ce ¡
L

4. How much water necds to be evaporatcd from a 30-gallon 2~'/o salt solution to result in a ¡'
¡
mixture of 40% salt?
!
(A) 5 ¡ (
(B) JO
'•
(C) 12
r
(D) 15 ¡
(EJ 17

1
t
1
1

1
S. When 30 poumls of sugar are atltled lo a mixture of 40'!'0 sugar, the concentration
increascs to 50°'0 sugar. \Vhat is the wcight of thc original mixture'?
(:\) 100 pou nds
fB) 125 pounds
(C) 150 pounds
(D) 175 pounds
(E) 200 pounds

ti A mixture of red beans ami black bcans are to be preparcd. The pricc of red beans is S2 per
pound. ami the pricc of black beans is $3 pcr pound. \Vhat is the ratio of red beans to black
beans if thc mixture is to be sold for $2.75 per pound?
(i\) l : l
(Bl l · '1
(Cl l: 3
(D) 2:3
(E) 3:4

1. Equal amounts of water ancl acid are added to 40 quarts of 80% acid solution. The new
mixture has a concentration of 70%. How m u eh acid is in thc rcsulting solution?
(A) 1O quartz
(B) 18 quartz
(C) 32 quartz
(D) 40 quartz
(E) 42 quartz

8. Coffcc beans from Ethiopia costa dollars per pound, and beans from Guatemala cost
b dollars per pound. In a ccrtain mixture, thcre are three times as many bcans from
Ethiopia as bcans frórn Guatemala. What is thc cost per pound of this mixture?

(A) b + 3a
--¿r--
(Bl g__:t-43/z

(C) g_~_ú_
(D) :}[!_:~}2.
2
(E) a 1 3lJ
- i
<'

('

i t·t thl' <tllJOtliJI !!l juin· ¡,¡ ih· ;Jtidt\! iJ,· _\",
_'·q¡¡¡, t' ¡IJ;¡¡ ¡¡,,. '., jttit·t· tdpilll' juit·•- ¡, ltHJ".,

% ct juico :< ~-1ixture Total juice


Drink 1 15% 20 3
Jctice 100% '< X

fvlixture 20% +x
20 0.2(20 + X)
()_:.'(:.:'() -1 X) :J X

1 + O.:!x 3 -' x
l -~ O.Bx
x = l.:.:':) litcrs
i\lternativcly, you can sct up a proportion.
. _ Amountofjuice
Concentral-Hl!l ~o - , 7 ----
---:--------- __
lotalmJ.\turc
Since thc new drink is 20";, juice.
1
1l:5 = ~-
Rcmembcr to add the juice to the total mixture in the dcnominator.
20 + x = 15 + 5x
4x = 5
x= ~ = 1.25 litcrs

2. B TI! e amount of salt in tlw 1st mix is 0.1 O· 25 == 25.


lhe amount ofsalt in tlw 2nd mix is 0.20 · 15 = 3.0.
lile <llllOlllll of \\'a ter lO be added ÍS ] () liters.
l\otice that thc pcrcC'nt salt for purc water is O'X,_

% of salt x Mixture == Total salt


tv1; ·- 1 10% 25 2.5
tvlix 2 20% 15 3.0
Water 0% 10
Mixture 50 5.5

Concentration '"" ¿\~_llOL!IltoLsi!ll_


l otalmtxlurc
2.'1 + 3.0 ') ')
----- ----------------- =-- -¡..)·(·)- == O. ll == l 1'!n
2'1-+15+10.
3. E Thc amount of alcohol in thc original drink is 0.12 · 300 = JG ce
Lct thc amount of water to be addcd be x.
Notice that thc "h alcohol of pure \Vatcr is 0%.

% of alcohol x Mixture Total alcohol

Drink 1 12% 300 36


Water 0% X o
Mixture 8% 300 +X 0.08 (300 + X)

0.03(300 + X) = J()
2-! + O.OB.r ~~ :IG
O.OBx = 12
x = E Qi! = 150 ce
8
Alternatively, you can set up a proportion.

Concentration = AI!]~2.l_l..!.!t ol: alcohol


I otal1mxture
Since the ncw drink is 8';{¡ alcohoL ~- = 2 -.
25 10
2 - 36
25- 3oo + ~\-
2(Joo + x) = 25 · 36
600 + 2x = 900
2x = 300
x =150 ce

4. D The amount of salt in the 1st mix is 0.20 · 30 = 6.


Since the amount of salt will not change, the rcsul'ing solution will still have 6 gallons
of sait and less water.
Let the water to be evaporated be x.
Concentration = 4Xl1.().Q11_1: _of ~<~1
.fotaJ 1111XtUre

--º---
30- x
= __:lQ_
lOO
6 - 2
J(f~\-- 5
6·5 2(30 X)
JO= 60 2x
2x = 30
x = 15 gallons

1
:). e 1 ,·[ tiw \\t'Í)~ilt olrllÍ\llllt' 1 i!t• .\.

0.4":>;

:5()(.! 0.::0(:30 ~- X)

ll.l.•. :;¡¡ ! .) . () .. l.\'


l.) 0.1 X
.r ¡;,(i potlll:!,.

ti. C lr1 tllis prohlt•rn. 1\t' 11illr1vl.'d to u·;e l\\0 di!Tet\'tll \~cri~thlcs cÍ!!Cl' tiH' ~tlllOtlllt olrni
[J('a!IS and bladz lwans <ll'l' 110( tkfint•d Íll ll'rlllS or each otlwr. In f<tl'l. that is tlll' ljllt'SlÍOll.

Let tlw <lllH>ttnt of red beans itl tlw tni;.;tme lw x.


T!Jc cost or ll'd lWUlllll'S S:2 IÍilll'S .\',

Lt•l the ~tmmtnt ofblack lwatls inthe lllÍ\lurc~ he y.


Tlw co~t of black bccnnws $:\times y
C)ucstion:
. y = <.
:S

Price X Amount Cost


Red $2/lb X 2x

Black $3/lb y 3y

Mixture $2.75/lb (X + y) 2.75(x +y)

cosl 1 + cnst" = total cost

2x + 3y "" 2.75 · (x +y)


2x + 3y -== 2.75x + 2.75y
3_1'- 2.75y = 2.75x 2x
0.:?5t' = 0.75x
,t = Q.:Z_;) = l
y 0.75 3

7. E Thc aJtJount of acid in tiH' origillalmix is -10 · 0.8 -== 32 quarts.


Thc amount of water in tlw original mi;.; is 40- 32 = B quarts.
Tlw 11('\\' concentration is /0''\, 70 -- 7
roo - Tú·
, . , ., ·
Conccnttdtton . Amount of acid_
~= -.--,--- -------·.----- ·
lo tal mt\t u re
3:! + .r _ {
::-1Ti ~~--_,: +~\· - To
10(:l2 + x) =-7 7(L10 + 2x)
320 ·+ lOx ,-_ 280 -1 J4x
40 = 4.r
x lO quarts Co back lo thc question ami find '· ut 1\ hat it is asking.
The rcsulling solution has lO + 32 o=- 42 quarts of acid.
B. A Let the amount of beans from Guatemala in the mixture be x. So thc total cost of
beans from Guatemala bccomes x times b $/lb.
l.et thc amount ortwans from Ethiopia in the mixture be 3x. So the total cost of beans
from Ethiopia beconws J.Y times a $/lb.

,\vcra•'e cost = c)<lt;~l_~~~ls_t_


,.., lo tal mtxture

-·_x·b+:\x·n
-:r·-~-3:(~-

x · ú + 3x ·a Factor out an x from the numerator ami the denominator.


--_~+:Ií.:---

x. ( h .L 3. (1)
-- ·-:¡:¡:---- Simplify thc x's
( !J + 3u)

·r·¡ 1c cost per puuml o t. t 11e nuxture


· · (/J
1s + 3 al
----;r--·

Tlw amount of simple interest can be calcu!ated as

Intcrest = Principal· Yearly interest rate ·Time in years.

This is commonly written as


I=P·r·t
If interest is compounded, the resulting balance (principal + interest) can be found as follows:
. . ( Yearly intercst )Numtwr ofpt'li<Hls pt:ry¡•ar < tÍIIW in p·ars
Balance = Pnnopal · 1 + --·---·---:----····----
N umber of penocls per year .

The fiJrmula is
B= P· (l+lir)". r

SAl'vJPLE PHOBLEMS

How much interest is earned if $12,000 is invest·é!d for 9 months at 4 percent simple
annual ínterest?

I=P·r·t
9
!= $12,0.500 · 0.0·! · 12 Nolicc th;~t wc cmwcrl9 months into -p2 = ~ years.
I = $360
What expression could be usc:cl to calculatc: the be-dance after 4 y(;ars if $2.000 is
invested <:t 6 percent in~erc:st compoundcd semia•wually?

"
l•
¡; 1
/'
1/
(1 'll( í
Ji Sinn· tiH· comptllttHiittg i~ d()!ll' ~t'llti<ttl!Jll<tli:-. 11
')
~~.!)()()'' 1
-
')

n s·...'.IHl!l. 11 (J.(l:)) S:..'.Oilll · 1 1.IUJ

!la t'l'II<!Íil amounr ot'molll'\' is di\·itlt>d ÍIJIO portiom to Íll\l'SI al ditlervttl itHerest rate-;:

S,\[\ll'l.E PHOBLF\1

Eve invested $3,000 at a certain simple interest rate and $4,000 at a simple interest
rate that is 2% higher. What was the interest in her first investment if tfle total interest
she earned was $360?

lntercst 1 + Interest, =Total interest


Principal x lnterest rate lnterest ($)
lnv. 1 $3,000 r 3,000r
lnv. 2 $4,000 (r 0.02) 4,000(r + 0.02)
-------- ----------------
Total 360

S3.000 r ' S-l,OOO ( r -f 0.0:..'1 S3GO


3,000r _, ·l,OOOr + 80 3GO
7,000r '~ 2BO
.' 2él0 -- 21!0 _) 70 - 4 -
' -- 7:üoo -- 7:ooo =- ?ü --- ·nm - 4';0

l. ÜO% ofS 10,000 \\aS in\TSted in srock. A \\'hich lo:,l 20<ló of its \'<11uc. Thc resl ,,·as
invested in stock B. which ¡:;ainecl30% in \'alue. \\'hat is the m·er1'll perccntloss or g;Jin
from this Íll\'C:.tnwnt7

(¡\) 20'l(, gain


(Bl lO'!(, gaín
(c:J N u loss or g<:in
(D) Hn;, luss
(E) 20')0 luss
2. A portfolio contains GO% bonds yielding 4% per year and c!O% stocks yielding 8'J{, per
year. What is the expccted yield of the portfolio'?
(¡\) 1.6%
cm -U3'/o
(C) 5.2'?\:,
(0) 5.G%
lE) (i.O%

3. Shelly depo,its $12.000 in a bank that pays 4% simple intcrcst per ycar. lf Shclly's intercst
carnings are taxecl at 15'!b, lww m u eh total money \\ ill shc have at the end of one year?
(A) $12,480
(B) $12,-108
(C) $l2,09G
(D) $-IOU
(E) $3B4

4. Nadcr has a crcdit card with an annual interest rate of 24% ami a balance of $3,200.
How much interest will he owe in one month?
(A) $3,264
(B) $640
(C) $64
(D) $32
(E) $24

5. Greg invests -k of his $15,000 at 4% simple intercst rate. He invests ~ of the remaining
amount at 6% simple interest rate. Finally, he keeps the remaining amount in cash with
no return. vv'hat is his average rate of return at the ene! of the year?

(A) 4t%

(B) 4-z%
1

(C) 5%

(DJ r 1 o:·
:JZ,o

(E) 5!%

6. U Teresa invests Tclollars atan annual simple intercst ralc of i%, how much moncy she
will ha\·e at the end of z years?
(A) T· i. z
T· i · z
(B)
-rocf-
(C) T·b
"'
T+ Ti;:,
(D) -Too·--
1001' + Tiz
(E) --fu o--
- Stit' ll:h 111in· ;¡,.. llllldl lllOIH'\ ;¡.., l ':trrit·. ll't :!!Ti,• IP:Ikt•, :l.:;",. ~!mp!t· inkn•-,¡ ¡wr \t'<ll'

dlld ~lit' lll::k,., )' .. '-itt!pl,· iiHt·n·-,¡ ¡w:· \t':!l. 1\lí:tt Í'> 'l1t• r;rri" 1)1 !lll' illi<'l'";[¡•;¡¡·;wd l11
( :IITit• [{) !h¡· i!ll<''<"·.t l',JI'!I'i! h1 \itt' irr tli'• , ¡·;¡¡·:

\ 1 ''
lil
¡; 11
:~ {)

( l!i
l1
:.'11
'1 );
11
¡j i 1
·¡

H. ( ;!oh:t! l::;mirlg,, ltw. op,·Idit", i11 lllll ,.t,ll!Iirit·~. r tJi!llll'\. \', t:r\ r:t!t' i•, !ll!'t't' linH·s ;¡·;

!mwlt ;¡, Ctllllltl\ /1'.-., t:t\ r;¡fe. 11'< d()il;tl', ''<lrtlirtg¡; ir1 t'tllllltl'\ !J ;m· !•xiu· ;¡s rnttdl :t-> its
t'<lillings in L'Olliltl'\ :\. 11 1!:11 i~. tiH· r:ttio uf( ;!ob:il's <l\L'r:tgt' t:t\ ratt' to l'lllllltr~·. \\ ta\ ratc'c
')
.)
(:\1

(Bl S
(i

(CJ 1
j
')
(!)) -
')
,)


(El
~}

l. D Gain/loss from a stock investrncnt is calculatcd exactly tiJC samc way as wc calcula te
intcrcst. Thc only diiTcn~nce is that thc loss is shown a~; a ncgati\l' v;duc.
BO"i, of S 1fl.OOO == fUl · S 1(1JJOO o~ S8.000

So SB,OOO is invcstcd in stock;\ and $2,000 is itl\'l'Slcd in stock B.


Gain/loss, -+ Gain/loss:, =-'Total gain or loss
2,000 · 0.3 -- 13.000 · 0.2 = 600 -- l.GOO =.- -1.000
TI 1cre fore t 1ll' pcrcent loss IS - l. OOO · 100
. Tó,OCló- - 10'.!'u

2. D lntcrcst 1 + lntcrest, = Total intcrcst


O.G · 0.0·! + 0.·1 · O.OB == 0.05G =~ 5.G"\,

AitL'I'Il<lti\·ely, assume thc portfolio starts \\Íth SlOO. Th<ll makcs bonds lOO· GO'?ó = StiO
and stocks 100 -- GO ==S lO.
Tlll' total yicld is
fi(' · 0.0·1 ·:- 40 · O.OB ~ S5.GO, '' hich is .'iR!;,,
3. B I = P · r · t
I = 12,000 · 0.0! · l = S-!80
She pays 15');¡ tax on S-WO ollly.
Tax e-= 0.15 · -IBO = Si~

\iet ínterest = S-fBO = S/2 = S-10!3


Totalnct amount = S12.000 + S IOB = S 12.408

4. e 1 =p. r. t

Note the time is 1 month, which is l~ ycars.

1 = 3,200 · 0.2-1 · ~
1 Simplify llrst.

I = 3,200 · 0.02 ce" SG 1


Nadcr \\'ill owe SlH of íntercst in otw month.
¡
¡
~-
5. A Average rate of return = :-T_cJ!<lllD!exe,2_t__
fotallnvcstmcnt
(P 1rJ) + (P,r2 l) + lP,r,tl
15.000 ---·---

Sl5,000 X t = $5,000

$5,000 at 4'?\J for l year


Remaining amount = $10,000
~ of thc remaining = $10,000 x ~- = $7,500

$7,500 at 6% for 1 year


$2,500 at 0% sincc it is cash \•Vith no return
(5,000. 0.04. 1) + (7,500. 0.06. 1) + (0)
---------ls,oüo ____________ _
200 + 450 - 0 043- -- 4 l ()
-rs.oüo-- - · -- 3Yo
6. E J= P · r · t
Notice that í% translates into ~ .
I= T·-L·z
100 10
I= L.L~
lOO
Total amount = T + LJ · z = IOO_L-l:...IL<.:
100 100
7. B Lct Carrie's moncy be x. Then Suc's moncy bccomcs 2x.
Intcrest earncd by Carríe _ x. o.O'í5 . 1
-Inict:C:st C<trricdTr¡' sí.le·- -- :zx-: ·a.cs-:-r
0.055- 55 - 11
-o~nr- Tüo -- 2o
8. E Lct country B's tax rate be t and country /3's earnings be 2E.
Country 1\'s tax ratc becomcs 3t, ancl its carníngs bccome E.

Awrauc tax = ;-;:-_]Q_ta_l__t;-\~--- = -;!~:"\_-~:r~~ 8 -


b lota! carnm,~s 1otal carnmgs
t · 2E + 3t ·E_ 5tE _ 5t
- -3E --
---,n~- +-7:.~-- -:r
5t
Average tax ratc -j- 5t 1 . 5
c~()(iJltry-/l'S\~::<-rat(, = 3T = ·:r · 3t ~~ 9
SECTiON 4.6: SET PROBLEr-.;s J\\ND VENN DIAGRA.f'ifS
A set is a collection uf things. These things are called elements or membcrs of the ser. The
easiest \\ay to so!n~ sct problcms is tu rcpresem the groups Venn diagrams as shmvn
lwluw.

The totalnumber of clemcttls in setA is n(r\) = x.


The totalnurnbt'r of ekments in sn n is n(IJJ y.
Tlle m·erlap = /1.
,-----------e-------·
13

\' -- ¡,
x - h llllllli.'l'r "f ·-~~- - - >· y ·- h oc numher nf
..:kmc:nts in 'd :\ rmh dcmcnts in ó'l'l B r>t1/r
11

'\\
\ numbc:r of dcments that
b = number of L'lcmc:nts that
bdong to ncither sc:t
btlong: to 'c'h A wul B (nvcrlap)

In general:

Total= Groupl + Group2- Bnth + Neither

If only the number of clements in either A or Bis askecl:

n(A or B) = n(A) + n(B) - n(A ancl U)

SAMPLE PHOBLEMS

In a group of 55 people, each person speaks French, German, both, or neither.


17 people spe¡c'< French, 22 people speak German, and 6 people speak both languages.
H<::>w many pe<..;>le in this group speak neither German nor French?

Always sUut by calculating tbe overlap ami placing it in the micldle.

r::-r:~:;\~
~0:)
Total== n(F) + n(G)- n(Pand G) + Neither = l7 + 22 -- 6 + n =55.

So!vc for n.

:n + n ='55
/1 e~ 55 - 33 = 22

22 peoplc speak neitl T Frcnch nor Gcrman.

Note th~:t there are l7 -· 6 = ll French-o:1ly spe<~kers aml22- 6 =-"' lG C('rman-ouly speah~rs.
------------------(¡~~~---------------~

Each of 35 people has an MBA degree, law degree, or both. lf 18 peop!e have t'IBA 1

degrees and 23 people have law degrees, how many people have MBA degree=:_j

Let m be thc number of peoplc holding both degrees (ovcrlap).


nC\1 or Ll = n(;\1) + 11([.) - HC\l and [.)

lB + 23 - /11 = 35
4l --m= 35

m = 6 is the number of people H·ith both dcgrees.


So there are lB - 6 = 12 peoplc who llave :\llH dcgrces only.

Alternalivcly. use a Venn diagram.

Since the total number of people in the group is 35,

18 - m + m + 23 - m = 35
41 - lll = 35
m 6 is the number of people with both degrces.
So thcre are 18 - 6 = 12 pcople have MBA degrees on!y.

A school library contains 4,500 books comprised entirely of fiction and nonfiction,
hardcover and softcover. In the library, %of the books are hardcover and 3,000 of
the books are nonfiction. lf 300 of the hardcover books are fiction, how many of the
softcover books are nonfiction?

Noticc that two separatc propcrtics are assigncd to each book: fiction/nonfiction and
softcovcr/hardcovcr. \Ve can organize these elata into atable as shown bclow.

~ of the books are harclcover.


%
:J
· 4,500 = 2,700 hardcm-cr books
Softcover Hardcover Total

Fiction 300

Nonfiction 600 2,400 3,000

Total 2,700 4,500

After inputting all gi\'l'll data (shown in bolcl), we can easily ca!culate the missing numbers:

I-Iardcovcr nonfiction: 2,700 - 300 == 2,tl00


Softcovcr nonfiction: 3,000 - 2,400 == GOO.

1'75
SECTION 4.6~PRACTICE PROBLE~1S

l. In a certain high school, al! studenrs lakt~ eithrr American or world bistory dassrs.
96 ~tltdents take :\merican history, am! 1-1 l takc \\orld histnry. lf:.'l studcnts takc !1oth
clas·;es, \'.hat is the ratio of students takin¡,; only .\merican hiswrv to those taking only
world history?
')

(:\)

(BJ :l
:i
( C.) 2
3
(D)}
lE) J
:J

2. Out of 90 CE Os surveycd, 32 previously worked in marketing, 40 of them \\·orked in


financc, and l il c~f them workccl in both. How many of them workcd in neither marketing
nor financc'?
(:\) 10
(8) 18
(CJ 26
(D) 32
(E) 36

3. In a group of 69 peoplc having breakfast ata diner, each person ordered coffee, orange
juice, or both. The number of people who ordered only coffee is 4 times as many as the
number of pcople who orclercd both. If 4 peop!c orderecl orange juice only, how many
peop!e ordercd only coffce?
(A) 65
(D) 52
(C) 26
(D) 13
(E) 13

4. A car lot contains 200 vehicles comprised entircly of cars ancl trucks, usecl and new.
60n1;J of the vehiclc~s CJre trucks, ami 140 of the ve!Jides are new. Ií ~- of thc cars are ncw,
how many ofthc trucks are used?
(AJ 30
(B) 40
(C) 50
(D) 60
(E) 80
5. 55% of the incoming l\II3A class play a musical instrumcnt, and 45°\, of them speak 2 or
more languages. to•;:¡, play an instrument and speak 2 or more languagcs. !f 20 studcnts
neither play an instrument nor speak 2 or more languages, how many of them speak 2
or more languages but do not play an instrument?
(:\) 35
(B) ,15
(Cl 55
(0) 70
(E) 90

6. A survey is conducted among 5GO memhers of a Llook club about two new books. The
number of pcople who likcd both books was equal to thc number of peoplc who liked
neither. The number of people who liked book A is 5 times as many as the number
people who liked both. The numlJer of people who liked book Bis 3 times as many as
the number people who liked both. How many peop!e liked book B only?
(A) 70
(13) 140
(C) 210
(0) 280
(E) 350

7.

Horror

A marketing survcy among moviegoers is shown above. If 85 pcople liked comed y, 4¿


people liked horror, and 60 peoplc liked action, how m;my people liked only one type
ofmovie?
(A) 187
(B) 119
(C) 6,1
(D) 38
(E) 16

8. There are 1,256 ncw cars in a parking lo t. Eacll car has eme or more of the follmving
items: a navigaliun system, ;; CD phyer, ora sunroof. If thcre are a total of 400 sunroofs
and 55-1 na\·igation systems ami if 1,002 cars ha\·e at mosl 2 itL~ms, how many cars ha ve
al! tlu-ce items?
(;\) 254
(B) 302
(C) 951
(! ) 1,208
(L) It cannot be cl~\;_~rmincd frum thc informatiun givcn
SECTION 4.6-SOLUTIONS

') :l
E B

l. B ,\mvrican history otlly:% - 2 i ,_


~·¡

\i'orld history only: l 1-1-21 = 120


_Z/.-
120
.--~ :!
:)
Henwmber to check tlw JllS\\Cr chuices tirst. Simplifying tlw fraction is sufllcient in this
case.

2. E Tlw overbp is alrcady giwn, so pul that in the middlL• of the Venn diagram.
Jf tl)e total numher of CEOs with marketing background is 32, thcn thc CEOs with a
marketing background only is 32 ·· · lB = 14.
If the totalmtmber of CE Os with financc background is 40, tlll'n the CEOs vvith a
finance background only is 40- 18 = 22.

11

Neithcr = 90 - 14- 18 22 = 36
Alternatively, use thc formula
Total = Group 1 + Group, - Both + Ncithcr
90 = 40 + 32 ·-18 + Neither
Neithcr = 90 -- 40 32 + 18 '= 36

3. 8 I.et the numhcr of people who ordercd both bcverages be 111. Thc numbcr of peoplc
who orclcred coffce 1 nly bccomcs 4m.

4 /11 + 1/1 -f 4 =o 69
5111 = G5
m,.. 13
CoiTce only ,,, tJ¡,· oc .¡ · 13 ,~52
4. A
New Used Total
-~- -- ----- -·- --~--- ---- --- -- -·------ -- ---·-·
Cars 50 30 80
Truck 90 30 120
~---~---------

Total 140 60 200

GO% of the vehides are trucks mcans O.G · 200 = 120 trucks.
The numhcr of cars is 200 - 120 = !lO.
Artcr linding thc total number of cars, f¡ of the cars are new, so ~ · BO = 50 new cars.
Since tlwre are l 10 ncw \eilide~. HO -~ 50 = 90 new trucks.
So there an~ 120 - ~JO ~o 30 uscd trucks.

5. O Instrument only • 55'!\i -JO% = 45"h


Language only ',E:i'i{¡ -10°(¡ = 35'!~)

Neithcr = 100% - 45% - 10%- 35'i'G = 10"{¡


If 10% of thc class is 20 s.tuclents we can set up a quick proportion to find the total dass:
10 - 20
lOÜ- x
X= 200

Thercforc, the number of people who spcak 2 or more languagcs but do not play an
instrument is
35%.200 = 70

6. B Let the numbcr uf pcoplc who liked both be P.


Thc number ofpeoplc who liked ncithcr is also P.
The number of pcoplc who liked A becomcs 5P.
The numbcr of people who liked A only is 5P -- P.
The numbcr of peoplc who likcd Bis 3P.
The numbcr of people who likcd B only 3!' P.

4P+ P+ 2P + P =--' SGO


81' = 5b0
p occ 70

Thc r;umbcr of pcople who Jikccl R only is 2P = 140.


¡\hcrnatively, ynu cuuld use the formula without the di;;gram:
Tot~il ·' Cmt,pl + Group 2 -- lloth ,. Nt•itl!er
:)liO ~ 5P ·~ :l P - fJ + ¡;
:íliO Bl'
J.!O :'.P

l J,HTor un!:.· is f2 -· t) ·· ~J il ·~ lti


.ktion un!·; is GO 5 - () ll '~ ~\g

ti5 1 l () L ~}H .., 11 ~J pc:opll' like one genre only

B. A Tllc· plH,i':>l' "at 11Hhl 2" mcaitS tlw su m of none, only one. or (\\'O uf t!le items.
In otLcr words. "~:l mosl" lilf'<li1S "not all three."
Su \\'C can siinply subtract 1.00:~ from the totalmunber to fincl thc humber of cars tlwt
llave al! thrce
l,25G -·- l.Oll:J.. = 25 l
Geometry

-+ 5.1 LINES, ANGLES, ANO TRIANGLES


-+ 5.2 QUADRILATERALS, CIRCLES, ANO SOLIOS
-+ 5.3 COOROINATE GEOMETRY ANO GRAPHS
-+ 5.4 DATA INTERPRETATION

SECT!ON 5.1: UNES, A~ Af\BD TRIANGLES


lines
m A !in e is made up of an infinite number of points. It is straight and
extends infinitely in both dircctions.
111 Unes can be named individually, such as line C1, or they can be referred
to using two points containecl in the line, such as line Kl~

K .(¡
T

m In the figure below, line C1 intersects line C2 at point K So La and Le are


vertical anglcs, ancl Lb ami L d are vertical angles. The me asure of angle
a (mL a) cquals the measure of angle c. The measure of angle b equals
the mcasure of angle d.

-{'¡

a = e ami b == d
• The su m of the nwasures of all anglcs on one side of a straight line is lB0°.

ll + b -= 180''
\L
a + b + e = 180"
• A right angle is indicated by a small square in the comer. The degree measure of a
right angle is 90°. lf two !ines interscct ata right ang!e, they are perpendicular to each
other. All angles formed by those intersecting lines are right angles as \\'el!.

L Right angle = 90°

• Thc sum of the measures of all anglcs around a point is 3G0°.

a+ lJ + e+ d +e= 3G0°
In addition, since KTis a line:

a + lJ + e== 180° ancl d + e== 180°

111 Lincs are parallel to cach othcr ifthey are on the same plane and do not intcrsect. To
indicate that el is parallel to e2, write itas el 11 e2.
111 If two parallellines are intersected by a thirclline as shown below, eight angles are
fonned. The four small angles (named a) have thc same degree measure. The four
large angles (named b) have the same dcgree measurc. In addition, a+ [; = 180°.

o h (/
"-1 - - - - - - - - - - - - -
a b

m A polygon is a closed planar figure made up of threc or more liue segments. Each !in e
scgment is callcd a side, ancl thcir intcrscction point is callecl a vertex of the polygon.
s A diagonal is a !in e segment that connccts two nonac!jacent vertices of a poly·gon.
Thrce 'ides: Four oidcs: Fi,·e siJc': Six sidcs:
Triangk (;•;adrilateral Pcnta¿:un lkxagon

\
\
\ \ ,¡¡~~
.----\- \
\ \
\ dia:: ':1al
\ ' 1
\ \
\
------.
• The perimeter of any polygon is the sum of all of its side lengths.
• A regular polygon is a polygon that has al! equal sides and equal angles:
Regular Regular Regular Regular
Triangle Quadrilateral Pentagon Hexagon

6 D o o
INTERIORAND EXTERIORANGLES
• Interior angles are angles inside a polygon.
a The su m of th.e interior angles of a triangle is 180°.

a+ b +e= 180°

111 The sum of the interior angles of a quadrilateral is 360°.

X + Y + Z + 1 = 360°

a In general, the sum of all interior angles of any polygon can be founcl with the
formula: (n- 2) · 180°, where n is the number of sides.
Example: For a hexagon, the sum of the interior angles (6- 2) · 180° = 4 · 180° = 720°.

m Yo u can pick one of the vertices of the polygon and draw al! diagonals from that point
only. Thc su m of thc interior angles of that polygon is equa! to 180° times the numbcr
of triangles formed.

The su m of the interior angles ofthis hexagon is


1 \ ',
1 \ 4 . 180° = 720°.
1 \
1 1 \
1 1
1 2 1
1 \
1 1

a Exterior angles ; re angles formcd by one sic! e of a polygon ancl extcnsion of another
side. h thc diagram below, n, !J, and e are exterior angles.
• In a lriangll'. anexiPrior angle is the su m of the two remole interior angles.

~ ~ti+/)

• The su m of all exterior angles of any polygon is always :lGO".

Triangles
• In ewry triangle, the longest side is opposite the largest angle and the shortest side is
opposite the smallest anglc.

e lf !J < a < e, then B < A < C.

~
If B < A < C. then ú < a< c.

B i\
e

• In every triangle, each si deis shorter than the su m of the other two si des amllonger
than the absolute val u e of their difference.

a + ú > e > 1a - bl

RIGHTTHIANGLE§

m A triangle with a 900 angle is callee! a right triangle. The longest side is called the
hypotenuse. The other sidcs (perpendicular sidcs) are called the Iegs.
• Thc Pythagorean thcorem states tlwt the square of the hypotenuse is equal to the
Slllll of thc squarcs of the legs:

(hypotenusc) 2 = lcg, 2 + leg/ or 11 1 = a 2 + ú 2

(/

m Severa! right trianglcs are frcqucntly ~;··en.

3-4-5 triangle: A triangle \\·ith sides proportionalto 3 : 4: Sisa right trianglc.

Examplcs: Thc l'ythagorcan tlworem for a triangle with sidcs


3, 4, S is 52 """4 2 + 3 2 • For a trianglc with sides, 6, 8, lO,
thc l'ythagorcan thcorem is 10 1 = H1 + G'
.'i.r
4.r

Jr
5-12-13 triangle: A triangle with sides proportional to 5: 12 : 13 is a right triangle.

Examples: The Pythagorean theorem for a triangle with sides 5, 12, 13 is 13 2 = 122 +5 2•
For a triangle with sides 15, 36, 39, the Pythagorean theorem is
39 2 = 15 2 + 362

• In a right triangle, the line segment drawn from the right angle to the midpoint of the
hypotenuse is equal to one-half of the hypothenuse:

A~--------------~c AE= BE= EC

SPECIAL RIGHT TRIANGLES

11 In a 30-60-90 triangle, the sides ha ve the ratio of x: x/3 : 2x.

Example: If DF = 12, DE must be half of that, D


which is 6, and EFis /3 times DE,
which is 6/3.
X

xf3 F

11 In a 45-45-90 triangle, the sides ha ve the ratio of x: x: x.fi.

Example: If AC = 7, BC must also be 7 and AB A


is .f2. times AC, which is 7.f2..
X _ X _ x.fi
AC- BC- /üT

AREA OF A TlHANGLE

D The hcight (or altitude) of a triangle is a line segmcnt from one of the vertices
perpendicular to the side oppositc that vertex. Thc side perpendicular to the height is
callee! a base.

1
b~i·•ln 1
. "' 1

-----~--·

bas.: 1
• The arca aml perimetcr of a triangle can be ealculated as follows:
1
Arca =~ x base x height = !bh
Perimeter = a+ ú+ e e

• The arca of a right trianglc beco mes:

Arca = ~ x leg 1 X leg 2


leg 2

leg 1

ISOSCELES TRIAlVGLES
• In <m isoscelcs triangle, two sides and their opposite
angles are equal.

11 The altitude between the two equal sides also divides the
angle of the vertex and the base into two cqual pieces.

111 In an equilateral triangle, <.i\l sides are equal and all


angles are 60°. a
60
a a

lll Each height (altitudc) of an equilaterallriangle


1
di\ id es lhc vertcx angle and the opposite side 1
1
into two equal picccs. 30" 30'
1
1
lll Notice that whenever you draw onc of the a 1
a
1
heights, you create a 30-G0-90 triangle. 1a/3
,--
m The area of an cquilatcral triangle whcn its 1 2
1
side is givcn cm be calculated as: 1
(iO"_ _ _ _ _L n 6Cf_
..______
t\rca of au equibtcral triangle = -0-- a 2 ([ a
2 2
SIMILAR TRIANGLES
• Similar triangles ha ve the same shape but not necessarily the same size. They have
equal corresponding angles, and their corresponding sides are proportional.

Two triangles are similar if


[)

m La= mLy, m Lb= mLx and


mLc= mLz
The corresponding si des of similar
triangles are proportional:
E
AB _ AC _ RC
DE-DF- EF F

B If KT is parallel to BC, then 6AKT is


similar to 6ABC.
e NOTE
If 6AKT ~ 6ABC,
lf a line segment parallel to one of
AK _ AT _ KT
AB- AC-BC the sides is drawn inside a triangle,
the two triangles become similar.
A

SAMPLE PllOBLEMS

What is the degrce measure of one of the exterior angles of a regular 9-gon?

(A) 140°
(B) 120°
(C) 80°
(D) 60°
(E) 40"

The su m of lhe interior angles of a 9-gon can be found by thc formula (n - 2) · lBOo = (9 - 2) ·
180° = 7 · lBOo
Sin ce it is a regular 9-gon, fine! the degree me asure of one of the interior angles by dividing
the s:nn by 9: 7 .l~Q_~ = 1400.
Finally, since an exterior ang!e andan interior angle at the same vertex always add up to
180°, the degree mcasure of an exterior ;:ngle is JHOO - 140° = 40°.
Alternative!y, reme'~1bcr that the sw : of al! exterior angles of any polygon ec¡uals 360°.
3(''10
Thercfore one exterior angle of a regular 9-gon is "-~'!- = 400.
The answer is (E).
---------- 4UJ:tQ!!» ------- -----------~
Based on the figure shown, 180° is how m u eh greater than 7x? 1

(A) 105°
(8) 75°

(C) 70°
(D) 15°
(E) 10°

To find thc value of x, set up an equation wherc all of thc angles shown add up to 3G0°.

7x + 4x + 10 + x- 5 + 90 + 85 = 3GO
12x + lBO = 360
l2x = lBO
X= 15
1
This means 7x = 7 · 15 = 105.

180 is 75 more than 105 (180 105 = 75).


1l
'
The answer is (13).

The height of a triangle equa!s (t + 4 ), and its base equals (t - 3). lf the are a of
triangle is 9 square inches, what is the value of t?

(A) 4
(8) 5

(C) 6

(D) 7

(E) 8

The arca of a triangle is fouml by 1· base· hcight, so 1 + 4)(1 -- 3) = 9.


(1

1 (t + 4)(1- 3) = 9 f\Iultiply hoth sides by:?_


(1+4)(t-3)=18 [)istributc.
12 + 1- 12 = lB Subtract lB from both sidcs.
/
2
+ t- 30 =o Factor.
(1 + 6)(1- 5) = O
t + G = O, 5=0 Since thc sidcs cannot be negative, r = 5.
1=-(j f~o5

The answer is (B).


SECTION 5.1-PRACTICE PROBLEMS

l. What is the degree measure of angle C?


(A) 40° B
(B) 50°
(C) 60°
(D) 70°
(E) 80°

2. In the figure below, what is the value of y?


(A) 20°
(B) 500 ,el
(C) 60°
(D) 70° 2x + 20
(E) 110° X+ 30

3. In the figure 111 m. What is the value of x +y?


(A) 30° m
(B) 60°
(C) 90°
(D) 120°
(E) 1800

4. In the figure, a = 2ú, b = 2c, and e = ~· What is the value of rl'?


(A) 18"
(l3) 36"
(C) 45°
(D) 5,¡o
(/
(E) 600

5. Ifthe minute hancl ofa dock movt~S 72°, how many minutt~s oftime passes?
(A) 7.5
(B) 10
(C) 12
(D) 15
(E) 45
(). In the figure, iW = ·1. What is Be?
(;\) ¿.¡~ ('
Wl :!.Hi
(Cl ·1.f:!.
(1 l) B
(1'.) 4j(i

,\

, . In the figure below, what is the area of ¿_\¡\JlV(P


,\/
(i\) 45
(B) 60
(C) 90 15
2x
(D) 120
(E) 225

8. In the figure below, what is the value of .r?


(A) 20°
(B) 30°
(
7a
(C) 40°
(D) 50° 1
(E) 60°
l
4a 60"

1 q h;!l is the valuc of z bascd on thc figure below'?


\1 ~, .
(B) ¿¿

(Cl T!
(!)) 42 -
(L) 74 ~l)(l'

10. T' l ,\·o si des or <lll isoscclcs trianglc ;l/3C are 17 and 15. What is thc grcatest
1ctcr of ;"'.;lBC?
(i\) 32
(Dl 35
(C) Ti
(DJ 39
(E) 4~l
1

190 Gi\lA'í 1\lJ\TH WOrtl< 'OOK


11. In right triangle DEF, E is the right angle. If DF = x + y and EF = :~: - y, what is the
length of DE in terms of x and y?
(A) [2:ry
(B) 2.JXY
(C) 2xy
(0) xy
(E) 4xy

12. In the figure below, which one of the si des is the longesr?
(A) AB .-\
(B) AC
[)
(C) BD
(0) BC
(E) CD

e
13. Points P, Q, and R are on the same line in that order. Point Sisa point that is not on the
same linc and is equidistant from Q and R. If mLSQR = 75° and mLPSQ = 20°, what is
the measure of LSPQ?
(A) 55°
(B) 50°
(C) 45°
(0) 40°
(E) 35°

14. Square ABCD is divided into 16 equal squares. If AR = 3, what is the are a of the shaded
triangle?
(A) 5
25
(B) }(j
45
(C) T6
(D) 15
"l
(E) ~f
15. The figure shows a dialthat is a regular hexagon. The arrowin thc middle turns clock-
wise 1500 evcry 20 seconds. Between which two numbers will it stop aftcr 2 minutes
and lO seconcls, if it starts by pointing to 1?
(A) 1 and 2 o
(!3) 2 and 3
(C) 3 nnd .¡ S

(D) ·1 and 5
(F) 5 andO
-1 2
1 l (). In !he figure shmvnlwlow, a. /J, antl e are ex!Prnal ang!es. If 2:l:io -~ /¡ f- e< 27:io and
a. h. ami e are integers. what is t!w gn'atest possihle ,·allll' of me Bi\C?
(¡\) !l(jo

{B) mr
(C) ~Jtr

(!)) q2o

(El 94° e

17. In triangle fJ()f? m,~ fJ is greater than 90°. lf PQ = :l and PI?= 5. what is the su m of al!
possible integer values of /U.)'?
(¡\) 12
(L) 13
(C 14
([ ) 17
(l· 19

lB. 1t the figure bdow, mLABC = mL CBD. What is the value of x?


(P 180 A
(t 20°
(C. 24°
([•) 3()0 lJ

(E, 42°

,_,¡;
19 . .6.ABC s a right trianglc whcrc mL C = 90°. lf thc arca of Úi1BC is 30 ami a- b = 7,
what i: thc lcngth of the hypotcnusc'? ....··'
·.·. ':11.

"'·
(/\) 11 ;\ .r;.

~
¡
(B) 13
(C) 1o.J2. h ·¡.
(Dl lO.J:l
(E) 19
e (/
-
¡¡
;l
.:r
.:t
20 . .6.ABCis ; . .1 isoscelcs right triangle (;W = AC). lf i\0 ~ 3 ami DC = 8, how long is Bt:? J'J
t.
(¡\) sfi ('

(B) 7
(C) 7fi E
·:j··
';'

(D) 9 ·:.1: .
n.J2
(E)

,\ fl
·sl
·:t¡
·l

_l
S
$
{
21. The si des of a triangle are 4k, 5k, and 6. Which of the follmving must be true for k?
(A) 1 <k< 9

(ll) ~<k< 9
'}
(C).:::. <k< 6
3 .
(D) 2 <k< 8
(E) 3 <k< 9

22. Triangle 1\1KTis an isosceles triangle. l3ased on the figure below, what is the ratio Í = ?

.J2
(A) M

(BJD~t
(C)2 ~
(D) 2.fi K y T
(El 3.fi

23. In the figure below, MNK and PQR are equilateral triangles. If the su m of their perimeters is
36 and the shaded area is 2l.f3, how much Jonger is one side of MNKthan one side of PQir?
(A) 6
(l3) 7
(C) 8
(D) 6[3
(E) 7 .f3

24. AEC is a right triangle, and FBDE is a rectangle. Based on the lcngths of the si des shown
in the figure, what is the val u e of x?
(A) 5 .4
(B) 6 6
(C) 8 F
(D) 10 6
(E) 12
E 8 [) e
25. l3ased on the figure, what is the value of n in terms of b?

(A) 3b ~--------
B e
(B) 30 -f 2a- :_¡,

(C) 30 b

(D) 90 - .J2
3 /)
(E) 30 + :~

26. In the figure below, with ( 1 11 ( 2 , vvhal is the Vil' · of x?


(t\) 30° A --..---2:::-_\·.-_-_~3(-::-l- - - ,f 1
(B) 35°
1-Hl" ¡¡
(C) 40°
(D) 45°
(El 50'
!
!
1
~

SECTION 5.1-SOLUTIONS 1
1
..,
1 2 3 4 5 (j ( 8 9 10 11 12 13 14 15 l6 17 18 19 20
1
e E B [) e e A e e E B E A e E E B [) B e
21 22 23 24 25 26
e B B [) E A

l. C LEFC is equal to 50° because LEFC and the 130° angle are on a straight line
(180 - 130 = 50). LFEC is 700 beca use it is a vertical angle to the given 70° angle.
Now that yo u ha ve two of the interior angles of 6FEC. L C is e as y to fin d.
mLC = lflO- 50- 70 = 60.

2. E Notice tbat 2x + 20, x + 30, and 2x + 30 add up to 180°.

2x + 20 + x + 30 + 2x + 30 = 1BO
5x + 80 180
=
5x = lOO
x= 20
Anglc y and the stun of 2x + 20 and x + 30 are vertical angles. Therefore, they have the
same measuremcnt.
2x + 20 + x + 30 = 40 + 20 + 20 + 30 = 110.
So y also equals 110.

3. B Since C is parallel to m, x + 2yon top aml 2x + yon the bottom add up to 180°.
Notice that this is the only equation that can be written. Therefore it is not possible to
sol ve for x and y separately. Oi1 the othcr hand, the question asks for x + y.
x+ 2y+ 2x+ y= woo
3x + 3y = 180° Divide both sides by 3.
x+y=GW

4. D Since all anglcs add up lo Hl0°, write the main equation first:
(1 + b + e + d = lU()o.

Ea eh anglc 'nr:cds to be written in terms of one single angle. For example, pie k e:
lJ = 2c . Gin:n.
il=c
3 ' This mcans d = 3c.
a= 2lJ Since lJ = 2c, replace 2c for b.
a= 2 · 2c =,le
Rcpbcc all inlo the equation above:

4c + 2c + e+ 3c = wcr
lOe= UlO
e"' lB
d = 3c = 3 · 13,., 51
5. e The minute hand sweeps 360° in 60 minutes. Set up a proportion:
72° _ X
360° - 60 minutes
60 .
11111 t ' . 7'> 0 7?
x= ~~ 0~ 5 ~ =
6
- = 12 minutes

6. e In arder to find. BC. you need to tlnd. one of the sides of 6BDCneeded. Since b.ABD
and 6BDC share DB, t1nd DB using the given length AB.
6ABD is a 45-45-90 triangle. Therefore, the ratios of the si des are AD: DB: AB = x: x: xfl..
This means:
DB_ AB
-1-- {2
DB _ 4
-1 - {2

DB = _±.:__ {2 = 4 .[2 = 2 {2
.[2 . .[2 2
6CDB is a 30-60-90 triangle. The ratio of its sides are DB: CD: BC = x: xf3: 2x,
This means:
DB_ BC
-~--2

BC = 2 · DB = 2 · .f2 = 4.[2

7. A The arca of a triangle is t·


base· height. We are looking for t ·(4x) · (2x) = 4.t-2.
No ti ce that we do not need to tlnd x but only 4x~.

Use the Pythagorean theorem:


152 = (4x)2 + (2xf
225 = 16.\-2 + 4.<-2 = 2(h-2

Since we need 4x~, divide both sides of the equation by 5:


225- 20.\-2
s--s·
45 = 4.<.;! = Arca

8. C In any triangle, an exterior angle is the su m of the two re mote interior anglcs.
7n=4n+60
3n = 60
{/ = 20

One of thc interior angles of the triangle is


4n = ·l · 20 = 80

The interior angles add up to HlO.


X -l 80 + 60 = 180
X= 40
\). C From tiH' figure ..: 1 :\0 1 .r J:lO 1 :l.r ~ :lliO. Add thl' like terms ami simplify to
gl'l z 1 'lx 1 1liO - :lliO.

SuiJ(ract 1liO fro111 l>otll sid!'s to get. :: 1 4.r ~ 200. Sin ce tlw qul'Siion prm·ides
.\ '" 10. sol\"l' tlll' l\\·o cquations simultaJJPously using elimination:

- ¡ 4.r = 200
+ .r-::-10
:i.r 70 2 1()
.r"" .¡¿

10. E There are two possible isosceles triangles: 17-17-15 allll J:i-15-17. Tlw one \\"ith the
longer perimeter is the first one: 17 + 17 + 15 ""49.

ll. B Use the Pythagorcan theorem: DP = EP + DE 2 •


+ y) 2 =
(.r (.r + yl 2 + DC
.r 2 + 2xy + y 2 = .\.z - 2xy + )' 2 + DP
2 2
Subtract x ami y from each sidc.

2xy= -2xy+ DE' Add 2xy to ea eh side.


4xy = D/:,. 2 Take the square root of both si des.
DE= 2,fxY

12. E In trianglc AIJD, the longcst side is !JD because ít ís across from the largest angle.

BD > AD >!lB sin ce 65 > GO > 55.


In triangle BCD, CD > IJIJ > IJCsince 70 > GO >50. Therefore CD must be the
longest síde.

13. A First, sketch thc poínts as dcscribcd:.

/ .

1'

NolÍlT tllat 75° is an extenwl anglc of lri<111glc f'S(!. Since an externa! anglc is the sum of
the two oppositc intemal angles.
75° =- 20" -f mL'ifJQ. Thcrcforc lllt~Sl'Q = 55°.
14. e
In this case, it is easier to count the number of squares that are not
shaded first.
There are 4 squares to the right side. Are a 1 is half of 4 squares, so it
equals 2 squares. Area 2 is also half of 4 squares, so it equals
2 squares. Are a 3 is half of 6 squares, so it equals 3 squares. In total,
4 + 2 + 2 + 3 = 11 squares are not shaded in the original problem.
This means 16 - ll = 5 squares are shaded.
Each small square has a side of ~ and therefore an area of(tt = r6
. Since 5 squares
were shaded, the total arca equals 5 · f6 = f~·

15. E First convert minutes into seconds since the turning speed is given in degrees per
second: 2 minutes and lO seconds = 2 · 60 + 10 = 130 seconds. Then set up a propor-
tion to find how many degrees the arrow will turn:
Degrees _ 150° Xo
Seconds - 20 seconds 130 seconds
Cross multiply 20x = 130 · 150. So x = 975°.
Every 360° is a complete turn, and 975° implies that there were two con)plete turns
(360 · 2 = 720°) pltr) some more. The additional turn after the arrow comes back to
number 1 is 975° - 720° = 255°

The angle bet:vveen each number is 60° because this is a regular hexagon and 3 ~
00
= 60°.

Finally, yo u can find how many numbcrs the arrows passes by dividing 255 by 60. 255 is
more than 60 · 4 = 240. So the arrow will stop after 5 but befo re O.

16. E The externa! angles of a triangle add up to 360°. Therefore a+ b + e= 360°.

Ifwe are trying to maximize the degree measure of LBAC. we will need to minimize the
value of n since they are on a line and add up to 180°. To minimize a, pick the largest
integcr value for b + e, which is 27·1°. Substitute b + e= 274° in a+ b + e= 3600 and
find n = 360°-274° = 86°. Since a+ mLBAC= 180°, ml_lHC= 180°- 86° = 9·! 0

17. B Compare this triangle toa right triangle vvith Iegs cqual to 3 and 5. The hypotenuse
in that case would be n =-~ f!F+-:;2 = f'f:L f:f1 is slightly less than Gsince G" = 3G.
1
In PQR, mLP is grcater tlnn 90°, which means that sicle RQ must be longer than
f.3-:r. Thc first integer value grcater than Dl is 6. Thcre's also an upper lirnit to the
valucs that RQ can take. Since the othcr two si des are 3 and 5, RQ must be less than 8.
1 Thcreforc RQ l an only be 6 or 7 sin ce it has to be an intcger, G + 7 = 13.
i

18. D Let mLAJJC = k, \vhich also means that m/ CBD = k. Write the sum of the interior
angles of trianglcs ABCand CHD. mLACH =-: 1BO- 3x and mLCBD = x +y.
For !}ADC: 2x +y+ 180- 3x +k= lFO
For [\CBD: 3x +X+ y+ k=' InOo
1 Set these equations equal to each other.

1
l
.J
2x + y + l BO - 3x + k = 3x + x + y + k
Subtract k and y from both si des, and simplify.
2x - 3x + lBO = 3x +x
-x + 1!30 = 4x
1130° = 5x
36° =X

19. B The arca uf 6ABCcan be written as a~ b = 30, so a· b = 60. If a- b = 7, you can use
substitution to solve for a ami b.

a b = 7 means a = b + 7, which can be substituted into a· b = 60.


(b + 7)b = 60
Ú2 + 7b = 60
Ú2 + 7 b- 60 = o.
Factor this equation as (b + 2)(b- 5) = O. b = -12 or b = 5.
Since bis a length. b = S. Use a- b = 7, to find that a must be 12. Use the Pythagorean
thcorem to find lz = ff2!. + 5 2 = 13.

Alternatively, start writing down 1! 2 + a 2 + b 2 , which means we are trying to get to


a 2 + b 2• Squarc both sides in (a- b) = 7 to yield a 2 - 2ab + b 2 = 49. Since we know
that a· b = 60 from the area equation, substitute it into our equation.

a2 - 2ab + b 2 = 49
a 2
- 2 · 60 + b 2 = 49
a 2 + b 2 = 49 + 120
a 2 + b 2 = 169
h = {'a-,-2 -+-cb:-=z = .JT6!J' = 13

20. C If AD = 3 anci DC = B, one leg of the isosceles right triangle is 11. Also vve know
that mL C = m LB= 45° in an isosceles right triangle (45-45-90 triangle). Using the
Pythagorean theorcm or the ratios of sidcs of a 45-45-90 triangle, we can calcula te BC
as 11 n.
Trianglc CDE is also a 45-45-90 triangle. From thc ratios of the si des of this spccial tri-
angle we can write:
CD_ CE
-:¡z-·r
B _CE
1I2- -1-
CE- = --·=-
8 · --"""
.f2 --,--,-
8.f2 = 4.f2
.fT2 ,!2 2

Sincc CF= 412, BEbecorncs 1lfl- 4{2 = 7.f2.

21. C In every triangle, one si deis less than the su m ami greater than the difference of the
?ihcr :oidcs. Therefore G < 4k + 5,1,: ami 6 > 5k- 4k. G< 9k and ~ < k, which means
~ < k a m! k< G. The two inequalil íes can be combined as follows: l.
1

32 /-- ,,l- <"' (') 1


.........
')')
B Draw the attitude from vertex M to point A on si de KT. This divides angle M and si de
KTinto two equal pieces. From the 30-60-90 triangle ratios:
AIT _ AT
;z- .fS
)'

~= }x Cross multiply.

Since x.fS = y, then i must be .fJ.


M

K A v T
2
23. B Let the length of the sides of MNKbe x, amllet the length of the sides of PQR be y.

Iftheir perimeters add up to 36, 3x + 3y = 36 and x +y= 12. The shaded area
represents the dil1erence of their areas. Since the area of an equilateral triangle
.. n2..f3
IS-;:r-,

1x -1 2
- y 2 = 2113

Factor out 1·
1 (x2- y2) = 2l..f3

Multiply each side by -!J·


X2 - )'
2
= 84

Since x 2 - y 2 = (x- y)(x + y) = 84

ami x + y= 12, substituting gives us (x--y) · 12 = 84 anc! x- y= 7.

24. D Sin ce FBDE is a rectangle, al! of its angles are 90°. That makes l'\AFB ami 6.BDC right
triangles. Additiunally, FB = 8 and BD = 6.

Using the Pythagore<ill theorem for f\..AFB gives us AIF = 6 2 + 82 = 100. So il.H = 10.

f\..AFB is similar to t,JilJCsince mf.A =m f. CBD amlmL:.ARF = nL'.C


AF __ AH
8i5 -· RC
66- 10
-- -_\'-
X= 10
A

6~-~
F

6
Fi

E s n e

J
''\
' 25. E The interior angles of triangles add up to 180°.
In óADC,
2a - 2ll + 3a -- lJ + a + lJ'"" lBOa
6a - 2ll = 1130°
3a- lJ = 900
3a = !JO" + [J

{/ = 30" + ~

26. A Draw a !in e tluough l3 that is parallcl to e ami e


1 2•

A...,......---------{¡
2x- 30
""-----''""'--~r------- ,(,

e¡_________ {z
LABX= 2x -30
LCBX=3x+l0

2x- 30 + 3x + 20 = 140
5x- lO= 140
5x = 150
X= 30°

1!1 A quo.drilateral is a polygon with four sL:cs.

111A parallelogram is a quadrilateral with two sets of paralld sides.


a Thc opposite sides ofa paral!clogram are parallcl ancl havc equallengths.
AB ~o DC ami AJJ "'' llC
J\B 11 OC aml AD illJC
lJ

f)

111 Diagonals uf a pz¡rallelogram b: ect e,1cb othcr.


DM = i\IU ;'m\ !l!lf = MC
• The area of a parallelogram is base x height = bh.

• The perimeter of a parallelogram = a+ a+ b + b = 2(a + b)

RECTANGLES

• A parallelogram with right ang!es is called a rectangle. All properties of parallelograms


apply to rectang!es as well.
• DB and AC are diagonals, and they are equal in length. ny using the Pythagorean
theorem, we can tlnd the length as DB 2 = a 2 + b 2 since ABD is a right triangle.

m The arca of a rectangle is length X width = a· b.


111 The perimeter of a rectangle is a+ a+ b + h = 2(a + b).

SQUJ.HE

a A squarc is a rectanglc with al! sidcs equal to each other.


m Thc area of a squarc is lcngrh X width = a· n = a 2 •
Ar--- B

/) <l e
m The pcrimeter of a square is n a + a "'~ ·la.
+a 1-
a Thc diagonal of a squarc, su eh as DH, equals af2.

r;; A trapezoid is a quadrilatcral with only two parallel sides.


AB!I DC
A a 11
----~--·-----

------- ~--"""---~~-------
/)
h e

j
• The area of a trapezoid is the product of its height and the a\"erage of the two parallel
si des (bases): a j_Y- · h = ~(a 1- b) 1!.

• The perimeter of a trapezoitl is a + b + e+ d.

CIRCLES

• A circle is a set of points in a plan e equidistant from a fixed point (the center).
• Any line segment that has its endpoints on the circle is called a chord.
DB and i\B are chord.s.

lil Any chord that passes through the centcr is called a diameter.
AB is a diamctcr.
a Any line segment from the center to the circle is called a radius.
OA, OB, and OC are radii (plural for radius.)
a The length of a diamcter is twice the lcngth of a radius.
111 Any line that has only one point common with a circle is called a tangent.
C1 is tangcnt to thc circle at K K is called the point of tangency.
{'¡

1111 A tangent linc is pcrpcnclicul:tr to thc radius drawn to thc point of tangency.
111 Thc arca of a cirde is TI r 2, whcre r is the r:tdius.
a Tbe circumference of a circle is 2TI r = Tid, where r is the radius and d is the diametcr.
n is
approximately Examples: The arca of thc circle bclow is TI · r2 = TI · 122 = l44TI.
22
3.14 or
7 . Thc circumfcrcnce of the circle is 2 · TI· r = 24TI.
THE RATIO OF THE AREAS
The ratio of the areas of two circles is the square of the ratio of their radii.
A1 r~
A2 = r;

Example: The ratio of the diameters of two circular rugs is 2 : 3. Wl1at is the ratio of
their areas?
If the ratio of their diameters is 2 : 3, the ratio of their radii is also 2 : 3.
Based on the previous information, the ratio of their are as is (~)" = ~-
2
A 1Tf r2
To check, compare the actual areas, -l = -~ = -j.
Az m:; 12
11 An angle with a vertex at the center of a circle is called a central angle.
u in the figure is a central angle.

e
B

m An are is a piece of a circlc.


m The dcgree meas u re of are ACB is equal to the central angle AOB (or n).
111 The degree measure of the entire circle is 3600.
111 The ratio of the length of are ACR to the circumference of the circle is
3g-0,;.
Thereforc, tbe length of are ACB can be calct~,1tcd as:
Length of are ACB = 2Tir · ~
30
Examplc: \Vhat is the Jength of an are with a central angle of 120° in a circle with a
radius of 8 in ches?
Are length = 2o · 8 · ~f;3 = 1~~~
1:1 The ratio of the area of the sector (slice) ilOH to the arca of thc circlc is :¡(~o"·
Thcrcfore, the arca of sector AOB is calculated as:
Arca of sector AOB = " · r2 •
3~0
Examph;: What is the arca of a sector with a central angle of 120"' in a circle with a
radius of 8 in ches?
Sector arca= "· 8 2 • l~Q = " · 61 · L = .0-l•r
3<lÜ 3 3

1
1
!

l
l
. .-,.!, ...
• An angle whose vertex is on the circle and sides are chords of the circle is called an
inscribed angle.
• If a triangle has three vertices on the circle (inscribed in a circle) and one of its sides is
a diameter, it is a right triangle. In other words, if an inscribed angle's sides intersect
the circle at the endpoints of a diameter, it is a right angle.

• If an inscribed angle anda central angle intersect the circle at the same points, the
degree measure of the inscribcd angle is half of the central angle (or half of the are
that the inscribed angle cuts from the circle).

111 If each vertcx of a polygon líes on a circle, the polygon is inscribed in thc circle or the
circle is circumscribed about the poygon.

o
a If each si de of a polygon is tangcnt toa circlc, the polygon is circumscribcd about thc
circlc or the clrcle is inscribed in the polygon.

o
REC LlNGULl\R PRt5MS
m A rectanguhr prism is a thrce-dimcnsional box with 6 rectangular faces. Edges are
the !ir: es where the f:•.ces mee t. su eh as lines F/3 or GIL Ea eh point where the ed¡;cs
intersect is called a vertex, su eh as points C or D.
• A rectangular salid has 6 faces and 8 vertices.
• Ta find the volume of a rectangular so lid, simply multiply the lengths af the three
dimensions.
• Volume of a rectangular sol id = Length x Width x Height = l X w X h
• To t1nd the surface area af a rectangular so lid, tlnd the su m of al! surface areas indi-
vidually and add them together. Notice that there are three pairs of equal rectan-
gular faces. For exarnple, the area af the gray fa ce abo ve (rectangle CDI !G) is lz X l.
Rectangle AEFB has the same area.
11 Total Surface Area af a Rectangular Salid = 2luu + 2wl + 2lzl = 2(/uu + tul+ /¡/)

CUBES

• A rectangular solid with al! edges equal is called a cube.


• Volume of a Cube = Length x Width x Height = a x n x n = n 3
e_,.__ _ ___.,8
G 1
1
1
1
a ~D.
.--&--- ---- A
/
...- /

(j
H a E
11 Total Sur fa ce Arca of a Cube = 6(a x n) = 6n 2

CYUNDEH§

¡¡¡ A right circular cylinder has identical circular bases. Its hcight is perpendicular to
its faces.
H Volume of a Cylindcr = Base Arca x IIeight = r. x r 2 x h = r.r 2 lt

w To find thc surface arca of a cylinder, find tlw arcas of thc bases ancl che surCan:. Thcn
add tllem toge;hcr. Noticc that the curved sicle surfacc is actually a rcctangle with
height egua] to the hcight ofthc cylinder ami wid:h equal to the circumference ofthe
base.
e Total Sc;rface :\rea of a Cylincb· = 2(r.r2) + 2r.rll

¡'
l
l
¡
¡
¡
l
..~L.
SAMPLE Pl.lOBLEl\lS

In the figure below, ABCD is a square. lf BD = 4./3, what is the length of CE?

(A) 4.J2 A D
(B) 4f6
(C) 6{2
(0) 6f6
<E> afG

B e E

In arder to find CE~ one of the si des of t:-.CDE is needed. Sin ce square ABCD ancl 6 CDE share
CD, start with finding CD using the given length BD.
Whcn a diagonal of a square is drawn, it e reates two special triangles with 45-45-90 angles.
Thcrefore, thc ratios of the si des of 6BCD are BC: CD: BD = .i: x: xf2.
CD_ BD
7- xfi
CD _ 4-!3
7- xfi
CD = 44
f2.
!:-.CDE is a 30-G0-90 triangle. The ratios of its sides are CD: CE: DE= x: x.f3: 2x.
co_ CE
-~~-- x-!3.
CE= .f3 · CD
CE= .[3 . iQ = l3, = 12 · .f2_ = 12-fl. = 6l2
,fi {2 f2. . f2. 2
The answcr is (C).
In the figure below, EAC is an equilateral triangle and KL WM is a square. What is the
ratio of the are a of b.L WC to the are a of b.EKL?

(A) .[3 E
3
(B)

(C)
[6
3
l3
K/\L
(D)

(E)
~
3
[6 ,.\
1 \ M IV e

Let one side of the square be x. So RT = LW = WM = AJK = k'E = LE= x.


mL C = 60° sin ce 6E'AC is equilateral, and mL LWC = 90° sin ce KLWiH is a square.
L\LWC is a 30-60-90 triangle.
LW_ WC
73--1-
x-
.f3- -1-
wc
wc =--=.f..= _:s_:_fS = xfJ
fJ fJ·fJ 3
1 xfJ X 2 fJ
Arca of 6L!i\1C = I~~--=-J- = 6 ___ = J?v'3. _:L = _?_
Area of 6EKL .? fJ x 2 fJ 6 K13 3
x-·4 4-
The answer is (D) _

A cylindrical container has a diameter of 14 m ancl is half filled with water. A cement
block in the shape of a rectangular prism is dropped into the water tank and is com-
pletely submerged. lf the sides of the cement block are 11 m x 7 m x 9 m, how much
does the water leve! in the tank rise? Use rr = ?:./'-.
(A) 2 (8) ~ (C) 3 (D) ~ (E) 4
4 2

-------------------------------·-------·-------------------

If the cemenl block is completely submcrged, the vol un '\:O of water that would rise is
11 · 7 · 9 m 1•
To find the height increase, think of this increase as the volume of a cy!inder with a base
diamcter of l·l m (r = 7).

\1 = "r-'! 1 = ')') · 7-' · 1z


-=..p- must equal 11 · 7 · 9.

:22 . 7 . /¡ = 11 . 7 . 9 Simplify.
2lz = 9 Divitk by 2.

! /¡ =o ~)-
2
T!Je ans\\·er is (D).
1

1
¡
_j_.
SECTION 5.2-PRACTICE PROBLEMS

l. lile ang!Ps of a quadrilaleralllaw a ratio of ¿::\:ti: 7. \\"hal is lile degn•c nwasurc of


lile largest ;lllgle"?
(¡\) ;!()"
(B) ·lll"
(C) ()00
(Dl l;!W
(!-:) 140"

L. In lhc figun· lwlow, whal is lhc val u e of .r"?

(¡\) 15°
(13) 4:1°
(C) 75°
(0) 105°
150"
(E) 195°

3. i\ rectangular water tank 5-fect wide, 10-feet long, amll2-feet high is fillcd to ~ of
capacity. If an additional50 c.ubic feet ofwater are added, what percentage of the tank
is filled?
(A) 50%
(13) GG.7'l):,
(C) 72'X,
(D) 75'Yo
(E) 96%

¡
4. If an are with lenglh l21r is ofthe circumfercnce o fa circle, what is the shortest
dislance betwccn the endpoints of thc are?
(A) 4
(13) 4.J2
(C) B
(D) Bfi
(E) Bf}

5. The length of each sidc of squarc A is increasecl by 1OO'X¡ lo make squarc B. If the length
of ca eh sidc of Bis doubled to make square C. the arca of A is \\'hat fraction of the su m
of the arcas of 13 ami C?
(!\) 1
4

(ll) h
(C) 116
( D) _I_
. 20

(Fl -(\-
)·,

20s GNAT NATH wom:soo:c


6. In the t1gure below, aH sides meet at right angles. Based on the sides given, what is the
perimeter of the figure?
(A) 140 J-l
(B) 120
(C) BO l.t
(D) 70
(E) GO

22

7. In the t1gure below, !in e KT is tangent to the circle at K and Mis the center of the circle.
What is the radius of the circle if KT = 15 ami MT = 17?
(A) 2
{B) 5
(C) 6 T
(D) 8
(E) 9

8. In the figure, FD equals FB and FE equals FC. If the degree measure of are CAB is 100°,
what is the measure of angle x?
(A) 140° E
(B) 120°
(C) 80°
(D) 70°
(E) 60°

9. vVhat is thc arca of the trapezoid sllown bclow?


(A) 9/3 p 6 o

_--~
(B) 27
(C) 27['J
(D) 36
(E) 3613 60°
,\/ 12 N
1O. In the figun>, ilflC[) ís a squarc. i\rc HIJ ís ,\ of a rircle with its renter atA !f HC = G,
what is tlw closcst approximation of tlw area of the shaded region?
(,\) ()
( !1) ]()
(Cl 14
(!)) lB
(L) 2B

11. The dimcnsions of a rectangular prism are 12 cm. B cm. aml4 cm. 1!ow many cubes
vvith edgcs 2 cm long can be fit into this prism'?
(i\) B
(ll) 10
(C) 12
(D) 24
(E) 4B

12. J\BC is a right triangle. A.C is the diametcr of one of the half-circles and BC is the diameter
of the other half-circlc. What is the ratio of !\ to 1\ as shmvn in the figure below?

(A) 23
A
(B) f2

(C) 2

(0) 25
(E) 3

l3. An edge of a sol id culw ís B inches long. lf thc cube is cut into two rectangular prisms
parallcl to onc of íts faces, hy how m u eh do es tlw surface arca increase?
(i\) lt st'ays the samc
(B) :32
(CJ G4
(D) 12B
(E) lt cannot be detcrmined from the illfurmation givcn

14. The volume of a sol id cube is Gci cm'. lf a hug is walking on the surface of the cube from
point Ato point 7. \\ hat is the shortest distance it can trm-cl?
(Al 8
(B) 4.JJ
(C) •l + 4/7
(D) ,¡/5
(F) 1GJ2
15. In the regular pentagon below, what is the mL BEC'?
(A) 15° B
(B) 18°
(C) 24° e
(D) 36° A
(E) -180

16. In the figure bclow, ABC is an equilateral triangle ami JTDC is a parallelogram. lf the
perimcter of L:.ABC is 48, what is the perimeter of EFDC?
(A) 12 A
(B) 16
(C) 24
(D) 32 B
(E) 36

D
e
17. In the figure below, !lCEFis a parallelogram ami BC = CD. If mLF = 40° anci
mLFGB = 165°, what is the mLGBD?
(A) 75° A lJ e

~y
(B) 83°
(C) 95°
(D) 105°
(E) 115° F E

18. In the circle below, O is the eenter. Three times the length of are AB equals twiee the
length of are CB. Are CA is twiee as long as are AB. What is the me él ;ure of angle x?
(A) 75° A
(B) 80°
(C) 85° ¡¡
(D) 90"
CE) gso

e
19. If AD = 12 and AB = 8, what is ¡; -:arca of tlw traprzoid AnClY?
(A) 22 + 6/3
(B) 22{3 + 6
ccJ 66 + wf:1
(D) GGfY + lB
(E) GGD + 5-l /) e

¡
1
•.. _:!&_.
20. ;\U:Fís a parallt>logram \\"lwre fU: i\11 and 2 · [){:" ..,_ ClJ.lf tlll' area of ,\HCIJ"'"' 12,
what is tlw arca oL·\fl[)f:F·?
(,\) l)(i (.

(11) B 1
(<:) ¡¿
({)) {)()
(E} 4ll

¡: F

2 l. Fin• íde.ntical circlL'S are tangent to ea eh other ami to the outer l"ircle as shmvn. \Vhat is the
ratio of the total area of tlw small cirdes to tht> area of the large cirde'?

(/\) l
9
')
(13) 9
(Cl ]'1
4
(LJ) 9
(E) -º.
9

22. Two identical quarter-circles are drmvn inside square !\BCD as shown. If AD = 2, what is
the shaded arca?
(A) 21T- 2 Ar---~~-.n
(B) 2.fl. - 1T
(C) 4/2 - 1T
(D) 4- 1T E
(E) 4.fl. - 1T
1
/) e
t
23. t\ rectangular sheet ofalumínum has a lcngth of 16 ami a witlth of31T. /\ manufacturcr
cuts thc maximum possihlc numhcr of idcntical circular tlisks with radií 2 from thís 1
sheet and díscards the rest. What is the arca ofthc discardcd sheet?
(í\l o 1¡
(B) 21T
(C) 31T
¡
(D) 81T
(E) lG1T

24. A cylindrical pícce of wood will be cut in lo ídcntical cuhcs. Thc diZJilH'tt'r of the C)'lindcr
is G.f2 ft amlthe hcight is IG ft. Ir thc cdge of the cubcs must be atleast ·t ft. what is the 1
t
greatcst total vol ume of the rcsulting cubcs?
(!\) 25G ft'
l
'
(B) 375 ft 1
(C) 4]2 ft 1
(D) 45~) ft 1

(E) 4:-JG rt'


SECTION 5.2-SOLUTIONS

...,
1 2 3 4 5 6 8 9 10 11 12 13 l·l 15 16 17 18 19 20
'
E D D D D A D e e B E e D D D D B B E D

21 22 23 24
E D E e
1
! l. E Let the angles of the quadrilateral be A, B, C. ami D. Then ~ = ~ = ~ = ~ = k.
i Express each angle in terms of k.
.4. = 2k, B == 3k, C = Gk, and D = 7 k. Sin ce the su m of the imerior angles of a quadrilateral
1 is 360, 2k + 3k + 6k + 7 k= 360 and Ulk = 360.

Sol ve for k to get k== 20. The largest angle is D, which is 7 k= 7 · 20 = 141Y.

2. D The su m of all interior angles of a pentagon can be found by using the formula
(11- 2) · 180 == (5 - 2) · 180 = 540. Add all the angles of the pcntagon and set them
equal to 540.
90 + 90 + 150 + X + X= 540
330 + 2x= 540
2x = 210
X= 105°
1
l 3. D The vol u me of the container is V= 5 · 1O · 12 = 600 cubic feet. If it is filled to ¡ of
capacity 600 · ~ = 400 cubic feet ofwater are in it. Once 50 more cubic feet are added, the
total volume of water beco mes 450 cubic fe e t. To find the percentage, divide 450 by 600:
1
l ~g§ = 0.75 =: 75%

4. D
Set up a proportion to find the entire circumference C.
X
3 - 12Ti
1 4- -c-;- Cross multiply.
l! r 3e = 48Ti
1 e== 1GTi
1

You can calculate the radius of the circle fmm its circumfcrence.
e= 2-;;r = HiTI

2Tir = 16-;; Divide both sides by 2-;;


r= 8

Thc original are is f of the circle. So the rcmaining angle, which is one of the angles of
the triangle in thc figure, is ~l of 360, which is 900.
Use the Pytbagorcan tlworem to find the distance between thc endpoints or the are:

1:2B == _\.e

x = nn
Notice that yo u can :llso use tlle ratios or tlw ·15---15-90 triangle to sr :ve this equation.

Gr·iAT ~·l¡.\TH vVOR~C100K i3


5. D Let tlw si de of square t\ he l. lf it is incrt•ased by l 00%, the si de of square B will he 2.
Tlw arca of B will be 2 · 2 = •l. One side of Checomes 4 since it is doubled from H. The area
of C is 4 · 4 ...., 1G.

ivlake sure you go hack to tlw question. read tlw last sentt·nre. ami ansm:r tlw rorrert
question.

(}. A _q
14
<1
~

22
"
h

Since all angles are right angles, a + ú must be 14 + 22 since they are parallel.
Additionally, e+ d + e must cqual 34.
The perimeter = 34 +e+ d +e+ 22 + 14 +a+ ú = 34 + 34 + 36 + 36 = 140
Altcrnatively, notice that the shape can be convertcd into a reétangle of sides 34 and 36.
(Imagine that it is made up of thin wire.) The perimeter is 2 · (34 + 36) = 140.

7. D Draw the radius from M to K. which will be perpendicular to KZ: Now yo u have right
triangle AIKT that has a hypotcnuse of 17 anda leg of 15. Find lHK(the radius) using the
Pyú1agorean Úlcorem:
15 2 + MK 2 = 17 2
MK 2 = 17 2 - 15 2
MK 2 = 64
MK=r=8

8. C If the measure of are CAH equals 100°, then anglc CEB will equal 50°. The question
also statcs that FE= FC. which makes CEF an isosceles triangle whcre anglc FCE is also
50°. Use thc interior angles of FEC (\\"hich equal lB0°) to fiml x.
500 + 500 +X= IB0°
X= !30°.

9. e p 6 ()
1
f
¡

M
¡1 3 6
h 3 N
J

f
~
Draw the heights of the trapezoid from fJ ami O. Notice that they create two 30-60-90
triangles \dth bases equal to J.

Thc ratio of thc hdght lo tlw [J¡¡sc of 3 can be oulili: :;d from the spccial triangle ratios
(.rl:\). Tlw hcight =e Ji:L

Arca~~~ ((i + 12) · JlJ = 2/.fJ


10. B Find the area of the quarter-circle and subtract the area of triangle ABD from it.
The area of the quarter-circle = ± r =± 6
1T 2 1T 2 = 91T.

The area of the triangle = ! ·6 · 6 = 18.

The area of the shaded region is 91T- 18 = 9 · 3.14- lB= 2B.27 - lB""' 10.

11. E Hnd the volume of the rectangular prism and divide by the volume of one cube:
12 . 8 . 4 = 6 · 4 · 2 = 4B
2. 2. 2 .

12. e Triangle ABCis a 45-45-90 triangle. So ~A~= f· Let BCbe 2x. ACcan be calculated
as 2xf2..
The radius of the circ!e on AC = xli, and the radius of the circle on BC = x.
____________
ri _
Al _ 1T • 2x 1T · (x.f2)2 _ 2 _ ')

Az 1T • r~ 1T • xz xz ~

Note that the ratio of the are as of two cil·c!es is the square of the ratio of their radii,

13. D

You do not necd to find either the initial or the final surface area ofthe solids. The only
addition to the surface area will be the shaded arcas in the figure where the cutis made. You
also do not need to know how far from the edge the cutis made. As long as it is parallel, the
cut crea tes t:wo additional surfaces that are squares.
So the adclitional arca is 8 · 8 · 2 = l2B.

14. D The shortest distance on the surface ofthe cube from Ato Zpasses through the
midpoint of thc cdge bctwcen them as shown in Figure l. An easier way to sec this is to
redraw the surfaces thc bug travcls on a two-dimensional planeas in figure 2.

Figure l

The volumc of r1.·:cuiJe is G-1. The length of one edge is .¡cm since 4 · 4 · 4 = Gl.
In Figure 2, use the Pytlwgorean thcorcrn to l'ind tlw distancc AZ:
i
l AT ~' 4' + 8' =, lG + 61 = BO
l AZ = -lfS
¡
J
l:J. D Find mw of tlw angles of a n'gular ¡wntagon using
(11 · 2) · IBO (:J 2J · 1~10 _ IOll"
¡¡ ;¡

l.ct IILJC/Jlw .r. ,·,LUJis an isosct>ll's triangle. since E/J ~ DC. So lll! <TlJis also x.
In ,~,~:·u J, .r l x ! 1OB -ce l BW ¡¡

e
,\
l\,WL= úUJ/:'
111/.. AUJ = !OH"
:Hi + :¡¡; ¡. 111/ 11/:'C = 1OW
111 L. JI];'C = ]()o

1G. D Lct !IF be x and FB be y. Then :l(x + y) = 4!l since the perimetcr of ABC equals 48.
That means x + y = 1G.
lf AF = x. then !lE= EF = AF = x.
If F/3 =y. thcn BD = DF = FB = y.
The pcrimcter of the parallelogram is
2(EF + FD) = 2(x + y) = 2 · 1G = 32.

17. B 1f mL F = 40°, then mL e= 4(r in a parallclogram. Additionally, mL e+ m LA =


U.30°. Tlwrcforc, m!_A = 140°.

6BeDis an isosceles trihngle. So mLeBD = mLeD/3 = osoo2- 40 ol = 70°.

mLAGB is 15° because mLfGB is given to be 165° and they add up to 180°.

In L\J\DG:
mLGlJA + 15° + 1400 = l!i0°
mL GBA = 180° 15°- 140° = 25°

Al vertex JI:
25° + m!. GlJD + 70, = lBOo
mL GBIJ = 85°

lB. B 31Xr31 = 21C/JI and IOll = 21ÍlHI are given.


Let are lcngth ¡,;\¡7¡ = 21. Substitute 21 for IA!ll. So J · 21 = 21Cl31 amiiCBI ==JI.
To find ¡c)f¡, substitute 21 for 1íífh So 2i;Üi'¡ = 2 · 21 = 41.

The entire circle is 21 +JI+ rlt = 91.


The ratio of ¡;;\jl¡ to thc cirClllllference of !he circle is r) 1 ,.= ~) 'which also is the ratio of
•1 •
the central angle lo 3G0°.
2 _ X
9 ·- 3(¡()"
X= 80°

216 GMAT NATH \.VORKBOOl(


19. E Draw the altitudes AX and BY. Sine e ;lJ) = 12, use the ratios of the si des of a 30-60-90
triangle to find AX, which is equal to BY.
AX _ 13
AD- 2
A\_ 13
12 -. 2

·\V=
,¡,l\. 12'2 13 = (')'/!J
.)

BY= 613
DX _ l
AD- 2
DX __ 1
12 - '2
¡?
DX= -2 = 6

YC is also 613 sin ce L>BYC is an isosceles right triangle.

D 6 X Y 613 e
h, + h,
The are a of the trapezoid is - ---=. · h.
2
8 + 1~ + 613. 6fJ = 22 +2§13. 613 = 6613 +54

20. D Divide the parallelogram into smaller regions parallcl to the sides as shown.
If the arca of D. BCD= 12, then the area of BCDX = 24. This mcans each of the small
parallelograms has an area of 12. Count the number of parallelograms included in
region AJWEF·: which is 5. Arca of ABDEF = 12 · 5 = 60.

F E

21. E Lel tl : radius of one of the small circles be r. Then the radius of the large circle
bccomes 3r.
The ratio of thc total arca of the small circles to the area of the large circle is
5orr 2 __ 5r 2 .... 5
·.;.(Xi·)" - 91·~ -- 9

1
¡
¡

l
1
,J:
22. D First draw diagonal A C. It is equal to twice the radius of each circle. Use the 45-45-90
triangle ratios to find AC = 2PI.
If the radius of one quarter-circle is {l., its area is

;\ = t1T(-/2)2 = ~
Sin ce there are 2 quarter-circles, the total are a uf the circular regions is rr.
The arca of the square is 2 · 2 = 4, so the shaded area is 4 - 1T.

23. E As you can see in the figure, only 8 disks could be cut from this sheet since the width
of 3n "" 9.42 docs not allow for a third row of disks.

31T"" 9.42

The area of the remaining sheet is 16 · 3n - 8 · n(2)2.


48n - 32n = 16n

24. C The cross section of tbe wood block looks like:

The largest cube that can be cut will ha vean eclge of G ft. (6.ABC is a
45-45-90 triangle.)
The question states that the cubes must be identical. You can cut only 2 of the
6 X 6 x 6 cubes. Since the length of the block is only 16ft, you cannot cut 3 cubes,
whícb would be 1!1 ft long.
Thc volumc of the 2 cubes is 2 · 6~ = 432.
You may decide to cut 3 cubes tbat are 5 X 5 x 5. In tlwt case, the V\ lume woukl
be 3 · s~ = 375.
So the maximum total cube volumc is 432 ft\
Cubes that are 4 x 4 x 4 would allow for 4 cubcs to be cut, but thc total \'ülume wo\'ld
still be Iess than 432.

218 GMAT i"l:\TH WOR?\BOOi<


SECTION 5.3: COORDINATE GEOMETRY ANO GRAPHS
Points on the Rectangular Coordinate System
• The rectangular coordinate system is u sed to loca te and identify points on a plane
relative toa tlxed location (the origin.)

y y-axis

-1

Quadrant /! 3 Quadrant 1
2
x-axis
/
--1 -3 -2 -1 o ' 3 4 5
-1
_,
Quadrant 111 Quadraw IV
-3

-4

a Each point on the plane is ic!entified by its orientation relative to the axes.
11 Point !'vi is 3 units above the x-axis.

Point J\f is 2 units to the right of y-axis.


111 The coordinatcs of point M are (2, 3).

m A pair of coorc!inates (x, y) is callee! an ordered pair.


lll The coordinates of the originare (0, 0}.

y
x-axis y-axis
T(0,4) 4

3--<--- 1 M(:?, 3)
\ 1
1
"K(-4, 2) 2 1
1
1
1
1 H (3, {))
-Q-·-··~~--··- 0.· - - · -·-
t
0··---?-~o·---~-o-- X
-4 -3 -2 -1 o 1 2 3 4
-- 1

o -··'
¡\(-2,-2)
-- J

--4 ..
C(l, -4)
Distance Between Two Points
• The distance between two points (such as P and Q in the figure) can be found using
the Pythagorean theorem. Notice that triangle PRQ is a right triangle.

Q (2, 3)
3
1
1
1
1
d 1
la
1
--<1>------+-~T--e-------- X
2
1
1
1
-2---·
P(-1,-2) b R(2,-2)

The lengths of a and b can be obtained from the figure simply by counting the units.

a= 5 and b = 3
d 2 = 52 + 32 = 25 +9 = 34
d = {34

11 A general formula to find the distance between two points A(x1, y1) and B(x2, y2) is
d ={(Y;- J;;f+ (x2 - xl

The distance between P( -1, -2) and Q(2, 3) is d = ~(3 - ( -2)) 2 + (2- ( -1))2 =
,[52+§2 = {34

111 The midpoint between two given points A(x1, y 1) and B(x2 , y2 ) can be found using the
following formula:

Af (~\;_::!, y1 ~Y~)

The midpoint between P and Q abo ve is (.::::-l t ~-, - + 2 3 ) = (~,


2
!).

m Graphing an equation means marking all points that satisfy thc equation. It is a visual
way of showing a relationship betvvecn y and x.

Example: Ccmsider thc relationship between the ages of two sisters \\he re one is
3 years older than the other. Thc relationship betwcen thc oldcr sister's age
(y) and thc youngcr sister's age (x) is simply y is th re e more th;n x. This rcb-
tionship can be rcpresented with a func1 :un or with a graph.

As a function il is y= x + 3. If thc agc of the younger one is given, su eh as x = 3, yo u can


casily figure out that the oldcr one's age is Gyears (y= 6).
Befare representing this relationship using a graph, take a look at a few points that
satisfy the equation and set up a table.

X. )'
When the younger one was born, the older one was 3 Jears old.
o 3 When x = O, y= 3.
l 4 Plug x 7 1 into y= x + 3 = l + 3 = ·l.
3 6 Plug x = 3 into y= x + 3 = 3 + 3 = 6.
4 7 Plug x = 4 into y= x + 3 = 4 + 3 = 7.

This means the ordered pairs (0, 3), (1, 4), (3, 6). ami (4. 7). as well as olher poinrs. are on
your graph. Plot these points on the rectangular system ami observe the nature of the
relationship. This particular function is a linear function, therefore the graph will be a line.
The graphical represenration of y= x + 3 is shown belmv.

y
/

7 ,-/H. 7l
/
/
6 / (3, 6)
5 //
/
/
.f /" (1, .f)
/
3 (0, 3)

o • ~-.----.-~X

-2 -1 o 1 2 3 4
-1

lll A graph can be used to fine! particular points thnt satisfy a rclationship. In some cases,
only a graph is given to define a relat;onship.

Examplc: Whcn the youngcr sister is 2 ycars old, (x = 2), you can read the older sis-
ter's age from the graph. Pincl x = 2 nn the x-axis. f\lo\·e perpendicular to
the x-axis (down or up) until you hit the graph. Head tbe y-vnlue. The older
sister is 5 years old.

y
/
/
7 .!"/
/

6 "/
/
- /
)+-/-+
-~ ,/ 1
/ 1
/{ 1
•. 1
1
2 1
1
1
1
-~- -.;,.---~o----1>---- .~
-2 -1 o¡ 1 2 3 -t
t -1
The same principie can be applied if only y is known. For example, if the older sister is
6.5 years okl, lind y= 6.5 on the y-axis. Move perpendicular to the y-a.xis (right or left)
until yo u hit thc graph. Read the x-value. The younger sistcr is 3.5 years old .

.v
/
/

7 /
------~
/1
6
/" 1
/ 1
5 // 1
/ 1
4 • 1
// 1
3 1
1
1
2 1
1
1

----:-t---+--.......,.-+--x
-2 -1 (} 1 2 3 4
-1

GRi\PUS OF FUNCTIONS
111 In a function in the form of j(x) = 3x- 4, j(x) repre·sents y, the output or the
range of the function.
111 Graphing a functionj(x) = 3x- 4 means graphing y= 3x- 4 on the coordinate
plane (j(x) =y).

Example: Sketch the graph ofj(x) = 3x- 4.


First find two points that satisfy the cquatirm. For example, plug in x =O, y= 3 ·O- 4 = -4,
ami x = 2, y= 3 · 2 - 4 = 2. So (0, -4) anc\ (2, 2) are on the graph.
Second, locate these points on the coordinatc a.xis.
Final! y, draw a line through the points.

¡; Equations lhat can be representcd in the fonn y= mx + L1 are callcd linear eí;uations,
amlthcir graphs are stuight lines.

Example: Thc graph uf y= -2x + 3 is a linc tbat passes tbrough (-·1, 5), (0, 3), and
(1, l). Thcsc points can be detumincd by crearing a tablc of values as
dcscribed above.
REMEMBER
lf a point is on the graph, it satisfies the equation of the line.

For example, if ( -1, 5) is on the graph of


y= -2x + 3, then
5 = -2(-1) + 3.

• Only one line can pass through any rwo given points. Since two points are enough to
define a line, it is suftlcient to find two points 011 the line to graph it.
• Mark (l, 1) and (-1, 5) 011 the coordinare system, and draw a line through these two
points.
111 As you can see, the line y= -2x +3 also passes through (0, 3), (1.5, 0), and other

points that can be found by looking at the graph.

y= -2x +3Y

(-1, 5) 5

1!1 Slope is defined as the ratio of the changc in y to the change in x.


Change in y Rise Y2 - Y1
Slope = -----------;--- = ---- = -;:--;:;_---~
Change m x Run .\2 xl
To find the slope of !in e PQ, yo u can simply find r1 and /;ami divide.

)'

Q t2, 3)
3
1
1
1
1
1

_,__._lf 111
1
--.-;-~X

1',-/-J ,,, --L,__ ,)


Slope = ELs.~ = i 1 = ~
Hun lJ 3
You can also use the formula:
SI Yz-Yt 3-(-2) 5
" ope = xz - xt = 2 - ( -1) = 3

• In a linear equation in the form y= mx = ú, m represents the slope.


Example: In y= -2x + 3, the slope equals -2.

SLOPE
. Rise
Slo pe 1s Run.
As you move to the right on a line from one point to the other, how far up you go versus
how far right you go is the slope.

lf the graph is going down, the slope is negative.

V )'

Zero slope Undelined slope


Positive slope Negative slope Example: y = 3 Exarnple: x = 2

lntercepts
• The y-intercept of a graph is the point where the graph crosses the y-axis.
To find the y-intercept from the equation, sct x =O.

/ x-interccpt

a If the cquation is givcn in the form y= //IX+ ú, ú is always the y-intercept.


Set x = O in y= n1x + ú = 111 · O + ú = ú. Thc y-intcrccpt is at y= ú.
Example: The y-intcrcept of y= -42x + 17 is 17.

u The x-intcrccpt of a graph is the point \vberc the graph crosscs the x-axis.
To find thc x-interccpt from the equation, set y= O.
Examp::..;: Find the y-intercept ancl x-intercept of y= 7x -- 10.

For the y-intercept, set x =O. So y= 7 ·O- 10 = -lO. The graph crosscs the y-axis at
y= -10.
For the x-interccpt, set y= O. So O = 7 · x- 10, x = ~0-. The graph crosses the x-a..xis at
.X= }_Q
7.
WRITING TIIE EQUATION OF A UNE
• If two points on a line, such as (x1, y 1) and (x2, y), are given, use the following steps to
tlnd its equation.
y- y
l. Find the slope using the slope formula: Slope = m= ~~
2. Use one of the points to write the equation: y- y 1 = m· (x- .t).
2 _ ~-

Example: Find the equation of the line that passes through (3, 3) and (2, -1).
'1 Yz - Y1 -! - 3 -4
s o pe = m = ~tz -.::- xl = 2=-3 = -=-r = 4
By using y- y1 = m · (x- x 1) and plugging in one of the points, yo u get y- 3 = 4 · (x- 3).
This can he rearranged to beco me y= 4x - 9.

• If the slope of a line ami one point on the line such as (x1, y1) are given, it is possible to
write the equation of the line. Since yo u know the slope, simply use y- )'¡ = m · (x- .1.).

Example: Find the equation of the line that pass es through (3, 2) and has a slope of- 2.

Using y- y1 = 111 • (x- x) and plugging in the point gives you y- 2 = -2 · (x- 3).
It can al so be rearranged to be y= - 2x + B.

PARALLELAi~D PERPENDICULAR UNES


¡¡¡ Parallellines have the same slope. If C1 11 f 2, then m2 • 111 =
1
1111 Perpendicular !in es ha ve negative reciproca! slopes. If C., _¡_ f 3 , then m2 = - __L.
- 111
J
1
=-
llh
- 2 ' 1113 = -'?
-
.{3 .{2

and
a Graphs of quadratic cquations are parabolas, as shown in the rigttrt;.
r

r ·· T ·- · ···1
1 1 1
. i ... T.. ·f. -1
i 1
.. 1 -· . i ·1
1 1 1
f·· + 1 ·1
1 1 1
l. -- .1 ... 1 ¡ .. 1
:
-~·-\}-
1 ¡ ~/
""'--·
:
-">--·*--· .\
1

-· 3 -2 -l o l 2
··1
• To graph a quatlratic equation, crcate atable of x-values antl y-values to fintl a few
points on the parabola.

Example: Graph y= (x + 1) 2

~
-2 l y= (-2 + 1) 2 = (-1) 2 =1
-1 o y= (- 1 + 1)2 = (0)2 = o
o 1 y= (O + 1) = (1) 2 = 1
2

1 4 y= (1 + 1)2 = (2) 2 = 4
Plot these points on the coordinate axis, and draw a parabola through them.

SAMPLE PH013LEMS

In the figure below, what is the area of MBC?


y
(A) 5 A ( -3, 2)
(8) 7.5
(C) 10
(D) 12.5
(E) 15
B(-1,-1) C(4,-l)

Base BC is 5 units long. The height is the vertical distan ce from BC toA (the difference in the
y-coordina tes). Therefore, the height is 2 - ( -1) = 3. The arca of the triangle = !3 · 5 = 7.5.
The answer is (I3).

lf pc.;1t K (n + 2, n - 3) is in Quadrant IV, how many integer values can n take?

(A) 1
(8) 2
(C) 3
(D) 4
(E) 5

If Kis in Quaclrant IV, then x >O ami y< O.


n+2>0
ll > -2

ami
n-3<0
n<3
Sin ce -2 < 11 < 3, 11 could be -1, O, l, or 2.
Tbe answcr is (D).
What is the x-intercept of the line that passes through ( -1, -3) and (3, 5)?

(A) 2
(8) 1

(C) l
2
(D) -21
(E) -1

To find the equation of the line, llnd the slope first:

Slope = m =
V - V
~;:~ =
5- ( -3)
:< _ 0
f
3_(_·
= = 2 i
Write the equation using y- y 1 = m (x- x 1): o

y- 5 = 2 o (x- 3) or y= 2x- 1
The x-intcrcept could be found by plugging in O for y:
O= 2x- 1
,-= l
·' 2
The answer is (C)o

l. If point B (m- 4, m) is in Quadrant JI, what is tl ,,~ product of all possible integer val u es of m?
(A) 2
(B) 3
(C) 5
(D) 6
(E) 8

20 In the rectangular coordinate systcm shown, 2 ON = 3 OMO If the area of triangle Ol\'lvl
o o

is '18, what is the x-coordinare of M?


(A) 4
(D) G
(C) B
(Dl 10
(E) 12
M
1¡.
¡
o \-
30 The cqtwtion ofline 111 that passes through points ri O. 2) ami n (2, --3) is ny = lix = 1-l.
What is the val u e of d?
(A) --5
(B) __ 3
-.)

(C) o¡

(D) ~i
3
(E) .S
4. The midpoint between A (3, 2) and B (5, -6) is on the line givcn by the equation
h + 3y- 6 = O. What is the value of k'?
(A) -3
(B) -2
(C) l
(0) 2
(E) 3

5. Which of the following could be the valuc of 11 if the distance between points A (3n, 5)
and B (5n, -3) is lO?
(A) 2
(B) 3
(C) 4
(0) 5
(E) 6

6. The slope of the !ine that passes through K (- 3, -4) and T (w, - 2) is -l. What is the 1
value of w? 1
(A) -5 t
(B) -3 1
¡

(C) -1 !
¡
(0) 1
(E) 3

7. In the figure, OA .l AB, mLAOB = 60°, and the coordinates of point B are (12, 0). What
are the coordina tes of point A?
(A) (4, 4!3) y
(I3) (3, 3.f3)
¡\
(C) (3.fJ, 3)
(0) (4.fJ, 4)
(El (2.fi, 3.fJJ .¡
l
1

o· J J - - - - - - - - - - - · -B o ----x
(12, O)

B. If /(2) = a, what must be the val ue of f(3a)?


(A) -1
y
(B)
5
(C) 2
(D) 3 4
(E) 4
J

- __,._,.._,___ ~-

-3 -2 --1 o 2 3
-l
9. ON = NM = AIK, and triangle MNP is an equilateral triangle. What is the x-coordinate
ofpoint T? y
l (A) 2/3
(I3) 4
(C) 4/3
(D) 8
1 {E) 8/3

1

1

10. In the figure below, OTis tangent to the circle at point T(l2, 5). If the center ofthe
cü·cle is at C (15, 0), what is the radius of the circle?
1 (A) 2.ff4
1 (B) 4/3
1 7'(1:2, 5)

l
1
{C) 8
{D) 4ft
{E) 8/3

1l
1

11. The vertices of parallelogram EFGH are E( -2, 0), F(2, 3), G {4, 1), and D(k, t). Which
one of the following is a possible val u e of k+ t?
1
(A) -2
l (B) -1

l (C) O
(D) l
1 (E) 2

12. ax + by= 42 is a line that passes through points P (l, 2) and Q (--4, ---3). What is -(/
1
! (A) -±
3
(B) -l

(Cl -~
(D) 41
(E) ~

B. Thc shaclcd arca shown in thc figure is equa! to ~1 sc¡uare units. Whal is thc valuc of d?
(t\) 2 y=x
A
(B) 3
(C) 4
(D) G
(El fl
14. The equation of the line on the graph is y= -2x + 12. lf the perimeter of rectangle
OMNP cquals 14, what is the are a'?
(A) 8
(13) lo
(C) 12
(D) 14
(E) 16

o 1'

SECTION 5.3-=SOLUTIONS

12 13
B D

l. D If Bis in Quadrant ll, then x < Oand y> O.


m- 4<0
lll < 4
and
m>O
That mcans O < 111 < 4.
Since mis a positivc integer, m could be 1, 2, or 3.
1·2·3=6

2. C Let Oi\1 be 2x. Thcn 2 · ON = 3 · 2x and ON = 3x.

2
The arca of triangle ONM is
1·2x·3r=,l'l
. . l
l
3.x 2 = 48
x 2 = H1 1
x= ::!:4 Sincz x is a lengtb, it cannot equal -4.
If x = 4, thcn OM = 2x = 2 · 4 = 8.

3. C Plug (1, 2) ami (2, - 3) into ny = úx + 14:


2n = h + 14
lf a !ine passes
-3a = 21J + 14
through a point,
the coordinates Multipl · the first equation by 2, ancl subtract.
of the point 4o=21J+28
wi!! satisfy the
- -3a = 211 + 14
ution of
7 a == 14
linc.
a=2
---·-·-----

230
4. E The midpoint can he found by:
1
X 1 + X" )'¡ +
M 1 -~-, -,)- - 1-..,-, Yzj-
1 3 + 5 2 - G) _ •
---->--- - (-!, -2)
l '- -~\- '-

If (-!, -2) is on the line kx + 3y- G =O,


4k + 3( -2) - 6 =o
-lk- 12 =o
·lk = 12
k=3

5. B Use the distance formula:


10 = ,ff=T-=-5l 2 +(5/z- 3n"P = lF8Y+Tinl 2 Square both sides.
lOO= 6-1 + -ln' Subtract G-1.
3n = 4n 2 Divide by4.
11 2 = 9 Take the square root.
n= :t::3

Only 3 is in the answer eh o ices.

6. A Use thc slope equation:


y,-)'¡
SI ope = ---=-----=-
xz .\¡
_-2-(-4)_ 2
- 1 - ------- - ------ Cross multiply.
w- (-3) lU + 3

-w-3=2
w= -5

l 7. B y

1
1
6 1
1
1

1i (i()·' 1
-----~----------:>-----X
¡ o T B (12, O)
!1
j

1 First use the 30- (}0-~JO spcciztl trianglc ratios for triangle ().\Ji.
¡
O!l: 0.-\: A!l =-' 2x: :r: x.ff
12 - 0:\
2:\: - -_.¡:-

i
l
l
0.-l = (-)
12 ____ AB
í
¡ 2:\-- :~-[:3

AIJ = 6-/3
Dr;nv a ¡wr¡wndicular linP from /\lo !he x-axis to determine !he x-ami y-coordinall'S.
Sin ce risa right angll'. usP !he :10-hO-!JO trianglt• ratios in triangle U:\l".
(),\ : ( Jr: o\/" ~ 2X : X: .r.f:l
(j - (){"
2x x
()[=:1

G - i\F
2.\· -- .\F\
:\T=:Ir!

Tlw coordinalcs of point i\ art' (:1. :l.f:ll.

B. E

..¡ .... ----1

:~ j~ t
o
1
• X
-1 () 1 2 3
-- 1

On the figure, draw a perpendicular line at 2 to !he x-axis ami read the y-coordinate on
the graph. /(2) = 1

lf a= 1, now you are looking for f(3a) = /(3), which is equal to 4.

9. e

X 1/'
1

!~
N
..¡
1 T
-->--~ <>----<r-
\ -
.
r

Since OK = 12, ON = NM = ¡\fK = í\ffJ e-= NP = 4.

In thc equi!ateral triangle lilNJ>, !he height XJ> beco mes 2Ü from the 30-60-90 trianglc.

Triangles KXfJ and KOT are similar triangles sin ce XP is parallclto OT:
KX _ Xl'
RO- -en
6 - 2fS
!2 - -DT
Of= 4D
10. A The length of OTean be found using the Pythagorean theorem:
OT= ~12 2 +5 2 = /169 = 13
If yo u draw the radius from C lo 1: CT is perpendicular ro OT sin ce T is the point of
tangency. Use another Pythagorean theorem to find r:
r = ff5 2 - l:F = .f56 = z/11

11. A Plot the points on the rectangular coordinare system as shfwm below. Point D could
be at (0, -2). so k+ t == -2.

12. B If a point on a line is given, it satisfies the equation of the line. Plug (l, 2) and
( -4, --3) into ax + by= 42 separately to get two separate equations.
a+ 2b 42 and -4a -- 3b = 42
=
Since they are both equal to 42, you can set them equal to each other:
a+ 2b = -4a- 3b
5a = -5b
!1=--1
b
Alte¡:natively, rearrange ax + by= 42 into slope-intercept form:

y= -];x + 1ff
This means- Eis the slope. You can find the slopc ofthe line using tvvo points:
1/1 = _:::::] :::::_~ = _g = - !1
-4- l 5 b
We are looking for fj = -l.

13. D The arca of a trianglc is J·base· hcight. In the figure, the base triangle AO!J is All ancl
the height is equalto a.

To fiml the base (distance :lB), plug o in<o the given line equations to find their
y-coordiilates:
yli = 12 a aml -v_\ = a

The diffcrcnce betwcen thcm is the hase.

J3;¡::;e == o-- -}a = 1o


lhl' arl'a is .~ · .1, 11 · 11 !l.
<.. <..

11' ~~ :Hi
¡ ¡ - ()

\' .. 1
,\'

1
\.- ·-"- \

14. B l.ct thc coordi11at('S of poinl ;VlJp (11, /J). lhalmcans tlw width of llw rPctangle is a ami

thc lcnglh is iJ. lfthc perimcler is :14, then '2.11 + '2.li = 14 ami a+ iJ = 7.
Yo u can also plug (a, /¡) into the equation uf the li11e sincc point N is 011 the line:
/1 == -'2.a + 1'2.
IJ+'2.a=l'2.
Solve the two equations simultaneously:
2a + IJ = 1'2.
a+ lJ= 7
(/ = 5

Since a = 5, 5 + ú = 7 so lJ = '2..
/\rea = 2 · 5 = 10

SECTION 5.4: DATA INTERPr?ETA TI ON


So me ofthc qucstio11s 011 thc Gí\11\T rcquirc you to analyzc certain types of graphs and tables.
Examples of so me of thc most common question types are give11 below.

Table

Energy Bars Calo?ies Protein (g) Total Fa~ (g)

A 105 14 7
8 220 12 9
-- -~--- "---·-
e 104 4 13
o 202 10 4.5
E 240 20 11
- - -- ~-·- ·- ··-

The tablc abm·c sho1\·s sclccl 11lllritional facts for S e11ergy bars. AnS\\'l'l' the following tluce
qucstions bascd on thc data gh-cn.

Questio:í 1: Whil-h of the cncrgy bars has the higlwst protcin ptT total fat?
(1\l i\ (B) IJ (C) C (Dl lJ (_El F

234 G·lAT i l "cf!i WOill<.BOCi(


This problem is a perfect caseto use the process of elimination. The ratio of protein to fat for
bar A is 14 : 7, which is exactly 2. Compare all other ratios to 2 and eliminate the ones that
are less than 2. For bar B, the ratio is 12 which is less than 2; eliminate B. The ratio for bar
9
C is t3"which is less than 1; eliminate C. The ratio for bar D is f~ which is slightly greater
than 2, sin ce -\Q is exactly 2; elimina te A The ratio for bar E is i~ which is slightly less than 2;
eliminate E

The answer is (0).

Question 2: The average protein content of al! t1ve bars is what percent greater than that
of bar D'?
(A) 120% (B) lOO% (C) 20% (D) 12% ·lE) 1.2%

The average protein content of all five bars can be found by adding their protein contents
and dividing by 5: 04 +
12
+ ~ + 10 + 20 } = 2Q = 12. The protein content of bar D is lO g.
5
Now the question reduces to 12 is what percent greater than 10'?
Percent difference = 12 - 10 ·100% = 20%
lO
The answer is (C).

Question 3: Which of the energy bars has the highest amount of protein per calorie?
(A) A (B) B (C) C (D) D (E) E

Once again, use elimination to compare the bars among themselves. In doing so, yo u will not
neec! to calcula te the exact ratios but only rough estimates. For examp!e, bar A offers 14 g of
protein per 105 calories. That is roughly 1\ =
10 /o~ = 0.14.

Now see if yo u can elirninate any answer choices by comparing them to 0.14. The ratio for
bar Bis ffo• which is roughly equal to z\1t = TBo = O.OG. Al! yo u need todo is a roug!t mental
calculation to sec if the ratio is even e! ose to 0.14. If it is signitlcantly lower, yo u can elimina te
it. If it could he close, yo u should calcula te the exact numbers. In this case, the percentage for
Bis very low compared with A; eliminate B. The ratio for bar C is T~h = T8o which is roughly
0.01; eliminare C. The ratio for bar D is toº-2 = ~{tr = 1g0, which is roughly 0.05; eliminare D.
Thc ratio for ;l;p6. Compare this ratio to 21fo = 0.10. The ratio -fB1 is less because it
b;l. Eis

20
has a larger denominator. Thercfore ~p < 0.1 O; elimina te E

Thc answer is (A).

BcWecl Tea Grams of Sugar per Serving

A 140 2.0

8 150 3.0
e 90 2.5
D 150 1.5
E 220 2.0

ll ,e; tablc aho\·e silO\\·::; thc sugar content per scrving of various bottled teas and the number
of scrvings incluJed in onc lJ, ·ttle. Use ;he data to answer tiw follo\\'ing thrce questions.
Question 1: What is the average sugar content pcr serving'?
1
1
(A) 140 (B) 150 (C} 300 (D) 324 (E) 750 1!

Average grams of sugar per serving can be fuund by simply tak.ing the average of the second
culumn:
Average= Hº-± 150 +_29 + 150 t 2~Q = l~O = 150
S ;,
rhe answer is (13).

Question 2: What is the average sugar content per bottle?


(A) 140 (B) 150 (C) 300 (D) 324 (E) 750

The average grams of sugar per bottle can be found by averaging the sugar content per bottle.
To t1nd !he amount of sugar in one bottle. multiply the sugar content per serving by the num-
ber of servings. For example, bottle A has 140 · 2 = 280 grams per bottle. Find the amount in
each bottle and average as follows:
(140 . 2) + (150. 3) + (90. 2.5) + (150. 1.5) + (220. 2)
Average= r:
J

= (2RO + 450 + 225 + 225 + 440) = _1620 =


324
5 5
The answer is (D).

Question 3: Which tea has the highest amount of sugar per bottle?
(A) A (B) B (C) e (D) D (E) E

Bottle i\ has (140 · 2) = 280 grams of sugar per bottle.


Bottle B has (150 · 3) = 450 grams of sugar per bottle.
Bottle e has (90 · 2.5) = 225 grams of sugar per bottle.
Bottle D has (150 · 1.5) = 225 grams of sugar per bottle.
Bottle E has (220 · 2) = 440 grams of sugar per bottle.

The answer is (13).

Circle Graphs
The circle graph shows the perccntage of each ingrcdient by weight included in Joy's Trail
l\Iix. Answer the following qucstions bascd on thc gmph .

.Joy's Trail i\ Iix

1
!
!

Question l: IfJoy's Trail Mix has equal amounts ofberries and M&~fs by weight, what is the
ratio of the weight of M&t-.ls to the weight of raisins?
11 (A) l : -l (B) l : 3 (C) 1 : 2 (D) 2 : 3 (E) 3 : -l
i

The percentages of al! ingredients add up to 100%. Add all given percentages: 20% + 30'Yo +
25% = 75% and subtract ti·om 100%. So lOO%- 75% = 25%. Since M&l\ls and berries have
equal weights, then each of them make up ~~% = 12.5% of the total. The ratio of the weight
of M&Ms to rhe weight of raisins is 12.5%: 25~1 = l : 2

The answer is (C).

Question 2: What is the degree measure of the central angle of the slice that represents
cashews'?
(B) 96° (C) 100° (D) lOBo (E) 112°

A circle contains 360°. Since cashews are 30% of the mixture, the cashew slice has a central
angle of 30'Yo · 360° = 0.3 · 360° = lO!r.

You can also set up a proportion to t1nd the angle:


30 _ X
Too- 36o
X= 108°

The answer is (D).

Question 3: If the en tire mixture weighs 40 ounces, hmv many ounces of nuts are used in
the mixture?
(A) 8 (B) 12 (C) 20 (D) 25 (E) 50

There are two types of nuts, peanuts (20%) and cashews (30%). The total percentage of nuts is
50%. Therefore, the weight of the nuts can be found by multiplying 50% by 40 ounces.

50'1il · 40 = 20 ounces

The answer is (C).

Question 4: Sanjay buys 64 ounces of Joy's Trail Mix. Sine,' he dislikes berries, he picks
them out. Which of the following is closest to the new percentage of M&l\ls in
Sanjay's trail mix?
(¡\) 25% (B) 2::% (C) JG% (D) 14% (E) 8%

Berries are 12.5% of the mixture initially. Tbat means the total amount of berries eq¡¡a!s
12.5')-ó · 6·1 = 8 ounccs aml thc total amount of t-.I&Ms equals 12.5'Yo · 64 = 8 ounces. After
he picks out the berries, the new mixture weighs G,l - 8 = 54 ounces. Thc pcrcentage of
l\l&Ms can be found using a proportion: s~f = ¡~ .
1 0
Cross mulliply n<;o = 5-lx, So X= ~')qp. Since the queslion is asking for the ap¡mnimatc per-
centage, HQp = 8~' (2 = 16. Since \Ve are aclually dividing 800 by 54, which is largcr than 50, our
'5 • J 0
answcr needs to be slightly snwller than 1G. Thc closest answer choice is 14%.

The answcr is (D).


Bar Graphs
Sodas Hcmaining at 1\litlnight

Ocol-.e

11 Dil'l Coke
O Cukt' !ero

Monda y Tuesday Wednesday Thursday Friday

The bar graph above shows the number of cans of soda remaining in a vending machine at
midnight after each workday. The cansare not replenished each night. Answer the following
questions based on the chart.

Question 1: What is the average number of cans of Diet Coke left in the vending machine
during the workweek?
(A) 1 (13) 5.6 (C) 6 (D) 6.7 (E) 7.6

The number of cans of Diet Coke lert in the vending machine each night is 15, 6, 1, 1, 15. To
fiml the average, add all the numbcrs ami divide by 5.

Average= (15 + 6 + 1 + 1 + 15) '


- - - - - ¡ : · - - - · - - - - = 7.6
,)

The answer is (E).

Question 2: l low many cans of Cokc were sold on Tuesday·?


(Al 7 (13) G (C) 5 (D) 4 (El 3

Since the number of cans ofCoke at midnight on Monday was 22 aml atmidnight on Tuesday
was 15, 22 - 15 = 7 cans of Cokc werc sold on Tuesday.

The answer is (/\).

Qucstion 3: \rVhat day did Diet Coke havc its lowcst sales?
(/\) i\londay (B) Tucsday (C) \\'ednesday (D) Thursday (E) Friday

On Thursday, the number of Dict Cokcs did not chaüge, so zero were sol d.

The answer is (D).

23J Gi\11\T i'V\TH V/)R¡z >oo:c


Cumulative Graphs
Year-EHd Sales of Silicon, In c.

"'~ ..HJ
:r.
O S..:rvices
~
·a 30 Hardware

..
.S
:n
::; 20
O Snftware
rJ1

!O

2002 2003 20tH 2005 2006

The bar chart above shows the cumulative year-end sales figures ofSilicon, Inc. from 2002 to
2006. Answer the following questions based on the cumulative sales graph above.

Question 1: What is the percent decrease in Services revenue from 2002 to 2003?
(A) 100% (B) 75% (C) 50'!{¡ (D)25% ' (E) 10%

Loo k at the height of the green sections. The services sales in 2002 was $4 mil! ion and in 2003
was $2 rnillion, therefore the percent change is:
New- Original ') - 4
% change = ----:--:--- · 100% ==---·lOO%=- 50%4 50% decrease
Ongmal 4

The answcr is (C).

Qucstion 2: In which ycar was the percent increase in the hardware revenue thc largest?
(Al 2002 (B) 2003 (C) 2004 (Dl 2005 (El 2006

Hardware revenues from 2002 to 2006 are 12, 23, 17, 19, and 21. The largest increase is from
12 in 2002 to 23 in 2003, which also is the higliest perceni ·1ge increase in this case. It is an
in crease of more than J OO';tJ.

The answer is (D).

Qucstion 3: I lmv much did total rcvcnue decline in 200-l'?


(A) $2mi!lion (D) $5 million (C) $7 million (D) $10 million (E) $12 million

The total revcnttL' in 2003 was $50 million and in 200"1 was $'!fl million. Tiw decline was
50 - 48 = 2.

The answcr is (A).

Questlon 4: What fraction of the m•crall rcvenues in 20% carne from hardware sales?
(i\)-l
4
(B) 1
3 (C) ~ (D) ; (E)~
'
In 200(), tata 1 sa les were $SG mi,11ion am1 11arclwarc sa ll'S werc ~21 mlt·¡¡·IOIL '1'1 w ratio
. .IS f-G-,
?l
J)
which simplifies to ~-

lile answer is (C).


Data Sufficiency

-+ 6.1 FORMAT
-+ 6.2 TWO TYPES OF QUESTIONS
-+ 6.3 INTERPRETING THE STEM
-+ 6.4 CHALLENGING THE STATEMENTS

If this is the tlrst time you are taking the GMAT, data suftlciency questions are probably
new to you. As the name implies, a data sufficiency problem asks you to determine if the
given information is sufficient to find an answer. Approximately 30-40% of the questions in
the math section of GMAT will be in tbis format. Data sufficiency questions do not actually
require yo u to t1nd the particular answer. They measure your abili ty to analyze a question,
identify irrelevant information, and be able to decide vvhich pieces of information are in fact
sufficient to answer that qucstion.
Al! data sufficiency questions ha ve the samc format. The first part (the stem) contains th.e
question and in so me cases some initial information. If information is provicled in thc stem,
it is definite. This piece is not in question.
After the stem, two statements, statement (l) and statement (2), are provided. They con- Do just
tain two separa te pk•ces of information that mayor may not be relevant/sufttcient to answer enough work
to deterrnins
the question. Your j,;]¡ is to decide ifyou can answer the queslionusing the first piece ofinfor-
if you can
mation only, using thc sccond piece of information only, or using both of tlwm togcther. In answer the
so me cases, ea eh statcment alone will be sufficient. In sorne, both statements wil! be needed. question.
In other cases, neithcr statement will be sufficient, even when taken togethcr.
Take a look al tlll' t•xample lwlow:

~
Kyle o¡wrwd a lll'\\' sa\·ings <llTO\IIll Tlw stem providt'S l\\'O pien•s of infor-
7 years ago ami llasn't madt• all\ mation. llw ti11w ¡wriod is 7 n•ars. and
tra11sactions siiiU'. Kyle made no furtlH'r transactions.

l~
llow llltll'll nrorwy does Kyle haw Ymr must decidl' if you ha\·e enough
toda:.,.? information to calculate the amount of
money today.

]
......
'~
( ll Tlw intnest rate is L:i";,, ¡:jrst analyze statement ( l) ami statement
(21 separately. Analyze them together
only if neither of them is su!Ticient alone .
(!)
.r: (2) 1\.yle deposited $2500.
1-

lt cannot be stressed enough that you do notneed to calculate an actual value or decide ifthe
answer is ycs or noto answer a data sufficicncy question. YolÍ simply need to disco ver if yo u
havc cnough information./\lmost everyone's initial reaction is to try to find an exact value to
makc surc, but doing that could be a big time trap for you. Si!~ce test preparers know pcople
have this tendency, some data sufficiency questions are designed to tempt you to spend time
solving for an answer.
The answer choices will always be the same. Memorize them before you start practicing
your data sufficiency problems to make sure you are very comfortable examining them on
thc test day. We have listed the official answcr choice language below.

(/\) Statement (1) 1\LONE is sufficient, but statcmcnt (2) alone is not sufficient.
(13) Statemcnt (2) i\LONE is sufficient, but statemenl (l) alonc is not sufficicnt.
(C) l30TI l statemcnts TOGETIIEH are suflident, but NEITHEH. stalement ALONE is sufficient.
(D) Ei\Cil statement 1\LONE is sufficienl.
(E) Statements (1) and (2) TOGETI !EH are NOT suflkient.

Use a mcthodical approach to review the stalc'menls prescnted.


1 alone A
(SreP}) Hcad thc qucstion very carcfully and rcphrase/simplify it if neL·essary (more on that
2 alone B
la ter). Think about what addition;:d information yo u would nccd to ans\\·er the qucstion.
Together e
@~ Considcr unly stalemcnt (l) \\'Ílh lhc stcm.
Each alo ,8 D
lf slalement (1) alone is ;;_ti{[Lcknt. thcn the only possiblc answer choiccs .}
Neither E A.D
are 11 or 12. You can cross out B. C. and E on your scratch pad.
or
If thc firsl statcmcnt (l) alone is !lQLsttfflt.~kn.L thc onlr possiblc atiS\\'cr Jl
B. C:. E
choiccs are lL___C_al1cLI:;. You ha ve eliminated A ami D.

242
(sreP 3) Consider only statement (2) with the stem, disregarding the t1rst statement.
Avoid the common mistake of thinking about the tirst statement as a given when
considering the second statement.
(sreP 4) If statements (l) ami (2) are not sufficient separately, then and only then consider
them together. Combine the stem with the two additional pieces of infonnation given
and ask the suftlciency question again.
The matrix shows the possible choices for answering the suft1ciency questions. Following
the steps above, sreps 2 and 3 ask you to consider statement (l) alone t1rst and statement
(2) alone next (the lighter region). Only if the answers are no-no would you end up in the
darker region (step -l) and consider them together.
As yo u can see, if (1) alone is sufficient, the only answer eh o ices are D andA (tirst column).

Sufticiency Qucstions

0) only?

Yes No

Yes D B
l2)
only?
. .
,._ "' ' ' " ' · " :~.

No A j~gether:? . :
·'Ye~:c
·No:E
·- . . . ·
. .. .·•.
It is a very good idea to use the elimination technique each time. It removes the det1nitively
wrong answers, thereby saving yo u time and easily avoidable mistakes.

TYPES
There are two main types of elata sufficiency problems, those asking for a specific val u e ancl
yes/no questions.

VAUJE QUESTIONS
Value queslions ask for one unique quantity. Some examplcs ofvalue questions includc:

;a I!ow old was i\li 2 years ago?


m \Vhat is thc val u e of 2x + 3y?
m By what pcrccnt die! Dana's salary incrcase?

Only one valnc can answer the question. The inforrnation is considered insufficient if it clocs
not allow for a val u e to be calculated or if il allows for multiplc val u esto be calculated.

In va!ue questions, the information is sufficient only if it is possib!e to determine a !,l[lÍQhlEl

quantity for th•2 question.

!
l
J
Example:
Tricia brings a box of doughnuts toa party. If a doughnut is picked at random, what is
the probability that it will be a chocolate doughnut?
(1) There are 16 chocolate douglmuts and 12 vanilla doughnuts in the box.
(2) There are 14 pcople at the party, and each of them ate a chocolate doughnut.

(sTEP1) We only know that there is a box of dougbnuts. We would either need the number of
each kind included in the box or the ratio of the number of each kind of doughnuts.
(sTEP2) Consider only statement (1) with the stcm.
What is thc probability of picking a chocolate doughnut if there are 16 chocolate
doughnuts and 12 vanilla doughnuts in the box?
Statement (1) alonc is not sufficicnt since we cannot assume that there are only choc-
olate and vanilla doughnuts in the box. Therefore, the possible answer choices are I1.
C. andE. Eliminate A and D.
(sTEP3) Consider only statement (2) with the stem, disregarding the first statement.
What is the probability of picking a chocolate doughnut if there are 14 people at the
party and each of them ate a chocolate doughnut (since we are ignoring statement 1)?
Statement (2) is clearly not enough to answer the probability question. Eliminate B.
(sTEP 4) Since statements (1) ami (2) are not sufficient separately, we need consider them together.
What is the probability of picking a chocolate dough'nut if tbere are 16 chocolate
doughnuts and 12 vanilla doughnuts iri the box anJ there are 14 people at the party,
each of whom ate a chocolate doughnut? We still do not have enough information
about the total number of doughnuts. Knowing how many doughnuts were eaten and
the number of available dougbnuts do es not help us calcula te the probability of picking
a chocobe doughnut in the first place.
No value can be calculated even when we use the two st<nements together.
The answcr is (E).

Example:
What is the valuc of n?
(1) l?zl is a prime numbcr.
(2) n is an cvcn number.
(srEP}) No initial information is provided in this stem. l

(srEP 2) Considcr only statemcnt (1) with the stem.


.t
18')
What is the valuc of 11 if -n"' is a prime number?
Find thc prime hctorization of Hl2 first. l82 = 2 · 7 · 13. So ll cou!d be 14, 26, and 91
. -1B2 13 1182 7 1 1B2 " 'I'l 3 .bl 1 f 1 f
smce
1T = , am -26 - = , am gy- = .... 1cre are poss1 e va ues or n, t 1ere ore
the first statement (1) alone is not_~fficiGD.t. The possible answer choices are .!1_C.LªL1l~
E. Eliminate J\ and D.
®iD Considcr only statemcnt (2) with th·· stem, disrcgarding the first statemcnt.
What is thc valuc of n if 11 is an cven numbcr? Statcment2 is clearly nL~ enough infor-
mation to find out about 11. Eliminate B.
(SrEP}) Si:1ce statemv; 1ts (1) and (2) me not su!Ticient separa: ·!y, \VC nccd cons:der thcm togcthcr.
\\ hat is thc value o[ n if l?f- is a prime number and ll is an even numbcr?
We still do 11 ll ha\ e enough information to find a unique vallt'! for 11 since two of the
three possible 11 valucs \.e identificd in Step 2 are evcn.
No unique val u e for 11 can be calculatec! evenwhcn we use the two statements togcther:
Thc answer is (E).
r
t
i
VES/NO QUESTIONS


!
Yes/No questions ask if yo u can give a definitive yes ora definite
no answer. Examples include:

• Is m> n?
IMPORTANT!
In yes/no questions, "sufficient" means
either an absolute yes or an absolute no.
i1
¡
• Is xeven?
• Does Sue make more money per month than Greg'?

¡ Sufficient means either an "absolute yes" oran "absolute no."
IMPORTANT!
Insufficient means sometimes yes and sometimes no.
In yes/no questions, if you can answer
This question type is potentially tricky for some students since the question either way, you have a
so me tend to confuse having enough information ro say no with not solution. "No" does not mean there's
having enough information. not enough information.
The following example is a straightforward question.

Is Carl's rent more than $750?


(1) Carl's rent is less than Sue's rent.
(2) Sue pays $650 for rent.

(srEP1) This is a yes/no question. We are only interested in t!guring out ifC~rl's rent is more
or less than $750.
(srEP 2) Consider only statement (l) with the stem.
If Carl's rent is less than Sue's, is he paying more than $750 for rent? We do not have
enough information to answer that question. Statement (l) alone is not sufficient to
answer yes or no since we clon't know how much Sue pays. Therefore, the possible
answer choices are B, C, andE. Eliminate A and D.
(sTEP 3) Consider only statement (2) with the stem, disregarding the first statement.
If Sue's rent is $650, is Carl paying more than $750 for rent? We do not have enough
information to answcr that question. Statement (2) alone is also not sufficient since
we are using only the information provided in statement (2) without statement (1).
Elimina te B.
(STS>4) Since statements (1) ancl (2) are not sufficient separately, we need to consider them
together.
Here's the new question. If Carl's rent is less than Sue's ami Su e pays $650 for rent, is
Carl paying more than $750 for rent? Now we ha ve enough infonnation to answer the
question, which mcans thc answer choice is C. BOT! 1 statements TOCETIIEH are suf- .
ficient, but NEITHER statement Al.ONE is sufficiet'i. That Carl doesn't pay more than
$750 does not mean your answer choice should he E.
The answer is (C).

Example:
Is x greater than y?
(1) -r= :J
-
-(!1:\
6/'
+?J)="
791 .
J'

.¡:j lO l )
0 - --
C-l x- 2- (;
l1;í- Too ·Y)
'2
This is a yes/ no qucstion. We are only interested in figuring out if x is greater than y.
( sTEP 1)

(sTEP2) Considcr only statcmcnt (l) with the stem.

ls xgreater than yif x = 2- (~~ +~~)+y?


Take a Iook at the 2- (~~ + ~~) piecc first. We do not neetl to know the exact value of
that number, but wc tlo nced to know if it is positive or negative. Since their numera-
tors are less than thcir tlenominators, ~~ and ~~ are both less than l. Therefore their
sum will be less than 2 antl 2 minus a number Iess than 2 is a positive number. The
question reduces to asking if x is greater than y if x ec¡uals a positive number + y? We
have enough information to answer the question.
So statement (l) alone is sufficient. The only possible answer choices are A or D. You
can eliminate B, C, and E.
(sr.EP3) Consider only statement (2) with the stem, tlisregarding the first statement.

Is x greater than y if x = ((!~ - i~6) ·Y) ? 2

Yo u can rewrite the second piece as(%~ - i~6 )y


2

2
• Sin ce we cannot determine if y is

positive or negative, the coefficient (!~ - i~6 )do es not really matter. We cannot answer
2

the question considering statement (2) alone.


(srEP4) Since statement (1) alone is sufficient, we do not need tQ consider them together.
Statemcnt (l) ALONE is sufficient, but statement (2) alone is not sufficient.
The answer is (A).

Rephrasing orwriting down your interpretation ofwhat the question is really asking is always
use fu! and time efficient. If the stem contains an expression, simplify itas muchas possible.
If the stem contains a statement, see if yo u can rephrase it to clarify the question. Consider
the following examples:

Is lQO ,-;_11~ an intcger? Simnlifv


tJ
as l9_Q
n + 2n/l = 1-º_Q
n + 2·
3x 2+ 6xy + 3y 2 • • . . 3(x 2 + Zxy + y 2) 3(x + y)2
- - - - - - · - - rntaht be g1ven.
x+y o
Sunpl!fy as------:----
x+y
= ------
x+y
= 3(x +y)
where x + y,p O
Is n2 - n > O? Simplify as n(n - 1) > O. In other words, is the
product of 11 ancl (n - l) positivc?
ls .Vodd? A.-< can be odd only if k is odd. So the question is
is k odd?
(n - 1) is an even numbcr. 11 is an odd number.

Sue's rcnt is not more than $750. Sue's rent is less than or equal to $750.

When n is didded by k, 11 is divisible by k.


the remainder is zero.

(1 - X) (1 + )') > 0 Either both (1 - x) ami ( 1 + y) are both positive or


both negative since their proc!uct is positive.
SECTION 6.4: CHALLENGING THE STATEMENTS
NUMBERS
In certain data sufficiency questions, you can test a range ofnumbers to see ifthe statemen~s
TOTRY
are sufficient or not. When testing numbers, it is easier (and definitive) to prove that the
Positive
statement is insuft1cient than to prove it is sufficient.
Negative
For example, consider the following question: 0 <X< 1
-1 <X< 1
Is (x - 1)2 < 3?
-1, O, and 1
(l)x<S Fractions
(2) X> 2

If you've decided to test so me numbers, make sure you try exceptions (within the
. parameters) for each statement to show that the statements are insufllcient. If yo u can-
not get contradicting results and for every exception you get a consistent answer (either
always yes or always no). you must accept that it is sufficient. The exceptions are num-
bers such as negative numbers, numbers between -1 andO, numbers between O and l,
other fractions, and the numbers -1, 1, andO.

For statement (l), try 2 and -2.


(2 - 1) 2 < 3
1<3
So 2 gives a true statement. However, it does not prove that x < 5 would work for any
real number. Try -2.
(-2-1) 2 <3
9<3
So -2 gives a false statement. Sin ce yo u ha ve tvvo contraclicting answers, yo u can
conclucle that statement (l) is not sufficient.

For statement (2), try 3.


(3- 1)2 < 3
4<3
So 3 gives a false stalement. Try a larger number su eh as 5.
(5 - 1) 2 <3
16 < 3
So 5 also givcs a false statement. They agrcc so far. Since thc stem does not indicate
that x is an integer, t1y 2.2 since you know that 2 gives you a true answer.
(2.2- 1)2 < 3
1.2" < 3
So 2.2 givcs a true statement. You have contradicting answers again. Statement (2)
also is not sufficient alone.
Consider statements (l) ami (2) together. x < 5 and x > 2 means 2 < x < 5. Is (x- 1)2 < 3'?
In this case, you can try 2.2 and 4. These numbers will give you contradicting answers again.
The answer is (E).

In so me cases, all of thc cxceptions will give yo u consistent answers. Then yo u can conclude
that the statement is sufticient.

Final Notes
In data sufficicncy problcms:
IMPORTANT!
• All numbers are real numbers. lf you succumb to the temptation to
• Figures conform to the stem but not necessarily solve for a value, know that statements
to the statemcnts. (1l.9nc::l (2) never contradict each other.
Therefore, if they are both sufficient,
• Alllines are straight.
they should individually lead you to the
• Al! anglc mcasuremcnts are positive. same result.
• All figures lie in the same plane.

FIGURES NOT DfV\\IVN TO SCALE


Do not guesstimate ba~ed on the dimensions of the illustrations given in geom,~try ques-
tions. Figures are generally not drawn to scale. Redraw them on your scratch pad based
on given information.

l. What is the product of n am! IR


(1) a ami b are consccutive intcgers.
(2) n 2 + b 2 = 613

2. PR ami RP are two diffcrent two-digit numbcrs. What is the value of P?


(1) PR+ RP= 132
(2) P> R

3. When k is dividcd by t, the quotienl is 6 amlthe remainder is 4. What is val ue of k?


(1) t = 4
(2) k= 2t + 20

4. k ami tare positivc intcgers where


k+~-= 3.75. v\;hat is the su m o[ k ami t'?
(l) kand tare both lcss than 5.
(2) t is an odd intcger.

5. x, y, and z are nonzero rcalnumhers. x ·y< O. ls -'jf ncgali\·c?


(l) x 2 z <O
(2) X· Z < 0
6. P, Q, and R are points nn the same !ine. PR = 6. What is thc distance between P and Q?
(l) RQ = 2
(2) Q is between P and R.

7. llow old is Clive today?


( 1) Deniz, Kaya, and Clive's ages are ,Proportional to 2, 3, and 5 toda y.
(2) Deniz is 5 years younger than Kaya.

8. In a certain test, a student makes three quarters for each correct answer and loses two
quarters for each wrong answer. No loss or gain is recorded for unanswered questions.
How m u eh money did the student make or lose on this test?
S
(1) The test had 60 questions.
(2) The student had 5 more correct answers than wrong answers.
e
9. What is the smallest number in a set of 7 numbers?
(1) The su m of the 7 numbers is I 26.
(2) All numbers in the set are even.

10.

What is the value of y?


(1) el and e2 are parallel.
(2) y= X+ 40

11. A, B, and C are positive integers. A is what% of C?


(l) A is 20% of B.
(2) Bis 30% of C.

12. There are a total of 150 doctors and nurses in a hospital. How many female doctors
are there?
(l) The ratio ofnurses to doctors is -!:l.
(2) The ratio of fcmale staff to mal e staff l : 2.

13. What is the ratio of i if xy = 6- ~?


(l) X· J' · Z = 20
(2) )'. z = 1 o
14. A certain amount of water is in a tan k. What fraction of the tank is fu m
(1) If k more gallons ofwater are addcd, ~ of the tank is full.
1
(2) !f kgalloJlS are removed, ofthe tank is empty.
15. If a = 33' + 1, what is the val u e of a?
(1) 3-' =!
(2) 27' = B

16. What is the ratio ofthis year's sales to last year's sales at store Q?
(1) Store Q's sales were $15 million last year.
(2) The sales increased 30% compared with last year.

17. After recciving his salal)', Tavares spent :\ of it the first day and kept spending t of the
remaining amount each day. Ilo,·: much was his salary?
(1) $800 was left at the end of 4 days.
(2) IIe spent $900 on the second day.

18.

q
F A
M

G ll

CIDB, ABHG, and ECAF are squares. Thcir arcas are K, M, and L, respectively. Is q
greater than 90°?
(1) K> L +M
(2) CB - AB < AC

19. ls x negativc?
(l) x 2y <O
(2) x-y> O

20. Is 4a + lJ odd?
(l) ú is cven.
(2) a is odd.

21. Whatisthcsumofx+y+z=?

(1) X_ Y_ 4
3-5-z
(2) X· Z - 12 = 0

. xz- vz
22. vV!Jat !S the valuc of -X}7'-- = ?
t V
(l) .:._=~-
1 3
(2) X =o 15
23. 5, 2, and x are the lengths of the sides of triangle CRA. lf x is an integer, what is its val ue?
( l) x is prime.
(2) CB:l is an isosceles triangle.

m, 11, and k are positive integers, and !Jl -~


24.
.2 1l = 3k. ls kan even integer?
(1) mis even.
(2) n is Ollcl.

25. e

A B

Is AB longer than BC?

(l) mLA =46° and mLB = 43°


(2) mLC = 91°

26. If ab ami ba are two-digit numbers where a anc! b represent digits, wh?t is the value of
(1- b?
(l) ab- ha = a - b
(2) (1 = 9

27. In the equation 2x 2 +(k- 2)x- k+ 4 =O, xis a variable and kis a constant. What is the
value of k?

(1) x = ! is a root of the equation.


(2) 2x 2
+ (k - 2)x- k+ 4 = O is divisible by (x + 3).

28. x and y are two nonzcro real numbers. Is x >y?


(l) y2 x 2 >O
1 1
(2) x-y< O

29. If a 2 ::5 o, what is the value of d?


(l) 2a + 3 is an intcger.
(2) (ll ::::; a'

30. Is the median age at thc family picnic 50?

(1) The oldest person at tht> picnic is B5, and the youngest person is 15.
(2) Half of the grnup are older than Jcrry, who is 50 years ole!.

31. In a particular high school, 60% of the teachcrs havc graduate degrees. ls the number
of female teachers with graduate degrees more than the number of male tcachers with
graduate dcgrees'?

(1) ·10% of the teacht'rs are male.


Ul There are 1B fe m ale teadwrs.
32. If k is a positive integer, is 2k + k 2 odd?
{1) 6k 2 + 2 is even.
(2) 5k + k 4 + 3 is odd.

33. m and n are two consccutive integers where 11 > m. \Vhat is the value of m2 - n 2?
(1) m 1· n = 35
(2) m- n = -l
. (a 2 c + clJ)
34. What 1s the value of ca ?
(1) -ªb = 3
(2) a= 5

35. (2' - 3)"' = l. If mis a nonzero integer, what is the value of x?


(1) tnisodd.
(2) x is even.

36. k, l, and m are tluee !ines on the same plan e. Is k parallel to l?


(l) k is perpendicular to m
(2) mis perpendicular to l

37. Amir, Bianca, and Carlos shared a certain number of questions. Bianca received how
many more questions than Carlos?
(1) Amir and Bianca received 24 questions in total.
(2) Amir and Carlos received 20 questions in total.

38. There are only red and blue marbles in a ha t. If a marble is to be picked at random,
what is the probability that it will be red?
(l) The number of red marblcs is 1of the numbcr of blue marbles.
(2) The number of blue marbles is 30 more than the number of red marbles.

39. Is a> 3?
(1) -a+ 2 < -1
(2) 7- 2a < l

40. A lmv firm has two different hourly rates, onc for junior lawycrs and one for senior
lmvyers. \Vas the average hourly rate on case Q grcater than $600'?
(1) The firm had 3 senior lawyers ami 7 junior lawyers vvork on the case.
(2) The senior Jawyer rate is $750/hr, ami the junio'r lawyer rate is $400/hr.
!
41. Kaml Tare positive intcgcrs. When 248 is divided by K, the quotient is Tand the 1
i
remainder is 10. What is the value of K?
(1) Kisodd.
(2) Kis a three-digit intcger.
l
{
42. Is y> x? !
'
(1) (l - x)(2 --y) <O
(2) X< 0

252 Gt<lAT t-1ATH \VORX.BOOK


·13. What is the val ue of J.?
(l) X- t = 2

(2) y- f = 3

44. ahal is a four-digit integer where a, h, e, ami d represent different nonzero integers. Is
a+h+c+d>ll'?
(1) abcd > 6,000
(2) ahcd is divisible by 5 ami Se= 4b = a

45. O is an operation defined in real numbers as a Oh= a 2 + b" - 2ah. What is the value
of a?
(l) aOb =O
(2) a= 4- b

46. What is the val u e of x + y?


(l) x 2 + y 2 = 17 ami x · y = 4
(2) x - y = 3 and x 2 - y 2 = 15

47. When a is divided by b, the quotient is 5 and the remainder is k. vVhat is the remainder
when 2a is divided by 5?
(1) k= 3
(2) b= 8

48. What is the value of y?


(1) y= 7- 2x
14- 2y
(2) x=--
4

49. What is the are a of rectangle ABCD?


(1) Perimeter = 24
(2) The lengths of AB anci BC are prime numbcrs.

50. Does n!J + be+ nc = !ir~/)(~?


(1) ]Ji + lii + l() = 20
n b e
(2) 4 + jc + .:!i: = se
n h
51. A is a 15-digit integcr. Is A divisible by 9'?
(1) Each digit of A is divisible by 3.
(2) The sum of al! digits of A is 72.

52. All cookies in a coffec slwp contain nuts, chocolate chips, or hoth. If there are a total of
90 cookies, how many of thcm have nuts only?

(l) The nurnbcr of cookies with chocolate chips only is equal to the number of cookies
with both.
(:2) The numher of cookies with nuts only b three times thc number of cookies with
chocolate chips only.

GllAT ~·IATH WO::i:l{BOOK 25:3


53. What is the value of 11?
(l) 21l + 2 ;;:::: lO
(2) 2- i;;:::: o
54. What is the ratio of m to 11 if m amln are nonzero numbers?
(l) 1611 2 - 2m 2 = 7111 2
(2) 4mn =3m 2

55. Two cars en ter a 6-mile lunnel from each eml at the same time. I Iow many seco mis
afler they en ter do they meet?
( 1) The su m of their speeds is 120 mi/hr.
(2) They meet at a point 4 miles from one end of the tunnel.

56.

BCDEFG is a regular hexagon, andA is the midpoint of EB (not shown). What is the area ¡
ofABCD?
(1) AB = 6
(2) FD = 6.[3
i
57. Is a greater than 8?
1
(1) a minus bis not more than 7.
(2) The sum of a and bis less than or equal to 9.

58. Both Martha ancl Pete received a raise last year. Do es Martha m(lke more than Pete 1
this year?
(1) Martha's raise was 6%, ancl Pete's raise was 8%.
(2) !3oth I\'Iartha and Pete reccived a raise of $4,800. 1
59.

In the figure, al! circles are identical and they are centercd on the vertices of quadri!at-
eral 'ABCO. vVhal is the su m of thc shadcd are as?
(1) Thc radius of each circle is B.
(2) AB = BC ~= CD = ~ AD
60. How many positive factors does m have?
( 1) m is the product of 3 prime numbers: a, b, and c.
(2) mis even and divisible by 15.

61.

A
J7Vc B

If ARCE is a parallelogram, what is the are a of L.ABD?


(l) D is rhe midpoint of EC.
(2) The area of ARCE is 60.

62. If x is a real number, is x 2 + x positive?


(l) x> -1
(2) X< 0

63. x is a nonzero real number. Is x even?


(l) _.¿ + x is even.
(2) x-2 ; 6 is an integer.

64. Does ZJ[~ = kif 11 =F O?

(l) mis 10% of n.


(2) 10 -1s. m= 10 -16. 11

(1) a + lJ = 11 oo
(2) e= 700

GG. Whal is the value of m 2 + 4mn + 4n 2 + !JL_l- 211 2m- 4n'?


3
(l) llz+2n=6
(2) !.!L = 2
211

67. What is the slope oflinc f 1?


(l) Une 3ax + 4ay = 3 is parallel to ( 1•
(2) Line 2ax + Sy= Sa is perpendicular to ( 1•

GB. ls J(l > l?


(l) a ami[¡ are negati\'c numbers.


(2) 1/J! > lo!
69. Sergei owns 7 aquariums and a various number of goldfish in ea eh of them. What is the
average number of goldfish per aquarium?
( 1) The median number of goldfish in all aquariums is 6.
(2) four of the aquariums ha ve the same number of goldfish, <md the rest each ha ve
double the number of fish.

70. There are only green and yellow marbles in a bag. If the probability of randomly picking
a yellow marble first ami a green marble second without replacement is {~, how many
yellow marbles are there?
(1} The ratio of yellow marbles to green marbles is 3 : 2.
(2) There are two more yellow marbles than green marbles.

1 2 3 4 5 6 7 8 9 10 11 12 13 14 15
e E D A A e e E E e e E B e D

16 17 lB 19 20 21 22 23 24 25 26 27 28 29 30
B D A E A E A D E D A D E E B

31 32 33 34 35 36 37 38 39 40 41 42 43 44 45
E B A e D e e A D E e e e D e
46 47 48 49 50 51 52 53 54 55 56 57 58 59 60
B A E e D B e e B A D e e A A

1 ~ ~ ~ ~ ~ ~ 1~ ~ ~ ~
1 1 1 1 1 1 1 1 1 1 1 1 1 1

l. C (1} a aml b could be any two consecutivc integers. NOT sufficient

(2) a 2 + b 2 = 613 is not enough to find a· b. For example, (a+ b} 2 = a 2 + 2ab + b 2, but
you need lhc valuc of a+ b lo solve that cquation. NOT sufficient.

Using (1} aml (2) togethcr, you can write a= b + 1 from (1) and plug that into a 2 + b 2 =
613 from (2}. (b + 1)2 + b2 = 613 can be solvcd for b ancla can be found through (1}.

The answer is C; both statements togethcr are sufficient.

2. E (1} If PR ami RP are two-digit numbers, their val u es can be represcnted as follows;

PR = lOP+ R
RP= lOR + P
lOP + R + lOR + P = llP + 1lR = 132
u(P+ m=
u2
P + R = 12 does not allow m to solve for P. NOT sufficient

(2) P > R. by itsclf do es not provide enough informal ion. NOT sufficienl
\Vhen taken together, (l) states
P + R = 12 and (2) states P > R. Therefore PR could be 75 or 84.
(2) This states that P > R, but that does not restrict the possibilities to just one. PR can be
still 75 or 83.

The answer is E; both statements together are still not sufficient.

3. D (1) From the division write k= t · 6 + 4. So if t is given, k can be calculated easily.


SUFFICIENT

(2) If k= 6t + 4 and k= 2t -r 20 are given, the two equations can be solved simultane-
ously. SUFFICIENT
There is no need to solve for k, but it would look like.

6t + 4 = 2t+ 20
4t = 16
t= 4

The answer is D; each statement alone is sufficient.

4. A First simplify the given equation by multiplying both si des by 4.


4(k+¡f)=3.75·4
4k+t=15

Since k and tare positive integers, a limited number of (k, t) pairs work for the equation
4k + t = 15. These are (1, ll), (2, 7), and (3,3).

(l) This states that k< 5 and t < 5.


k= 3, t = 3 is the only solution. SUFFICIENT

(2) Sin ce the possible pairs of (k, t) are (1, 11), (2, 7), and (3,3), knowing that t is odd means
tcan be 11, 7, or 3. NOT sufficient

The answer is A; statement (1) alone is sufficient.

5. A (1) If x'z <O, you can conduele that z <O since .~ will always be positive and the prod-
uct of .\~ and z must be negativc. Also, from the question, yo u know that x · y< O. If the
product of two nonzero numbcrs is negative, their quotient must be negative as well.
Thcrefore =-y=)~· zmust be(-)(-)=(+)

The answer to the qucstion is no. SUFPICIENT

(2) lf yz <O, yo u cannot conduele if y and z are negative or positive. Without any informa-
tion about z, yo u cannot decide if ~Y is negative or positive. NOT sufficicnt

The answer is A; statcment (1) alone is sufficient.

6. e __,_"___
6 _
1' R
(l) Q can be on eithcr si de of R and still be 2 m va y from R. NOT sufficient
(2) The exact location of Q is not provided. NOT sufficient

Using (l) a!lll (2) togcther, PQ = 4.


4 2
_ _, __ _· G - - z - - -

p Q R
Thc answcr is C; both statcmcilts togcthcr are su!Ticient.
'· e (1) Hatios alone are not enough lo condude Uin~'s age. The ages could be 2:3: !i or
4: (): 1O or any multiple of lhese. NOT suflkient

(2) Only lhe relalionship lwlween Dt•niz am!Kaya is giwn. NOT sulTicienl

By using ( 1l and (2) togelher. you can sel up equalions lo solve for all agPs. Fquations are
given lwre for reference. You do nol ¡wed lo solw' 1lwm for lhe purposes of lhis qm'slion.
D_K_C
;¿- - :¡ -- ~

Sincc lJ = K !i from (2). replan' it in equation ( 1).


K- 5 _K
--y---]
:lK- 15 = 2K
Salve for f.:.

The answer is C; both statements together are sufficient.

8. E ( 1) No information is givcn about the number of correct. wrong, and blank answers.
NOT sufficient

(2) You can sct up C = W + 5. You cannot sol ve this equation since the total number of
questions and the number ofblank answers are unknown. NOT sufficient

By using (1) and (2) together, there's still no way of finding out how many of each the stu-
dent answered since thcrc is no indication about blank answers or about another equa-
tion betwcen corrcct ami wrong answers.

The answcr is E; both statements together are stillnot sufficient.

9. E (l) The su m does not provide enough infonnation to find any of the numbers.
NOT sufficient

(2) An infinitc numbcr of diffcrent sets of even numbers can add up t.o 132.
NOT sufficient

Using both (1) ami (2) still does not provide the relative size of the 7 numbers.

(l) ancl (2) could have provided enough info if (2) hacl said that the 7 numbers are
consecutivc. Howevcr, that is not given.

Thc answcr is E; bolh statcments togcther are stillnot suffident.

10. e (1) lf ( 1 11 ( 2 , x +y= HlW, which is not cnough to solve for y. NOT sufficicnt

(2) y= x + 40. 1!owevrr, we are not givcn that [ 1 11 ( 2 • NOT sufficient

By using (1) aml (2) together, we can concludc x +y= 180 aml y= x + 40, which can be
sol ved together lo find y.

Thc answer is C; both statements logether are sufficient.

11. C (1) A perccntagc (ratio) relationship bct\\W'll A and nis givcn. Nol enough information
is gi\Tn to relate C toA. NUT suf!lcicnt

(2) A pcrccntage rclationship bctwecnl3 aJ)(l Cis giH'll. Not cnough information is gh-cn
lo reliltc Cto A NOT sufficicnt

258 GMAT l'>l.\TH WO!ctKBOOX


By using (l) and (2) together, the percentage relationship between A and e can be
calculated.

(1) A= 0.2B
(2) B = 0.3e, plug this into (!)
A = o.2(0.3l e
A= 0.06C
A is 6% of e

The answer is C; both statements together are suft1cient.

12. E (l) The statement does not provide infonnation about maJe to female ratio.
NOT suftlcient

(2) The statement does not provide information about the doctor to nurse ratio.
NOT sufficient

By using (l) and (2) together, yo u can write:


N_4
D-T
4D= N
J..!- 1
F-2
M=2F
Even with the total of 150 doctors and mu·ses, it is not possible to solve for female doctors
sin ce the ratio of female doctors to mal e doctors can be different than the ratio of female
nurses to maJe nurses. Only when put together is the ratio l : 2.

The answer is E; both statcments together are still not sufficient.

13. B Multiply all tenns of the first equation by z to simplify the denominator:
z · (xy) = z · (6)- z · (~)
.qz = 6z- x
(l) If xyz = 20, plug that in: 20 = 6z- x
In this equatinn, it is not possible to isolate :~· NOT suftlcient

(2) If y· z = lO yo u can plug in 1O for y· z.


x·10=6z-x
Add x to both si des.
llx = 6z
z- 11
x --tr
SUHICIENT

The answer is B; statcmcnt (2) alone is su!Ticient

H. C (l) l.et thc initial amount of water in the tan k be t and thc total capacity of thc tan k be A.
Statemcnt (l) translatcs into:

t+ k= ~~l
¡

Although you only nccd thc ratio:~· yc'u cannot gct that from this equation alone.
NOT sufikient

GMAf í"L".Hl WORlü300K 25;)


(2) This translatps into t k--~;\. \rhich cannot lw soln'tl for _\ eitiH•r. NOT suftkient
\Jsing ( 1) ami (2) togl•tlll'r:
1 ! k~ ~ ,\

k J.\
1
i\dtltlll' Pquations to elilllinalP k.

21"' ~ ¡\
1
{ ,¡
,-\ -
1

lile <lllS\H'r is C; both statenwnts togetlwr are suffkiPnt.

15. O 3'' is equi\·alPntlu (T) 1 • lfyoucan get :\' fromtlw statenwnts, tlwn 3'' + 1 can be
calculated.

(1) ±·
lf 3 ' = yo u can raise both si des to -1 power to gel 3'. (3 ') 1
= (~ )
1
aml3' = 2.
SUFFICIENT
(2) lf27' =U, then (3')' = B aml3'' =B. SUFFICIENT

The-answer is D; each statement alone is suffkient.

16. B (l) This provides only a dollar amount but nota comparative measure between years.
NOT sufficient

(2) Let the last ycar's sales be x. This ycar's sales become x + 0.3x = l.3x. The ratio can be
found by dividing this year's sales by last year's sales 1_~-I = 1.3. SUFFICIENT
The answer is !3; statement (2) alone is sufficient.

17. O (1) lf he spent 1


of his moncy each da y, ~ of the beginning amount remains at the ene!
of each da y. Let his salary be S ami sct up (but do not sol ve) an equation in terms of the
remaining amount:

(2) Let thc salary be S again. So~ S rcmains aftcr the first day. lavares spcnt ~ · !s
on thc second day.
~ ·1· S= 900 can be soh-cd for S. SUFFICIENT
Thc ans\\'Cr is D; cach statemcnt alonc is sufficicnt.

lB. A (1) This states that !\------f.+ ¡\J. Thercforc (Cfi)' > (;\C)' + (:\lW. If AHC\\Tre a right
2
triangle, (CB) = (;\C)' + (i\li)" would be true. In this case, since (CB)' is grcatcr. r¡ must
be grcatcr than 90". SUITICIENT

(2) Cll -- ;\!! < J\Cmcans CB < AC ·~ 1\U. In l'\'Ct} triangle. any side is slwrter than the
sum ofthc othcr I\YO sidcs. This statement docs not pro\·idc any addition<ll information.
NOT suiTicient

Thc answcr is ¡\; statcmctll ( 1) alot1c is sufl1cient.

260 GMAT llATi! \VORi<BOOK


19. E ( 1) This does not pro vide any information about whether or not x is negative because
.t-2 is ahvays positive. On the other hand, if .t.;¡y< O, y must be negative. NOT sufficient

(2) x- y> O means x > y, which does not provide any infonnation about whether or not
x is negative. NOT sufficient

By using (l) and (2) together, yo u know that x is greater than y and y is negative.
However, that is not suft1cient to find out if x is posilive or negative. It could be either
and still be greater than a negative y.

The answer is E; both statements together are still not sufficient.

20. A Regardless of a being odd or even, 4a will always be even. If yo u can tell whether lJ is
even or odd, yo u can answer the question.

(l) lJ is even. SUFFICIENT


(2) As discussed above, knowing a is odd is not enough to answer the question.
NOT sufficient

The answer is A; statement (l) alone is su!Ticient.

21. E (l) This provides information only about the ratios of x, y, ami z. The' statement does
not help solve for x, y, z or for x +y+ z. NOT sufficient

(2) This provides only the product of x and z. NOT sufficient

By using (1) and (2) together, there's still not enough information sin ce the ratios of al!
variables are provided but no reference is given to their absolute sizes.

The answer is E; both statements together are still not sufficient.

22. A (l) Cross rnultiply to get 3x = y. Replace 3x for y in the equation given in the question:
2 2
X - Y _ X2 (3x) 2 _ .\-2 - 9.\-2 _ -8.\;; _ -8
-xy-- x(3x)-- -3:\-2- - -3.t-2 - 3-
SUFFICIENT

(2) To finct the required ratio, yo u need the val u e of y al so or the ratio of y ami x.
NOT suftkient

Theanswer is A; statement (1) alone is sufl1cient.

23. D The si des of the triangle mus! satisfy the following inequality: 5 + 2 > x > 5 - 2.
So 7 > x > 3. Therefore x cou!d be {4, 5, G}.

(1) Only 5 is prime among the eligible integers. SUFFICIENT

(2) If x = {-1, 5, 11} and CB:I is an isosceles triangle, x must be 5. Note that x cannot be 2
since :?., 2, and 5 cannot be si des of a triangle. SUFFICIENT

Thc answer is D; cach statement alom; is sufficient.

2-1. E FiN rem·ite the giH~n equation as:

!!!. -~ 2:_1! = !~l + ~1 = !~~ + JI "' 3k


(1) Knowing mis even is 110! enough infonnation since l~l could be either even or odd.
!
For example, is even but ~ is odd. NOT sufficient

(2) Knowing n is odd does not provide enough information since mis also needed to
decide whet her or not k is even. NOT sufficien t

Even when (1) aml (2) are taken together, a conclusion cannot be reached since !!J
remains unknown.

The answer is E; both statements together are still not sufficient.

25. D (l) AB is longer than Be if angle e is larger than angle A Since the measures of angles
A and B are given, the me asure of angle e can be calculated. A, B, ami e are interior
angles of a triangle, and they add up to 180°. Since we can get the measure of angle e, we
can answer the question. SUFFICIENT

(2) If the measure of angle e is 91°, it must be the largest angle in the triangle since
A+ B +e= 180°. Ifyou plug in 91° for G A+ B + 91° =180°.
So A + B = 89°. Neither A nor B can be greater than C. SUFFICIENT
The answer is D; each statement alone is sufficient.

26. A If ab and ba are two-digit numbers, their val u es can be written as:
ab =lOa+ b
ba=lOb+a

(1) If ab- ba = a- b, replace the val' es from above into this equation:

ab- ba =a- b
lOa+ b- (lOb + a) = a- b
9a- 9b =a- b
9(a - b) = (a - b)

The cquation can be true only if (a- b) = O. SUFFICIENT

(2) a= 9 by itself is not information to determine a- b. NOT sufficient


Thc answcr is A; statcment (1) alone is sufficient.

27. D (l) If x =! is a root, it can be pluggcd in for x ami k can b~ determined. SUFFlCIENT
(2) ff thc equation is divisible by (x + 3), that mcans x = -3 is a root of the equation. Yo u
can plug in -3 for x and solvc for k. SUFFICIENT

Thc answer is D; cach statemcnt alone is sufficient.

28. E (1) y 2 - x 2 >O means )' 2 > x 2 • This is not sufficicnt since x aml ycóuld both be ncga-
tivc, both be positivc, or one of cach. Note that y= 3 ancl x = 2, ancl y= -3 and x = 2
work v\·ith statcmcnt (1) but are conflicting results. NOT sufncient
' x-y<
(2) l 1 O mcans
' l x <y1
Try x = 3 ami y= 2:

x < y, such as t<!


262
Try x = -3 and y = 2:

x < y, such as - ft < !


NOT suftkient

Using (l) and (2) together is not sufficient to decide.

The answer is E; both statements together are still not sufl1cient.

~9. E If rr ::; a, then a must be a number between O and l. Only the squares of numbers
between Oand 1 are less than the original number. Notice that d can equal a as well,
so O ::; a ::; l

(1) If2a + 3 is an integer, a can be only O,!· or l. NOT sufficient


(2) a 3 ::; a 2 does not provide any additional information other than O ::; a ::; l.
NOT sufficient

The answer is E; both statements together are still not sufficient.

30. B (l) Mínimum and maximum values can be used to find the range but not the median.
NOT sufficient

(2) The initial statement means that if yo u list all the ages from least to greatest, 50 will be
in the middle. That is the definition of median. SUPFICIENT

The answer is B; statement (2) alone is sufficient.

31. E (1) This statement does not provide enough information on how graduate degrees
are distributed among male and female teachers. You cannot assume that the 60% rate
applies to both genders. NOT sufficient

(2) This statement does not provide additional information about the distribution of
graduate degrees either. As discussed in (l). 60% is an overall rate. NOT sufficient

By using (1) ancl (2) to~ether, yo u can conclude that 60% of the teacbers are female and
set up a proportion: %- = 1~. Cross multiply and find x = 30, which is the number of
1 0
teachers. The number of mal e teachers is 12. You can also find out that GO(Yo · 30 = 18
of the teachers ha ve graduate degrees. However 1 there's still not enough infonnation to
calcula te ma!e ami female gracluate degree holders separately.

The answer is E; both statcments together are sti!l not sufficient.

32. B (l) G.t~ + 2 does not pro vide enough infonnation about k since 6,\:.! + 2 is always even
regardless of k. NOT sufficienl

(2) lf 5k + k~+ 3 is odd, then Sk + k~ must be even because even + odd = odd.

Sk + k 1 is even. All powers of odd numbers are odd, so Sk must be odd. That makes k~ odd
since odd + ocld is even.

If k 4 is odd, k must be odd since powers of only odd numbers are odd. SUFFICIENT

Thc answer is B; statement (2) alone is sufficient.

33. A (l) lf 111 ancln are conserutive integers and 111 > n, tlw!l 111 - 11 = l. Sin ce this state-
ment provides 111 + n =: 35, two equatiuns can be solved simultaneouslj'. SUFFICIENT.

Gfvii\T 1\li\TH WORl\BOO:< 263


1 dm•s not provide IH'\\' information sinn• the question already indirall's
(2) 111 -- 11....,
that111 amln are const>cutin' mJmlwrs. NOTsuflkiPnl

Tlw answer is !\; statement ( l) alonc is surllcient.

:l4. e First simplify tlw expression intlw question by splitting tlw dl'llOIIlillator:
a'c + d1 a 1e eh {¡
··- ¡:¡¡-- ""' . i'ii ¡ ú/ = (/ +- il

( l) This provides {; = 3 or ~ -= j hut not the val u e of a. NOT suflkient


(2) Thc value !J is also ncedcd. NOT sufficient

By using ( 1l ami (2) together. ~ = j and 11 = :i can be plugged in to gct the value.
The answer is C; both statements togethcr are suflkient.

35. D (!) If a power of (2'- 3) is 1 then (2'- 3) is either 1 or -1:

(-1)'"1" = -1, (-l)"""" = l, (1)" 11 1'"''"""= 1:

If m is odd, then:
(2'- 3) =1
2' =4
x=2
SUFFICIENT

(2) x has two solutions sin ce 2' - 3 could be 1 or -1 depending on m.


2'- 3 = 1 2'- 3 = -1
2X = 4 2X = 2
2' = 22 x=l
x=2
If x is evcn, it must be 2. SUPFICIENT

Thc answer is D; each statement alonc is sufficicnt.

3G. e (!) Not cnough information is givcn about/. NOT sufftcicnt


(2) Not enough informa !ion is given about k. NOT sufftcient
Using (l) ami (2) togethcr, if k .l 111 aml { l 111, k must be parallel to {sincc they are per-
pendicular lo the same line on thc same pla11e.
111

The answer is C; both statemcnts togethcr are sufficicnt.

37. C (1) Knowin¡; how many i\mir amll3ianca rccei\·ed is nol enough since wc do not kno11·
the total number of qucstions or how many Carlos reccivcd. NOT stifficient
(2) Knowing how m ay Amir and Carlos received is not enough because of the same
reasons. NOT sufficient

By using ( l) and (2) together, yo u still cannot get the number of questions Amir, Bianca,
and Carlos received individually. lf yo u subtract the two.equations si de by side:

.tl + B = 24
- A+ C= 20
B- C= -l

Bianca received 4 more than Carlos.


The answer is C; both statements together are suftlcient.

38. A The probability of picking a red marble when the hat contains only red (r) and blue (b)
marbles is
r _ r
total - r + b
(1) Yo u can write r = 1· b or ;f = #· This means r = 4 units and /J = 7 units (or r = 4x and
b = 7x). The total is ll units (l1x).

P(red) = 4 X =..L. SUFPICIENT


llx 11
(2) lJ = 30 + r by itself is not sufficient to t1nd -=-L [J, since the total number of marbles is
1
not provided. NOT sufficient
The answer is A; statement (1) alone is sufficient.

39. D (1) -a+ 2 < -l Add -2 to both sides.


-a< -3 Multiply both sides by -1 and switch the inequality sign.
a>3 SUFFICIENT
(2) 7- 2a < l Subtract 7 from both sides.
-2a < -6 Divide by -2 ancl switch the inequality sign.
a>3 SUFFICIENT
The answer is D; each statement alone is sufficient.

40. E (1) The overall average rate cannot be calculated since the average rates ofjunior ancl
senior lawyers are not given. NOT sufficient

(2) This docs not provide the number of lawyers and hours spent. NOT suftlcient

Using (1) and (2) together still does not provide enough information since we still do not
know how many hours werc spent by junior and senior lawyers. Note that yo u need thc
total hours for each since the hourly rate is requircd.

The answer is E; both statements together are stillnot sufricient.

41. C The remainder is 10. Ifyou subiract the remainder from 2-!B, the resulting number is
divisible by K ami T 248 1O = 238.

Al:crnatively, writc K· T + 10 = 2c18 and !\. · T = 23B. The factors of 2:'lfl are 2 · 17 · 7.
1 23B
2 119
7 3-l
14 17
(1) TI1ere is more than one odd integer. NOT sufficient

(2) Using the same analysis, notice that there are 2 three-digit integers (119 and 238) that
could be K NOT sufficient

By using ( l) and (2) together, yo u can conclude that K= 119 sin ce it is the only three-digit
odd integer that divides 238.

The answer is C; both statements together are sufficient.

42. e (1) If (1 - x)(2 - y) <O, then two cases are possible:

Case I: (1 - x) is positive ami (2 - y) is negative.


l - x > O and 2 - y< O
1< x ami 2>y

Case II: (l - x) is negative ami (2 -y) is positive.


l - x < O ami 2 - y< O
1> x and 2 <y
NOT sufficient

(2) x < Ogives no infonnation about y. NOT sufficient

When using (1) and (2) together if x <O, then case I is not P<?Ssible. Case II must be
correct, which means 2 < y, therefore y> x.

The answer is C; both statements together are sufficient.

43. e (1) Both si des of the equation can be multiplied by y to get xy- 1 = 2y. This equation
alone is not enough to get y· NOT sufficient
i
(2) Both si des of thc equation can be multiplied by x to get xy - 1 = 3x. This equation ¡-

alone is not enough to get NOT sufficient
'

By using (1) and (2) together, xy- 1 = 2yand xy- 1 = 3x. Since the left sides ofthe equa-
tions are equal, sct the right si des equal to each other.
2y= 3x
x_2
y-3.
The answcr is C; both statcmcnts together are sufficient.

44. O (1) lf aúcd > 6,000, a has to he 6, 7, 8, or 9. Although cach digit cannot be iclentified,
you know that a is at least 6. The other digits are at least 1, 2, aml 3, so the sum is at lcast
6 + 1 + 2 + 3 = 12. SUFFICIENT

(2) If aúcd is divisible by 5, d is either 5 orO. It must be 5 sin ce a, ú, e, and d are nonzcro.

lfflc = 4d = a, e must be 1 sin ce ifyou pick any number greater than l, a and d beco me
grcáter than 9.

If a= l, e= 8, d = 2, then a+ ú + e+ d = 1 + 8 + 2 + 5 = 16
SUFFICIENT

The answer is D; each ~tatement alonc is sufficient.

45. e lf a O ú = a2 + ú2 - 2aú, thcn


a O ú = (a - b)2.

2S6 GM.4.T i'tATH WORiDOOl\


1 If a Oh= O, then (a- W =O.
1 (1)
So a = h, but this is not enough to find the value of a. NOT sufficient

(2) a= -l - h is not enough information to sol ve for a. NOT suft1cient


By using (1) and (2) together, a = h anda= 4 - b. Substitute the llrst equation into the
second one:
a=4-a
2a = 4
a=2
The answer is C; both statements together are sufficient.

46. B Hemember that (x + y) 2 = x2 + 2xy + y 2•


(1) Since you know x 2 + y 2 = 17 ami x ·y= 4, plug them into the equation:

(x + y)2 = 17 + 2 · 4 = 25 Take the square root.


x+ y= ±5 NOT sufficient

(2) Factor x 2 - y 2 as (x- y)(x +y).


2 2
Since x - y = (x - y)(x + y) and x - y= 3, plug them into the equation:
15 = 3(x +y)
5= x+y SUFFICIENT

The answer is B; statement (2) alone is sufficient.

47. A a= 5h +k

(1) If the remainder is 3:


a= 5h + 3
2a = 2(5b + 3) = lOb +6
Since the remainder cannot be greater than 5 when dividing by 5, the remainder is
6 - 5 = l. SUFFICIENT

(2) If b = 8, then a= 5 · 8 + k= 40 + k. Multiply each side by 2 to get 2a = 80 + 2k.


Without any information about k, the remainder cannot be determined. NOT sufficicnt

The answer is A; statement (1) alone is sufficient.

48. E (l) Not enough information is given to flnd the value of y. NOT sufficient

(2) Not enough information is given to tlnd the val ue of y. NOT suff1cient

Using (l) and (2) together:

y= 7- 2x
14- 2y
x= ---4- l\lultiply both sides by 4.
4x=l4--2y Subtract 14 from both sil!:::;.
4x- 1-1 = -2y Divide l:oth sides by -2.
y= 7- 2x This is the same equation as (1).

Since both (1) and (2) provide thc same information, you canuot find the value ofy

The answer is E; both statemeuts togethcr are still not suftkient.

GMAT NATH WO::!KBOOK 267


49. e (l) Let the length be l and the width be w. 2(1 + w) = 24 means l + w = 12. which is
not enough to find the arca (arca equals l· w). NOT sufficient

(2) Not enough information is given to calculate the arca. NOT sufficient

By using (l) and (2) together, l + w = 12 and l and w are prime numbers. The options for
l ~nd w are only 5 and 7. Arca= 5 · 7 = 35.

The answer is C; both statements together are sufficient.

50. D (l) Factor out 16 and make the denominators equal.


16(1 + l + ~) = 16 ( be + ae + .i!lz_) = 20
a b e abe abe abe
be + ae + ab _ 20
abe -16
be + ae + ab = i abe Rearrange
ab+ be+ ae= Sabe SUFFICIENT
4

(2) Multiply both sides by ab to eliminate the denominators.


4 ab + 4abe + 4abc = Sabe
a b
4ab + 4bc + 4ae = Sabe Divide both sides by 4
ab + be + ae = Sabe SUFFICIENT
4
The answer is D; each statement alone is sufficient.

51. B (l) To check if a number is divisible by 9, add al! of its digits and see if the sum is divis-
ible by 9. If the su mis divisible by 9, the number is also divisible by 9. Even though all
digits are multiples of 3, their su m may not be a multiple of 9. For exarnple, 14 digits with
3's and 1 digit with 6 add up to 14 · 3 + 6 = 48, which is not divisible by 9. If al! the digits
are 9's, the number would be divisible by 9. NOT sufficient

(2) lf the sum of the digits is 72 which is 9 · 8, A must be divisible by 9. SUFFICIENT


The answer is B; statement (2) alone is sufficient.

sz. e (1) N

The first statemcnt alone providcs the information above. NOT sufficient
(2) N

Thc sccond statemcnt alone provides thc information abo\·c. NOT suff1cienl
\Vhen (l) and (2) are taken together:

3x+ x + x= 90
5x= 90
X= 18
Cookies with nuts only: 3x = 54.

The answer is C; both statements together are sufficient.

53. e m 211 + 2;:::: 10 Subtract 2 from each side.


2n;::;: 8 Divide each side by 2.
11;::::4 NOT Sufficient
(2) 2- ~;::::o Add qto each side.
2 >fl
-2 Multiply each side by 2.
4;::::/l NOT Sufficient

When using (1) and (2) together, n;:::: 4 and n :s; 4. So n must be 4.

The answer is C; both statements together are sufficient.

54. B (1) 16n 2 - 2m 2 = 7m 2 Add 2m 2 to each side.


16n 2 =9m 2 Divide each side by 9n 2•
16 _ m 2
g- "7z2 Take the square root ofboth sides.
1
~ could be or 1 -1- NOT sufficient

(2) 4mn = 3m 2 Divide both sides by 3mn.


4mn _ 3m 2
3t111i- 3mn
4 - 111
SUFFICIENT
3-n
The answer is B; statement (2) alone is suftkient.

55. A (l) The time it takes two cars to meet is tbe samc as the ti lile it takcs one rar traveling
at thc su m ofthe speeds of the two cars to pass through the tunnel. lf a car is traveling at
a s¡1ecd of 120 mi/hr:

D= r· t
6 = 120. t
t=-JL¡m SUFF!CIENT
120
(2) When they meet, the total distance traveled equals 6 miles:

6 = r1 • t + r2 • t
r1 • t = 4 an, r2 • t = 2 are also given. Wc can divide these two equations side by side ancl
,.
get -/- = 2 but not the individual s¡wcds. NOT sufficienl
2

The answer is A; starement (1) alnne is sufficient.

Gi'1JH 1•iATH WOR!<:SOOK 269


56. O (1) If you divide a regular hcxagon into triangles as shown, eaeh triangle beco mes an
equilateral triangle. Sincc AB = 6, each side equals 6. ABCD covers 2 of thcse equilateral
trianglcs, so its arca is twice the arca of one equilateral triangle. Find the arca of an
equilateral triangle using the formula crp.
SUFFICIENT

f
l

(2) f7J is twice the height of one of the cquilateral triangles that are shown in the figure. If
the height of an equilateral triangle is known, its sides can be calculated using the ratios
of thc sides of a 30-60-90 triangle. Once the si de length is known, the area can be calcu-
lated as well. SUFFICIENT

The answer is D; each statement alone is sufficient.

57. e (l) This statement means a- b ~ 7. NOT sufficient

(2) This statement means a+ b ~ 9. NOT sufficient


By adding (l) and (2) together:

a-b~7
+a+b~9
2a~ 16
a~ 8 SUFFICIENT

The answer is C; both statemcnts together are sufficient.

58. e (l) Since the last year's salaries are not providecl, this year's salaries cannot be calcu-
lated even though the raise percentage is given. NOT sufficient

(2) Knowing the dollar increase in salaries is not enough to calculate who is making more
this year because last year's salaries are not known. NOT sufficient

By using (1) ancl (2) together, both rvlartha ami Pete's salaries can be calculated.

For Martha, if 6% corresponds toa $4,800 in crease, yo u can set up the proportion:
6 4,BOO
Tiro = -:x- to find hcr salary as of last year.

1 ~0 =
4
You can set up the proportion ·~t2Q to find Pete's salary last year. Then compare.
The answcr is C; both statements together are sufficicnt.

59. A (1) The shadecl arca in each circle has a central angle that is equal to 3600 minus
the corresponding interh'r angle of the quadrilateral. It is not possible to determine
each angle individually. However, the su m of the interior angles of a quadrilatcral
is 3600. Jt is possible to find the sum of the central angles of the shacled regions as
follows:

4 · 360° - 3GOO = 3 · 3GO"


Essentially, the shaded areas add up to 3 full circles. Since (1) provides the radius, the
total area becomes 3 · 1r. 8 2• SUFFICIENT

(2) The sides ofthe quadrilateral are irrelevant to the shaded arcas. NOT sufficient

t The answer is A; sratement (1) alone is sufficient.


t
60. A (1) If mis the product of3 prime numbers, m= a· b ·e and its factors are l

1 l andm
a and be or (l, m, a, b, e, ah, he, ae)
1
¡ b ami ae
l
1 e ami lJa
¡
Alternatively, pick 3 prime numhers ami find their product. For example, use 2, 3, ami 5.

l
m= 2 · 3 · 5 = 30. The factors of30 arel, 2, 3, 5, 6, lO, 15, and 30. SUFFICIENT
(2) If mis even anda multiple of 15, it could be 30, 60, 90, and so on. NOT sufficient
1
1i The answer is A; statement (l) alone is suftlcient.

l
! 61. B (1) This does not provide enough information since no dimensions or areas are given.
1 NOT suft1cient

(~)
The area of 6.ABD is half of the area of ABCEbecause they have the same base and the
same height.

Area 6.ABD = ~ · J¡ • AB
Area ARCE = h · AB

SUFFICIENT

The answer is B; statement (2) alone is sufficient.

62. C (1) Try severa! values for x.


Try l: F +l = 2 Tme
Try -0.5: (-0.5]2 + -0.5 = -0.25 F.!lse

We get contradicting answers. NOT sufficient

(~) Try severa! values for x.


Try-2: (-2)2+(-~)=2 True
Try -0:5: (-0.5)' + -0.5 = -0.25 False

We get contradicting answer5. NOT sufficient

Using (l) and (2) together gives -1 < x <O. So x 2 + x will ahvays be negative sin ce x 2
will always be positive but its absolute value will be lcss than x, which is negative. For
example:

( -0.9)2 + -0.9 = 0.81 - 0.9 <o

The anS\\~~r is C; both statements together are sufficient.


63. E (1) Factor x2 + x = x{x + 1). This does not provide enough information.
If x is even, x + lis odd. So x(x + l) is even.
If x is odd, x + 1 is even. So x(x + l) is even. This means x(x + 1) is always even regard-
less of x. NOT sufficient
(2) x2 + 6
X
= x2X + §,X = x + .§.X
To make this expression an integer, x could be.±: 1, .±:2, .±:3, or .±:6. NOT sufficient

Using (l) and (2) does not provide enough information either.

The answer is E; both statements together are still not sufficient.

64. O First simplify the question by cross mulliplying, 10m= n.

(1) Statement 1 states that m = 0.1 n. If yo u multiply both si des by 10, yo u get 10m= n.
SUFFICIENT

(2) In 10- 15 • m= 10- 16 • n, multiply both sides by 10 16 and simplify.


10 1s. 10 -15. m= 10 1s. 10 -1sn
10m= n
SUFFICIENT

The answer is D; each statement alone is sufficient.

65. D (1) The first statement allows you to calculate LACB.


a+ b + mLACB = 180°
110° + mLACB = 180°
mLACB= 70°
Since x and LACB add up to 180°, x must be 110°. SUFFICIENT

(2) Sin ce BC 11 DE, mLACB must be the same as e, which is 70°. Sin ce x and LACB a cid up
to 180°, x must be 110°. SUFFIClENT

The answer is D; each statcment alone is sufficient.

66. A First simplify the question:


m2 + 4nuz + 4n2 = (m+ 2n) 2
21
(111 + 211)2 + fl!_~ ~ - 2(111 + 211)
(1) Heplace each m+ 2n with 6.
62 + Q- 2. 6
3
SUFFIClENT

(2) 11l = 2 means 111 = 4/l. This clocs not give a numerical value for the expressiun giren
2 11
by itself. NOT sufficient
The answer is A; statement (1) alone is suff1cient.

67. A In orcler to find thc slopc of ( 1, we need to find the slopes of the given !in es first.

(1) Sol ve for y in 3nx + 4ay = 3.


3ax+ 4ay= 3 Subtract 3ax from both si des.
4ay = -3ax + 3 Divide both sides by 4a.
The slope is ~ 3 .
y= -3r+--ª--
4 · 4a 1
The slope of e is also -~sin ce they are parallel. SUFfiCIENT
1

(2) Solve for y in 2ax + 5y = 5a.


2ax + 5y= 5a Subtract 2ax from both si des.
5y = -2ax + 5n Divide both sides by 5.
Y = .=2av
5 -~
+ .5.ª
5 The slope is -;a.
Since we do not know the value of a, the slope of f 1 cannot be determined. NOT suftlcient

The answer is A; statement (l) alone is sufficient.

68. C (1) This statement does not indicate whether a or lJ is greater. NOT sufficient

(2) If IJJI > 1al, O < a< IJ or b < n <O. Try different numbers:

0<2<3 !>! w !>!


2 3 l1 b
-4 < -2 <o _l_ > ...!_ so l >l
-4 -2 b {l

The results are inconsistent. NOT sufficient

When using (1) and (2) together, there's only one possible case in which is b <a< O. That
is sufficient to answer the question.

The answer is C; both statements together are sufficient.

69. C (l) The median of a set does not indica te anything about the average of a set of num-
bers. NOT sufficient
(2) No actual numbers of fish are given. NOT suff1cient

Use (l) and (2) together. lf the median is 6 aml4 out of 7 aquariums ha ve the same mtm-
ber of !ish, those 4 aquariums rnust have 6 fish each to make the median G. If 4 out of
7 numbers are the same, that number must be the median. Those 7 numbers could be
arranged from least to grcatest as follows:

xxxxnlic
nxxxxlJc
abxxxxc
nbcxxxx
In ea eh case, the medinn is 6. So the rest of the aquariums ha ve 12 fish ea ch.
;¡·¡1e aver<~ge
. •• 6 . 4 + 12 . 3 - 60
. > 1s -·--··,r------ - - 7
Thc answer is C; both statcments together are sufficient.

70. A (1) Thc ycllow marblcs are }~ ancl thc green marbles are C.
1'=1
e; 2
Let i' be 3x and G he 2x.
The proba}Ji!ity of picking yellow first is ~:~ and of picking green second without replace-
ment is r:: 2 X . The probability of picking yellow lirst and green second is
:JX- 1

Multiply both sides by~·

5. 3. 2x _ 4 .5
3 5 5x- 1- T5 3
2x _ 4 Cross multiply.
5x .::¡- 9
l8x = 20x- 4
x=2
The number ofyellow marbles is 3x = 6. SUFFICIENT

(2) LetG be t and Y beco mes t + 2. The probability of picking yellow first is it+/ and of
2
picking green second without rcplacement is t ~ . The probability of picking yellow first
2 1
and green second is
t+2 t _4
2t + 2 . 2t +1 - 15
t2 + 2t - 4 Cross multiply.
4t 2 + 6t + 2 - T5
15t 2 + 30t = 16t2 + 24t + 8
t 2 -6t+8=0 Factor.
(t - 2)(t - 4) = o
t = 2 or t = 4

There are two possible answers. NOT sufficient

The ansvver is A; statement (1) alone is sufficient.


Model Tests
L.~
7
-+ 7.1 MODEL TEST 1 AND ANSWERS
-+ 7.2 MODEL TEST 2 AND ANSWERS

Gl\1AT NAT:l WOfC300K 275


ANSWER SHEET
Model Test 1

1. @@@@® 11. @@@@® 21. @@@@® 31. ®®@®®


2. @@@@® 12. @@@@® 22. @@@@® 32. @@@@®
3. ®®@®® 13. ®®®®® 23. @@@@® 33. @@@®®
4. @@@@®. 14. @®@®® 24. @®@®® 34. @®@®®
S.@@@@@ 15. ®®®®® 25. ®®@®® 35. @®@®®
6. @®@®® 16. @®@®® 26. @®@®® 36. @®@®®
7. ®®@®® 17. ®®@®® 27. ®®®®® 37. @®@®®
a.@®@®® 18. ®®@®® 28. @®@®®
9. ®®@®® 19. ®®@®® 29. @®@®®
10. 0®0®® 20. ®®@®® 30. @®@®®
--------------------

-------
MODELTESTl
2
l. 2x and (y+ 2) are invcrsely proportional. \Vhen x = 1, y= 3. \\11at is the value of 8x
whcny= 2?
(A) 10
(B) 5
(C) ~
2

(0) 21
(E) 25
4
2. It takes 12 seconds to cut a uniform metal bar into four equal pieces. How long would it
take to cut the same bar into 7 cqual picces?

(A) 28 scconds
(B) 24 scconds
(C) 21 seconds
(0) 18 scconds
(E) 16 scconds

3. What is the val u e of k+ m if k ami m are integers?


(1) k· m- 1 = 4
(2) k'"= 64

4. Spam filtcr A catches 90% of al! spam e-mails sent toan e-mail box. Spam filter B catches
only 75% of all spam e-mails. IfOlivia uses both spam filters consccutivcly and the
effecth·cncss of ea eh filter stays constant under any scenario, what percent of the sparn
e-mails will make it into her e-mail account?

(A) None
(B) 1%
(Cl 2'7o
(D) 2.5%
(E) 5%

5. Barbara takes 3 hours to finish a task. The same task takcs Bcn 6 hours ami Daría 9
hours to complete individually. Working at their constaiÚ rates, first, Bcn ami Barbara
start \Vorking together ami finish half of the task. Thcn Daría joins thc two. They finish
thc rcst of the task all togcther. I Iow long did it take to finish the en tire task?

(:\) 1t hours
(B) 2 hours

· (C) f+ hours
20 .
(D)-lf hours

(E) {t hours
6. Is x = 3?
(1) (X - 2)X + 2 = l
(2) (X-
x+2
3)x =O
7. The sum of four consecutive odd integers is -96. Which ofthe following is the second
largest numhcr in the set?
(A) -31
(B) -29
(C) -27
(D) -25
(E) -23

8. What is the ratio of r to s?


(1) ris 32% more than s
(2) r is 34 lcss than 2 times s

9 -.!6.25 + {().64 - J0.09 = ?


. 192- n21- ·
(A)~
{B) t
(CJ 1
(DJ 3
(E) 9

1O. .t\B and B.t\ are t:wo-digit integers whcre A and B eaeh representa nonzero digit andA>
B. Js AB divisible by 5?
(1) AB- BA is divisible by 9.
(2) AB + BA is divisible by 11.

11. It takes 10 workers 24 days to paint 8 iclcntical buildings. Working at consistent rates,
how many days would it take 6 workcrs to paint 4 of those buildings?
(t\) 18
(B) 12
(C) 18
{D) 20
(EJ 2·1

12. ¡\fNKis thc largcst three-digit positive intcgcr wherc M =F N =F K. and PQ is the largest
two-digit ncgatiYc intcgcr >vherc P =F Q.
M;VK- PQ =,?
(A) 910
(B) 997
(C) 999
(D) 1,000
(E) l.O~)B

230 Gr'iAT 1'<1ATH WORKBOOK


13. When 64 is divided by y. the quotient is ~r: and the remainder is zero. What is the
val u e of y+ 1 if x and y are both positive integers?
(A) 9
(B) 8
(C) 7
(0) S
(E) 4 e
14. A is the center of the cirde shown. ,\BCD is a square where CD
and CB are tangent to the circle. What is the are a of the cirde'?
(1) The area of ABCD is 12.
(2) The length of are BED is .,..[3.

15. How many pounds of apples can Sydnie buy with all her money?
(1) She can buy either 8 pounds of apples and 6 pounds of pears or 2 pounds of apples
and 1O pounds of pears with her money.
(2) Apples cost $1.50 per pound, ancl pears cost $1.30 per pound.

16. If 2' + }-_--; = 80, what is the val u e of x?


2
(A) 2
(B) 4
(C) 6
(D) 8
(E) lO

17. 3. h>.fFf =
(A) 1J
(B) f3
(C) 3
(D) 3D
(E) 9

18. The smallest of a set of consccutivc integers is -12. If thc su m of all integcrs in thc Sl't is 27,
how inany intcgers are in the set?
(A) 27
(B) 26
(C) 25
(D) 15
(E) 1·!

19. M dollars are split benvecn t\\'O brothers, \\·ho are 15 ami 1G years old, proportional to
thcir ages. \ Vhat is the val u e of .\/?
( 1) The youngl'r brothcr gcts S 13 lcss.
(2) Thc older brotlwr get~ S20B.

Gi"lf\T ¡>,JATH \'/ORi{BOOK 28i


~0. Fm e~:: r<'al mt1~1lwrs, !\:t;l·:ion ~ is ddined as follm\·s:
aB h a·· Ir' i f u : ¡,
ll f /¡ if ¡{. ¡,
\Vhat is tlll' \·altll' of 1 l • :2\ B t:2 • S)'l
(;\) 5
(!l) ¡.¡
(U 15
(!)) J<)

(El 95

2 l. .-\. B. C D. /;', E-: ami (; ;m• 7 consecutin• odd integns. \ \'hat is tlw an•r;1gl' of these 7 intcgers'1
(1) The an•rage of /), 1:'. F. ami (;is li-t
(~l e+ 1:' = 122

22. If} < ~~ + ~· what is tlw largest possible integer val u e of ¡¡'?
(¡\) 22
(B) 23
(C) 24
(D) 25
(E) 2G

23. Carl wants to buy 3 applications for his smart phone. lf he picks 3 ;\pplications ran-
domly among (i ganws am14 utility applications, what is the probability that he will get
2 games antl 1 utility application?

1
i'·
(t\l T2
(B) i
(C:) )~)
(D) ,\
t..

(F) §
2-1. llow many students are in the bh'?

( 1) lf studcnts are grou¡wd so that thcre are 3 studcnts ¡wr tablc, .¡ studcnts are left owr.
(2) lf students arc grou¡wd so tltat thne are.¡ students ¡wr tabll'. tlwre are 2 Pmpty tablcs.

\Vhat is tlll' \'aluc of 11 · e?


(1) (/ 5/J; /){/ 1; d- 2
(' 3
{:!) (/ ~ 1); J¡ = rl ¡ 1; cd ,~ b

2li. 2()";, of a to\\n's population is infected by a \·irus. lf l'\'l'ry \\'l'l'k, :.'O";, nf thc lwalthy ¡wo·
pk are getting infccted, ;tpproximately \\'hat perccnt of tlw popubtion \\·ill be infected
by tlw end of tlíL' Sl'Ullld \\'l'l·k'?

L\) no";,
(Bl (i()";,
(C:) 50'';,
(Dl .¡()";,
(F) :\0''(,

282 Gl'lAT MATH WORKBOO:<


27. A piggy bank contains Q quarters, D dimes, and N nickels. What is the ratio of the do llar
amount of quarters to the total do llar value?
Q
(A) (D +N)

Q
(B) (Q + D +N)
25Q
(C) (Q + D +N)
25Q
(D) (lOD + 5N)
25Q
(E) (25Q + lOD + SN)

28. How many days does it take to fill half of the pool with water?
(1) There are 200 gallons of water in the pool initially. Every da y an amount cqual to
the existing water is added to the pool.
(2) The entire pool takes 10 days to fill.

29. A 39-year-old dad has two sons who are 3 years apart. Three years from now, the
dad's age will be twice the su m of the ages of his sons. flow old is the youngcr
son today?
(A) 3 years old
(B) 6 years old
(C) 9 years old
(D) 12 years old
(E) 15 years old

30. Ift>l,ism<O?
(1) t- mt >O
(2) m - tm > O

31. A document is k pages long with an average of k \rorcls pcr pagc. lf Pat delctes a total
of k words from thc documcnt without changing thc number of pagcs, what is the.new
average numbcr of words pcr pagc?
(A) k
(B) k- l
(C) k:~- k

32. p ancl r ca eh rcpresent a digit. ls Gprprp diYisiblc by r¡f?


(1) r=G
(2) J7 = o
:n. Tlw ratio ot tlw \·oltmws pf nm splll'n'~ is s. \\'ll;tt is tiH' ratio of tlw radii uf tlw t\Yo
1
sp 1wrcs'? (\'olttll1l' of a splwrc: \ · 7>1' l
3
(:\) S'

(B) S:

(C) s
([)} ~S
lEl 'n
:l-L lf each side of a square is inncascd by -1 inclws. tlle arca of tlw squarl' incrL\tsl'S h\· -10.
\Vhat is tlll' arca of the original squarc?
(:\) 3
( l3) -1
(C) 9
(0) 16
(El 49

35. :\ 14-inch piccc of wirc is bcnt into a right trianglc \\'ith a leg of ()in ches. \Vhat is tlw
arca of tlw triangle?

(A)~
21
(B) -;¡·

(C) ~]_
- 4
(D) -"-
75
2
(E) ~s_

36. If y ;t 0, is X == 0?
5x- v
(l) -y-·--= ()
(2) 3xy - ()x =O

:17. Equilatcral trianglc AHCis placcd on tlw rl-ctangubr coordinatc systcm so tll<tl tlw com
dinates ofpoint A are (-2, Ol and tlw coordina tes of point B are (:1, 0). lf tlw _¡·-cnordinat-
ofpoint Cis ncgatin•, huw far is it fromthc origin?
(:\) --4~J
(Bl :zFI:>
(C:) 6h
(Dl 12
(E) Bh

284 GMAT MATH WORKBOOK


ANSWER KEY
Model Test 1

1. D 11. D 21. D 31. B


2. B 12. B 22. B 32. A
3. e 13. D 23. D 33. E
4. D 14. D 24. e 34. e
S. D 15. A 25. A 35. B
6. B 16. e 26. e 36. A
7. E 17. E 27. E 37. B
8. A 18. A 28. e
9. B 19. D 29. B
10. E 20. E 30. B
ANSWERS TO MODEL TEST 1

l. O lf x ami y are inwrsely proportional. tlwir prodtKt is constan!:

Di\·ide by:.! ami rt'pb(T X


1
l. y 1 :l. and y ')

(1) (3 ~ 2) e~ X~ (2 f- :_!)
5 e' X~(-!)

~ = \
·l -2
Br~ =B.(:->)... ~ •e B. 25 - 25
- 2 4 1() ·• ')

2. B To divide a bar into -1 picces, \H' rwed to rnake :1 cuts. lf3 cuts take 12 secomls. each
cut takes 4 seconds.

To cut the bar into 7 picccs, we need to make G cuts. That mnrld take G · -1 = 2-1 scconds.

3. e Olk·m 1
=k· .l
111
·1

1\lultiply each side by 111. k= -t11z. NOT sutTicient

(2) k"'= G4 is not enough information since G4 1 = !F = -t' = 2' 1 = 6-1. NOT sutTicicnt

By using (1) and (2) togcther, we can substitute k= -.1111 into k"' to get (·1111)"' ~e G·l. This
equ.ation will work only wlwn 111 2.

The answcr is C; both staternents together are-sutTicicnt.

4. O Thc tlrst spam tlltcr catches 90%, which means it lets in 10% of the sparn. Similarly,
t11tcr B lets in 25'!'0 of the spam. In the cnd, 25% of 1O'ft, of al! sparn c-mails willmake it
into the inbox:

25% · 1O'Yo = 2.5%

5. O 1f Barbara (.r) all(l Bcn (y) are \mrking togethcr:

1.-t-L=J
X _\' t
l-t-l=~-t-1=}=1
3 G G G G.Z
lt takcs tlwm2 hours to finish thc en tire tao-k. So tlwy spent 1 hour to llnish orH:-half of
thc task. Once Daría(:) joins tlwrn all(l tlwy start \\'Orking togetlwr:
·:
;•
l_¡_l_ !_ 1
~\·.y+-;:-¡
1 .L j . 1 _ () '\ ') 11
3' (~ t ~)·-- IH t ÍTi_¡_ f!¡ j¡j

lt \\'Ollld take tlwm ul hottrs to tlnish thl' l'lltirc task. To tlnish tlw Sl'CO!ld half of thc

task takcs ~ · : ~ ~~ ? hours


;
1
1 Add tlw t\\'o times:
20 lwurs
11 .
6. B (1) If (x- 2¡x+ 2 = 1, then x- 2 = 1 or x + 2 =O. Any number exceptO raiscd to the
zero power equals l.

x-2=1
x=3
or
x+2=0
x= -2
or
x-2=-1
and x + 2 is even

NOT suffident

(2) rr(x- 3)x =o ' then x- ~ =o. The only way x- 3 could be zero is when x- 3 =O
x+2 x+2 x+ 2
and x = 3. SUFFICIENT

The answer is B; statement (2) alone is sufficient.

7. E The average of equally spaced numbers, such as consecutive odd integers, is the mid- ·
dle number.
-96 = -24
4
Sincc we have consecutive odd intcgers, they must be:
-27 -25 -23 -21

Notice that -24 is in the middle of -25 ami -23.

The second largcst number is -23.

8. A (1) r is 32% more than s means:


r = 1.32 · s Divide by s.
sr = 1.32
.
SUFFICIENT-
(2) r is 34 less than 2 times s mcans:
r = 2 · s + 34 This cannot be sol ved for f. NOT suffJcient
The answer is A; statcment (l) alone is sufficient.

9. B -/6.25 = n6 ~ = t-~ = 2.5


-/O.ful = 0.8
-/0.09 0.3

hz - fiiT = .r9T=TI = f8l = 9


2.5
__ + O.B - 0.3 -_ 9-
9_____ 3 _ 3
1

1O. E (1) AB- R\ is always di\"isiblc by 9, rcgardlcss of the val u es oL-\ and B.

;\B=IOA+/3
- BA = IOB+ A
---~---··--··----------
AB- D.\""' 9:1 913= 9(:1- B)
AB could be 65 nr 71. NOT sufficicnt

Gl"iAT MATH WOR:moo;< 287


.\B !0. \ ; /!
/?\ lll/! .. \
----- ---- --
~-

. \H , 1!.\ 1 1.' . 1 1 H 11 (. 1 , m
:\gain . .·W rould lll' ti:i or 71. :'\( l !' ~ulficient
lhe aib\\l'r ¡-; ¡.:: hoth :-;talL'IlWllts togetlwr are still110t suflident.

!l. D First rakubtL' tlll' total anwunt ofdays that l person \\ould takL' to paint B buildin,:
takes 2-l · lO = 2-10 days for 1 person to paint B buildings. '

From here. \·ouun!ld either write a prnportion or find the unit rate. Jfyou cakulate
the rate, it r; kes 2JO days ror B worh·rs to paint 1 building. So.¡ buildii.lgs \\'ould take
1
:\0 · .¡ = 120 days for 1 ¡;erson. It will take 6 workers _!_~O= 20 days to paint .¡ buildings.

12. B The largest tluce digit positin~ integer \\·here .\ l ~' N~' 1\. is 9B7.
The largest t\\o-digit negatiH' intcger -.dll'I'C /'1' Q is -10.

¡\/,\'!\. PQ "'"' ~)!\{ - (- 1()) e= 997

1:l. D Jf the rl'!naindcr is ;.ero. 6-l must be the product of the dividcnd ami the quotient:

2x 2x ·l ,,
-v · -v = -~;::· == G-l
x'- IG
y'
Take the squarc root of both sides.

;\==:'::-l
Since xand y are both positivc. :f.= ·L
:~ -~ 1 = ·l + 1 •ce :i

1-1. D ( ¡) l'-Jotin· tll;tt. \/!. \\hirh is one sidc nf tlw squarL', is a radius of thc cirrk. Sin ce th•
firsi statl'llll'llt giH'S thc arca ofthc squarc. its side. \\hich is tlw radius of thc cirde. e

be cakulatcd:

Then use tlw radius to find thc arca of the rirde .

.-\ ~- ~or'

SUITIUF:'\T
(2) SitKL' .\l)C[) is a squarl'. thc ccntcr anglc fl.\[) is ~JO". That nw;m-; are HF/)is l of 1:
e;}~~:
cirdl'. Tlll' L'Í!"Cll!llfl'rl'llCl' oftlll' circle l!lliSt lw -!times tlll' ll'ngth oftlll' are. . >

\\'hich is L'qu;d to 2 .. r. So sincc WL' knO\\. tlw radius. \\'l' can tllL'rl'forL' calcubtc tlw ,:;

SlllTlU 1':'--:T
llw ans\\"L'r is D: c;¡ch st;l!L'llll'lll ;doiH' is sutTicÍL'Il!.

288 GMAT MATH WOR::somc


15. A Let Sydnie's total money be M, the price of apples per pound be A, aml the price of
pears per pound be P.
(1) M= 8A + 6Pand M= ZA + lOP. We can set these two equations equal to each other:
8A + 6P = 2.-1 + lOP
6A= 4P
3A..= p
2
Substitute this Pinto the first equation:

M = BA + 6P = BA + 6 · 3~1 = 17 A
SUFFICIENT

(2) The prices of apples and pears alone are not enough information to answer the ques-
tion. NOT sufficicnt.

The answer is A; statement (1) alone is sufficient.

16. e l\fultiply both sides by 2 2 - ' to simplify the fraction:


22-x
22-x. 2-< + - = 22-.<. 80
22-x
22 + l = zz-x . 80

__!?_ = 22-x
80
/6 = 22-x
2-1 = 22-x
-4 = 2- X
x=6

17. E Sta:~_!:y working the inner square root, ..f!Y2 = J!i = :}. The overall equation beco mes
3· p ·~· t'vfultiply the expressions under the squarc root to get 3fT', which is cqual to
3. 3 = 9.

18. A Start by adding 12 to 12, --11 to 11, and so forth ami rcalize that tlwy ca eh add up to zero:

-12 -11 -10


'------y-----------"~
+ ... + ll + 12 + 13 + 14 = 27
25 intcgers add up to zero 2 integers

The su m of all integcrs from 12 to 12 will be zcro. Rcmcmbcr that zero is an intcgcr
betwcen -12 and 12. Thcrc are 25 + 2 = 27 integcrs.

19. D M dollars are split proportional to 15 ancl 1G, \\·hich mcans thc youngcr brother gets
l5x ancl the older brother gets 1Gx. M= l5x + !Gx = 31 x. lf\\·e can find x, \W' can anS\\·er
the question.
(l) 1Gx !Sx = $!3
x= $13
SUFFICIF.NT
f!:
l"'';':;.
~T~;r
-;;¡...
}f"

·;'
1~1 ll1x
.r
S:20H
$ I:l

D
ti~¡ f~
''
. SlllTICIE;--;T

llw an~\\l'r is D: each statl'llll'llt ;tl<;lll' i-. :-.utficil•nt.


~··
~) ~

~o. E For (·1 • ~). \H' use a .. - li' '>Íill'l' .¡ · ·>


t·l• ~l V '>.' 1~

For (~ • :Jl. \H' use 11 + /¡ sinn• ~ · :i.


(2 • :J) =2 ~·· :J e ¡

l'lll' t]Ul'Stion then reduces to:


(12. 7) = 12' - 7' =95

21. D ( 1) lf the an.>ragc of ·1 of the ronsecutiH~ ndd integers is G·L tlwir su m must be
li·l · ..¡ = 256. which can be used to find each number. There is no need to set up an
j.
equation, but it would look like:

X+ (X+ 2) + (X+ .f) + (X+ ·1) = 251)

Alternatively. since thc ;l\wage is G·l. the numbcr in the middle (bet\H'en E ami F) must
be G·l. So the numbers are Gl. 6:3, (6·1). li5. Gl. SUHICIENT

(2) In A, H, C. D, E. F. and G integers C ami E are equidistan! from D. This nwans the a ver·

,, agc of C antl E is equal to D.


122 '
D=--r=Gl
''
1; D is al so in the middle of the en tire se t. r\11 tlw numbers are equally spaced lwcausc they
~.
¡. are conscnlti\·e odd integers. So the an'rage ofthe entire sct is also fJ ~e Gl. SUFFICIENT
¡•;
~ ·~ Thc ans\H'r is [); each statement alone is suiTicil'llt.
i
1
!. ,·

22. B First. subtract } from cach side:

3 1 {) 1
¡ 2 11 i. i.
()
J< 11
·1
To Lornpan· tlll' fractions easily. e\pand }¡by
G to make the numcrators hot!J ti. Notice
that \H' cmnot multiply both sidcs h\' 11 sincc \\l' do not knm\· if 11 is positi\·c or nl'gatin•.
{i . ()
~.¡ ·. ,¡
Tlw largest 11 could lw is ~3. [f 11 m·rl' 2·!. tlw fractions mndd hl' cqual ami any intcgcr
grcatcr than ~-1 m>tild nwkc tlw right side less than}

')')
._,).
D Tlwre an· :; diiTert•nt \\ays to gl't 2 g;mws amllutilit\· sincc the ordl'ring docs not mattn.

(;1/(;, (;c;u, or uu;. l.et's cakulate tlw proh;¡lJjlity ofgetting (;(J(;;


(i ·1 :i ·- 1
1Ü. ~l . H . ¡;

\'otin· that probahilitil'S fm cach sn•n;lrio are the sanw. Tlw dl'IHllllÍil<ltors \\·ill ;il\\'a\·s h.·
tlll' sanll' ( 1O. ~l. lll. amlthl' nurnnatms \\ÍII changc onh· tlH'ir ordvr. So tlw 0\'l'Ldl proh-
·¡· ... 1 1 . 1
a 111 11\ rs . · ¡¡ · · .
.
2

290 Gl"L"-T M.f\TH \'.'ORKBOOK


Alternatively, ifyou prefer to use the combination formula:

(Pie k 2 out of6) · (Pick l out of 4)


{J¡{:k 3 out of 6: all possibilities

Q ~ 6·5·~
(6- 2)!. 2!. (4- l)!. l! -:v~· 4
. lO! -- = lO· 9 · 8!
(lO- 3)!. 3! 7!~

2-1. C (l) This do es not give enough information sin ce the number of tables is not given.
One equation could be written; S= 3t + 4, where S is the number of students and 1 is the
number of tables. NOT sufficient.

(2) This also does not give the number of tables. One equation could be written;
S= 4 · (l- 2) = 4t- 8. NOT sufficient.

By taking (l) and (2) together, set the equations equal to each other.

S= 3t + 4
S= 4t- 8
3t + 4 = 4t- 8
t= 12

If we know the number of tables, the number of students can be calculated.

The answer is C; both statements togcther are sufficient.

25. A (l) Start from the last equation and keep plugging in un ti! all variables but n and e are
eliminated:

d_2
e-3
d=~ Plug into bd = 7
3
b ·~)e= 7 Solve for lJ

l;= ~~ p!ug into a= 5lJ

n=s-.?l
2e
ne =_lO;)_
2
SUFFICIENT

(2) Similar to (1 ), start frorn the lcft:

ed ,." G
rt=tf Plug into lJ = d +7

b= iie + 7 Pluu into (/ =


o
_::>_
ú
{/e~_?__ = __?_f--
§ .L 7 G + 7e
e .

NOT sufficicnt

Thc ansm:r is :\; statc·ml'Ilt (l) a!onc is suft1cient.

G1'1AT ~iATH V/ORKBOO:< 291


~li . e l.t•! tlll' !mm population lll' 100;
.\t tlll' lwginning. lll(). ll.~ :20 ¡wopll' arl' inft'l'(t•d alllf BO ~liT not. [)uring tlll' l[rr.,t
lHl · ll.:'- ¡¡·> morl' peoplc gt•t infected .

.\t tlll' t·ml uf till' tlr ... t l\l'\'k, a (()tal of :20 , ¡ li :;¡; pcopll' an· infl'Ctl'd ..\ftl'r utH' \lt, ·

tlww ~tre li llw~IIthy peopil' ll'ft. During tlll' :-rt'ltliHI 1\l'l'k. ¡.;.¡ · 0.~ 1:2.B l:l llH>l'l'
¡woplt• gl't inft'l'tl'll.

.\t illl' l'tlll oft110 \H'l'ks. thl'rl' are:;¡; - 1,, ,)

~7. E 1) quartl'rs are 1mrth ~51) n·nts.


{)ditm·s aw mmh 10/Jcents .
.\'nickl'ls <tre mmh :1S Cl'Ilts.

Tlw total 1·aluc is !.5() + l{)/) + :J.\'.

Tlw ratio of thc value of quarters to thc total is


!.:J()
t!.:il/ + T(l7J +.'3:\i
¡'
;:
¡
l 2B. e (1l \\'e do not know the cap;1city of tlw pool. f'\OT surticil'llL
(2) This statcmetlt dol's not mcntion ¡f tlll' filling procl'SS adds tlw same an1ount l'\'l'l':
da y or not. NOT sullicicnt.

By taking r 1) al1ll (2) together, the lllling proccss is <tS follows:

Start l)ay 1 Da1· 2 Day :1 Dm· .¡


200 ·~ :.'00 + .100 • BOO + 1,liOO + ...

\Ve can go up toDa y ¡O aiHI tlnd out thl' capacity of tlll' pool. Therc is no necd to gct
¡:,, numlwr hut \l'l' knm1· we can. .
i'¡·
,, The ;msm·r is C: both sta!L'I11l'l1ts togcthcr are su!Ticient.
!;


!.'l. B Lt·t tlw youngl'r ;.o 11 be 11 \'l'ars old toda\. Tlw older <;on is 11 + :) years old. Thrl'e yt
frotn 11!\\\·. till'\' ;m· ;¡JI :1 Yl'<II:S oldt·r. .
¡ :\~) : :\ '!. · (// ; :\ e 1/ '- :1 e :l)
·1!. 2(211 ··- ~))

l
.
fl j
~.;·...
.
' 1:.'
:.'1
11
·111
·111

lB

llw \'tlllllgl'r ~011 i~ li \t'ars old inda\.

H:
~.' :HJ. B ( 1) 1 ... !11/ () llll'~JIJS 1. ;, 1- 111) . O.

~: llll' pmdlll'( of 1 ~lllll


111 . ! .

~\
( j .. 11/l is posÍtÍ\l'. 1 is airt'<tlJ\· gin'll (O lll' po:-.ÍlÍ\l', SO]

... ..
'' hicll tlll\tlls 1 · 111. So 11/could lw. or .~· \OT suftlcit•nt.
H~!
~r (:.') 111 1111 · () llll'illlS /11 · (] fl . O.

i:,,.,.
lfr:
lhl' product of 111 ;ttHI ( 1 ... f) is positilt'. 1 is alrc;ttil gin·n to lw grl'atcr th~m l. so ti r·
lllll<..! lw Jll'g<IlÍ\l'. llll'll 111 is tll'g~tlÍil' ~inn· tlwir prodtwt is po'itill'. So 111 · O. Sl!ITIU
~~: ~~
~·'·

~~~~~$l:l¡i;í:
·¡ ht• <tth\lt'r is 1\: st~tlt'tlll'llt :~1 ~tlotll' is sttlficit'tll.

i 292 Gl.,>jAT r.1ATH WORKBOOK

¡;¿~ULll
31. B Initially the total number of words equals the number of pages times the number of
\VOrds per page, k· k = k 2• If Pat de tetes k words, the new total will be k2 - k..
2
Sin ce the number of pages is still k, the new average will be (k ; k) = k( k; 1) = k - l.

32. A (l) lf r = 6, the question beco mes is 6p6p6p divisible by 6p?


()ptJpGp = 10101
6p
65
For example, ~~ 65 = 10101.

Any digit you replace for pwill give a quotient of 10101. SUFFICIENT
(2) lf p = O, the qucstion becomes is 6 · OrO rO divisible by rO? Not necessarily, because for
r = 1 the answer is yes antl for r = 7, the answcr is no. NOT Sutncient
The answer is A; statement (1) alone is sufficient.

-t_r3
33 • E V1 --3"
' 1

'~"-4-r:~
\-'2- 3 ,¡ 2

V 1 .. r]
-l. =
V,.
_3__
4 -r
1 = S
Simplify 1and TI.
3 .. 23
Take the cubc root ofboth sides.

34. C Let one si ele of the original squarc be 11.

Original area = 11 2
Increasecl arca = (11 + 4f
+ 4) 2 =
(n 11 2 + 40 Square the left side.
n' + 8n + 16 = 12 2 + 40 Subtract n 2 ancl 1G from both si des.
811 = 24
11=3

Thc original area is 11 2 = 32 = 9.

35. B lf onc of the legs of a right triangle is 6, the su m of the othcr lcg ;md thc hypotcnuse
must be 1-l -- G = 8. Lct the sccond lcg be 11, ami thcn thc hypotcnusc bccomcs 8 - n.
Use tlw Pythagorcan theorcm:

Suhtract 11 2 from both sides.

36 o= G·l - l Gn

lGn = G·l :\G = 28


_ 2B _ 7
ll--lG-:1
Thc sccond ll'g is Jinchcs long.
' ' is. J
Tlw ·¡rn 2 · leoh¡ · le<>02 = _1
2 · G · ·l7__ =e 21
.¡ .

Gl>lAI t>1ATH V/ORKBOOK 293


:( , .. :Hi. A ti) Splít tlw fraction to gL'I ::i;· O.

This nwans ::i: l. Tlll'rdt>n' x ; O. The ans\\L'r ís tÍo. Sl!ITIUF\T

1:.!) :l.tT · li.r :l.tlr :.!) O. .rcuuld be ;ero nr _rcould l't¡ual :.!. Thís st~tlt'l1Wilt is
incondusÍ\l'. :\< lT ~ultirít'llt

The ans\\'er is .-\; ~tatt'llll'llt ( l l alom' ís ~uiTicÍL'Ilt.

37. B First sketch tlll' poitth on tht' .ty-coordinatL' systL'Ill.

.-\ 1-2.0) T H (6,0)


2 () 2 6
1
,
-
-3
--1

:\R equals B, \\·hich is one side of the equilateral triang!P. Since the x-coordinate of
Chas to be in tlw middle of:\ and H, it must he 2. Draw /Cin the figure to show the
hcight of the cquilatcral triangk. lJsing the 30-G0-90 triangle ratios or information from
the geometry section, \\'l' can conclude that re= 4rL That means the coordinates of
point Care (2, 4P3). -

By using the Pythagorean theorcm in triangle OTC or the distance formula, we can
calculate the length of OC:
.¡¡¡:r:¡¡:·+-2-; ~ & = 2 [¡ 3

J
! .

;l

294 Gt·lAT MATH WORKBOOK


ANSWER SHEET
r Model Test 2

1. @@@@® 11. @@@@® 21. @@@@® 31. @@@®®


2. @@@@® 12. @@@@® 22. @@@@® 32. @@@@®
3. @@@@® 13. @@@®® 23. @@@®® 33. @@@@®
4. @@@@® 14. @@@@® 24. @@@@® 34. @@@®®
S.@@@@@ 15. @@@@® 25. @@@®® 35. @@@@®
s. @@C9®® 16. @@@@@ 26. @@@@® 36. @@@@®
7. @@@@® 17. ®®@®® 27. @@@@® 37. ®®@®®
8. ®®@®® 18. ®®@®® 28. @@@@®
9. @@@®® 19. ®®@®® 29. ®®@®®
10. @@@@® 20. @@@@® 30. @@@@®
--
l. 10% of an island's population is infected with arare type of flu. A researcher discovered
that each sick person infects two healthy people in one week. What percent of the
island's population will be infectcd two weeks from now?
(A) 40%
(B) 60'-''o
(C) 90'Yo
(0) 99%
(E) The cntire island

2. \Vhat is the perimeter of the isoscelcs triangle ABC?


( 1) Two of its si des are 4 ami 6 inches long.
(2) Two of it angles are 50° and 80°.

3. In a rugby tournament with 12 teams, each team must play every other team once in
the first round. How many total games are in the first round?
(A) 11
(B) 55
(C) 66
(0) 110
(E) 111

4. When €0.642 was worth $1, what was the approximate value of$280 in euros?
(A) €1GO
(B) €180
(C) €200
(0) €400
(E) €4·!0

5. t\ndy is a years olcl, and his son Drew is dyears old. In how many years (in terms of a
ami d) will Andy be 4 times as old as Drew?
(¡\)
a-· 411
--;¡--·
(B) 3n + d
--,¡----
( C) g__::-:f!~{

(D) {1__::-._ (1
4
- 4a - d ·
(L) ---:f·--

6. Is integcr P di\·isihle by B-1?


(1) Pis di\·isiblc by 6.
(2) Pis di\isiblc by 1-1.

Gl'V>.T MATH V/ORXBOOK 297


~. ''

7. FYe and t;rpg togethPr ha\'c SS. lfl;reg has Srn1norl' than FYe. ho\V 1nuch 1110l1t'Y doPs
Fn· han- in terms of S ami nz'?
(:\) S · '.!.m
'B) (\ .. '.!.11!)
\ 2
lU S · 111

\Dl (S m)
')

\ 1) S f 111

B.Gr=-\
.
8

:\!? is tangent to the circle at :\. \Vhat is the radius of the circle \\·ith center /J?
( 1) . tB = 2-1
(2) HC = lB

9. If f..: cookies sell for h dollars, how many cookies can be purchased fnr 111 dollars?
m/J
(t\)
-e
[¡f.:
lB) 11/
(C) 111/Jf.:

,[2
(.\)
T
(B) !2
2
(C)

lDl-~
\Fl
·n2
11. \\'hat is thc ¡wrccnt profit from thc s;lle of one pair of shocs?

( 1) Tlw storc huys :1 pairs for S:1Tand sclls 2 pairs for S(iT.
('.!.) The s;11l's price of one pair of shocs is ~ of thl' cost of o m• pair.

12. lf 111 ·, 11 ·.O· r. \\hich oftlw follm\ing must be falsc:>


(.\) /1/: n ·. ()

(B) !_1 + ll1 ·~ ()


1
(C) 111 /l. ·. ()
IT' 1/1
(()) 111/1- 1111 - ()
( ¡:) 111 -· /1/1/ . ()

298 GMAT MATH WORi<SOOK


13. What is ~he value of ~ + -fi + -~?
(l) a= 2b and b = 3c
(2) ab + ac + be = 2abc

l-l. If the terms 2t- 6 and t + 8 represent two consecutive even integers, what is the
greatest possible value of their su m?
(A) 38
(B) 42
(C) 46
(D) 50
(E) 56

15. What is the value of ~-~~-=_ t~:?


a- b _
(l)
(ij}- -52
(2) a+ b = ª-ab
5

16. x and y are integers, ami -6 < x :::; 8 and -3 :::; y < 5. The greatest possible val u of e
3x- 2y is how m u eh greater than the least possible val u e of 3x- 2y?
(A) 63
(B) 53
(C) 48
(D) 46
(E) 37

17. The roots of the quaclratic equation x2 - 4x + m- 3 = O are X 1 ami X 2 • If X 2 = 3x 1,


what is the value of m?
(A) 4
(B) 5
(C) 6
(D) 8
(E) 9

lB. lJ is what fraction of the surn of a ami IR


(l) The ratio of the su mofa anclb to a is G.
(2) a is 20% of lJ.

19. If 5" · 3 - 5''' 1 = 3,000, \\hat is the val u e of 11?


(:\)
(B) 2
(C) 3
(D) 4
(E) 5

GNAT ¡; \TH VVORXBOOK 299


20. One-thinl of the sum of two positive integers equals their difference. If the sum of the
squares of the two numbers is 2-15. what is the di!Ierence of their squares?
(A) 7
(B) U
(C) 98
(D) 147
(E) 17·1

21. Is b- a negative?
(1) a·b<O
. (2) a- b >O

22. Thc cost of a purse toa retailer is $120. At what price should the retailer list the purse to
make a 20% pro lit aftcr offering a 20% discount?
(A) 514-l
(B) $168
(C) $172
(D) $180
(E) $196

23. On Junc 11•11, Investor :l had 2,400 shares of stock X ami began selling those shares at
a constant rate of 60 shares pcr da y. On the same day, Invcstor B bcgan buying stock
X ata constant rate of lOO shares pcr da y. If Jm·estor B had 320 sharcs of stock X prior
to his purchase on June ¡¡•h, after how many days did the two investors have the sarne
numbcr of sharcs of stock X?
(A) lO days
(B) 11 days
(C) 12 days
(D) 13 days
(E) 14 days

24. There are k bluc ha lis all(! t red balls in a bag. Tvvo balls are pickcd consccuti\·cly with-
out replaccmcnt. What is the probabilíty of gctting a blue ball in thc first pick ami a red
ball in thc sccond pick?
(A) _jg__
k+ t
k
(B) _k +t + + t - 1
k t
ee) r:t:r ·1-:t:-í-=-I
k+t
(Dl F¡:-{:---¡
(E) _j:~ . _L -
k+t k+t

1
25. Is LJ-=~ positin:?
) - (/

(l) O<a<l
(2) ah> IJ

300 GMAT ~!ATH WORX300K


26. What is the ratio of rto s?

(1) :!_+S= lB
r
(2) .:} +r= 12

27. There are twice as many girls as boys on a water polo tcam. If t\vo playcrs are pickcd
at random, thc probability of picking t\vo girls is ~i. \\'hat is the probability of picking
t\vo boys at random?
5
(A) 51
11
(B) 51
16
(Cl SI
(D) 19
51
(E) 29
51

28. At the end of30 days, Jan made $2,600. How many days did he actually work?
(l) For every day he worked, Ian made $140 net. For every day he did not,
he spent $60.
(2) The number of days he workcd is 6 more than twice the numbcr of days he did
not work.

29. A senior consultant charges M dollars per hour for his services. His associate chargc:s ·~
of that rate. If for a ccrtain projcct the associate worked twice as many hours as the
senior consultan t. in tcrms of Mwhat is the average hourly rate for the project?

(A) 2A1
5
(Bl lM
5
(Cl ~-M
5
(Dl QIY
10
(E) §gl

30. Thc cost of a cab ride in a certain city is $t for thc first mil e and n<t for caeh 0.2 miles after
that. lf Sandy paid $P, how far W<~s her ride in terms of n, t, and P?

(:\) ~~(P- t)

(B)
P-t
-511
(C) E__--:__1_ + l
511
(D) f-=_1 - l
511
(E) ~~(P -- t) + l

GMAT MJ\TH WORKBOOK 3\.11


31. Every day, Joe's daily wage increascs by an amount cquivalent to his total wagcs from
the day before. How m u eh does Jo e earn on the 11th day?
(1) He made $573.44 on the 14th day.
(2) He m a de a total of $4.-t l in the tirst 6 days.

32. Ronaldinho invests part of his $10,000 in an account earning 6% simple interest per
year. He invests the rest in another account earning 7.5% simple interest per year. If
after 3 years the total interest carned is $1,980, what is the ratio ofthe amount ilwested
in 6% to the amount investcd in 7.5%?

(A) 23
(B) 4
3
(C) l

(D) J.4
(E) ~
3

33. A retail clothing store is planning Oil sending out questionnaires via e-mail. The
marketing department estimates that 40'Yo of al! e-mails get discarded as spam and
only 20% of the people who receive the e-mail actually start the survey. Finally, the
department es!imates only -l- of the people who start the survey actually finish it. If
the marketing department wants at least 360 completcd responses, how many e-mails
should be sent at mipimum?
(A) 3,000
(B) 6,000
(C) 9,000
(0) 15,000
(E) lB,OOO

34. In the figure, the circle is tangent to the y-axis with a center at O. The line that passes
through D, O, ancl B (not shown) is parallel to the y-axis. If the coordinares of point B
are (n, m). which of the follmving could be the coordinates of point D?
(A) (n,- m)
(B) (n, n - m)
(C) (n, 2n + m)
(D) (n, n + m)
(E) (n, 211- m)

·•',¡
~
~ e

302 GMAT MATH WOR:<BOOK


35. Is};t = !!!:.1
Tl •

(1) k _ m
t:::::-f - n - m
{2) _k_ = _!!]_
t+k n+m

36. The dimensions of rectangle ABCD are x antl y, and the dimensions of rPctangle EFGll
are (x + y) and (X- y). What is the ratio of the diagonal of EFGII to the diagonal of
ABCD?

{A) y
{B) X
x+y
{C) .f2
{D) _X_
x-y
{E) i
1
37. If in amln each representa nonzero cligit, what is the val u e of !B!_Z...
m.n + -0'!J!.!:_-
.mn !L10'B?
(1) ll = 2
{2) m= 3

Gi'liAT t·1ATH WORl(BOOK 303


ANSWER KEY
Model Test 2

1. e 11. D 21. B 31. D


2. e 12. D 22. D 32. A
3. e 13. B 23. D 33. e
4. B 14. D 24. e 34. e
S. e 15. B 25. e 35. D
6. E 16. B 26. e 36. e
7. D 17. e 27. A 37. B
8. e 18. D 28. D
9. D 19. B 29. B
10. e 20. D 30. E
l. e ,\ssume there are lOO people on this island.l11at means lO ofthem are infcctcd in
the beginning.

1O will infect 20 in one wcck. At the cnd of l week, thcre will be 30 sic k pcople.

30 sick peoplc will infect 60 people during thc second week. At the cnd of the second
week, there will be 90 sick pcople. Therefore, the percentage of sick people will be
90_ = 90'''
100 .o.

2. e (l) The triangle could have Jegs of either -t-4-6 or 4-6-6. NOT suft1cicnt
(2) The statement docs not provide any side lengths. NOT sufficicnt

Taking (l) and (2) togethcr shows_ that two angles are soa aml80°. This mcans the third
angle is 50°. Then we can conclude that the side opposite the soo angle is 4 sin ce it is the
shorter side. Therefore, the triangle must have Icgs of 4-4-6. Thc perimetcr cquals 1·!.

The answer is C; both statements together are sufficient.

3. e Team 1 \\ill play 11 games; Temn 2 will play an additional 10 games sincc we've
already counted its match \\"ith Team l, and so on.

Add 11 + lO + 9 + 8 + 7 + 6 + 5 + 4 +- 3 + 2 + l.
For easy addition, rearrange as l + 9, 2 + 8, 3 + 7, and 4 + 6 to make four lOs.
11 + 10 + .10 + 10 + 10 + 10 + 5 = 66 gamcs

4. B Set up a proportion:
-~ --7 0.64g_ = _:!_
$ l 280
X= 280 · 0.642
x= €179.76
X= €180

Alternatively, use the process of elimination. 0.6,!2 is slightly largcr than 0.6.

0.6 · S2BO = $168

Your ans\\'er should be sli¡;htly more than S1G8. The closest answcr is SlB O.

5. e Let the llUI\Jber ofyears be x. In xyears, Andy will be a+- xyears old ami Drcw will be
d + xyears ole!. Set up the equ:: ion as follows:

a +- x = 4 · (d + x)
a+- x = 4d + -lx
a 4d = 3x
a- -Id_ .
---:r···- -· X

6. E ( 1) P could be 3G, 1GB, or any numl 'Cr divisible by· G. Hcme\·er, 3G is nol di\·isiblc by B·L
i\OT sufficient
(2) P could be 2B, 8-1, or any n: mber di\·isiblc by J.l. HmH'\·cr, 2B is not di\·isiblc by B·l.
NOT sulTicimt

GMAT i'cll\TH WORl<BOOK 30::;


Taking (1) and (2) together is not sufficient because Pcould be 42, which is divisible both
by 6 and 14 but not by 84. Pcould also be 8·l, which is divisible by 6, 14, aml84.

The answer is E; both statements together are still not sufficient.

7. D Let Eve's money be E. Then Greg's money becomes E+ m. Set up the following
equation:
E+E+m=S
2E+ m= S
2E= S- m
E= (S- m)
2

8. C (1) AB alone does not provide enough information since we cannot create an equation
for the radius. \Ve are givcn AB. but we do not know the lcngth of either BD or BC. We
know that ifwe draw Di\, ABDwould be a right triangle. However, we do not have enough
information to use the Pythagorean thcorem. NOT sufficient
(2) BC alone is not enough infonnation sin ce we cannot crea te an equation for the radius.
We are given BC. but we do not know the length of AB. We do not ha ve enough informa-
tion to write a Pythagorean thcorem. NOT sufficient
By taking (1) a~1d (2) together, we can use the Pythagorean theorcm for triangle ABD since
DAis perpendicular to AB. Let DC =DA = r and Jet BD = 18 + r:
(18 + r)2 = 24 2 + r 2
324 + 3Gr+ r2 = 576 + r 2
36r= 252
r= 7

The answcr is C; both statemcnts together are sufficient.

9. D Set up a simple pro portian to find the number of cookics:


__$_ _ fJ_m
numbcr - 7:: - T Cross multiply.
? = !!_~~
. b
The answcr is D.

10. e
hf¡ + }~) 1ff +

=L/ 17 ~:?n) ;t-


7
+ + = (}z + 72) ·!J-
(-J~). ~~ = 1

305 Gt"'AT HATH WORKBOOK


T 11. D (l) The cost of one pair = 5/ .

The sales prire of one pair = 6 l = 3'1: so the perccnt profit:


5T
3 T - _,.f
New - Original
- . - . --·lOO% = ~,r- ·IOO%
0 ngma1 -º--
3
The T's can be factored out in the numerator and cancelled out. So a numerical value can
be calculated. SUFFICIENT

(2) Let thc cost be x. The sales price beco mes 9 5t. The percent profit is:
9x
New - Original ·S - x
---,-.-_-""-- · 100'!-h =--.-·lOO%
Ongmal x
Similar to (l), the x's can be factored out in the numerator and cancelled out. So a
numerical value can be calculated. SUFFICIENT

The answer is D; each statement alone is sufficient.

12. D Review each answer choice separately:

(A) Since m< n <O, 1mi> In l. For example, -5 < -3 <O ami 1-51 > 1-31. So m 2 - n2 is
always positive. TRUE

(B) q is negative since n < O < tanda negative divided by a positive is always negatiw.
ft
Similarly, is also negative. A negative plus a negative is always negative. TRUE

(C) 11 and 1,~ are both positive sincc a negative dividcd by a negati~e is always positive.
'J
Remcmber that a positive plus a positive is always positive. TRUE

(D) mn - mt = m(n - t). Sin ce n < O< t, n - t must be negative. Remembcr that mis
also negative. A negative multiplied by a negative is always positive. FALSE

(E) 1lt- 11111 = n(t- m). Since m< O< t, t - m must be positive. Remcmbcr that n is
also ncgati\'C. A positivc multiplicd by a ncgatiw is always negativc. TRUE

Once you rcalize that (D) is thc answer, you do not havc to rcview answcr choice (E).

13. B (l) Wc can \Hite each variable in tcrms of one of thc othcrs, but that does not giw a
numcrical valuc. For example, a = 2iJ and iJ = 3e, which also means
2/J = 6eand rz = 2/J = Ge.
-(}-
)e
+ -,3Le + le cannot be solved for a numerical valuc. NOT sufficient
(2J 1_ + 1 + _L = _/zc:_ + .!It: + _!.1_/z_ = !!_r;__±_fif..:i-_!i!!.
a iJ e abe abe rz/;c alJc
Sincc ab + ac + /Je = 2al;c, !!.L±-rzJo:_±_{z_lz. = ~!.1-~ = 2. SUFFICIENT
abe abe
Thc ans\n;r is B; statement (2) alone is sufflcient.

1-L D lf thc two givcn cxprcssions are consccutivc ewn integcrs, their differcncc must be 2.
llmw\·cr, \\·e do not know \\·hich one is thc greatvr eme. Try two different sccnarios.
First assume t + B > 2 t - 6:

(l + B) (2 t G) = 2
t + B- 2t + G = 2
t =: 12

GMAT NATH WORKBOOK 307


If t = 12, the numbers are 18 and 20. The sum must be 38.
111en assume 2t- 6 > t + 8:

(2t- 6) - (t + 8) = 2
2t- 6- t - 8 = 2
t - 14 = 2
t= 16

If t == 16, the numbers are 2·! and 26. The sum is 50.
The second option gives usa higher sum.

15. B Simplify the question first. We are looking for:


j__j_
a-z - b-z al bz b2 - az. ab
(r 1 - b- 1 = -1-l = a2 b2- ·Ti- a
a-b
_ (b- a) · (a+ b) · ab _ b + a
- ab· ab· (b- a) - (iJ)
(1) This statement provides a- b = -~. which does not give us the valuc of bf+lba ·
ab 5
NOT sufficient

(2) Since a+ b = ~ab is given, divide both siclcs by ab to gct b ;b a=~· SUHICIENT
The answer is 13; statement (2) alone is sufficicnt.

16. B To find the highest possible value, pick the largest x = 8 and lowest y= -3.
3x- 2y = 3(8)- 2(-3) == 30
To find the lowest possible val u e, pick the 10\vest x. Since -6 < x and x is an intcger,
x = -5. The highcst y= 4.
3x- 2y= 3(-5)- 2(4) = -15-8 = -23
The maximum minus the mínimum is
30- (-23) =53

17. C 13oth roots must satisfy the equation. Plug thcm in for x a11CI create two equations.
Instc;-¡d of using X2 , plug in x 1 and 3x1•

x 12 - 4x1 + 111 3 == O

(3.1.) 2 - 4 · 3 · X 1 + 1/l - 3= 0
9.t/- 12x1 + m- 3 = O Subtract the first equation
.=_:x-tz - 4xl +_!!!_-:]__:__~
8x 12 - Bx 1 = O
8x 12 (x1 - 1) =O
X1 = 0 Of X1 = 1

lf .r1 = 1, then x2 = 3. Plug onc of tlwm into thc original cquation to find m. Note that X1 ""O
docs not satisfy the initial conditions:

12 4·1+m-'3=0
/ll e= 6

Alternativcly, try ans\\Tr choices until ym1 get 1\\·o roots \\·hcrc x2 = 3x 1•

308 Gi"lA1' MATH WORXBOOK


18. D The question can be ttanslated as follows into b = x. (a+ b). So the fraction we are
Iooking for is _jz__b.
a+
(1) 111e statement translates into a ~ b == 6. If yo u cross multiply, you will get a + b == Ga,
1
which means b = 5a. So
a-.
: b = - 1 +Sas = : :>Gc:a =
( a a
tb
SUFFICIENT

(2) a is 20% of b means a= 0.2b or Sa = b. So, _b_ = ....2(1_ = _!?ª = t SUFFICIENT


a + b a + Sa 6a 6
The answer is D; each statement alone is sufficient.

19. B

5" • 3 can be written as 5" · 53 = 5" · 125.


5" • 1 can be written as 5" · 5.
125. 5"- 5. 5" = 3,000
5"(125 - 5) = 3,000
5" = ~.ooo = 2 c:) = "'J2
120 ' '
n=2

20. D Translate the first statement:

1(a+ b) =a- b 1\lultiply by 3.


a+ b = 3a- 3b Add 3b and subtract a frorn both sides.·
4b = 2a Divide both sidcs by 2.
2b =a

Translate the sccond statement:

az + [J2 = 245 Substitute a = 2h.

(2l7)2+ b 2 = 245
4b 2 + Ú2 = 245
5b2 = 245
b2 = 49

h=7 The qucstion says thcy are positivc intcgcrs.


a= 14
Thc tliffcrence of squares = 14 2 - 72 = 147.

21. B (1) a· [; < Odo es not providc cnough information. If a is positive, lJ is ncgative. If bis
ncgath·e, a is positi\·c. NOT sufficient

(2) If a - b >O, yo u could multiply hoth si des by -1 ami flip the incquality sign to get
b -- a< O. SUITICIENT

The answer is B; statemcnt (2) <1lonc is sufficicnt.

2'J D If thc cost of thc purse is S120, the retailer must scll it at S1H to make a 20% profit:

120 · 1.2 =e $14-l or


120 + 120. 20"(, == 120 + n = SJ.!.l

Thc rctailcr W<mts to rccl'i\"C Sl-11 aftcr ofkring <l20''b discount. So the list price less thc
20"[, off tlH' list prin' must be S 11·1. Ll'l tlw list prin' lw .\~

X-· O.:!x 11·1


O.H.r 11-1
,. 1·11
'' X-- SlHO
tl.B
¡:'

2:\. D Let tlll' numlwr ofshares ofstock .\in pmtfolio .\ lw .\.So.\ 2 .. 100 · tíOtf. \dll'rt'
tlw numlwr of days.

Let tlw 1Himlwr of sharcs of stock.\ in portfolio B be 15. So ll e 320 +- lOOti.

lf they beco me equal in d days. set tlwsc t\YO expressions equal to cach otlwr:

2.-100 -- GOd :120 + 1OOd


2,100 = .]20 + 1GOd
2,()(10 1tíOd
U=d

2·1. e The probability of pirking a blue hall in the first pick is


Number of hlue _ k
- --,f'()t;!T"_____ - T:+~r

Sincc there's no rcpbccment, thc ncw total is k+ r - 1. The probability of picking"


ball in the sccond pil'k is

I+r___ _
(

Picking a bluc first ami a red second is the product of thc t\\'o:
k l
¡::.¡:-{ . T :~--i-=1

25. e (1) \Ve know that o< (/ < l. So(/ is a positi\'C fraction lcss than l. but wc still do l'

hm·e any information about h. (f IJ = :1 ami o= ~. tlw r~sult would lw positi,·e. but '
ami n = ~ \\'ould result in a ncgati\·c \·alue. NOT sutlicicnt __ _
(2) o/1 > !J docs not prm·ide cnough infonnation h\' itsl'lf sin ce \\T do not kno,,· d f¡ ;·:
1
positin' or negati\e.lf /Jis negati\·c, thcn di\iding by i1would llip thc incquality so ',
1
lwcomes less than l. lf l1 is positi\·c, thc incquality stays thc sanw and 11 bcconws g
than L NOT sullicicnt

By using (! l ami (21 togcthcr. \\'l' can condudc that /1 is a ncgatÍ\'!' number. \\'hcn"
1
multiply any positiYc numbcr l.?} a numlwr lwt\H'en o ami 1. tllc mllnbn gcts snJ<: '

For cxamplc :\ · ~ · :l or :} ·}-:.} lf f¡ is rwgatin•, -3 · l '- --3 as in statt•rm'nt l2l.


rr [¡ is a ncgati\'l' llllllll)l'r, 1,L-
' 1-- {/
1 \\"Ollld ah\-a\'S be positiH'. For l'\;!lllpll'. :_:l.. -
. -3 ')
r.
Tlw ans\H'r is C: hotll statL'llll'llts togc·tiH'r are sufllcit'Ilt.

2(), e (1) -f, + S ce :¡__;_ '/ "" ·l ~- rs 1B. You could a !so cross rnultiply. llo\\·cn'r. -1 ¡_ ,._,

docs not pro\-itk' cnough inform;1tion. l\01 su!Ticient

(2) ~! + r ·~
+ '-~' • 1 ~,-L' ~ 12. '\o u rould a !so r1 oss
·_t multipl~. 110\\l'\ cr. -1 -+ rs ·~
1
:
does not prmidc l'Ilough infomwtion. :\Ol suffirit'Ilt

310 GMAT t':ATH Vil/'KBOCZ


By using (l) and (2) together, we can say l + rs = 18r and 4 + rs = 12s. Set them cqual to
each other.
18r = l2s so r.:
S
= l.?.
16
The ans\ver is C; both statements togethcr are sufficient.

27. A Let the number of boys be x. The number of girls beco mes 2x, and the total players
beco mes 3x.

We can calculate the probability ofpicking two girls as ifwe are picking them consecu-
tively (obviously without replacement):
2
X is picking a girl in the first round.
3X

((2x-
x _ l) · pie
) IS · k'mg a gu
· 1m
· t11e seconll ro u m 1.
3 1
~-I. ~x- 1 = ~~ Simplify thc x's on the left.
3x 3x- 1 51
:t
2 2t- 1 22
3 · 3 -l = ~~
. . 3
l\lultiply both s1dcs by 2.
]_ . ~ . ~X - l = } . I?_ = _ll
2 3 3x - 1 2 51 17
2x- l _ 11
3x =-r -
17 Cross multiply.
3ü l 7 = 33x - 11
x=6
There are 6 boys aml12 girls.

The probability of picking two boys consccutivcly is


6 5 5
T8. 17 =51

28. D (l l Lct thc numbcr of days Ian workccl be x. The number of days he el id not work is
(30- x):

2,600 = HOx + (30- x) · (-60)


2,600 = 140x -- l.BOO + GOx
-uno = zoox
22 =X
SUFFICIENT

(2) Let thc numbcr of days Tan mnkcd be x. The numbcr of days he did not \HJrk is (30 -- x):

x = G+ 2 xJno -
x = G + ()() -- 2x
3x = 66
X= 22
SUFFIC!El\T

The ans\rer is D; cach statenwnt alone is sufficicnt.

29. B
Total ch~If"l'
:\n·ranc rate = -- --- -- --"--
" Tot;1l hours
Let thc time spent by thc senior con~ultant lw 1 hours. ·¡he a:;sociate spcnt 2thours. ami
tlw assm·iatl''s ratc is ,\[ · ~ ""' 2nl
:) 5

GMJ\T i'1AHl WOHi<:BOOX 3ll


Tlw tutalcllarge is

.\1· t ; :!.;'f. :!.r ':i~(t e .1.\lt ~l.\lt


:J :J :J :J
~l.\ Ir
~).\[t .l.\1
':i :\1 :J

:m. E

Tlle total cost -- Cost of first mi le ~ Cost of tlle rest

llw cost uf tlle first mile is St. For the rest of tlw tri p. use tbe fee ¡wr mi le times tlw miles.
TI ll' rl'l' ¡wr o·-·¡ 1111·¡ t'S .IS:)- f(j(}
n SlllCl'
. . .
/liS llllTillS.
n r.l't l t1l'
n = :.>_{j·
·¡·¡1l' COSt per 1111·¡ l' .IS :1- · J(f(j

total miles \\T ~1re trying to find be 111:

11
P -"" r+ [111- 1) ·
20
-

Notice that we uc-e (111 - 1) sinn' the first mile is already paid for by f.

p 1
t+ (111 - l) · -' - Subtract t from both sides
20
fl t = (111 ·- 1) ·J.¡~ t\lultiply both sides by '1?·
20
¡¡ (P -- -
r) -- 111 - l Add 1 to both sidcs.
~() (P - f) + 1 = 111
1/

:n. D (1 l l.et's assume )oc m a de Sx on tbc llth da y. Sin ce on thc 12th da y tbe earnings wilf
incrcase by Sx. be will makc S2x. Basically, bis earnings doublc ewry day.

11th day: Sx
12th day: S2.r
13th day: S·lx
J.¡ th tby: SB.r

Sin ce B.r = $:173.-l·l. .r can be casi! y fouml. SU FFIC! ENT

(21 lrt's ;¡<;sume )oe madc SyPn thc first day:

1st da y: y
2nd day: 2y
:lrd day: ·ly
lth day: By
Sth da y: 1f1y
lith day: T!y

T!w Slllll for thc G days is y L :!.\' L ly +-By f· J(iy" :l2y ''$.J..! J. lhis can be easih· sohnl
for 1'. S!JFFICILNT

·¡he ans\\Tr is 11: cach statenwnt a!mw is su!Ticicnt.

312 Gl'-Ló,T MATH WORi<BOOi(

1 ~
32. A IfRonald makes $1,980 in 3 years at simple interest, he makes !l ·;no = $660 per year.

Principal X Interest rate = Interest ($)


lnv. 1 SP 0.06 0.06P
lnv.Z $10.000- p .0.075 (10,000- Pl0.075
Total 660

Interest 1 + Intercst2 = Total intercst


0.06P + 0.075(10,000 - P) = 660
0.06P + 750 - 0.075P = 660

90 = 0.015P
P = $6,000 at 6%
So he earned $-1,000 at 7.5%.
. . 6,000 3
Th e ratiO IS .f,OOO = z·
33. e Let the number of e-mails to be scnt be 11. Thc number of e-mails that reach inboxes
is 60% · n = 0.6 . 11. Only 20% of the pcople die k the links. So 0.2 · O.G · n people go to the
website. Finally,! (0.2 · 0.6 · n) surveys get completed.
i (0.2 . 0.6 . 11) = 360
n = 9,000 e-mails

3,1. e The x-coordinate of thc centcr must be !l since thc x-coordinate of point R is n.
Furthermore, since the circlc is tangcnt to the y-axis atA, its radius is 11. So OA =
OB = OD = n.

If the y-coordina te of point Bis m, then 111 must be a negative number. The distance
from the ccnter to the x-axis is 11 + m. Finally, sin ce OD = 11, the y-coordinate of lJ must
be n + n + m = 211 + m.

So the coordinates ofpoint D are (n, 2n +m).

35. D Cross multiply the quest ;on. \Ve are looking for kn = 1111.

(l) Cross multiply to get k(n- m)= m(t- k).


kn- km = mt- m k
kn = mt SUFFICIENT

(2) Cross multiply to gct k(n + m) = i/z(t + k).


kn +- km = mt + nzk
kn = mt

Thc ans\\'er is D; cach statcnwnt alone is sufficient.

GMAT Mi\TH 'NvRK<>O:< 313


:l!i. e llwlengthoftlll'diagonalof.\/!(:JJis~.\' ' \ .. tll.
llll' kngth uf tlll' diagunal of /·H ;¡¡ is ~(X ¡ 1'!" · (X yl.' ·e cl ..

tl, ~':!. · ~'x·· y


tf 1 ~X y·

JI. B Simplil\ tlw queqiontirst:


/1111 1 11111 lOO
¡¡¡_,¡ = 10 am O. 1111Í

'W? cannot be simplilled \\ithout knm\"ing 111.

( l) 11 = :!. is not enough since all \\'l' twed is 111. :\OT suflkient

(2) 111 == 3 is enough infonnation since ' {G 1


= ~~ = 3.3. SUrT!CIENT
The answer is B; statement (2) alone is sutlicient.

314 C·lAT i'1ATH WOi \800:<


INDEX

Absolute val u e, 19 Counting, 26,74-77 translation of word problems


Addition Cubes, 205 to, 135-1-113
of algebraic expressions, 92 Cumulative graphs, 239 Equilateral triangle, 1H6
of decimals, 3G Cylinders, 205 Equivalen! fractions, 30
of fractions, 31 Even integers, 20--21
Data interpretation, 23-l-239
ofradicals, 112 Experimental qucstions, 6
Data sufficiency questions
of signed numbers, 21 Exponents, 23-2-!, 110-111
challenging the statement,
Admissions, 3 Expression, 131
247-2413
Age problems, 1313 Exterior angle, IH3
description of, 5, 10
Algebra, 91-IJ.!
format of, 241-2-13 Factorial, 75
Algcbraic expressions, 91-95
interpreting the stem of, 2-16 Factoring, 83, 121
Altitude, l8G
val u e questions, 243-2-14 Factors
Anglcs, 1131-113-1
yes/no questions, 245-246 detlnition of, 2·!
Anxiety, 11
Decimals of quadratic equatic;:1s, 122
Arca
addition of, 36 5-12-13 triangle, Hl5
of circle, 202
conversion into fractions, 32 -15-·15-90 triangle, IH5
ofparallelogram, 201
definition of, 36 Fractions
of rcctangle, 201
division of,.37 addition of, 31
ofsquare, 201
fractions corwerted into, 32 comparing, 3-1
of trapezoid, 202
multiplication of, 3fi comp!cx, 32
of triangle, 1BS-186
subtraction of, 3G decimal conwrsions, 32
Arithmetic, 19-90
DescriptiYe statistics, G6-7-1 detlnition of, 30
Arithmetic scquences, 12,1
Diagonal, 1!32 clivisinn of. 31-32
Associatiw propcrty, 20
Diamcter, 202 equiva!mt, 30
:\ verage, 6G--67
Diffcrence of l\VO squarcs, 12 mixrd numbcrs conYertcd
Bar graphs, 23!3 Direct proportions, 51 into, 30
Base, 1135 Discrete probability, /G--77 mu!tiplication o f. 31
Breaks, 6, 1·1 Distributi\·c pro¡wrty, 20 rcclucing o f. 30
Divisibi!ity, 25 "splitting." 32
Central angle, 203
Di\'ision subtraction of, 31
Chord, 202
of algcbraic cxprcssion~. 93 Functions, 123
Cirde graphs, 2:l6-237
of drcimals, 37 [(x), 123
Circles, 202--203
of exponcnts, 110-111
C:ircumfcrcnce, 202--20:l Geomctric SL'qucnccs, 12-l
of fractions, 31-32
C<wflkicnt, a1 Gcomctry, llll-210
of radicals, 112-113
Combination, 76 Grat!u;1tc l\lanagcnll'nt
of signcd numlwrs, 21
Common ditTcrence, 12-t t\dmission Council, 2, 5
Domain, 123
Conmwn ratio, 12-l Graphs, 220--222, 23G--2:!9
C:ommutatin~ propcrty, 20 Edges, 20-1 Gucssi'1~. 1-l
Complcx fractions. 32 Elimination, 101-102
Compound intcre,t, lliH Fquation ofa linc, 225 Hypotcnusc, lB·!
Computch1d~1ptin• test. 7-9 Equations lndcp,·ndent cn·nts, 77--/H
C:onsecutÍ\T intcgcrs, 21, 13'1 linear, !Í~J-1 02, 22:!-22:\ lncqualitics, 103-101
C:oon!inate gt·onwtry, 21 ~J-2:~0 quatlratic, 1!0--122 !nscribcd anglc, 20 l

GlYlJ\T teJAr;.¡ WORK300;( 315


PctiJn\·lt'i. t:n s;,·qlt•.'tll't''· l :.' 1 L':,
llllt'~;t•!'i. 1,, .'.\. 1.\'1
l't'llll \1 [;¡[ i ll\ ~:. s;, ·t 1' r"l ,¡, ·1 11'. 1 ;:\ 1 ~'1
llllt'l t q>l,, ~~ 1
l't·rpt·lldictd.lrlint·s. '<1:~. :~:.>:, ~idc. i H~
lntni"r .tn¡;lt·. JIU
s;i~;l \t ·ti 11 llllllll'r'. ~ 1 .,.,
l:l\t'J"-...t' pn~~lrJ!{itlll"'. ~d .-1:! l'i .. :I!.C
I'Lil·,· '-"ltw . .:, "imil.rr t 1i;111gk. 1;~;
lt\'.l''--\1!\l'li: il1!t'lt",{ ',",\l!"d
1'(11~\~~t~ll. \H~ l~t). ~\l l
<;iml'!,· illll'l't''t. 11;¡¡
111'1 lhlt'll 1'. 1h:\ 1;-'
l'tlh 11<>11\i.ll. 11:.' '.i111piit\ing r;ldit·:ds. 11:.'
lnatitJILtl lllllldlt'l'. 1'1
l'n·p;lrdtitlll ,lr;llt·,j•·'· ~~- 1:.' ~lopt', ~~:~ ~~:2·l
""" ,.¡.., tri;lll~lt·. 1/:ti
l'ri1m· fat'ttJril;lti"ll. ~~, :.'ti s;"lids. ~() \ :.'tl:i
ll';"( l tllllllltlll lllliltipk. ~li s;oluti<Hl "f l't¡lldtion. ~1'1
l'rii!H' 1\\lllllll'r'.-' l
lt•g,;, uf 11 i.u1gk. lf\ 1 l'l<>l>;thilit\' St¡u:m·. ~lll
1.Í!H';!Il't¡\1;11 Ítlll'i, ~1 1 1-lll_', di .L'fl'{l\ ",""!)-77 Squart• rllol. lll. 1 n
of l ~,.,_o or !llore t'\'t'lltS. J""; 7H '.t.rlrd<!ld dt·\·i;rtion. <il-liB
Lint•ar ilwqualitil's. 103 -lO l l'rllhl·:lll·,;ul\·in¡; t¡tll''tlol\'i, :> :.tudy plan. ~J .. lO
l.ilh'S
Produr! uf 11. l :.' l Substitution. ~ll. 101
l't¡\latiull ora. _,_,,-, Sul>trart io11
l'ro 1•o rl ion-;, .!11- :>:.'
in graphs. :!:.':!-:.'.':\ l'\'thagorl'al\ tl:•·on·m .. lB-l. uf :rlgl'brair exprcssions. 92
l\'JH'S oí'. l ll 1-l ¡¡_> :.'Ol, 220 of dl'cimals. :111
of fr;1ctions. :11
\ll'dian.li/ ()uadratir equations, l :.'0-122 of radicals. 112
\lidpoints. :2:20 ()li<Hir<llir formula. 121 of siglwd llllll\lJCrS. 22
\lixl'd numlwrs. :\0 ()uadratic funrtions, 225-:.'21) Su m of al! intnior angh>s. !In. lB/
\ lixt\lfl' prohkms. 1(i 1-l tiB Quadrilatt•rals. 200-20 l Su m of 11 ¡crms, 1:.' l
\lodt'. ti/ <)uantitatÍ\L'Sl'Ction. l:i-l!l Sy1nholis111. l :!3
\lonomial. B l <)ut•,tions
0-lultipks. 2-l d;lta sutlirit'IH'Y· Sce Data Tahle. 2:1-\-:.>:Hi
\fui ti plica t ion sullícit·nc\' qut•stions Tangcnt. :.>02
of algl'hraic l'Xpressions. 92 lcrm. Bl. 12-l
l'>:¡wrimcntal. li
of decimals. :lli problem-so\'.·ing. S Test-day str:ncgic~. 12-15
of t'XpOtll'lll '· 1 l 0-l l l :10-ti0-!10 triangle. lBS
of rranions. :\ l Hadirals. lll-113 3--l-:i tri<lllgle, lB·l
ofradirab.ll:2--ll:\ Hadius, 202 Tra¡wwids. 201--202
of sig1!L'd lltlll\lwrs.:! 1 ICurge. li7, 1:':\ Triangll's. lBl-lBB
\lutually t•:-;dusiw L'\'l'lliS. lB Hall· prohlt'I\\S, 1-lH- l SS
Hatio \'aluc qw•stions. 2-n-2-\-l
:\egatÍ\1' ¡•:.;pont·nts. 111 ('Oilli!Hlll, 12·1 Variahlt-s. Bl
111 J¡ ll'J'Ill. ¡:_>.¡ \'en11 di<rgrams. l 7:\--119
dt•,;niption of. 'í0--:>1
:\u mlwr,; Hat!unaiiHII\\hn'i, 1'l \'t·rt•'\, l H2 . ..'0-l
i rrat ional. 1~~ l{;rt io of tl!t• arcas. 211:1 \'ol\11\ll', ..'O:i
1\lÍ\t'd. :10 lkalnumlwrs, l ~~-~0 \ \'t>rd proll!Pms. l :l:i-1 BO
pri llll'. :.' 1·
lkriprocal. ~:.' <tgP. L\B
f<lliPll<J!. l ~~ lkrtanglt•s. :.'01 t'\rhange. !:lB
lt'<!l. 1:J<~Il Ht·,·langular ,·oordin;\lt' ill\'t'Slllll'l\t/Ínll'l'l'St, ltlfl- [/2
S\Slt'11!, :_> [<) mixture. l!il l(iB
Odd ilriq;t·r'. ~O<'l 1\tTtangubr prisms. :!O-l-2():i ratc. l \B l :i:i
( lrdt'll'd p;1ir. :.' l 'l.:.':.' l 1\t·ducing fr;ll 1i< >n·;, .HI ~~·t. 17:1-179
<lrdt•ring. ;:, lkgi,tning. lB ~oh ing. l :>B-- U~l
llrdn oft'\ll'rati,,ns. "'' lk111:1indns. :.:1; tr;1n~L!tion to l't¡\lation:'.
<lrigin. :.' l ~~ l\qw;1ting 'l'l}\ll'I\Ct's. 1:.':> l:l'í--llB
1\ight al\·r~lt•. lH:! \\tlrk. l:iti--ltil
l';¡¡;J!)()I;rs. ~~:>-~:.'li
1\ight tri<lnglt•. lH-1 I:Hi \ \·ork problcms. l :;¡; .. lli 1
l\u;~llt·lli11l''i. le\:.' . .>:':, H<;(l[S or t¡llddr;ltir
1\lr<tlil'ltlgl;!lllS. :,'()() ~l)l l't¡\!:\lÍllllS, l'l'l x-intt·rn·pt. :.':.'-!
J'arti;il Íll\'l'Slllll'lllS, \(i~l
Huunding. ~7 Yl'< no q\w'ilions. :_>.¡:; ·-' lfi
Pvrrt•nlagl':-. . .-)~i~ :l~l
_l'·Íillt'I'L't•pt. :.':.'·\
1\•rft-t·t sq11:1rt'"· 1:.'

1 316 GMAT t>'lATH WC;KBOOK


1
1
~-

Vous aimerez peut-être aussi